Path questions

Ace your homework & exams now with Quizwiz!

A 16-year-old boy has a 6-month history of type 1 diabetes mellitus and requires daily insulin injections. His blood glucose control has been poor recently, which the physician suspects may be related to the patient's reluctance to self-administer insulin. Which of the following questions is most appropriate to broach this issue with the patient? "You seem to have missed your shots. How often has this happened?" "You are clearly having difficulty with insulin injections. Would you like to arrange for a family member to give you your shots?" "Giving yourself insulin injections can be difficult. What's it like for you?" "It's been my experience that most poorly controlled diabetics have trouble giving themselves insulin. Do you have this problem?" "I get the feeling you have not been taking your insulin regularly. Would you like me to arrange some training for you by our nurses?"

"Giving yourself insulin injections can be difficult. What's it like for you?"

A 4-year-old boy is brought to the emergency department because of a 1-hour history of headache, nausea and vomiting, and dizziness. Physical examination shows severe lethargy. His venous blood is bright red; carboxyhemoglobin saturation is 30°/c, (reference range < 5°/c,). Which of the following steps in the electron transport pathway is primarily affected in this patient? Reduced A B substrate ., NAO+ ., Flavin C D E -• coa-• Cyt b • Cyt c-• Cyt aa3 • 0mononucleotide 2 0 A) (cyta + a3)

(cyta + a3)

A 42-year-old woman comes to the physician because of a 3-week history of numbness of the fingers on the left hand. Neurologic examination shows loss of touch graphesthesia and loss of two-point discrimination in the left hand. This patient most likely has a lesion in which of the following locations of the hemispheres shown? (somatosensory cortex)

(somatosensory cortex)

A 54 year old woman comes to the physician with a persistent cough she's had for the past 3 months. She has also lost weight recently and has woken up in the middle of the night drenched in sweat. Further history reveals that she has ulcerative colitis which is being controlled with medications. Following the physical exam, the physician orders a chest xray. After seeing the xray results, the physician immediately prescribes an antibiotic regimen and asks the patient to discontinue one of the drugs used to treat her ulcerative colitis. Which of the following drugs increased her risk of developing her disease? (tumor necrosis factor alpha inhibitors) ex: infliximab

(tumor necrosis factor alpha inhibitors) ex: infliximab

An investigator has conducted an experiment to determine whether certain environmental exposure morbidity is eliminated if a person carries a specific allele of three different genes on three separate chromosomes. The frequencies of an individual having the allele for these respective genes are 0.6, 0.2, and 0.1. All three alleles arenecessary to confer protection. The probability that a randomly selected individual will have all three alleles is closest to which of the following? 0.6 0.012 0.2 0.1 0.9

0.012

A 4-year-old boy (111-2 in the pedigree) has had a clumsy gait for the past year. Examination shows calf hypertrophy and proximal muscle weakness. Serum creatine kinase activity is increased. Muscle biopsy shows loss of muscle tissue, regenerating muscle fibers, and fibrosis. A maternal uncle (1 1-3) had similar findings and died at age 15. The patient's 20-year-old sister (111-1) is pregnant; ultrasonography identifies a male fetus. Which of the following is the probability that this male fetus has inherited this disorder? 0.5 0 0.25 0.125 0.75

0.5

A 55-year-old man is admitted to the hospital for treatment of sepsis. He appears anxious and confused. Treatment with vancomycin and ceftriaxone was initiated in the emergency department. His temperature is 39.8° C (103.6°F), pulse is 132/min, respirations are 28/min, and blood pressure is 85/48 mm Hg. Physical examination shows warm, flushed skin that is moist with diaphoresis. There is no edema. It is most appropriate to administer which of the following solutions at this time? 0.9% Saline 5% Dextrose in water and 0.45% saline 3% Saline 0.45%, Saline only 5% Dextrose in water only

0.9% Saline

A 71-year-old woman comes to the physician for a follow-up examination. She has a 30-year history of type 2 diabetes mellitus currently treated with insulin. She is 160 cm (5 ft 3 in) tall and weighs 59 kg (130 lb); BMI is 23 kg/m2. Her blood pressure is 116/78 mm Hg. This patient is most likely to have an endogenous fasting serum insulin concentration (N=5-20 μU/mL) closest to which of the following? 20 μU/mL 10 μU/mL 80 μU/mL 40 μU/mL 1 μU/mL

1 μU/mL

A 40-year-old African American woman comes to the physician because of a 2-week history of fever, malaise, and dyspnea. Her temperature is 36.7°C (98° F), and respirations are 20/min. Physical examination shows erythema nodosum, parotid enlargement, and hepatosplenomegaly. Her serum calcium concentration is 16 mg/dL. A CT scan of the chest shows bilateral hilar adenopathy. Serum studies are most likely to show an increased concentration of which of the following? Cholecalciferol Parathyroid hormone 1,25-Dihydroxycholecalciferol 24,25-Dihydroxycholecalciferol Calcitonin

1,25-Dihydroxycholecalciferol

Genetic counseling and cystic fibrosis. What is the probability that a child born to a mother from the Caucasian community and a father from that Asian community will have the disease? 1/900 1/12,000 1/3,000 1/6,000 1/1,000

1/12,000

Which of the following represents the risk of misscarriage from amniocentesis? 1/100 - 1/200 1/500 1/50 - 1/100 1/200 - 1/400 1/1000

1/200 - 1/400

Amniocentesis is typically performed during which of the following periods of gestation? 10 -18 weeks 14 - 20 weeks 12 - 16 weeks 10 - 14 weeks 12 - 14 weeks

14 - 20 weeks

A female newborn is delivered at 38 weeks' gestation. Apgar scores are 8 and 8 at 1 and 5 minutes, respectively. Physical examination shows a bulging, fluid-filled mass approximately 5 cm in diameter in the midline over the lumbosacral region. There are no spontaneous movements of the lower extremities. This abnormality most likely occurred because of abnormal embryonic development during which of the following periods postconception (in days)? 60 to 75 100 to 120 15 to 40 150 to 180 1 to 10

15 to 40

A 78-year-old male with syncope feels heaviness in his chest with exertion and breathlessness when lying down. A crescendo/decrescendo systolic murmur is heart best at the second right intercostal space with radiation to the carotid arteries. Which of the following measurements of left ventricular and aortic pressure is most likely to be found on cardiac catheterization? 100/10 and 150/90 150/10 and 150/80 100/18 and 100/50 150/18 and 150/80 150/18 and 100/50

150/18 and 100/50

A 5-year-old girl is brought to the physician because of a chronic cough that is worse at night and productive of thick green sputum. She has a history of intermittent cramps in the right lower quadrant of the abdomen and frequent respiratory tract infections. Her 8-year-old brother is healthy. Physical examination shows clubbing of the fingers. There is hyperresonance on chest percussion. Diffuse medium-pitched inspiratory crackles and scattered pulmonary wheezes are heard during expiration. A chest x-ray shows evidence of diffuse hyperinflation, which is especially prominent in the right upper lobe. Which of the following best represents the likelihood of this patient's brother being a carrier of this condition? 2 of 3 1 of 1 1 of 4 1 of 3 1 of 2

2 of 3

In a clinical study, a polymorphic marker with three alleles, 1, 2, and 3, is found to be tightly linked to the gene for polycystic kidney disease. A pedigree is shown. If Ill, 1 is unaffected by this disease, this patient is most likely a carrier of which of the following markers? 2,3 1, 3 1, 2 2, 2 1, 1

2,3

A 25-year-old woman comes to the physician because of a 3-year history of irregular menses. Menarche was at the age of 14 years. Puberty began at the age of 12 years and progressed normally. Her blood pressure is 116/62 mm Hg. Physical examination shows increased hair growth on the face and chest. Pelvic examination shows clitoromegaly and a normal-appearing uterus. Serum studies show the following. Item Concentration Na+ 139 mEq/L Cl- 90 mEq/L K+ 7 mEq/L HCO3- 17 mEq/L Glucose 42 mg/dL BUN 25 mg/dL Creatinine 0.4 mg/dL 17-Hydroxyprogesterone Increased This patient most likely has a deficiency of which of the following enzyme activities? 11 (3-Hydroxylase) O 8) 17a-Hydroxylase O C) 2 1 -Hydroxylase O 0) 3(3-Hydroxysteroid dehydrogenase O E) 5a-Reductase 21 hydroxylase

21 hydroxylase

A 4-day-old boy is brought to the pediatrician's office early in the morning because he had vomited throughout the night after breastfeeding. Physical examination is normal except for lethargy and dry mucous membranes. Laboratory studies show: Serum Na + cIK + HC0 3- Glucose Urea nitrogen (BUN) Creatinine 17-Hydroxyprogesterone The most likely explanation for these findings is a deficiency in which of the following enzymes? 21 hydroxylase 20,22-Desmolase 17-Ketoreductase 17a-Hydroxylase 11 -Hydroxylase

21 hydroxylase

A 24-year-old female with a familial history of congenital heart disease, intellectual disability, and psychiatric illness presents for prenatal genetic counseling. History reveals her father was born with a heart defect and developed schizophrenia as an adult, while her brother has cleft palate, a heart defect as well, and intellectual disability. Cytogenetic analysis of the father and brother will most likely reveal which of the following? 22q11.2 microdeletion 4q16.3 microdeletion 7q11.23 microdeletion Maternal 15q11-13 microdeletion 5p15.2 microdeletion Paternal 15q11-13 microdeletion

22q11.2 microdeletion

During a clinical study of calcium and phosphorus metabolism, a 50-year-old man undergoes a series of laboratory studies. His serum calcium, phosphorus, and parathyroid hormone concentrations are within the reference ranges. He is given an infusion of 2 g of calcium chloride over 2 hours. His serum calcium concentration now is 11.5 mg/dl. Compared with the preinfusion level, the serum concentration of which of the following substances is most likely to be increased at this time? Vitamin D3 7-Dehydrocholesterol 1,25-Dihydroxycholecalciferol 24,25 dihydroxycholecalciferol Previtamin D3

24,25 dihydroxycholecalciferol

A 30-year-old woman and her 25-year-old husband come to the physician for advice about their risk for having another child with Tay-Sachs disease. Their 6-month-old son was recently diagnosed with this disease. There is a history of this disease in both the woman's and the man's families. Which of the following best represents the risk for this couple's next child to have Tay-Sachs disease? <1 % 10% 50% 100% 25%

25%

A 45-year-old woman comes to the physician for a follow-up examination 8 weeks after beginning tamoxifen therapy for estrogen- and progesterone-positive invasive ductal carcinoma of the breast. Her 50-year-old sister also has hormone-sensitive breast cancer treated with tamoxifen. Physical examination shows no abnormalities. Serum studies show decreased concentrations of endoxifen, the active metabolite of the prodrug tamoxifen. Genetic analysis shows the homozygous presence of cytochrome P450 206*4 alleles. Which of the following best represents the likelihood that this patient's sister has the same alleles? 25% 0% 100% 50% 75%

25%

A previously healthy 77-year-old woman who resides in askilled nursing care facility is brought to the emergencydepartment 6 hours after the onset of acute midback pain thatbegan while lifting a box. The pain does not radiate, and shehas no other symptoms. She continues to carry out her dailyactivities. She appears to be in mild distress. She is 157 cm(5 ft 2 in) tall and weighs 47 kg (104 lb); BMI is 19 kg/m2.Examination shows mild tenderness over T11. There is notremor. Serum studies show a calcium concentration of9.1 mg/dL, a urea nitrogen concentration of 12 mg/dL, and acreatinine concentration of 0.5 mg/dL. An x-ray of the dorsaland lumbar spine shows an anterior wedge fracture of T11.In addition to treating the pain, supplementation with whichof the following is most likely to improve this patient'sunderlying condition? Vitamin C Levothyroxine Selenium Vitamin E 25-Hydroxycholecalciferol

25-Hydroxycholecalciferol

A 25-year-old woman comes to the physician for counseling prior to conception. She has seizure disorder well-controlled with valproic acid. She asks the physician if her fetus would be at risk because of her drug therapy. The fetus would be at greatest risk for a drug-related adverse effect at which of the following stages of pregnancy? 12 to 18 weeks 3 to 8 weeks 0 to 2 weeks 9 to 12 weeks 18 to 24 weeks

3 to 8 weeks

A 24-year-old woman is brought to the emergency department after taking an overdose of a drug (Drug X). There is no specific treatment for this type of overdose, but it is known that Drug X follows first-order kinetics. Laboratory studies show a serum concentration of 32 μg/ml of Drug X; 6 hours later, the serum concentration is 16 μg/ml. Current guidelines indicate that the patient should not be moved from the intensive care unit until the serum concentration of the drug is 1.1 μg/ml or lower. Which of the following best represents the minimum number of hours from the time of the first blood sample (32 μg/ml) that the patient must remain in the intensive care unit for monitoring? 36 24 18 30 42

30

A 65-year-old man with a history of ischemic heart disease comes to the physician because of fatigue and pain and swelling in his legs. His blood pressure is 135/80 mm Hg and pulse is 80/min. End-systolic volume is 80 ml and end-diastolic volume is 120 ml. Which of the following best represents the ejection fraction in this man? 50% 33% 40% 25% 67% 36%

33%

A 39-year-old female presents for prenatal counseling prior to conception. Which of the following karyotypes would puts this individual at greatest risk of having a child with Down syndrome? 46,XX,del(7)(q11.23) 46,XY,del(22)(q11) 45,XX,t(14q;21q) 47,XX+13 47,XX+18

45,XX,t(14q;21q)

Case of Robertsonian translocation-mediated Down syndrome. Which of the following karyotypes is most likely to be found in this infant? 47,XX,+18 45,XO 46,XX,t(14;21) 46,XX,del(22)(q11) 46,XX,t(9;22)

46,XX,t(14;21)

An adolescent male with a ventricular septal defect and a history of delayed developmental milestones, e.g., language and speech. Physical examination reveals dysmorphic facial features, e.g., small mouth, narrow nose, and upward slant of the eyes. Which of the following is the most likely chromosomal abnormality of this disorder? 46,XX,del(7)(q11.23) 47,XX+13 46,XY,del(22)(q11) 47,XX+18 46,XY,t(11;22)(q23;q11.2)

46,XY,del(22)(q11)

A 22-year-old man comes to the physician because of infertility. The photomicrograph shows tissue obtained on biopsy of the scrotal testes. Which of the following is the most likely genetic abnormality? 46,XY 45,X/46,XY 47 XXY 47,XXX 46,XX

47 XXY

Case of Edwards syndrome. Which of the following is the most likely chromosomal abnormality in this infant? 47,XX+21 22q11 deletion 5p deletion 47,XX+13 47,XX+18

47,XX+18

A 21-year-old woman who is a medical student comes to the physician for evaluation of recurrent throbbing headache associated with sensitivity to light and sound. Neurologic examination shows no abnormalities. Sumatriptan is initiated for suspected migraine. This drug is most appropriate for this patient because of its ability to activate which of the following types of receptors? 5HT3

5HT3

To decrease his risk for cardiovascular disease, a 24-year-old man begins a weight-loss diet. He is 153 cm (5 ft 8 in) tall and weighs 95 kg (210 lb); BMI is 32 kg/m 2. He intends to lose 16 kg (35 lb) by limiting his daily caloric intake to 2000 calories. In order to maintain the recommended protein intake (56 g daily), a balanced decrease in carbohydrates and fat is required (caloric ratio of fat to carbohydrate is 30:55). Which of the following best describes the number of calories that should be provided by fat in this patient's diet each day? 510 430 370 740 630

630

A 35-year-old woman is given 500 mg of drug X intravenously. Several minutes later, the serum concentration of drug X is 12.5 mg/L. Two hours later, the serum concentration is 10 mg/L. Assuming that drug X has first-order elimination kinetics, which of the following will be the serum concentration of drug X (in mg/L) in another 2 hours? 6.5 6 8 8.5 7.5 7

8

A study is conducted to assess a new prenatal test for Down syndrome that is minimally invasive. The number of fetal chromosomes is predicted based on the analysis of multiple single nucleotide polymorphisms for chromosomes 13 and 21. The test is considered positive if the ratios of fetal to maternal DNA for the two chromosomes are different at p < 0.05. The test is administered to 120 women at 12 to 20 weeks' gestation. A definitive diagnosis of Down syndrome is made by either amniocentesis or postnatal examination. The results are shown: Maternal Plasma DNA Analysis Positive Negative Total Amniocentesis or Postnatal Examination Down Syndrome Normal 4 1 2 1 1 3 6 114 Based on these data, which of the following best represents the specificity of this new test for detecting Down syndrome? 33% 99% 67% 57% 80%

99%

A 2-year-old boy is brought to the physician because of increased thirst, urinary frequency, and failure to thrive. He appears frail. He is at the 5th percentile for length and 7th percentile for weight. His temperature is 38°C (100.4 °F). Physical examination shows dehydration and decreased muscle tone. The diagnosis of Fanconi syndrome is made. Which of the following sets of changes in fractional reabsorption in the kidneys is most likely in this patient? AA: decreased, glucose: decreased, phosphate: decreased, HCO3-: decreased

AA: decreased, glucose: decreased, phosphate: decreased, HCO3-: decreased

A 35-year-old man comes to the physician because of severe midline back pain during the past 4 weeks. He has gained 8 kg (18lb) during the past 6 months. Visual field testing shows decreased peripheral vision in both eyes. An x-ray of the chest shows compression fractures at T 10 and L 1. There is a healing left rib fracture posteriorly at T6. Imaging studies show a pituitary macroadenoma. The most likely cause of his condition is a tumor producing which of the following hormones? Luteinizing hormone Growth hormone ACTH Thyroid-stimulating hormone Prolactin

ACTH

A 51-year-old man comes to the physician because of a 2-week history of cough productive of blood. He also has had a 3-kg (6.6-lb) weight loss during the past month. He has smoked 2 packs of cigarettes daily for 32 years. He has hypertension treated with metoprolol. Physical examination shows clubbing of the fingernails. Wheezes are heard over the right upper lobe anteriorly; there are no crackles. Laboratory studies show: Serum Na+ 125 mEq/L K+ 3.9 mEq/L Cl- 85 mEq/L HC03- 25 mEq/L Urea nitrogens mg/dl Creatinine 0.4 mg/dl Osmolality 262 mOsmol/kg Urine osmolality 564 mOsmol/kg A CT scan of the chest shows a 2.4 x 1 .2-cm, spiculated noncalcified mass in the right upper lobe eroding into the right bronchus. This patient's laboratory findings are most likely caused by an excess of which of the following hormones? Adrenocorticotropic hormone Cortisol ADH Thyroxine (T 4) Brain natriuretic peptide

ADH

A 2-year-old male has higher blood pressure in the right arm than in the left arm, decreased femoral pulses, and a systolic ejection murmur heard best below the left clavicle. Which of the following is the most likely cause of this congenital anomaly? Persistent fetal circulation Incomplete partitioning of the atrioventricular canal Transposition of the great vessels Abnormal development of the aorta Dextrocardia with situs inversus

Abnormal development of the aorta

A 57-year-old woman comes to the physician for a routinefollow-up examination. She has a 5-year history ofhypertension treated with captopril. She has had a 6.8-kg(15-lb) weight gain since her last visit 1 year ago; she is165 cm (5 ft 5 in) tall and currently weighs 72 kg (160 lb);BMI is 27 kg/m2. Her temperature is 37°C (98.6°F), pulse is88/min, respirations are 14/min, and blood pressure is 160/86mm Hg. The lungs are clear to auscultation. Cardiacexamination shows no abnormalities. The abdomen is softand nontender with no palpable masses. Pedal pulses andsensation to pinprick are diminished bilaterally. Fastingserum studies show a total cholesterol concentration of240 mg/dL and glucose concentration of 182 mg/dL. Whichof the following is the most likely cause of these findings? Excess glycogen breakdown Abnormal insulin secretion with insulin resistance Insulin allergy Excess release of glucagon andcatecholamines Autoimmune β-cell destruction

Abnormal insulin secretion with insulin resistance

A 32-year-old woman undergoes an operation after the diagnosis of hyperparathyroidism is made. During the operation, three parathyroid glands are found on the posterior surface of the thyroid gland, but one does not appear in its normal superior location on the right side. Which of the following embryologic events is the most likely cause of the findings in this patient? Failure of neural crest cells to migrate through the pharyngeal arches Absence of inductive factors in occipital somite mesoderm Incomplete formation of the third pharyngeal pouch Abnormal migration of endoderm from 4th pharyngeal pouch Premature involution of the thyroglossal duct

Abnormal migration of endoderm from 4th pharyngeal pouch

A 12-year-old boy is brought to the physician because of a 6-month history of difficulty walking and leg pain. Physical examination shows mild atrophy of the lower extremities and hammer toes. He walks with a high-stepping gait. Examination of a nerve biopsy specimen from this patient will most likely show which of the following findings? Degeneration of oligodendrocyte Lymphocytic infiltration Absence of Schwann cells Degeneration of astrocytes Abnormal myelin sheaths

Abnormal myelin sheaths

A 3-year-old girl is brought to the physician because of a 1-year history of short stature. She has no history of major medical illness. She is below the 3rd percentile for height and at the 10th percentile for weight. Physical examination shows coarse facial features and contractures of the large joints. X-rays show dysostosis multiplex. Plasma lysosomal enzyme analysis shows increased -hexosaminidase, - glucuronidase, -galactosidase, and a-fucosidase activities. Which of the following mechanisms is the most likely cause of the lysosomal enzyme findings in this patient? Abnormal targeting of these enzymes to lysosomes Storage of these enzymes within the cytoplasm Degradation of these enzymes within the cytoplasm Degradation of these enzymes within the lysosomes Deficiency of the other lysosomal enzymes in the cytoplasm

Abnormal targeting of these enzymes to lysosomes

A 75-year-old female is admitted for treatment of a small right pleural effusion. Physical examination shows dullness to percussion at the level of the 7th rib and below on the right. A thoracentesis is performed. The patient is positioned, and the catheter insertion site is prepared and anesthetized. Which of the following represent the most appropriate location for the insertion the catheter? Below the 5th rib in the midclavicular line Below the 9th rib in the midscapular line Below the 5th rib in the midaxillary line Above the 5th rib in the midscapular line Above the 7th rib in the paravertebral region Above the 5th rib just to the right of the sternum Above 9th rib in the midscapular line

Above 9th rib in the midscapular line

A 19-year-old primigravid woman comes to the physician for her first prenatal visit. Her last menstrual period was 18 weeks ago. Her pregnancy has been complicated by cocaine use. She has no history of serious illness and takes no medications. Her temperature is 36.7°C (98°F), pulse is 90/min, respirations are 16/min, and blood pressure is 110/50 mm Hg. Examination shows scleral icterus and poor dentition. The fundal height is 20 cm. The remainder of the examination shows no abnormalities. This patient is at greatest risk for which of the following conditions? Macrosomia Abruptio placentae Gestational diabetes Postdates pregnancy Pituitary infarction

Abruptio placentae

An 18-year-old man has yellow nodules on the Achilles tendons of his feet and extensor tendons of his hands. Examination of tissue obtained on biopsy of one nodule shows a collection of foamy histiocytes within the dermis. His serum cholesterol concentration is 980 mg/dL, and serum lipoprotein electrophoresis shows a selective increase in LDL. Which of the following is the most likely underlying disorder? Absence of functional LDL receptors in hepatocytes Partial reduction in functional LDL receptors of hepatocytes Defective receptor- binding 8-100 protein in LDL particles Diminution of receptor-independent LDL uptake by mononuclear phagocytes Increased gastrointestinal absorption of dietary fat

Absence of functional LDL receptors in hepatocytes

A 63-year-old man comes to the emergency department 30 minutes after the sudden onset of right-sided chest pain and shortness of breath that began after a 10-hour drive in his truck. The pain is more severe with inspiration. He does not smoke cigarettes. His pulse is 1 10/min, and respirations are 32/min. The lungs are clear to auscultation. Which of the following findings is most likely on ventilation-perfusion lung scans? Decreased perfusion gradient from the apices to the bases of the lungs Absence of venti lation in some areas that are perfused Increased venti lation-perfusion ratios at lung bases compared with those at the apices Absence of ventilation-perfusion mismatching Absence of perfusion in some areas that are ventilated

Absence of perfusion in some areas that are ventilated

A 25-year-old woman comes to the physician because of a 2-year history of intermittent, diffuse, cramping lower abdominal pain. The pain is usually associated with 2 to 6 days of loose, watery stools, and is typically relieved with defecation. Between these episodes, her stools are normal. Her vital signs are within normal limits. Physical examination shows no abnormalities. Laboratory studies, including complete blood count, metabolic panel, and thyroid function tests show no abnormalities. A drug targeting which of the following mechanisms of action is most appropriate for this patient? Inhibition of 5-hyd roxytryptamine 1 receptor Inhibition of colonic water reabsorption Accentuation of mu-opioid myenteric plexus receptor Inhibition of tumor necrosis factor-a Accentuation of bile salt reabsorption

Accentuation of mu-opioid myenteric plexus receptor

A 60-year-old man has had episodes of fluctuating weakness for the past 2 years. The weakness was first noticed when he had difficulty lifting books. At times, he has had drooping of the eyelids and double vision. Recently, he has had difficulty climbing stairs and regurgitation of liquids into his nose. On physical examination, the weakness increases with repeated exercise. An x-ray of the chest shows a mass in the mediastinum. Which of the following laboratory assays is most likely to be positive? Rapid plasma reagin Acetylcholine receptor antibody Acetylcholine receptor antibody Botulinum toxin C-reactive protein Rheumatoid factor

Acetylcholine receptor antibody

A 65-year-old man with hypertension volunteers to participate in a clinical trial of a newly developed loop diuretic. As part of the study, his acid-base/volume status is monitored. After 3 days of treatment, which of the following sets of findings is most likely in this patient? Acid base: metabolic alkalosis, volume contraction: yes

Acid base: metabolic alkalosis, volume contraction: yes

A 45-year-old man comes to the physician because of an enlarging face, shoulders, and trunk and thinning of his arms and legs. Physical examination shows a round, plethoric face, fat pad over the upper thoracic spine, and purple striae on the abdomen. Serum studies show an undetectable adrenocorticotropic hormone (ACTH) concentration and an increased cortisol concentration. Administration of low-dose dexamethasone would most likely result in which of the following sets of serum findings? Acth; no change, cortisol no change

Acth; no change, cortisol no change

A 42-year-old farmer has a 7-mm red scaly plaque on the helical rim of the left ear. A photomicrograph of tissue obtained on biopsy of the plaque is shown. Which of the following is the most likely diagnosis? Actinic keratosis Merkel cell carcinoma Malignant melanoma Keratoacanthoma Basal cell carcinomal

Actinic keratosis

A 45-year-old man has a 2-day history of severe abdominal pain. The patient admits to consuming large amounts of alcohol over the past few days. Laboratory studies show serum amylase activity of 700 U/L, serum lipase activity of 850 U/L and serum calcium concentration of 7 mg/dl. Appropriate therapy is initiated; however, the patient dies after 48 hours. The photomicrograph shows the mesenteric fat as seen at autopsy. Which of the following best explains the appearance of the fat? Severe malnutrition associated with alcoholism Acute alcoholic hepatitis Direct toxic effect of alcohol on adipose tissue Action of pancreatic enzymes on adipose tissue Acute appendicitis with rupture

Action of pancreatic enzymes on adipose tissue

A 35-year-old man is brought to the emergency department because of a 2-hour history of severe fatigue and dizziness. He has had profuse, watery diarrhea for 8 hours despite a lack of oral intake. He recently returned from a medical relief trip to a remote village in Honduras. His temperature is 36°C (98°F), pulse is 122/min, and blood pressure is 90/50 mm Hg. Physical examination shows dry skin and decreased capillary refill. Test of the stool for occult blood is negative; the stool is gray and turbid. A Gram stain of the stool shows predominant gramnegative, comma-shaped bacteria; there are no erythrocytes or leukocytes. Which of the following best describes the mechanism of the toxin that caused these findings? Activation of guanylyl cyclase Deactivation of ADP-ribosylation factors Activation of phosphodiesterase Activation of adenylyl cyclase Activation of Na+ -K+ ATPase

Activation of adenylyl cyclase

A 10-year-old boy who has had type 1 diabetes mellitus for 1 year is receiving insulin. One hour after his morning dose of insulin, he becomes tremulous and diaphoretic and has tachycardia. Several hours later, his symptoms resolve. His blood glucose concentration is now increased. Which of the following is the most likely cause of this patient's hyperglycemia? Activation of hepatic adenylyl cyclase Activation of muscle protein phosphatase Activation of muscle glycogen synthase Activation of muscle phosphorylase Inhibition of hepatic protein kinase A

Activation of hepatic adenylyl cyclase

A 47-year-old woman comes to the physician because she wants to "do something about my wrinkles." Physical examination shows fine wrinkles in photodamaged skin on the face. Treatment with tretinoin is started. Collagen synthesis will most likely be increased in this patient by which of the following mechanisms? Increased sebum production Activation of nuclear gene transcription Decreased cAMP production Protection of keratinocytes from UVB irradiation Displaced vitamin A from cellular stores

Activation of nuclear gene transcription

A 39-year-old man has the acute onset of pain, corneal clouding, and diffuse redness in the left eye. There is no discharge. Vision is 20/20 in the right eye and 20/100 in the left eye. The left pupil is dilated. Which of the following is the most likely cause of these findings? Acute hordeolum Subconjunctival hemorrhage Corneal abrasion Acute glaucoma Bacterial conjunctivitis

Acute glaucoma

An otherwise healthy 29-year-old male presents with severe fatigue and mild weight loss, and mild periostitis (bone pain) over the past ten days. Physical examination reveals left quadrant tenderness and enlargement consistent with splenomegaly and a palpable hepatomegaly. Laboratory studies show: Hemoglobin 9.6 g/dL; Hematocrit 39.7%; Leukocyte count 35,000/mL; Platelet count 550,000/mL; and circulating CD34+ 350 cells/mL. Which of the following is the most likely diagnosis? Acute myelogenous leukemia Papovavirus infection Thrombotic thrombocytopenic purpura Epstein-Barr viral infection Aplastic anemia Acute megakaryocytic leukemia

Acute myelogenous leukemia

An 80-year-old woman is being evaluated for suspected temporal arteritis (TA). Her erythrocyte sedimentation rate (ESR) is 100 mm/h. Treatment of TA involves the use of glucocorticoids, which could have serious adverse effects. The pretest probability for TA is 50%, in this patient. In the evaluation of TA, ESR has a sensitivity of 99% and a specificity of 60%. Based on the results of the ESR testing in this patient, which of the following is the most appropriate next step in management? Additional testing confirm the diagnosis of TA Corticosteroid therapy, since the diagnosis of TA has been established with 99% certainty Elimination of TA from further diagnostic consideration Repeat ESR; if again positive, corticosteroid therapy Repeat ESR; if normal, additional testing to confirm the diagnosis of TA

Additional testing confirm the diagnosis of TA

A 35-year-old man comes to the physician because of several episodes of squeezing chest pain during the past 2 months. Physical examination shows no abnormalities. An angiogram is done that shows normal coronary arteries. During the angiogram, the patient receives an intravenous infusion of norepinephrine (NE), and his total coronary blood flow is monitored continuously for 5 minutes. The graph as shown depicts the results following NE infusion (arrow). Which of the following substances is the most likely cause of the increased total coronary blood flow observed 1 to 2 minutes after the NE infusion? Angiotensin II Adenosine Epinephrine Thromboxane A2 Histamine

Adenosine

A 23-year-old woman comes to the physician because of a 1-day history of chills, nausea, and vomiting. She also has a 4-day history of flank pain, urinary frequency and urgency, and pain with urination. Her temperature is 38.9°C (102°F), pulse is 125/min, and blood pressure is 96/60 mm Hg. Physical examination shows right costovertebral angle tenderness. Urinalysis shows 25 WBC/hpf. A urine sample grows pink colonies on lactose-MacConkey agar, produces a characteristic green sheen on eosin-methylene blue agar, and is indole positive. The production of which of the following by the bacterium aids in the virulence of the causal organism in this patient? Thick peptidoglycan layer Teichoic acid in the cell wall Antihemolytic factors Adhesins Pyrogenic exotoxin

Adhesins

A 35-year-old man who uses crack cocaine daily comes to the emergency department because of a 2-hour history of substernal chest pain. He last used cocaine 6 hours ago. His temperature is 37°C (98.6°F), pulse is 110/min, respirations are 14/min, and blood pressure is 160/100 mm Hg. Which of the following is the most appropriate next step in patient care? Admit the patient for possible myocardial ischemia Educate the patient about the risks of cocaine use and send the patient home Transfer the patient to a psychiatric facility Admit the patient to the hospital for possible myocardial ischemia

Admit the patient for possible myocardial ischemia

A 19-year-old woman is admitted to the hospital for antibiotic treatment of meningococcal meningitis. She is stabilized. Three days later, her pulse is 120/min, and blood pressure is 60/30 mm Hg. Physical examination shows bilateral flank tenderness. Serum studies show a sodium concentration of 128 mEq/L, potassium of 5.4 mEq/L, and bicarbonate of 20 mEq/L. Which of the following is the most appropriate next step to determine the cause of this patient's hypotension? Thyrotropin-releasing hormone stimulation test Urine culture and antibiotic sensitivity test Dexamethasone suppression test Blood culture and antibiotic sensitivity test Adrenocorticotropic hormone stimulation test

Adrenocorticotropic hormone stimulation test

A 60-year-old man who emigrated from China to the USA is brought to the physician because of a 1-month history of progressive confusion and swelling of the abdomen and legs. He has no history of major medical illness. He was a peanut farmer in China. He has smoked 1 pack of cigarettes daily for 40 years; he does not drink alcohol. Physical examination shows mild ascites and ankle edema. The liver is palpable and firm at the right costal margin; the spleen is not palpable. Serum studies show: Albumin 4 g/dL Total bilirubin 2.5 mg/dL Alkaline phosphatase 200 U/L AST 45 U/L ALT 60 U/L lgG anti-hepati tis A virus positive Anti-hepatitis B virus negative Anti-hepatitis C virus negative Ultrasonography of the abdomen shows a 10-cm hypodense lesion in the right lobe of the liver. A biopsy specimen of the lesion shows dysplastic hepatocytes in small clusters and rows three to four cells in thickness. No normal architecture is seen within the lesion. Exposure to which of the following best explains the liver biopsy findings in this patient? Aflatoxin Nicotine Nitrosamines Hepatitis A virus Mercury

Aflatoxin

A 51-year-old woman with Graves disease develops ulcerating pharyngitis after 6 months of daily propylthiouracil therapy. Which of the following is the most likely underlying cause of ulcerative pharyngeal disease in this patient? Leukocytoclastic vasculitis Tumor necrosis factor-a blockade Thrombosis Agranulocytosis Thyroid necrosis

Agranulocytosis

A 77-year-old woman is visited by the home care nurse whonotes that the patient is more lethargic than usual. Her skin and mucous membranes are dry. An increase in the serum concentration or activity of which of the following provides the strongest indication that the patient is dehydrated? Albumin Uric acid Bilirubin Calcium Alkaline phosphatase

Albumin

A 9-year-old girl is brought to the physician by her adoptive parents because they are concerned about her increasing difficulty at school since she began third grade 7weeks ago. Her teachers report that she is easily frustrated and has had difficulty reading and paying attention. She also has had increased impulsivity and more difficulty than usual making and keeping friends. Her biologic mother abused multiple substances before and during pregnancy, and the patient was adopted shortly after birth. She is at the 20th percentile for height and 40th percentile for weight. Examination shows a flattened nasal bridge and a long philtrum. During the examination, she is cheerful. Psycho educational testing shows an IQ of 82. The most likely explanation for these findings is in utero exposure to which of the following? PCP (phencyclidine) Cocaine Ecstasy (3,4-methylenedioxymethamphetamine) Marijuana Heroin Alcohol

Alcohol

A previously healthy 32-year-old man is brought to the emergency department because of a 1-day history of intermittent flashing spots and blurred vision, vomiting, confusion, and difficulty walking. He is a painter and says that yesterday he cleaned up a spilled bottle of paint thinner. Ophthalmologic examination shows dilated pupils with hyperemia of the optic disc and retinal edema. Physical examination shows tachypnea. Serum studies show an anion gap metabolic acidosis. The most appropriate pharmacotherapy for this patient is most likely to inhibit the activity of which of the following? Lactate dehydrogenase Alcohol dehydrogenase Aldehyde dehydrogenase Pyruvate dehydrogenase Formaldehyde dehydrogenase

Alcohol dehydrogenase

A 55-year-old woman who is a physician is admitted to the hospital because of a fractured femur sustained in a motor vehicle collision. Two days after admission, she develops tachycardia, restlessness, diaphoresis, and anxiety. She says that "vague shapes" are coming out of the walls. Which of the following is the most likely cause of this patient's current condition? Acute stress disorder Alcohol withdrawal Panic disorder Histrionic personality disorder Bipolar disorder

Alcohol withdrawal

A 4-month-old boy is brought to the emergency department because of a 1-day history of respiratory distress. His parents say that he has had progressive weakness and difficulty feeding during the past month. His respirations are 50/min. Physical examination shows an enlarged tongue and generalized hypotonia. Serum concentrations of lactate, pyruvate, glucose, and uric acid are within the reference ranges. A chest x-ray shows cardiomegaly. A biopsy specimen of skeletal muscle shows increased glycogen. Impairment of which of the following enzymes is the most likely cause of the abnormal glycogen storage in this patient? Branching enzyme Alpha 1, 4 glucosidase Phosphoglucomutase Glycogen synthase Glucose-6-phosphatase

Alpha 1, 4 glucosidase

A 17-year-old boy has septic shock that is unresponsive to intravenous ADH (vasopressin) therapy. This treatment is discontinued, and high-dose dopamine therapy is initiated. Stimulation of which of the following receptors is most likely to be of benefit to this patient? 1 -Adrenoreceptors D7receptors 7Adrenoreceptors Alpha-1 adrenoreceptors a7Adrenoreceptors

Alpha-1 adrenoreceptors

A 67-year-old man is brought to the physician by his wife because of a 1-year history of progressive difficulty writing and walking. The patient is stooped and talks slowly. Physical examination shows a bland facial expression. There is a fine resting tremor in both hands, but not in the lower extremities. The tremor is not apparent when the patient is moving. When asked to walk, the patient has some difficulty starting and stopping. There is cogwheel rigidity in all four extremities. Testing of postural reflexes shows moderate retropulsion and mild propulsion. Gross and microscopic appearances of brain tissue from a patient with a similar condition are shown. The midbrain on the left is a normal control. The inclusion indicated by the arrow is predominantly composed of which of the following substances? Amyloid Prion protein Tau Alpha-synuclein Ataxin

Alpha-synuclein

A previously healthy 25-year-old man is admitted to the hospital for treatment of viral pneumonia. He is mechanically ventilated because of respiratory failure due to hypoxemia. An open-lung biopsy specimen is obtained and shows a pattern of acute lung injury. The presence of which of the following in the biopsy specimen most clearly suggests diffuse alveolar damage rather than bronchiolitis obliterans-organizing pneumonia in this patient? Uniform fibroblastic proliferation Alveolar hyaline membrane Foamy macrophages Masson bodies Interstitial collagen deposition

Alveolar hyaline membrane

A 60-year-old man comes to the physician because of progressive shortness of breath during the past 3 months. He states that for many years he has worked in a shipyard. His respirations are 25/min. Bilateral basilar crepitant crackles are heard. An x-ray of the chest shows reticulonodular pulmonary infiltrates consistent with interstitial fibrosis. Microscopic examination of sputum shows occasional elongate structures as shown. The fibrosis was most likely initiated by interaction of these structures with which of the following cell types? Clara cell Kulchitsky cell Goblet cell Chondrocyte Type I pneumocyte Squamous epithelial cell Alveolar capillary endothelial cell Alveolar macrophage Ciliated columnar epithelial cell Type II pneumocyte

Alveolar macrophage

A previously healthy 27-year-old woman comes to thephysician because of a 3-month history of moderateabdominal pain that improves for a short time after she eats.She has not had any rectal bleeding. Her temperature is36.4°C (97.5°F), pulse is 80/min, and blood pressure is110/70 mm Hg. Abdominal examination showsmidepigastric tenderness. Her hemoglobin concentration is12 g/dL, and leukocyte count is 8000/mm3. SerumHelicobacter pylori antibody assay is positive. Which of thefollowing is the most appropriate next step in management? HIDA scan Ranitidine therapy Omeprazole, magnesium hydroxide, andmetoclopramide therapy Ciprofloxacin therapy Amoxicillin, clarithromycin, and omeprazoletherapy

Amoxicillin, clarithromycin, and omeprazoletherapy

An 18-year-old man is brought to the emergency department because of anxiety and confusion for 6 hours. He has not had visual or auditory hallucinations. He appears cachectic and older than his age. His temperature is 38.2°C (100.8° F), pulse is 130/min, respirations are 14/min, and blood pressure is 180/110 mm Hg. Physical examination shows diaphoresis, pupillary dilation, and muscle weakness. The patient undergoes gastric emptying followed by charcoal treatment. Toxicology screening will most likely be positive for which of the following substances? Barbiturates Marijuana Opiates Benzodiazepines Amphetamines

Amphetamines

A 35-year-old metal worker collapses after exposure to an unknown chemical at a factory. He is dyspneic but is noncyanotic and smells of burnt almonds. The most appropriate next step is to administer which of the following? Physostigmine Ethanol Erythropoietin Hyperbaric oxygen Amyl nitrite

Amyl nitrite

A 67-year-old woman comes to the physician because of a 1-month history of low back pain. She has hypertension well controlled with a thiazide diuretic. Her pulse is 140/min, and blood pressure is 140/85 mm Hg. Physical examination shows tenderness to palpation over the L2-3 vertebrae. Laboratory studies show: Hemoglobin: 13.5 g/dL Hematocrit: 41% Leukocyte count: 10,500/mL Segmented neutrophils: 65% Eosinophils: 1% Basophils: 1% Lymphocytes: 30% Monocytes: 3% Platelet count: 250,000/mL Serum electrophoresis shows a monoclonal spike of lgG kappa. A chest x-ray shows cardiomegaly with bilateral pleural and pericardial effusions. An x-ray of the spine shows a lytic lesion. Echocardiography shows an echodense thickened left ventricle and poor diastolic compliance. A photomicrograph of a specimen obtained on myocardial biopsy is shown. Which of the following is the most likely cause of the cardiac findings in this patient? Acute myocarditis Amyloid infiltration Cardiomyopathy Acute infarction Plasma cell infiltration

Amyloid infiltration

A 37-year-old male dies suddenly. He had a history of fatigue, dyspnea, dependent edema, and decreased ejection fraction. At autopsy, the structure of the valves is normal, but all four chambers of the heart are enlarged. A mural thrombus is attached to the wall of the left atrium. Histologic examination of the heart show hypertrophied and atrophied muscle cells. Which of the following is the most likely diagnosis? Myxedema heart Restrictive cardiomyopathy Hypertrophic cardiomyopathy Amyloidosis Dilated cardiomyopathy

Amyloidosis

A 1-year-old girl is brought to the physician for a follow-up examination. She has had numerous infections of the skin and oral mucosa since birth; the infections have been slow to respond to antibiotic therapy. Her temperature is 38°C (100.4 °F). Physical examination shows multiple erythematous lesions of the skin, some of which have superficial ulceration. Laboratory studies show a leukocyte count of 21,000/mm 3 with 77% segmented neutrophils, 6% bands, 14% lymphocytes, and 3% monocytes. A biopsy of one of the lesions shows no neutrophils within the dermis or epidermis. Culture of the lesion grows Staphylococcus aureus. This patient's recurrent infections are most likely caused by defective production of which of the following? A prostaglandin A component of complement An interleukin An integrin An addressin

An integrin

A 32-year-old man comes to the physician because of back pain and joint stiffness for 3 months. His symptoms improve after mild exercise. During this illness, he also has had fatigue and a 3.2-kg (7-lb) weight loss. Physical examination and x-rays of the sacroiliac joints confirm a diagnosis of ankylosing spondylitis. Which of the following laboratory tests would be most appropriate for the patient's two children to evaluate their susceptibility to this disease? Analysis of class I MHC allele expression Measurement of 8 -lymphocyte mitogen responses to pokeweed mitogen Measurement of T-lymphocyte mitogen responses to phytohemagglutinin Polymerase chain reaction assay for bcl-2 gene expression Polymerase chain reaction assay for tyrosine kinase gene express

Analysis of class I MHC allele expression

A 76-year-old man with a 1-month history of a pulsatile abdominal mass is diagnosed with an abdominal aortic aneurysm. CT scans of the abdomen with contrast also showed an incidental finding of a horseshoe kidney. In this setting, the surgical approach to repair of the aneurysm in this patient is most likely to be further complicated by the presence of which of the following? Multiple ureters with abnormal courses Tumors with malignant degeneration Anamolous origins of multiple renal arteries to each kidney Friability of vascular tissue as a result of collagen synthesis abnormalities Abnormal origin of the superior mesenteric artery

Anamolous origins of multiple renal arteries to each kidney

A 67-year-old man is brought to the physician because of a 2-day history of double vision and drooping of his left eyelid. He has no history of trauma to the eye. His vital signs are within normal limits. Physical examination shows ptosis of the left eyelid. When the eyelid is raised manually, the eye is fixed in the out position and the pupil is dilated. The visual acuity of the left eye is within normal limits. Which of the following is the most likely cause of these findings? Tumor of the optic nerve Aneurysm of the posterior communicating artery Compression of the superior cervical ganglion Damage to the trochlear nerve Occlusion of the scleral venous sinus

Aneurysm of the posterior communicating artery

A 68-year-old man comes to the physician because of a 1-month history of light-headedness and tightness in his chest with exertion. He adds that the pain is worse after arguing with his wife, and the symptoms resolve with rest. He has a past history of lower gastrointestinal bleeding; evaluation at that time was negative on upper endoscopy and colonoscopy. His temperature is 37°C (98.6°F), pulse is 85/min, respirations are 15/min, and blood pressure is 110/75 mm Hg. Physical examination shows no abnormalities. His hemoglobin concentration is 8.2 g/dl, and hematocrit is 24%. Test of the stool for occult blood is positive. An ECG shows no abnormalities. Repeat colonoscopy shows no abnormalities. Which of the following is the most likely cause of this patient's gastrointestinal symptoms? Adenocarcinoma of the colon Ulcerative colitis Angiodysplasia Peutz-Jeghers syndrome Diverticulitis

Angiodysplasia

A 30-year-old woman comes to the physician for a routine examination. She has a 10-year history of type 1 diabetes mellitus. Examination shows normal findings. She has microalbuminuria; her hemoglobin A1c is 7%, and serum creatinine concentration is 1.8 mg/dL. Which of the following is the most appropriate intervention? Insulin Aspirin daily Angiotensin-converting enzyme (ACE) inhibitor Calcium-channel blocking agent Oral hypoglycemic agent

Angiotensin-converting enzyme (ACE) inhibitor

A healthy 5-year-old boy is brought to the physician by his parents for a well-child examination. He has a history of motion sickness. Physical examination shows no abnormalities. The parents are planning a vacation to Australia, and they ask the physician if they should give their son diphenhydramine during the trip for his motion sickness. Which of the following is the most likely mechanism of action of this medication for motion sickness? Antagonist at serotonin receptors Agonist at 1-adrenoreceptors Antagonist at M3 receptors Antagonist at histamine-2 (Hi} receptors Agonist at N-methyl-o-aspartate receptors Agonist at a 1-adrenoreceptors

Antagonist at M3 receptors

A 22-year-old woman comes to the physician 3 months after she noticed a painless, slowly enlarging mass on the left side of her neck. Physical examination shows a freely mobile, soft, cystic mass with a cutaneous surface opening. The physician explains that it is from incomplete fusion during embryonic development. Which of the following is the most likely location of the opening of the duct leading to the mass in this patient? Submental Postauricular Posterior to the parotid gland Anterior to the sternocleidomastoid muscle Midline on the neck

Anterior to the sternocleidomastoid muscle

A 15-year-old boy has had pain in the knee since sustaining an injury in a high school football game 6 weeks ago. The high school trainer has been treating him with heat and ultrasound, without significant improvement. Physical examination shows tenderness of the medial femur approximately 7.5 cm above the joint. There is no ligamentous instability, joint swelling, or effusion. Which of the following is the most appropriate next step in diagnosis? Arthroscopy Measurement of serum calcium concentration Anteroposterior and lateral x-rays Arthrography Ultrasonography

Anteroposterior and lateral x-rays

A 10-year-old boy is brought to the physician because of a 4-day history of nosebleeds and easy bruising. Three weeks ago, he had an upper respiratory tract infection. Physical examination shows ecchymoses over the upper and lower extremities. Laboratory studies show a hemoglobin concentration and leukocyte count within the reference ranges and a decreased platelet count. A bone marrow smear shows an increased number of megakaryocytes. Which of the following platelet abnormalities is the most likely cause of the findings in this patient? Decreased concentration of the Gp1b/iX complex Decreased synthesis of thromboxane A 2 Decreased concentration of the Gp Llb/llla complex Decreased binding of Gpla/lla (aJ3 1) to collagen Deficient binding of von Willebrand factor to the Gp1b/iX complex Antibodies directed against the glycoprotein IIb/IIIa complex

Antibodies directed against the glycoprotein IIb/IIIa complex

A 34-year-old man comes to the physician because of a 3-month history of a progressive rash on his feet. The rash is not itchy or painful. A photograph of the feet is shown. A rapid HIV antibody test result is positive. In addition to treatment with highly active antiretroviral therapy, which of the following is the most appropriate pharmacotherapy for this patient? Antihistaminic Antineoplastic Antifungal Antibacterial Antiparasitic

Antineoplastic

A 22-year-old woman comes to the physician for a follow-up examination. One year ago, she was diagnosed with a pulmonary embolism. Two years ago, she delivered a female stillborn at 23 weeks gestation. Physical examination today shows no abnormalities. Laboratory studies show a platelet count of 250,000/mm3, a prothrombin time within the reference range, and an increased partial thromboplastin time. The findings in this patient are most consistent with which of the following conditions? Factor V Leiden mutation Antiphospholipid antibody syndrome Prothrombin G20210A mutation Increased factor VIII (antihemophilic factor) concentration Protein C deficiency O E) Prothrombin G20210A mutation

Antiphospholipid antibody syndrome

A 16-year-old girl is brought to the physician for a school physical examination. She has no history of serious illness and takes no medications. Menarche has not yet occurred. She is 155 cm (5 ft 1 in) tall and weighs 66 kg (145 lb); BMI is 27 kg/m2. Her blood pressure is 140/80 mm Hg in the left arm and 105/70 mm Hg in the left leg. Vital signs are otherwise within normal limits. A grade 2/6 systolic murmur is heard best over the upper back to the left of the midline. The extremities are well perfused with strong peripheral pulses. Breast development is Tanner stage 2, and pubic hair development is Tanner stage 1. This patient most likely has an abnormality of which of the following vessels? Renal artery Coronary artery Renal vein Pulmonary artery Aorta

Aorta

A 16-year-old girl is brought to the physician for a schoolphysical examination. She has no history of serious illnessand takes no medications. Menarche has not yet occurred.She is 155 cm (5 ft 1 in) tall and weighs 66 kg (145 lb);BMI is 27 kg/m2. Her blood pressure is 140/80 mm Hg inthe left arm and 105/70 mm Hg in the left leg. Vital signsare otherwise within normal limits. A grade 2/6 systolicmurmur is heard best over the upper back to the left of themidline. The extremities are well perfused with strongperipheral pulses. Breast development is Tanner stage 2,and pubic hair development is Tanner stage 1. This patientmost likely has an abnormality of which of the followingvessels? Pulmonary artery Coronary artery Renal artery Renal vein Aorta

Aorta

A 46-year-old man receives the diagnosis of squamous cell carcinoma of the esophagus. A barium swallow is shown. Esophagectomy at the region indicated by the arrows is most likely to involve ligation of arterial branches of which of the following vessels? Thyrocervical trunk Superior mesenteric artery Internal thoracic arteries Aorta Pulmonary arteries

Aorta

A 68-year-old male with a history of coronary artery disease undergoes cardiac catheterization. A via the right femoral artery. As the pig-tail catheter is advanced into the apex of the left ventricle, which of the following valves will be crossed? Pulmonary Aortic Mitral Tricuspid

Aortic

A 68-year-old man is brought to the emergency department because of a 3-day history of increasingly severe deep chest pain. He also has had shortness of breath, stridor, hoarseness, difficulty swallowing, and a nonproductive cough. He has a long-standing history of hypertension. He has smoked 2 packs of cigarettes daily for 45 years. His temperature is 37.2°C (99° F), pulse is 80/min, respirations are 15/min, and blood pressure is 160/94 mm Hg. The lungs are clear to auscultation. Physical examination shows a visible pulsation above the manubrium of the sternum and displacement of the trachea to the right. A murmur is heard in the second right intercostal space. Which of the following is the most likely diagnosis? Aortic aneurysm Pulmonary regurgitation Pneumothorax Acute bronchitis Goiter Aortic stenosis

Aortic aneurysm

A 53-year-old male presents with severe chest pain, diaphoresis, and severe dyspnea. He reports that the pain radiates to his back near his interscapular region. His heart rate is 130/min and regular, his blood pressure 128/62 mm Hg and 94/52 mm Hg in his left and right arms, respectively. Auscultation reveals diffuse, inspiratory crackles in both lung, distant heart S1 and S2 sounds, and a grade 2/6, blowing, early diastolic murmur best heard between the second right and third left intercostal spaces adjacent to the sternum. A chest x-ray would most likely reveal which of the following in this patient? Aortic dissection Myocardial infarction Ruptured chordae tendineae Cardiac tamponade Pulmonary embolism Ventricular septal defect

Aortic dissection

A 73-year-old female presents to the emergency room after collapsing at home. She is 5' 4" and weighs 198 lbs with a BMI of 34. Medical records reveal a long-standing history of hypertension. She reported experiencing severe, excruciating, stabbing-like chest pain that radiated to her left shoulder before she collapsed. Upon admission, her pulse is 112/min and blood pressure is 162/84 mm Hg. Auscultation reveals distant heart sounds and a drop in systolic blood pressure of >10 mm Hg upon inspiration. ECG reveals ST-T wave changes. Chest X-ray shows a widening of the mediastinum. Which of the following represents the most likely diagnosis? Pulmonary embolism Myocardial infarction Cardiac tamponade Aortic dissection with cardiac tamponade Ventricular septal defect

Aortic dissection with cardiac tamponade

The largest proportional increase in left ventricular afterload and myocardial oxygen consumption is most likely to occur following a 20% increase in which of the following? Heart rate Coronary blood flow Stroke volume Right ventricular pressure Aortic pressure

Aortic pressure

A 38-year-old man comes to the physician because of a 1-week history of shortness of breath with exertion. His respirations are 12/min. Light palpation of the carotid artery shows the upstroke to be abnormally brisk and the downstroke to fall precipitously. Which of the following is the most likely cause of these physical examination findings? Aortic stenosis Mitral stenosis Ventricular septal defect Mitral regurgitation Aortic regurgitation Aortic coarctation

Aortic regurgitation

A 64-year-old female has a two-year history of progressive shortness of breath. Blood pressure is 174/74 mm Hg with rapid collapse of the peripheral pulse. She has a high-pitched, blowing, diastolic decrescendo murmur heard best at the second right intercostal space. Which of the following is the most likely diagnosis? Mitral stenosis Mitral regurgitation Aortic regurgitation Tricuspid regurgitation Pulmonic regurgitation Aortic stenosis

Aortic regurgitation

A 62-year-old man dies suddenly while playing tennis. He had no known cardiac risk factors and no history of coronary artery disease. At autopsy, examination shows a cardiac valve defect and concentric left ventricular hypertrophy. Which of the following valve abnormalities is most likely involved in his sudden death? Pulmonic insufficiency Mitral stenosis Mitral insufficiency Aortic insufficiency Pulmonic stenosis Aortic stenosis

Aortic stenosis

A healthy 56-year-old man comes to the physician for a pre-employment physical. Cardiac examination shows a systolic heart murmur that is best heard in the second interspace at the right sternal border and is transmitted to the carotid arteries. The murmur begins immediately after S1, rises in crescendo before falling in pitch, and ceases before S2. Which of the following is the most likely cause of the murmur? Atrial septa I defect Tricuspid stenosis Mitral regurgitation Aortic stenosis Pulmonary regurgitation

Aortic stenosis

An 18-year-old woman with mild mental retardation is brought to the physician because of a 3-day history of decreased ability to see in reduced light. She has a lifelong history of chronic diarrhea. Two years ago, she developed a lack of muscle control of her arms and legs, and generalized weakness. Her 16-year-old brother has had similar symptoms Ophthalmologic examination shows bilateral reti nitis pigmentosa. There is ataxia and loss of deep tendon reflexes. Laboratory studies show erythrocytes with spiny projections and serum total cholesterol concentration of 40 mg/dL. Which of the following apolipoproteins is most likely deficient in this patient? Apo C Apo B Apo A-1 Apo E Apo A-11

Apo B

A 40-year-old man is brought to the emergency department after being involved in a motor vehicle collision. He is pronounced dead on arrival. He had a history of fever, headache, chills, and pain in the right upper quadrant of the abdomen 5 days ago. During autopsy, histologic examination of a liver section shows disruption of the normal hepatic lobule. Small shrunken hepatocytes with intense eosinophilic cytoplasm, fragmented nuclear chromatin, and cytoplasmic bleb formations are noted. Which of the following processes is most likely occurring in the hepatocytes described? Metaplasia Liquefaction necrosis Fatty change Caseous necrosis Atrophy Dysplasia Heterophagy Apoptosis Coagulation necrosis

Apoptosis

A 55-year-old woman with colon cancer and a healthy 55-year-old woman are participating in a study of colon cancer. Malignant cells are obtained from the tumor in the affected patient, and normal colonic epithelial cells are procured from the healthy subject. After both cell types are treated with transforming growth factor-beta, the number of normal cells decreases, whereas the number of tumor cells remains unchanged. The tumor cells most likely express a mutation that inhibits which of the following physiologic processes? DNA repair Necrosis Cell cycle progression Apoptosis Migration

Apoptosis

A previously healthy 32-year-old man comes to theemergency department because of a 3-day history of painand swelling of his right knee. Two weeks ago, he injuredhis right knee during a touch football game and has hadswelling and bruising for 5 days. One week ago, heunderwent extraction of a molar for severe dental caries.He is sexually active with one male partner and usescondoms consistently. HIV antibody testing was negative3 months ago. His temperature is 38.6°C (101.5°F), pulse is100/min, and blood pressure is 120/60 mm Hg.Examination of the right knee shows warmth, erythema,and a joint effusion. Flexion and extension of the right kneeare severely limited. An x-ray of the knee confirms thejoint effusion. Which of the following is the mostappropriate next step in diagnosis? Bone scan Venous Doppler ultrasonography Arthroscopic exploration of the knee Arthrocentesis MRI of the knee

Arthrocentesis

A 19-year-old woman comes to the physician because of increasingly severe pain in the right hip during the past month. She has a 4-year history of refractory inflammatory bowel disease with arthritis in the hands treated with corticosteroids for the past 2 years. An x-ray of the pelvis shows collapse of the superior half of the right femoral head with preservation of the articular cartilage. The left hip appears normal. Which of the following is the most likely cause of the hip abnormality? Gouty arthritis Aspetic necrosis Ankylosing spondylitis Septic arthritis Multiple myeloma

Aspetic necrosis

A 25-year-old male and 23-year-old female with achondroplasia present for prenatal genetic counseling. Because achondroplasia is an autosomal dominant disorder, this couple has an increased risk of having offspring with this same disorder. Which of the following best describes this type of mating? A2 A3 °α√γβε Founder Effect Consanguineous Mating Disassortative Mating Assortative Mating Stratification

Assortative Mating

A 42-year-old woman comes to the physician for a routinehealth maintenance examination. She says that she feelsfine. She is 170 cm (5 ft 7 in) tall and weighs 86 kg (190 lb);BMI is 30 kg/m2. Her blood pressure is 131/82 mm Hg.Physical examination shows no other abnormalities. Fastingserum studies show:Glucose 105 mg/dLCholesterol, total 210 mg/dLTriglycerides 185 mg/dLC-reactive protein 0.35 mg/L (N=0.08-3.1)This patient is at increased risk for developing which of thefollowing conditions? Chronic lymphocytic (Hashimoto) thyroiditis Atherosclerosis Type 1 diabetes mellitus Systemic sclerosis (scleroderma) Systemic lupus erythematosus

Atherosclerosis

A 60-year-old female develops palpitations over a six-week period. A rhythm strip from her ECG is shown. Which of the following is most likely responsible for the palpitations in this woman? Ventricular fibrillation Atrial flutter Atrioventricular reentrant tachycardia Sinus tachycardia Ventricular tachycardia

Atrial flutter

A 57-year-old male presents with complaints of fatigue, dyspnea, and recurrent dizziness over the past five-weeks. An ECG is recorded (see image). Which of the following is most likely accounts for this ECG pattern in this patient? Atrioventricular conduction disease Ectopic focus in the atria Sinus node disease Atrioventricular re-entrant pathway Ectopic focus in the ventricle

Atrioventricular conduction disease

A 2-year-old girl is brought to the emergency department 20 minutes after she accidently ingested insecticide. Her father reports that he was spraying his lawn and accidentally left the open bottle on the ground. His daughter walked over, picked up the bottle, and started to drink, but then she immediately threw down the bottle. She is in moderate respiratory distress. Her respirations are 18/min and shallow. Pulse oximetry on room air shows an oxygen saturation of 82%. Physical examination shows copious oral secretions. In addition to securing an airway, the most appropriate immediate step is administration of which of the following medications? Atropine Diphenhydramine Pralidoxime Succinylcholine Physostigmine

Atropine

A 35-year-old man who is a farmer is brought to the hospital by his wife because of difficulty breathing, sweating, excess salivation, and diarrhea for the past 2 hours. His wife says that the symptoms started when he was applying a new insecticide to his crops. Temperature is 36°C (96.8° F), pulse is 50/min, respirations are 22/min, and blood pressure is 90/60 mm Hg. Which of the following is the most appropriate treatment? lpratropium Propantheline Neostigmine Mecamylamine Atropine

Atropine

A 48-year-old woman comes to the physician because of a 2-month history of pain and stiffness in her hands, elbows, knees, and hips when she gets out of bed in the morning. Physical examination shows multiple swollen, tender joints. Presence of which of the following would best support the diagnosis of rheumatoid arthritis in this patient? Autoantibody reactive with a ri bonuclear antigen (Ro) Autoantibody reactive with desmoglein 3 Autoantibody reactive with type IV coll agen Autoantibody reactive with a ri bonuclear antigen (Ro) Autoantibody (IgM) reactive with IgG Antinuclear antibody

Autoantibody (IgM) reactive with IgG

A 56-year-old man has had fever pleuritic chest pain for 1-day. He was admitted to the hospital three week ago because of an acute transmural anterior myocardial infarction. He has taken a β-adrenergic blocking agent and aspirin since being discharged. A friction rub is heard over his precordium. Which of the following is the most likely cause of the current findings? Adverse drug effect Aortic dissection Post viral inflammation Recurrent ischemia Autoimmune reaction

Autoimmune reaction

A 67-year-old man comes to the physician for a follow-up examination. Eight months ago, he received a diagnosis of primary lung carcinoma involving the adrenal glands, liver, and bone. He has had an 8-kg (17.6-lb) weight loss during the past 3 months. Physical examination shows cachexia and significant muscle wasting. Which of the following intracellular components is most likely increased in this patient's muscle cells? Autophagic vacuoles Mitochondria Endoplasmic reticulum Mitotic spindles Golgi complex

Autophagic vacuoles

The blood flow through an organ is measured while theperfusion pressure is varied experimentally. An abrupt,sustained increase in perfusion pressure increases flowinitially, but over the course of 1 minute, the flow returnsnearly to the baseline level despite continued elevation of theperfusion pressure. The organ under study is exhibitingwhich of the following? Ischemia Reactive hyperemia Active hyperemia Autoregulation

Autoregulation

A 20-year-old woman has multiple neurofibromas. Her mother, a maternal uncle, and one of her two brothers have similar lesions. Which of the following is the most likely mode of inheritance? Autosomal recessive X-linked recessive X-linked dominant Mitochondrial Autosomal dominant

Autosomal dominant

A 30-year-old woman is brought to the emergency department 6 hours after the onset of severe abdominal pain and confusion. Her brother, father, and paternal grandmother had similar problems. Two weeks ago, she began a low-calorie diet. Her pulse is 120/min, and blood pressure is 160/110 mm Hg. Physical examination shows restlessness, excess sweating, and tremors. There is no abdominal tenderness. Urine studies show an increased porphobilinogen concentration. Which of the following is the most likely pattern of inheritance causing this patient's condition? Mutifactorial Autosomal recessive X-linked dominant Mitochondrial DNA mutation X-linked recessive Autosomal dominant

Autosomal dominant

A 4-year-old boy has respiratory tract colonization with Pseudomonas species. Sweat chloride concentration is 120 mEq/L (N<60 mEq/L). Which of the following is the most likely mode of inheritance of the underlying disorder? Mitochondrial Autosomal dominant Autosomal recessive X -linked Multifactorial

Autosomal recessive

A 15-year-old boy is brought to the physician because of right wrist pain that began when he landed on his hands after falling backward while skateboarding 4 hours ago. Examination of the right upper extremity shows no bone deformities of the wrist. There is point tenderness to palpation of the radial aspect of the wrist between the abductor pollicis longus and extensor pollicis brevis tendons. This patient is at increased risk for which of the following sequelae as a result of this injury? Subluxation of the distal aspect of the trapezium bone Damage to the median nerve Spiral fracture of the ulna Damage to the ulnar nerve Rupture of the flexor carpi ulnaris tendon Avascular necrosis of proximal aspect of scaphoid

Avascular necrosis of proximal aspect of scaphoid

A 26-year-old man is brought to the emergency department by ambulance 2 hours after injuring his right arm during a skiing accident. His right upper extremity is immobilized in a sling. Sensation to pinprick is absent over the lateral aspect of the shoulder. An x-ray of the right shoulder is shown. Damage to which of the following nerves is most likely in this patient? Long thoracic Axillary Radial Suprascapular Musculocutaneous

Axillary

A 13-year-old male with growth retardation, microcephaly, sun-sensitive skin rash, and recurrent infections presents for genetic testing. Family history reveals, the patient is the second child of first cousins, his parents and siblings are health and unaffected, while two cousins on the maternal side, demonstrate similar symptoms. Genetic analysis reveals a defect in the BLM gene that codes for DNA helicase. Which of the following is the most likely site of action of this enzyme in the DNA replication fork shown below? B F A C E D

B

A 20-year-old college student develops fever, severepharyngitis, hepatosplenomegaly, and lymphadenopathy.The pathogenesis of this syndrome most likely involves adouble-stranded DNA virus infection of which of thefollowing cells? B lymphocytes Neutrophils Kupffer cells T lymphocytes Macrophages

B lymphocytes

Ten years after a total gastrectomy, a 60-year-old man has difficulty walking. Mentation and cranial nerve function are normal. He has diffuse spasticity in his arms and legs, markedly impaired proprioception in his feet, increased muscle stretch reflexes in his arms and knees, absence of muscle stretch reflexes in the ankles, and bilateral extensor plantar responses. The most likely cause is deficiency of which of the following vitamins? B6 (pyridoxine) C E Biotin B12 (cobalamin) K2 A Retinal

B12 (cobalamin)

A 78-year-old man comes to the physician because of fever, chills, fatigue, shortness of breath, and a 5.4-kg (12-lb) weight loss over the past month. He underwent urinary catheterization 6 weeks ago for prostatic obstruction. His temperature is 38.5°C (101.3° F), pulse is 100/min, respirations are 14/min, and blood pressure is 160/80 mm Hg. Examination of the chest shows a soft S1 and a normal S2. A grade 2/6 early diastolic murmur is maximal in the second left intercostal space and is accentuated when the patient leans forward with held expiration. The lungs are clear. Which of the following is the most likely diagnosis? Pulmonary embolus Peritonitis Viral pneumonia Prostatitis Bacterial endocarditis

Bacterial endocarditis

An otherwise healthy 18-year-old man comes to the physician because of cracked lips and peeling sunburned skin. He began working as a lifeguard at an outdoor pool 2 weeks ago. Physical examination shows desquamation of sunburned skin of the face, neck, and extremities. His lips are dry and cracked. Application of petrolatum to his lips may aid in reducing lip symptoms by which of the following effects of the compound? Antimetabolic Keratolytic Ultraviolet light blocker Barrier Anti-inflammatory

Barrier

A previously healthy 25-year-old woman, gravida 2, para 2, comes to the emergency department because of a 3-day history of painful swelling of her vaginal area. Her last menstrual period was 2 months ago. She is sexually active with one male partner and uses depot medroxyprogesterone for contraception. Her temperature is 38.4°C (101.2°F), pulse is 96/min, respirations are 16/min, and blood pressure is 125/82 mm Hg. Examination shows a 4-cm, exquisitely tender mass in the inferior aspect of the left labium minus. The swelling and tenderness prevent insertion of either a speculum or fingers into the vagina. Which of the following is the most likely diagnosis? Chancroid Bartholin gland abscess Herpes simplex Lymphogranuloma venereum Condylomata acuminata

Bartholin gland abscess

A previously healthy 25-year-old woman, gravida 2, para 2,comes to the emergency department because of a 3-dayhistory of painful swelling of her vaginal area. Her lastmenstrual period was 2 months ago. She is sexually activewith one male partner and uses depot medroxyprogesteronefor contraception. Her temperature is 38.4°C (101.2°F),pulse is 96/min, respirations are 16/min, and blood pressureis 125/82 mm Hg. Examination shows a 4-cm, exquisitelytender mass in the inferior aspect of the left labium minus.The swelling and tenderness prevent insertion of either aspeculum or fingers into the vagina. Which of the followingis the most likely diagnosis? Herpes simplex Condylomata acuminata Lymphogranuloma venereum Chancroid Bartholin gland abscess

Bartholin gland abscess

A previously healthy 7-year-old boy is brought to the physician by his parents because of a tender lump in his right armpit for 6 days. Two weeks ago, his parents noticed a small red, raised, and tender bump on his right index finger. The family has several pets, including cats. His temperature is 37.8°C (100° F). Physical examination shows a 2x2 cm, tender lymph node in the right axilla. Microscopic examination of a biopsy specimen of the node shows necrotizing granulomatous adenitis with stellate microabscesses. Which of the following is the most likely causal organism? Bartonella henselae Epstein-Barr virus Toxoplasma gondii Mycobacterium tuberculosis Histoplasma capsulatum

Bartonella henselae

A 56-year-old man has had a small, slowly growing nodule on his chin during the past 3 years. The lesion is 1.3 cm in diameter, the center is ulcerated, and the border is waxy. Examination of tissue obtained on excision of the lesion ismost likely to show which of the following? Basal cell carcinoma Squamous cell carcinoma Melanoma Actinic keratosis Seborrheic keratosis

Basal cell carcinoma

An investigator is studying epithelial repair in the small intestine of an experimental animal. The plan is to identify the location of the most active cell division. This cell activity is most likely to be found in which of the following regions? Brunner glands Peyer patches Top of the villi Base of the crypt

Base of the crypt

A 25-year-old man is admitted to the hospital because of severe crush injuries to the chest and extensive burns over 30% of his body surface area. Three hours later, he develops tachypnea and dyspnea. Arterial blood gas analysis on room air shows a decreased Po2 and Pco2. A chest x-ray shows bilateral interstitial and alveolar infiltrates. The patient is intubated and mechanically ventilated. Damage to which of the following is most likely to preclude restoration of normal tissue architecture and pulmonary function in this patient? Capillaries Type I pneumocytes Basement membrane Mast cells Macrophages Fibroblasts

Basement membrane

A 68-year-old man has a serum creatinine concentration of 2.3 mg/dl due to chronically increased hydrostatic pressure in Bowman space. Which of the following disorders is the most likely cause of these findings? Nephrotic syndrome Type 2 diabetes mellitus Congestive heart failure Benign prostatic hyperplasia Hypertension Interstitial nephritis

Benign prostatic hyperplasia

A 45-year-old woman who is intubated and mechanically ventilated develops fungemia with Candida albicans during a prolonged hospital stay. Treatment with caspofungin is started. Which of the following features of the causal organism will most likely be targeted by this drug? Beta Glucan carbohydrates in the cell wall Porin proteins in the outer membrane Fungal RNA polymerase N-glycosidases involved in capsule synthesis Lanosterol in the cytoplasmic membrane

Beta Glucan carbohydrates in the cell wall

A 56-year-old woman develops a restrictive cardiomyopathy, proteinuria, and renal failure. She has a 35-year history of rheumatoid arthritis. Renal biopsy shows glomerular deposition of eosinophilic hyaline material; a Congo red stain shows a characteristic birefringent pattern when observed under polarized light. Which of the following features best explains the staining properties of this material? Beta pleated sheet structure Multiple disulfide cross-links High hydroxyproline content A-Helical structure Helix-loop-helix structure

Beta pleated sheet structure

A 42-year-old woman comes to the physician for an examination prior to employment. She has no history of major medical illness. Her vital signs are within normal limits. Physical examination shows no abnormalities. Laboratory studies show: Which of the following is the most likely diagnosis? Aplastic anemia Beta-thalassemia minor Folic acid deficiency Vitamin B12 (cobalamin) deficiency Sickle cell disease

Beta-thalassemia minor

A 55-year-old male develops shortness of breath. Examination shows a harsh systolic ejection murmur at the base that radiates to the neck. ECG shows evidence of prominent left ventricular hypertrophy. Which of the following cardiac abnormalities is most likely? Aortic dissection Mitral insufficiency Tricuspid insufficiency Mitral stenosis Bicuspid aortic valve

Bicuspid aortic valve

A 60-year-old male presents with a 3-day history of abdominal pain and nausea. History reveals the patient was diagnosed with non-Hodgkin lymphoma 6 months ago. Today, his temperature is 37.6°C (99.7° F), pulse is 100/min, and blood pressure is 130/80 mm Hg. Physical examination reveals abdominal tenderness of the flanks and left and right lower quadrants. Lab results reveal the following: BUN of 34 mg/dL and serum creatinine of 3.8 mg/dL. An abdominal CT scan reveals bilateral hydronephrosis and lymphadenopathy compressing the ureters. Which of the following is most likely to improve this patient's renal function? Bilateral stents in the ureter Stent in right renal artery Suprapubic tube Foley catheter Stent in left renal artery

Bilateral stents in the ureter

A previously healthy 52-year-old man is brought to the physician because of a 2-month history of progressive difficulty swallowing, double vision, and slurred speech. His vital signs are within normal limits. Physical examination shows ptosis of the eyelids and a waddling gait. Muscle strength testing shows fatigable weakness of the neck, arms, hands, and fingers. Repetitive nerve stimulation shows a 25% decrease in muscle action potentials in several muscles. Which of the following mechanisms is the most likely cause of this patient's disease? Binding of antibody to the acetylcholine receptor Decreased reuptake of acetylcholine Binding of acetylcholine by an antibody Decreased release of acetylcholine from the receptor Decreased acetylcholine production

Binding of antibody to the acetylcholine receptor

A 20-year-old woman is brought to the emergencydepartment 20 minutes after being stung by a wasp. She saysthat she feels a lump in her throat and chest tightness. Shehas a history of allergy to wasp venom. Her pulse is 120/min,and blood pressure is 80/40 mm Hg. Physical examinationshows eruptions that coalesce into giant urticaria. There isaudible wheezing. Which of the following best describes thecause of this patient's reaction? Induction of a cytotoxic reaction by CD8+T Lymphocytes Activation of macrophages by soluble immunecomplexes Formation of IgG antibodies againstextracellular matrix antigen Binding of antigen to preexisting cell-fixed IgE antibodies Formation of IgM antibodies against cellsurface receptor antigens

Binding of antigen to preexisting cell-fixed IgE antibodies

A 25-year-old man with cystic fibrosis comes to the physician because of a 3-day history of fever, cough, and progressive shortness of breath. His temperature is 38°C (100.4 °F), and respirations are 32/min. Bilateral crackles and rhonchi are heard on auscultation of the chest. A nutrient agar plate seeded with bronchial secretions grows an aerobic, oxidase-positive, gram-negative rod that forms slimy colonies and produces pyocyanin. The organism does not ferment lactose. Treatment with multiple appropriate antibiotics has not led to eradication of the causal organism. Which of the following is the most likely explanation for the persistent colonization of this patient by this organism? Overgrowth of acapsular strains Synthesis of exotoxin A Pyocyanin synthesis Biofilm formation in the lower respiratory tract Bacteriophage superinfection

Biofilm formation in the lower respiratory tract

A 55-year-old man comes to the physician because of two episodes of painless, blood-tinged urine during the past week. His last episode was 2 days ago. He has smoked 1 pack of cigarettes daily for 35 years. Vital signs are within normal limits. Physical examination shows no abnormalities. Urinalysis shows normal findings except for microscopic blood. Which of the following is the most likely diagnosis? Bladder cancer Nephrolithiasis Urinary tract infection Bladder diverticulum Interstitial cystitis

Bladder cancer

A previously healthy 72-year-old man comes to the physician because of decreased urinary output during the past 2 days; he has had no urinary output for 8 hours. Examination shows suprapubic fullness and an enlarged prostate. His serum urea nitrogen concentration is 88 mg/dL, and serum creatinine concentration is 3.5 mg/dL. Which of the following is the most appropriate next step in management? CT scan of the abdomen Ultrasonography of the prostate Ultrasonography of the kidneys Intravenous pyelography Bladder catheterization

Bladder catheterization

An 18-year-old woman is brought to the emergency department 2 hours after she ingested approximately 100 aspirin tablets in a suicide attempt. Physical examination shows tachypnea. Two days after admission to the hospital, her hemoglobin concentration is 12 g/dL. Test of the stool for occult blood is positive. Which of the following additional hematologic findings is most likely abnormal in this patient? Prothrombin time Bleeding time Fibrin degradation products Partial thromboplastin time Platelet count

Bleeding time

A 48-year-old man is brought to the emergency department because of difficulty opening his mouth and stiffness of his neck, shoulders, and back for 4 hours. He sustained a puncture wound to his arm 1 week ago. He has received no medical care for 30 years. Physical examination shows trismus and opisthotonos. Which of the following best describes the process in this patient? Demyelination Cell death in central nervous system neurons Cell death in muscles Blockade of inhibitory neurotransmitter release Cell death in peripheral nervous system neurons

Blockade of inhibitory neurotransmitter release

A 23-year-old nulligravid woman comes to the physician because she has been unable to conceive for 1 year. Physical and pelvic examinations show no abnormalities. Hysterosalpingograms are shown. This patient most likely has which of the following conditions? Blockage of the right fallopian tube only Rupture of the left fallopian tube only Rupture of the right fallopian tube only Blockage of the left fallopian tube only Blockage of both fallopian tubes

Blockage of both fallopian tubes

An 18-year-old man comes to the physician 6 hours after he twisted his ankle while playing football. Physical examination shows looseness of the lateral collateral ligament, suggestive of a tear. Lack of which of the following components in the ligament will most likely limit healing of this injury? Lymphatic vessels Blood vessels Blood vessels Elastic fibers Collagen fibers Nerves

Blood vessels

Two sets of patients (Groups X and Y) of similar age, weight, gender, and health status are given general anesthesia by inhalation. In Group X, anesthesia is induced by administering isoflurane at a concentration 1.5 times the minimal alveolar concentration (MAC) for this drug. In Group Y, anesthesia is induced by administering a combination of isoflurane and nitrous oxide, each at 0. 75 times the MAC for that drug. The onset of anesthesia is found to be significantly faster in Group Y than in Group X. The different response rates between the two groups is best explained by differences in which of the following properties of isoflurane and nitrous oxide? Lipid solubility Molecular weight Brain: blood partition coefficient Blood: gas partition coefficient Hepatic metabolism Brain: gas partition coefficient Minimal alveolar concentration

Blood: gas partition coefficient

A 10-year-old girl is brought to the physician by her mother for a well-child examination. She has not yet had a menstrual period. She is at the 50th percentile for height and weight. Physical examination shows absence of breast bud development and no pubic or axillary hair. The mother asks how she will know when her daughter begins puberty. It is most appropriate for the physician to tell the mother that the first objective sign of puberty will be which of the following? Development of pubic hair Rapidly increasing height Onset of menses Development of axillary hair Breast bud development

Breast bud development

A previously healthy 62-year-old man comes to thephysician because of a 2-month history of cough. He hassmoked two packs of cigarettes daily for 40 years.Examination shows no abnormalities. An x-ray of the chestshows a 2.5-cm noncalcified pulmonary nodule in the rightupper lobe. A CT scan of the chest confirms the x-rayfindings. An x-ray of the chest obtained 1 year ago showedno abnormalities. Which of the following is the mostappropriate next step in diagnosis? Bronchoscopy with transbronchial biopsy Ventilation-perfusion lung scans Measurement of serum calcium concentration Second x-ray of the chest in 6 weeks Median sternotomy

Bronchoscopy with transbronchial biopsy

An 83-year-old man comes to the physician because of a 3-day history of painful blisters on his torso. His pulse is 72/min, and blood pressure is 125/85 mm Hg. Physical examination shows numerous 0.5- to 1-cm, clear-fluid-filled, tense blisters over the trunk. Histopathologic examination of a biopsy specimen from the affected area shows a subepidermal blister. Production of autoantibodies directed against which of the following structures best explains the findings in this patient? Collagen, type VII Plakoglobin Cytokeratin Bullous pemphigoid antigen Desmoplakin

Bullous pemphigoid antigen

A 67-year-old man comes to the physician because of a 2-month history of pain in his feet. The discomfort is more severe in bed at night and is relieved by taking a hot bath. He has type 2 diabetes mellitus treated with glipizide. His pulse is 60/min, respirations are 12/min, and blood pressure is 130/88 mm Hg. Strength is normal and symmetric in the distal and proximal upper and lower extremities. The Achilles deep tendon reflexes are decreased, and quadriceps deep tendon reflexes are normal. Sensation to pinprick and vibration is decreased from just above the ankles distally. This patient is most likely to describe his pain as which of the following? Aching Cramping Squeezing Sharp Burning Colicky

Burning

A 75-year-old man with chronic obstructive pulmonary disease is admitted to the hospital 6 hours after an acute exacerbation of his disease. He is intubated and mechanically ventilated, and treatment with an increased dosage of corticosteroids is begun. Five days later, his temperature is 38.3°C (100.9°F). His leukocyte count is 15,000/mm3 (90% segmented neutrophils). A Gram stain of sputum shows gram-negative rods. A chest X-ray shows a new patchy right lower lobe infiltrate. Which of the following endogenous chemoattractants primarily caused the recruitment of leukocytes to this site of acute inflammation? C5a Filamin Filamin Protein kinase C Phospholipase A2 N-Formyl methionine terminal amino acid peptides

C5a

A 21-year-old woman comes to the physician because of a 10-day history of difficulty walking. Two years ago, she had loss of vision in the left eye which improved over 2 months. Neurologic examination shows decreased visual acuity in the left eye with pallor of the optic disc. She has past-pointing on a finger-nose test. She has a broad-based gait. An MRI of the brain shows lesions in the white matter of the cerebellum. Which of the following best describes the pathogenesis of the disease process in this patient? Complexes of rheumatoid factor and lgG are deposited in the choroid plexus CD8+ T lymphocytes kill astrocytes CD4+ T lymphocytes are activated by myelin basic protein Antibodies with specificity for axonal microtubules bind to the surface of axons Inflammatory vasculitis affects perforating arterioles

CD4+ T lymphocytes are activated by myelin basic protein

A 32-year-old woman comes to the emergency department 3 hours after the sudden onset of a severe headache. The pain is associated with nausea and vomiting. Medical history is noncontributory. She is drowsy but easy to arouse. Her temperature is 37.1°C (98.8°F), pulse is 92/min, respirations are 10/min, and blood pressure is 130/70 mm Hg. Examination of the head shows no abnormalities. Flexion of the neck produces pain. The optic fundi are normal. Motor and sensory examinations show no abnormalities. Cranial nerves are intact. Deep tendon reflexes are symmetric. Babinski sign is present bilaterally. Which of the following is most likely to confirm the diagnosis? EEG Biopsy of the temporal artery X-rays of the sinuses CT scan of the head Carotid duplex ultrasonography

CT scan of the head

A 63-year-old man comes to the physician because of a 6-month history of exertional chest pain that is relieved by rest. He has smoked 1 pack of cigarettes daily for 45 years. He has mild hypertension, but he takes no medications. Which of the following types of lesions in the left anterior descending coronary artery is the most likely cause of his condition? Vasospastic 100%, occlusion Embolism from the left atrium Calcified 80% stenosis Ruptured plaque overlying a 30% stenosis Fresh thrombus on a 50% stenosis

Calcified 80% stenosis

A 45-year-old woman has a thyroidectomy because of asymmetric enlargement of the thyroid gland, first noticed during a physical examination 6 weeks ago. She underwent an adrenalectomy for pheochromocytoma 3 years ago. Microscopically, the bilateral thyroid lesions are composed of spindle cells arranged in small clusters. Deposits of amyloid are present between neoplastic cells. Foci of C-cell hyperplasia are also present. The lesions are confirmed as malignant. Which of the following markers is most appropriate to monitor for the development of recurrence of the thyroid neoplasm? Norepinephrine Human chorionic gonadotropin Calcitonin A-Fetoprotein Thyroid-stimulating hormone

Calcitonin

A 50-year-old man comes to the physician 3 days after his first generalized tonic-clonic seizure. He has a 1-month history of frequent episodes of a pins-and- needles sensation around the mouth and in the hands and feet. He also has had occasional episodes of involuntary contraction of the muscles of the hands or feet leading to wrist flexion or foot plantar flexion. His pulse is 80/min, and blood pressure is 150/90 mm Hg. Neurologic examination shows mild, diffuse hyperreflexia. An abnormality of which of the following serum electrolyte concentrations is the most likely explanation for his symptoms? Calcium Potassium Bicarbonate Chloride Sodium

Calcium

A 66-year-old woman is brought to the emergency department by her daughter because of a 2-day history of fever, flank pain, pain with urination, and nausea. Ten days ago, she was admitted to the hospital for similar symptoms and was diagnosed with acute pyelonephritis. She was discharged with instructions to take oral ciprofloxacin after a 3-day course of intravenous ciprofloxacin resulted in improvement. She also has hypertension, hyperlipidemia, and osteoporosis. Current medications also include alendronate, calcium carbonate, ezetimibe, hydrochlorothiazide, and simvastatin. Her temperature is 39.1°C (102.4°F), and blood pressure is 130/80 mm Hg. The most likely cause of this patient's current condition is an interaction between her current oral antibiotic and which of the following medications? Hydrochlorothiazide Calcium carbonate Simvastatin Ezetimibe Alendronate

Calcium carbonate

A 20-year-old woman comes to the physician because of severe dysuria and a painful vulvar rash for 2 days. Examination shows vesicular lesions on an erythematous base on the right vulva. Which of the following is the most likely causal organism? Neisseria gonorrhoeae Parvovirus Chlamydia trachomatis Group B streptococcus I ) Treponema pallidum D) Hepatitis B J) Trichomonas vagina/is E) Herpes simplex virus K) Varicella-zoster virus F) Human papillomavirus r [!] Candida albicans

Candida albicans

A 70-year-old woman is found to have persistent fever despite intravenous broad-spectrum antibiotic therapy 3 days after undergoing operative excision of the anterior pelvic organs for recurrent cervical carcinoma. Her temperature is 38.5° C ( 1 0 1 . 3° F). Physical examination shows the presence of a central venous catheter that was placed on the day of the operation. There is also a well-healing abdominal wound without erythema and with two drains in place. Blood cultures and cultures of the tip of the central catheter on a sheep blood agar plate grow the organism shown in the photograph. A Gram stain of one of the colonies shows 4-μm, elliptical, purple, budding organisms. Which of the following is the most likely causal organism? Candida albicans Sporothrix schenckii Escherichia coli Staphylococcus aureus Candida albicans

Candida albicans

A 22-year-old woman comes to the physician because of a 2-day history of pain with urination, intense vaginal itching, and a thick discharge. She has no history of serious illness. She is sexually active and uses an oral contraceptive. Her temperature is 37°C (98.6°F). Abdominal examination shows no abnormalities. Genitourinary examination shows erythema of the vulva and vagina with an odorless curd-like discharge. The cervix appears normal. Bimanual examination shows no abnormalities. The pH of the vaginal discharge is 4. Wet mount preparations of the discharge with saline and with KOH are obtained. The saline slide shows mature squamous epithelial cells, and the KOH slide shows multiple budding yeasts with pseudohyphae. Which of the following is the most likely diagnosis? Lichen sclerosus Candidiasis Bacterial vaginosis Chlamydia trachomatis infection Trichomoniasis Scabies

Candidiasis

A 22-year-old woman comes to the physician because of a 2-day history of pain with urination, intense vaginal itching, anda thick discharge. She has no history of serious illness. She issexually active and uses an oral contraceptive. Hertemperature is 37°C (98.6°F). Abdominal examination showsno abnormalities. Genitourinary examination shows erythemaof the vulva and vagina with an odorless curd-like discharge.The cervix appears normal. Bimanual examination shows noabnormalities. The pH of the vaginal discharge is 4. Wetmount preparations of the discharge with saline and withKOH are obtained. The saline slide shows mature squamousepithelial cells, and the KOH slide shows multiple buddingyeasts with pseudohyphae. Which of the following is the mostlikely diagnosis? Bacterial vaginosis Scabies Trichomoniasis Chlamydia trachomatis infection Candidiasis Lichen sclerosus

Candidiasis

A 68-year-old man with alcoholism comes to the physician because of fever, chills, and a cough productive of purulent sputum for 3 days. Blood cultures are positive. Gram stains of sputum and blood show gram-positive, lancet-shaped diplococci. Administration of a vaccine directed against which of the following bacterial compounds most likely would have reduced the disease severity in this patient? Endotoxin Capsular polysaccharide Coagulase Lipoprotein(A) Peptidoglycan

Capsular polysaccharide

A 37-year-old woman is brought to the emergency department after her husband found her unconscious. Her temperature is 36°C (96.8° F), pulse is 128/min, and blood pressure is 70/40 mm Hg. Physical examination shows cool, pale extremities, jugular venous distention, faint peripheral pulses, and crackles over the bottom two thirds of both lung fields. Heart sounds are normal, and there are no murmurs. She withdraws to painful stimuli in all four extremities. This patient is most likely experiencing which of the following types of shock? Neurogenic Cardiogenic Septic Anaphylactic Hypovolemic

Cardiogenic

A 77-year-old man comes to the emergency department 1 hour after a 15-minute episode of right arm weakness and an inability to speak in sentences; the symptoms have now resolved. He has a 30-year history of hypertension treated with hydrochlorothiazide. He does not have diabetes mellitus or coronary artery disease. During the past 8 weeks, he has walked 3 miles daily without symptoms. His pulse is 80/min, respirations are 14/min, and blood pressure is 144/88 mm Hg. Examination, including cardiopulmonary and neurologic examinations, shows no other abnormalities. A carotid bruit is heard on the left. An ECG shows a normal sinus rhythm with a ventricular rate of 76/min. Echocardiography shows no abnormalities. Carotid duplex ultrasonography and follow-up MR angiography show an 80% left internal carotid stenosis. Which of the following is the most appropriate next step to prevent cerebral infarction in this patient? Percutaneous coronary intervention Extracranial-intracranial bypass surgery Prophylactic atorvastatin Prophylactic warfarin Prophylactic naproxen Carotid endarterectomy

Carotid endarterectomy

A 77-year-old man comes to the emergency department 1 hour after a 15-minute episode of right arm weakness and an inability to speak in sentences; the symptoms have now resolved. He has a 30-year history of hypertension treated with hydrochlorothiazide. He does not have diabetes mellitus or coronary artery disease. During the past 8 weeks, he has walked 3 miles daily without symptoms. His pulse is 80/min,respirations are 14/min, and blood pressure is 144/88mm Hg. Examination, including cardiopulmonary and neurologic examinations, shows no other abnormalities. A carotid bruit is heard on the left. An ECG shows a normal sinus rhythm with a ventricular rate of 76/min. Echocardiography shows no abnormalities. Carotid duplex ultrasonography and follow-up MR angiography show an 80% left internal carotid stenosis. Which of the following is the most appropriate next step to prevent cerebral infarction in this patient? Prophylactic naproxen Prophylactic warfarin Percutaneous coronary intervention Prophylactic atorvastatin Carotid endarterectomy Extracranial-intracranial bypass surgery

Carotid endarterectomy

A new compound is taken up by bacterial cells. No energy is necessary for uptake, and the compound is not concentrated in the cell. Which of the following best describes this mechanism of transport? Phosphorylation-linked transport Proton-gradient-mediated transport Group translocation Carrier-mediated diffusion ATP-dependent active transport

Carrier-mediated diffusion

A 26-year-old man develops loose, foul-smelling stools that float. Histologic analysis of tissue obtained on biopsy of the proximal small intestine indicates loss of villi (flat mucosa). Which of the following is the most likely diagnosis? Ascariasis Ulcerative colitis Inflammatory bowel disease Shigellosis Celiac disease

Celiac disease

A 58-year-old male presents to the emergency department with chest pain. ECG reveals ST-segment elevation. A coronary angiography reveals a coronary thrombus in the left anterior descending coronary artery. Biopsy of the myocardial cells that border the infarct demonstrate swelling. Which of the following represents the most likely cause of the swelling? Increased production of H2O2 Loss of intracellular Na+ Increased intracellular pH Loss of intracellular Ca2+ Cellular depletion of ATP

Cellular depletion of ATP

A 62-year-old man with alcoholism is admitted to the hospital because of somnolence and hepatic failure. On admission, his serum sodium concentration is 109 mEq/L. Hypertonic saline is administered, and the next day his serum sodium concentration is 138 mEq/L. Three days after admission, he has severe weakness; neurologic examination shows flaccid paresis of both the upper and lower extremities. Which of the following is the most likely cause of this patient's acute neurologic symptoms? Central pontine myelinolysis Hepatorenal syndrome Vitamin B1 (thiamine) deficiency Alcohol withdrawal Hepatic encephalopathy

Central pontine myelinolysis

A 62-year-old man with alcoholism is admitted to the hospital because of somnolence and hepatic failure. On admission, his serum sodium concentration is 109 mEq/L. Hypertonic saline is administered, and the next day his serum sodium concentration is 138 mEq/L. Three days after admission, he has severe weakness; neurologic examination shows flaccid paresis of both the upper and lower extremities. Which of the following is the most likely cause of this patient's acute neurologic symptoms? Hepatorenal syndrome Vitamin B1 (thiamine) deficiency Hepatic encephalopathy Central pontine myelinolysis Alcohol withdrawal

Central pontine myelinolysis

A 59-year-old woman comes to the physician because of the gradual onset of lack of muscle control in her left arm and leg. Her symptoms began 1 month ago after she was diagnosed with breast cancer that has metastasized to other regions of her body. Physical examination shows ataxia of the left upper and lower extremities. Muscle strength, deep tendon reflexes, sensation, and proprioception are normal. The most likely explanation for this patient's condition is a metastatic tumor in which of the following locations on the left? Lumbar spinal cord Cervical spinal cord Thoracic spinal cord Cerebellum Cerebrum

Cerebellum

An investigator is studying the human immune response to tumor cell antigens in malignancies. Which of the following sets of cancer types and tumor antigens is most likely to produce the highest antibody titer? Prostate cancer B-cell lymphoma Melanoma Breast cancer Cervical cancer

Cervical cancer

A 45-year-old man has had a 1-week history of increasing neck pain when he turns his head to the right. He also has had a pins-and-needles sensation starting in the neck and radiating down the right arm into the thumb. His symptoms began 3 months ago when he developed severe pain in the neck and right shoulder. Neurologic examination shows limitation of motion on turning the neck to the right. There is 4+/5 weakness of the right biceps and decreased pinprick over the right thumb. Deep tendon reflexes are 1+ in the right biceps and brachioradialis; all others are 2+. Which of the following is the most likely diagnosis? Ulnar nerve compression Carpal tunnel syndrome Thoracic outlet syndrome Cervical root compression Multiple sclerosis

Cervical root compression

A 4-month-old boy is brought to the physician because of recurrent sinopulmonary infections since birth. Physical examination shows hypopigmentation of the skin, eyes, and hair and numerous ecchymoses over the trunk and extremities. A peripheral blood smear shows giant granules in neutrophils and eosinophils. Which of the following is the most likely underlying cause of these findings? Acute myeloblastic leukemia Ataxia telangiectasia Leukocyte adhesion deficiency Sezary syndrome Chediak-Higashi syndrome

Chediak-Higashi syndrome

A 10-day-old male newborn is brought to the physician by his mother because of a 1-day history of red eyes with discharge. Pregnancy and delivery were uncomplicated, but the mother did not have prenatal care until the third trimester. The mother recalls no contact with ill people since his birth. Physical examination shows bilateral conjunctival injection with a watery discharge. Which of the following infectious agents is the most likely cause of the findings in this newborn? Neisseria gonorrhoeae Streptococcus pneumoniae Parainfluenza virus Cytomegalovirus Chlamydia trachomatis

Chlamydia trachomatis

A 27-year-old man comes to the physician because of painwith urination for 3 days. He has had no fever or chills. Heis sexually active with three partners and does not usecondoms consistently. He began having sexual relationswith his most recent partner 1 week ago. His temperature is37°C (98.6°F). Physical examination shows noabnormalities except for a clear, watery urethral discharge.Urinalysis shows 10-20 WBC/hpf but no bacteria. Which ofthe following is the most likely causal organism? Chlamydia trachomatis Escherichia coli Entamoeba histolytica Helicobacter pylori Shigella flexneri

Chlamydia trachomatis

A 25-year-old man who just returned from work as a Peace Corps worker in Africa begins oral chloroquine therapy for malaria caused by Plasmodium vivax. His initial therapeutic response is good, but he develops recurrent parasitemia 2 months later. Which of the following best explains the recurrence? Chloroquine is only effective against P. vivax when combined with metronidazole The patient is infected with a chloroquine-resistant strain of P. vivax The patient has a second previously occult malarial infection Chloroquine is ineffective on the exoerythrocytic malarial tissue stages Chloroquine is ineffective as oral therapy for P. vivax malaria

Chloroquine is ineffective on the exoerythrocytic malarial tissue stages

A 42-year-old woman comes to the physician because of a 1-month history of abdominal pain, especially after eating fatty meal s. She is 180 cm (5ft 11in) tall and weighs 100 kg (220 lb); BMI is 31 kg/m2. Physical examination shows jaundice and tenderness of the right upper quadrant of the abdomen. An increase in which of the following liver functions is most likely in this patient? Secretion of angiotensinogen Glycogen synthesis Conjugation of bilirubin Deaminalion of amino acids Cholesterol synthesis

Cholesterol synthesis

A 39-year-old woman comes to the physician for a follow upexamination because she recently was diagnosed withhypertension. Her blood pressure is 156/100 mm Hg.Physical examination shows no other abnormalities. Serumstudies show normal findings. A 24-hour urine collectionshows three times the normal excretion of epinephrine andmetanephrine. The excessive epinephrine production in this patient is most likely caused by which of the following celltypes? Zona glomerulosa Zona reticularis Zona fasciculata Juxtaglomerular Chromaffin

Chromaffin

Case of DiGeorge syndrome. These findings are most common with which of the following mechanisms? Mutation of tumor suppressor gene Chromosomal microdeletion Nucleotide expansion Defect in fibrillin synthesis Genomic imprinting Abnormal ciliary motility

Chromosomal microdeletion

An experimental animal is created that has a defect in an innate gastrointestinal defense mechanism. The animal is found to have decreased gastric hydrogen chloride production. After 2 months, endoscopic biopsy specimens of the gastric fundus and body show decreased mucosal thickness and hyperplasia of enterochromaffin-like cells. The disease process in this experimental animal most closely resembles which of the following? Chronic gastritis Zollinger-Ellison syndrome Whipple disease Gastric polyp Barrett esophagus Peptic ulcer disease

Chronic gastritis

A 42-year-old man with hepatitis C undergoes a liver biopsy. Pathologic examination shows lymphocytic infiltration and collagen deposition. These findings are most characteristic of which of the following inflammatory processes? Tissue necrosis Abscess formation Granuloma formation Purulent exudate Chronic inflammation

Chronic inflammation

A 67-year-old woman comes to the physician because of an8-month history of progressive shortness of breath. Theshortness of breath initially occurred only with walking longdistances but now occurs after walking ¼ mile to hermailbox. She also has a daily morning cough productive ofwhitish tan sputum. She has had no chest pain, palpitations,orthopnea, or paroxysmal nocturnal dyspnea. She hassmoked one pack of cigarettes daily for 52 years. Her pulseis 88/min, respirations are 20/min, and blood pressure is144/90 mm Hg. Examination shows a barrel-shaped chest.Breath sounds are decreased, and faint expiratory wheezesare heard in all lung fields. There is no peripheral edema. Anx-ray of the chest shows no abnormalities except forhyperinflation. Which of the following is the most likelydiagnosis? Panic disorder Congestive heart failure Asthma Chronic pulmonary embolism Angina pectoris Chronic obstructive pulmonary disease

Chronic obstructive pulmonary disease

A 20-year-old woman is found to have a blood pressure of 140/100 mm Hg at a routine annual examination. Fasting serum studies show a urea nitrogen concentration of 50 mg/dl, and glucose concentration of 90 mg/dl. Urinalysis shows numerous WBCs and WBC casts; protein excretion is 3000 mg/24 h (N<150). The estimated glomerular filtration rate is 20% of normal. Ultrasonography shows small asymmetric kidneys with broad scars and blunted calyces, and voiding cystourethrography shows a vesicoureteral reflux. Which of the following is the most likely diagnosis? Acute glomerulonephritis Bilateral hydronephrosis Chronic pyelonephritis Diabetic nephropathy Renal amyloidosis

Chronic pyelonephritis

A 21-year-old college student comes to student health services requesting medication to help her sleep. Four days ago, she returned from a 1-year trip to India where she studied comparative religions. Since her return, she has been unable to fall asleep until 4 or 5 AM and has difficulty awakening before noon. She constantly feels tired, has difficulty concentrating, and does not feel ready to begin classes. Her appetite has not decreased, but she has an aversion to eating meat since following a vegetarian diet in India. She has no history of medical or psychiatric illness. She takes no medications and does not drink alcohol. She appears sleepy. She is 173 cm (5 ft 8 in) tall and weighs 54 kg (120 lb); BMI is 18 kg/m2. Her pulse is 60/min, and blood pressure is 115/70 mm Hg. She is alert and oriented to person, place, and time. When asked to subtract serial sevens from 100, she begins accurately but then repeatedly loses track of the sequence. Which of the following is the most likely diagnosis? Malingering Bipolar disorder Adjustment disorder with depressed mood Primary insomnia Circadian rhythm sleep disorder Major depressive disorder

Circadian rhythm sleep disorder

A 21-year-old college student comes to student health services requesting medication to help her sleep. Four days ago, she returned from a 1-year trip to India where shes tudied comparative religions. Since her return, she has been unable to fall asleep until 4 or 5 AM and has difficulty awakening before noon. She constantly feels tired, has difficulty concentrating, and does not feel ready to begin classes. Her appetite has not decreased, but she has an aversion to eating meat since following a vegetarian diet in India. She has no history of medical or psychiatric illness. She takes no medications and does not drink alcohol. She appears sleepy. She is 173 cm (5 ft 8 in) tall and weighs 54 kg (120 lb); BMI is 18 kg/m2. Her pulse is 60/min, and blood pressure is 115/70 mm Hg. She is alert and oriented to person, place, and time. When asked to subtract serial sevens from 100, she begins accurately but then repeatedly loses track of the sequence. Which of the following is the most likely diagnosis? Adjustment disorder with depressed mood Circadian rhythm sleep disorder Bipolar disorder Primary insomnia Major depressive disorder Malingering

Circadian rhythm sleep disorder

A 5-year-old girl is brought to the physician because of listlessness, fatigue, and dull pain in the right upper quadrant of the abdomen. Her height and weight are below the 25th percentile. Laboratory findings indicate that the content of her 13-globin chain is 15% to 20% of normal. Sequencing of the 13-globin gene shows a point mutation in a sequence 3' to the coding region in which AATAAA is converted to AACAAA. Consequently, the amount of mRNA for 13-globin is decreased to 10%, of normal. Which of the following functions in mRNA synthesis and processing is most likely encoded by the sequence AATAAA? Transport of the mRNA out of the nucleus Capping with GTP Splicing of the initial mRNA transcript in the nucleus Silencing of the promoter Cleavage and polyadenylation

Cleavage and polyadenylation

A 27-year-old nulligravid woman comes to the physician because of the inability to conceive for 2 years. Menses have occurred at 3-to 4-month intervals since menarche at the age of 14 years. Physical and pelvic examinations show no abnormalities. Her serum follicle-stimulating hormone, luteinizing hormone, and estrogen concentrations are within the reference ranges. Which of the following is the most appropriate pharmacotherapy for this patient? Metformin Low-dose estrogen Medroxyprogesterone Clomiphene Tamoxifen

Clomiphene

A 53-year-old man develops a fever and hypotension while being treated in the hospital for a crush injury to the right lower extremity. The leg appears infected. During surgical debridement, extensive myonecrosis is found. Gram stain of the excised tissue shows large gram-positive rods with few neutrophils. Which of the following is the most likely causal organism? Staphylococcus aureus Listeria monocytogenes Bacteroides fragilis Clostridium perfringens Bacillus subtilis

Clostridium perfringens

A 39-year-old man comes to the physician because he has had an intense, stabbing pain behind his left eye for 5 nights. The headaches generally begin 1 hour after he goes to bed and last for about 1 hour. During the pain, he has left-sided nasal stuffiness and tearing from his left eye. He had a similar series of nightly headaches for 6 weeks several years ago. Which of the following is the most likely diagnosis? Sinusitis Tension-type headache Factitious disorder Cluster headaches Temporal arteritis Generalized anxiety disorder

Cluster headaches

A 65-year-old man dies 4 days after complete occlusion of the left coronary artery. The left ventricle contains a large irregular area that is soft and pale. Which of the following processes is most likely responsible? Dystrophic calcification Fat necrosis Gangrenous necrosis Coagulation necrosis Liquefaction necrosis Caseation necrosis Metastatic calcification

Coagulation necrosis

A 2-month-old girl is brought to the physician for a well-child examination. Her pulse is 110/min, respirations are 22/min, and blood pressure is 95/50 mm Hg. Cardiac examination shows a grade 3/6, systolic murmur that is best heard in the interscapular region. Femoral pulses are decreased bilaterally. Which of the following is the most likely cause of these findings? Ventricular septal defect Coarctation of the aorta Atrial septal defect Total anomalous pulmonary venous return Patent ductus arteriosus

Coarctation of the aorta

A 2-year-old boy with Down syndrome is brought to the physician by his mother for a follow-up examination. His blood pressure is increased in the upper extremities and decreased in the lower extremities. Physical examination shows cyanosis of the lower extremities. An AP x-ray of the chest shows notching of the inferior surfaces of the 3rd through 8th ribs. The diagnosis of left ventricular hypertrophy is suspected. Which of the following is the most likely underlying abnormality in this patient? Tetralogy of Fallot Transposition of the great arteries Persistent truncus arteriosus Atrial septal defect Coarctation of the aorta

Coarctation of the aorta

A 52-year-old man has chronic pancreatitis. Deficiency of which of the following is the most likely cause of this patient's inability to digest triglycerides? Phospholipase A2 Colipase Cholesterolester hydrolase Monitor peptide Carbonic anhydrase

Colipase

A male newborn has macrocephaly with poor skull mineralization, shortened extremities with misshaped long bones, and several fractures that occurred during birth. A defect in which of the following is most likely? Collagen Sulfatase Glycosyltransferase Chondroitin sulfate Glycosidase Elastin

Collagen

A 5-year-old girl is brought to the physician by her parents for evaluation of recurrent injuries. Her parents say that she started walking at the age of 14 months and since then has always seemed clumsier and had more injuries than other children. She has had increasingly frequent pain with exertion since starting a soccer program 3 months ago. She usually has pain or swelling of her knees or ankles after practice. She has been taken to the emergency department three times during the past 3 weeks because of concern about possible fractures; x-rays showed no abnormalities. Today, the patient walks with a limp. Examination shows numerous paper-like scars over the torso and upper and lower extremities. A midsystolic click is heard at the apex. The left ankle is swollen and tender; range of motion is limited by pain. The hips and the joints of the upper and lower extremities are hypermobile, including 25 degrees of genu recurvatum, thumbs that may be extended to touch the forearms, and flexibility at the waist, with palms easily touching the floor with straight knees. Which of the following is the most likely explanation for this patient's physical findings? Complement deficiency Vitamin D deficiency Immune complex deposition Defect in the migration of segmented neutrophils Collagen abnormality Cerebellar degeneration

Collagen abnormality

A 5-year-old girl is brought to the physician by her parents for evaluation of recurrent injuries. Her parents say that she startedwalking at the age of 14 months and since then has always seemed clumsier and had more injuries than other children. She hashad increasingly frequent pain with exertion since starting a soccer program 3 months ago. She usually has pain or swelling of herknees or ankles after practice. She has been taken to the emergency department three times during the past 3 weeks because ofconcern about possible fractures; x-rays showed no abnormalities. Today, the patient walks with a limp. Examination showsnumerous paper-like scars over the torso and upper and lower extremities. A midsystolic click is heard at the apex. The left ankleis swollen and tender; range of motion is limited by pain. The hips and the joints of the upper and lower extremities arehypermobile, including 25 degrees of genu recurvatum, thumbs that may be extended to touch the forearms, and flexibility at thewaist, with palms easily touching the floor with straight knees. Which of the following is the most likely explanation for thispatient's physical findings? Defect in the migration of segmented neutrophils Collagen abnormality Complement deficiency Cerebellar degeneration Vitamin D deficiency Immune complex deposition

Collagen abnormality

A 32-year-old woman comes to the physician because of a 6-month history of increased urinary frequency, especially at night, and increased thirst. She tells the physician that she is always thirsty despite frequent water intake. She has bipolar disorder treated with lithium carbonate. Physical examination shows no abnormalities. Her serum sodium concentration is 154 mEq/L, and urine osmolality is 180 mOsmol/kg. Abnormal function of which of the following structures is the most likely explanation for these findings? Glomerulus Collecting duct Afferent arteriole Loop of Henle Proximal convoluted tubule Efferent arteriole

Collecting duct

A 3-year-old boy is brought to the physician for a follow-up examination 7 days after he was diagnosed with severe chronic diarrhea due to Giardia lamblia infection. Immunologic studies are done. Results show that peripheral leukocytes that express both CD3 and CD4 completely fail to express CD40 ligand. Based on this finding, the immunoglobulin isotype that predominates in this patient's serum most likely has which of the following biologic properties? Complement activation Attachment to Fe receptors on mast cells Ability to pass across the placenta Attachment to Fe receptors on eosinophils Easy secretion across mucosa! epithelial cells

Complement activation

A 43-year-old woman comes to the physician because of a 6 -month history of altered consciousness. During the interview, she stops talking in mid-sentence, turns her head to the right, and extends and stiffens her right upper extremity. She has a blank look and does not respond to any questions. She then has some repetitive lip smacking and picking movements of the hands. The episode lasts approximately 30 seconds. She slowly returns to her normal state during the next 4 to 5 minutes. The most likely diagnosis is which of the following types of seizures? Generalized tonic-clonic Absence Simple partial motor Simple partial sensory Complex partial

Complex partial

A female newborn delivered at 32 weeks' gestationdevelops severe respiratory distress within hours of birth.Despite resuscitative efforts, the patient dies. Examinationof the lungs at autopsy shows lung alveoli with radii of lessthan 50 μm (N=100). Which of the following is most likelydecreased in the lungs of this newborn? Vascular resistance Compliance Surface tension Elastic recoil Airway resistance

Compliance

A 67-year-old woman comes to the physician for a follow-up examination. She had a pulmonary embolism and required treatment in the hospital for 3 weeks. She had a retroperitoneal hemorrhage; anticoagulant therapy was temporarily discontinued, and she underwent placement of an inferior vena cava (IVC) filter. She had a hematoma that was resolving on discharge from the hospital 2 weeks ago. Today, she says she has had a persistent sensation of tingling and numbness of her left thigh that she did not report in the hospital because she thought it would go away; the sensation has improved somewhat during the past week. Her only medication is warfarin. Vital signs are within normal limits. Examination of the skin shows no abnormalities. Muscle strength is normal. Sensation to light touch is decreased over a 5 × 5-cm area on the lateral aspect of the left anterior thigh. Which of the following is the most likely cause of this patient's decreased sensation? Vitamin B12 (cobalamin) deficiency Cerebral infarction during the hospitalization Compression of the lateral femoral cutaneous nerve Hematoma of the left thigh Spinal cord infarct Complication of the IVC filter placement

Compression of the lateral femoral cutaneous nerve

A 67-year-old woman comes to the physician for a follow-up examination. She had a pulmonary embolism and required treatment in the hospital for 3 weeks. She had a retroperitoneal hemorrhage; anticoagulant therapy was temporarily discontinued, and she underwent placement of inferior vena cava (IVC) filter. She had a hematoma that was resolving on discharge from the hospital 2 weeks ago.Today, she says she has had a persistent sensation of tingling and numbness of her left thigh that she did not report in the hospital because she thought it would go away; the sensation has improved somewhat during the past week. Her only medication is warfarin. Vital signs are within normal limits.Examination of the skin shows no abnormalities. Muscle Strength is normal. Sensation to light touch is decreased over 5 × 5-cm area on the lateral aspect of the left anterior thigh.Which of the following is the most likely cause of this patient's decreased sensation? Spinal cord infarct Vitamin B12 (cobalamin) deficiency Complication of the IVC filter placement Cerebral infarction during the hospitalization Hematoma of the left thigh Compression of the lateral femoral cutaneous nerve

Compression of the lateral femoral cutaneous nerve

A 28-year-old female primigravida at 38 weeks gestation, presents for a routine evaluation. Her blood pressure recorded in the sitting, standing, and supine positions is 120/75, 115/68, and 88/58 mm Hg, respectively. Which of the following most likely accounts for these findings? Preeclampsia Uterine contraction Peripartum cardiomyopathy Compression of the vena cava Gestational diabetes mellitus

Compression of the vena cava

A 25-year-old woman undergoes a radical mastectomy with removal of the axillary lymph nodes on the left side. After the operation, she develops painful edema of the left upper extremity. Which of the following therapies is most likely to decrease the swelling in this patient? Passive movement Diuretic therapy Exercise therapy Application of heat Compression sleeve

Compression sleeve

Cholera toxin catalyzes the transfer of an ADP-ribose to an arginine residue present in the stimulatory G (Gs) protein resulting in the inhibition of the GTPase activity of Gs on intestinal cell membranes. Which of the following is most likely to increase in these cells as a result of this process? Rate of cell division Release of Ca2+ ions from the endoplasmic reticulum Concentration of cAMP Concentration of inositol 1,4,5-trisphosphate Tyrosine kinase activity of the cholera toxin receptor

Concentration of cAMP

A 3799-g (8-lb 6-oz) female newborn is born by cesareandelivery because of a breech presentation. Apgar scores are7 and 9 at 1 and 5 minutes, respectively. Initial examinationshows a palpable clunk when the left hip is abducted,flexed, and lifted forward. Posterior pressure on the flexedhip elicits a similar clunk. The remainder of theexamination shows no abnormalities. Which of thefollowing is the most likely diagnosis? Legg-Calvé-Perthes disease Osteogenesis imperfecta Osgood-Schlatter disease Congenital hip dysplasia Slipped capital femoral epiphysis

Congenital hip dysplasia

A 2-month-old girl is brought to the physician for a follow-up examination 2 days after a complete blood count was found to be abnormal. Her blood group is A, Rh-positive. She was born at term to a healthy 23-year-old woman whose blood group is 0, Rh-positive. Birth weight was 3890 g (8lb 9oz). Pregnancy and delivery were uncomplicated. The patient has been active and feeding well. Her temperature is 37.6° C (99.7°F), pulse is 120/min, and respirations are 30/min. Physical examination shows no abnormalities. Laboratory studies done since birth are shown. Severe combined immunodeficiency DiGeorge syndrome Alloimmune hemolytic disease of the newborn Congenital neutropenia Congenital cytomegalovirus infection

Congenital neutropenia

A 4-year-old boy is brought to the physician by his parents because he has had two bacterial urinary tract infections during the past year. Physical examination shows no abnormalities. Radiologic studies of the urinary tract show marked dilation of the left ureter and renal pelvis , and minimal left-sided renal function. A left nephrectomy is performed. A photograph of the resected specimen is shown. Which of the following is the most likely cause of the renal disease in this patient? Congenital ureteral obstruction Nephroblastoma (Wilms tumor) Adrenal cortical hyperplasia: Unilateral Systemic lupus erythematosus Benign prostatic hyperplasia

Congenital ureteral obstruction

A 62-year-old woman comes to the physician because of a 4-week history of progressive shortness of breath. Her temperature is 100 F, pulse is 96/min, respiration rate is 28/min, and blood pressure is 162/86 mm Hg. Her jugular venous pressure is 20 cm H20, carotid upstrokes are decreased, and the trachea is midline. There is dullness to percussion at the base of both lung fields, breath sounds are decreased at both bases, and there are crackles halfway up both lung fields. Cardiac examination shows a normal S1 and S2, and S3 gallop, and a grade 2/6 holosystolic murmur heard at the apex. Which of the following is the most likely diagnosis? Bronchospasm Atelectasis Congestive heart failure Consolidation Pulmonary hypertension

Congestive heart failure

A 2-week-old boy is brought to the physician because of a 3-day history of persistent discharge from his eyes. He was born at term following an uncomplicated pregnancy and delivery. Examination of the eyes shows tarsal inflammation and a thin mucopurulent discharge. Testing of scrapings from the tarsal conjunctivae is positive for Chlamydia trachomatis. The remainder of the examination shows no abnormalities. Which of the following is the most likely mode of transmission of this patient's infection? Transplacental transmission Poor hand washing by caregivers Exposure to household pets Contact with maternal cervical secretions Aerosol exposure

Contact with maternal cervical secretions

A 36-year-old woman who was recently diagnosed with HIV infection comes to the physician for a follow-up examination. Two months ago, an antiretroviral therapy regimen consisting of efavirenz, emtricitabine, and tenofovir was initiated. Laboratory studies show a CD4+ T-lymphocyte count of 352/ml and an undetectable plasma HIV viral load. Two months ago, her CD4+ T-lymphocyte count was 158/ml, and plasma HIV viral load was 5500 copies/ml. Which of the following is the most appropriate next step in management? Continue tenofovir and efavirenz and discontinue emtricitabine Continue emtricitabine and discontinue tenofovir and efavirenz Continue tenofovir and discontinue efavirenz and emtricitabine Continue efavirenz, emtricitabine, and tenofovir with no changes Continue efavirenz and emtricitabine and discontinue tenofovir Continue tenofovir and emtricitabine and discontinue efavirenz Continue efavirenz and discontinue tenofovir and emtricitabine

Continue efavirenz, emtricitabine, and tenofovir with no changes

A 52-year-old woman with breast cancer comes to the physician for a follow-up examination. She had a 4-week course of radiation treatment 6 months ago. Her respirations are 26/min. Physical examination shows no recurrence of the cancer. A CT scan of the chest show Resorption Compression Contraction Obstruction Consolidation

Contraction

A 68-year-old man comes to the physician because of a 10-month history of shortness of breath and swelling of his feet. He has a family history of cardiovascular disease. He has smoked 2 packs of cigarettes daily for 50 years. His pulse is 80/min, respirations are 24/min, and blood pressure is 150/80 mm Hg. Physical examination shows 3+ pitting edema of the lower extremities. Diffuse, scattered wheezes are heard bilaterally on auscultation of the chest. Cardiac examination shows a grade 2/6 pansystolic murmur heard best at the lower left sternal border, which increases on inspiration. The point of maximal impulse is palpated in the sub-xiphoid area. S1 and S2 sounds are distant. Abdominal examination shows a liver span of 14 cm. Which of the following is the most likely diagnosis? Cardiac amyloidosis Primary pulmonary hypertension Aortic stenosis Coronary artery disease Cor pulmonale

Cor pulmonale

A 33-year-old woman comes to the physician because of a 16-kg (35-lb) weight gain during the past 6 months despite dieting. She is 168 cm (5 ft 6 in) tall and now weighs 104 kg (230 lb); BMI is 37 kg/m2. Her blood pressure is 156/96 mm Hg. Physical examination shows a round, ruddy face with hirsutism, excess weight predominantly i n the shoulders and abdomen, and purple striae over the abdomen. An MRI shows a mass in the pituitary gland. A CT scan of the abdomen shows enlarged adrenal glands. Which of the following histologic changes would be expected in this patient's adrenal glands? Cortical metaplasia Cortical hyperplasia Medullary hypertrophy Cortical hypertrophy Medullary metaplasia Medullary hyperplasia

Cortical hyperplasia

Homologous pairing of chromosomes in prophase I is essential for which of the following normal processes? Centromere division Chromosome uncoiling DNA replication Crossing over

Crossing over

Which of the following tests would prove the patient is infectious for herpes simplex virus (HSV) type 2? Culture of lesions Detection by IgM specific antibodies Direct immunofluorescent assay Tzanck smear PCR amplification

Culture of lesions

A 21-year-old woman at 30 weeks' gestation is admitted to the hospital because of contractions for 1 hour. Fetal ultrasonography shows that the ductus arteriosus has closed. Inhibition of which of the following enzymes is the most likely cause of this finding? 5-Lipoxyenase Cyclooxygenase Guanylyl cyclase Glutathione S-transferase Adenylyl cyclase

Cyclooxygenase

A normotensive 54-year-old man with normal renal function receives a heart transplantation. One year later, he has a blood pressure of 170/110 mm Hg and a serum creatinine concentration of 2.1 mg/dl. Which of the following immunosuppressive drugs is most likely to have caused these findings? Muromonab-CD3 (OKT3) Cyclophosphamide Azathioprine Cyclosporine Corticosteroid

Cyclosporine

A 70-year-old woman comes to the physician for a routine pelvic examination. During speculum examination of the vagina and cervix, the Valsalva maneuver causes a bulge of the anterior vaginal wall. Which of the following is the most likely cause of this finding? Obturator hernia Cystocele Bartholin gland cyst Uterine prolapse Rectocele

Cystocele

A 73-year-old man has had a poor appetite and has lost 11.5 kg (25 lb) over the past 4 months. He has been otherwise asymptomatic. Laboratory evaluation shows normochromic normocytic anemia. An x-ray of the chest shows a 2-cm perihilar mass. Biopsy of the mass shows small cell carcinoma of the lung. Physical examination and imaging studies show no clinically detectable metastases. Which of the following is the most likely explanation for the weight loss? Infection Cytokine effect Superior vena cava syndrome Tumor hormone production Renal failure

Cytokine effect

A 4-year-old boy is brought to the physician by his father because of a 2-month history of progressive weakness and loss of muscle function. He has had difficulty running, climbing stairs, and rising from a sitting position. Physical examination shows hypertrophy of the calf muscles. Serum studies show a markedly increased creatine kinase activity. The most likely cause of these findings is a defective or absent protein in which of the following areas of muscle? Nucleolus Nucleus Golgi apparatus Cytoskeleton Mitochondria Endoplasmic reticulum

Cytoskeleton

A 25-year-old man is diagnosed with advanced testicular cancer. Combination chemotherapy is being considered. Cisplatin is included in the regimen because of its effect on which of the following? Purine ring biosynthesis Microtubules Ribonucleotide reductase DNA Dihydrofolate reductase

DNA

A 76-year-old woman who is a resident of a skilled nursing care facility is brought to the physician because of a 2-day history of fever and cough. Her temperature is 38.5°C (101.3°F). Treatment with ciprofloxacin is begun. Two days later, she is admitted to the hospital because of a progressive cough and a temperature of 39°C (102.2°F). Culture of sputum at this time grows Streptococcus pneumoniae. A mutation in genes encoding which of the following enzymes is the most likely cause of the organism's resistance to ciproftoxacin? Transpeptidase Acetyltransferase DNA gyrase Tetrahydrofolate reductase 3-Lactamase

DNA gyrase

A previously healthy 59-year-old man comes to the emergency department because of a 2-day history of fever and headache. His temperature is 39.4 °C (102.9° F), pulse is 84/min, respirations are 14/min, and blood pressure is 110/70 mm Hg. Physical examination shows no other abnormalities. He is oriented to person and place but not to time. A lumbar puncture is performed, and the results of polymerase chain reaction testing of cerebrospinal fluid show herpes simplex virus. It is most appropriate to administer a drug that inhibits which of the following viral enzymes? Neuraminidase RNase A Reverse transcriptase DNA polymerase Protease

DNA polymerase

A 65-year-old woman with ovarian cancer is treated with cyclophosphamide and other chemotherapeutic agents. Cyclophosphamide affects which of the following targets in this patient? Epidermal growth factor receptor tyrosine kinase Bcr-Abl tyrosine kinase DNA polymerase a DNA replication DNA topoisomerase II Caspase 3 DNA histone deacetylase

DNA replication

A 35-year-old woman participates in a study of a new diagnostic test of steroid hormones in saliva. Menses occur at regular 28-day intervals, and she has been shown to ovulate on day 14. When her saliva is tested on a particular day of the cycle, laboratory studies show an increased concentration of estradiol and decreased concentration of progesterone. On which of the following days in this patient's menstrual cycle were these hormones most likely measured? Day 18 Day 6 Day 1 Day 12 Day 26

Day 12

Two days after undergoing a right hip replacement operation, a 67-year-old woman has shortness of breath and pain on the right side of her back with deep breathing. She appears anxious. Her pulse is 100/min, and respirations are 24/min. A pleural friction rub is heard in the right lung base. Examination of the right calf shows erythema and induration. Immediate treatment with oxygen, 4L/min via nasal cannula, is initiated. Arterial blood gas analysis shows: pH 7.54 PCO2 29 mm Hg PO2 61 mm Hg CT angiography of the chest shows a large embolism occluding blood flow to the right lower lobe. Which of the following terms best describes the current ventilation-perfusion relationship in the right lower lobe of this patient? Dead space Diffusion abnormality Hypoventilation Shunt Low Fio2

Dead space

A 60-year-old male comes the physician for a follow-up examination. He was diagnosed with essential hypertension four-weeks ago. His blood pressure at that time was 150/90 mm Hg. He has been treated with an angiotensin-converting enzyme inhibitor since his diagnosis. His blood pressure today is 120/80 mm Hg. In addition to blocking the conversion of angiotensin I to angiotensin II, this drug also produce systemic vasodilation by which of the following mechanisms? Stimulation of nitric oxide synthesis Increased catecholamine release from the adrenal medulla Increased prostacyclin (PGI2) synthesis Decreased bradykinin degradation Decreased atrial natriuretic peptide release from the atria

Decreased bradykinin degradation

A 25-year-old man who has a history of facial flushing and rapid heart rate each time he consumes a small amount of alcohol participates in a study of alcohol intolerance. Molecular analysis shows the presence of a lysine (K487, oriental variant) for glutamate (E487, native variant) substitution in aldehyde dehydrogenase. The kinetic characteristics of the enzyme variants are shown in the table. Enzyme E487 K487 KM (NAO+) (μM) 37 5600 Based on these findings, which of the following is the most likely cause of this man's condition? Increased catalytic efficiency of E487 Weaker binding of NAO + to E487 More rapid turnover by K487 Less rapid turnover by E487 Decreased catalytic efficiency of K487 Tighter binding of NAO + to K487

Decreased catalytic efficiency of K487

A 22-year-old man comes to the physician because of an 8-year history of episodes of yellow-tinged eyes. He is a medical student, and he says that he has noticed that the symptom occurs during studying for final exams and other periods of intense stress. He has had no pain. Physical examination shows mild scleral icterus. Serum studies show: Decreased conjugation of bilrubin Hepatitis C virus Surreptitious acetaminophen abuse Intermittent obstruction of the common bile duct Increased hepatic copper concentration

Decreased conjugation of bilrubin

A previously healthy 67-year-old woman comes to the physician with her husband because of a 4-month history of a resting tremor of her right arm. Her husband reports that her movements have been slower and that she appears less stable while walking. Examination shows increased muscle tone in the upper extremities that is greater on the right than on the left. There is decreased right arm swing. Her gait is slow and shuffling. Which of the following is the most likely explanation for this patient's symptoms? Decreased serotonergic activity in the brain stem Bilateral frontal lobe degeneration Excessive output of oxytocin Excessive thalamic output of norepinephrine Decreased dopaminergic input to the striatum

Decreased dopaminergic input to the striatum

A 40-year-old woman is brought to the emergency department 1 hour after sustaining a gunshot wound to the abdomen. Despite appropriate intervention, she develops ventricular fibrillation and dies 30 minutes later. At autopsy, abdominal examination shows an entry wound over the upper midline. There is 3 L of fresh blood in the peritoneal cavity; the organs appear normal otherwise. Specimens of tissue from the liver and kidneys show centrilobular pallor and swelling of the hepatocytes and proximal convoluted epithelial cells, respectively. Which of the following mechanisms is the most likely direct cause of these renal findings? Binding of Fas ligand to its receptor Decreased function of Na +-K + ATPase Activation of aspartate-specific caspase Ribosomal disaggregation Stimulation of phosphofructokinase

Decreased function of Na +-K + ATPase

A 43-year-old man with a 10-year history of alcoholism comes to the physician because of a change in ski n color. The patient says that he has been taking two extra-strength acetaminophen tablets every 4 to 6 hours for the past 3 days for a severe headache. Physical examination shows jaundice. Laboratory studies show an increased prothrombin time and a markedly increased serum AST activity. Alteration in which of the following metabolites within hepatocytes is most likely associated with his illness? Increased glucuronide conjugates Decreased NAO• Increased NAD• Decreased NADH Decreased glucuronide conjugates Decreased glutathione Increased NADH Increased glutathione

Decreased glutathione

A 25-year-old man receives a bupivacaine injection for median nerve block as treatment of carpal tunnel syndrome. Which of the following is the mechanism of action of this drug on the membrane? Increased reuptake glutamate Decreased permeability to ca2• Inhibition of the Na•/Ca2• anti porter Decreased permeability to Na+ Activation of the Na•-K• ATPase

Decreased permeability to Na+

An 18-year-old woman develops sepsis after an abortion. Within 24 hours she becomes dyspneic, oliguric, and develops petechiae, ecchymoses, and bleeding from venipuncture sites. Which of the following is the laboratory finding most consistent with this disease process? Increased plasma hemolytic complement activity Increased plasma factor VIII (antihemophilic factor) concentration Decreased bleeding time Increased antineutrophil antibody titers Decreased plasma fibrinogen concentration

Decreased plasma fibrinogen concentration

A 48-year-old woman comes to the physician because of a 2-month history of fatigue and intermittent headaches. Her blood pressure is 180/110 mm Hg. Physical examination shows no other abnormalities. Serum studies show a decreased potassium concentration and increased aldosterone concentration. ACT scan of the abdomen shows a tumor on the adrenal gland. Which of the following additional findings in this patient most likely supports the diagnosis of an aldosterone-secreting adrenal adenoma? Decreased urine potassium concentration Increased serum sodium concentration Decreased serum sodium concentration Increased plasma renin activity Increased urine metanephrine concentration Decreased plasma renin activity

Decreased plasma renin activity

A 59-year-old male presents to the emergency department immediately after experiencing chest pain. He reports feeling the chest pain while running to catch a bus. His temperature is 98.7, pulse is 87/min, and blood pressure is 154/92 mm Hg. Chest auscultation reveals a normal S1 and faint splitting of the S2 heart sound. The lungs are clear bilaterally. ECG reveals normal sinus rhythm. The patient reports that the chest pain remains constant and does not change with respiration or position. The patient is administered a drug and within a few minutes he reports a significant decrease in chest pain. Which of the following represent the mode-of-action of this drug? Increased preload Decreased afterload Increased afterload and preload Increased contractility Decreased preload

Decreased preload

A 60-year-old man comes to the physician because of a 2-year history of progressive shortness of breath with exertion. His two younger siblings have had similar symptoms. His mother died of dilated cardiomyopathy. Physical examination shows jugular venous distention and ankle edema. There is hepatomegaly. Inspiratory crackles are heard over both lung bases. A chest x-ray shows cardiomegaly and pulmonary congestion; angiography confirms dilated cardiomyopathy. An atrial endocardial biopsy specimen shows abnormal sarcomeres with paracrystalline inclusions in mitochondria. Sequencing of mitochondrial DNA shows a G-to-A transition. This results in an added A:T base pair to the 3' end of the tRNA, thus shortening the C loop. Which of the following is the most likely effect of this mutation on mitochondrial function? Decreased protein synthesis Increased mRNA stability Decreased replication Increased replication Decreased mRNA stability Increased protein synthesis

Decreased protein synthesis

A 68-year-old female presents to the emergency department with acute substernal chest pain. She reports that it occurred while she was walking up four flights of stairs at work. Auscultation reveals basilar crackles in the base of both lungs, a normal S1 and decreased S2 heart sounds. A grade 3/6 systolic murmur is heard at the upper right sternal border and radiates to the neck. Cardiac catherization was performed at rest and during exercise (see data). The patient reports feeling short of breath during exercise. Which of the following is the most likely cause of the shortness of breath? Tachycardia Increased aortic pressure Decreased pulmonary blood flow Decreased cardiac output during exercise Increased end-diastolic pressure of the left ventricle during exercise

Decreased pulmonary blood flow

A 36-year-old man undergoes elective liposuction under general anesthesia. The operation is terminated prematurely when the anesthesiologist notices that the patient has developed hyperthermia, tachycardia, and marked muscle rigidity. The drug that should be administered has which of the following actions? Increases plasma Ca2+ concentration Enhances renal tubular excretion of Ca2+ Shifts Ca2+ from the extracellular to the intracellular space Opens cellular membrane Ca2+ and Na+ channels Decreases release of Ca2+ from the sarcoplasmic reticulum

Decreases release of Ca2+ from the sarcoplasmic reticulum

A 22-year-old man is brought to the emergency department 30 minutes after he was hit by an automobile while crossing the street. The bumper of the automobile struck him on the lateral side of his left leg. On physical examination, he is unable to dorsiflex the left foot. Inversion and eversion of the foot are normal. Which of the following nerves is most likely damaged in this patient? Femoral Obturator Deep fibular (peroneal) Lateral femoral cutaneous Tibial

Deep fibular (peroneal)

A 67-year-old woman has the sudden onset of dyspnea and chest pain and dies within minutes. Stage IV ovarian cancer was diagnosed 1 year ago, and she has been bedridden for 2 months. Which of the following is most likely to be found on autopsy? Deep venous thrombosis Renal papillary necrosis Ruptured berry aneurysm Acute myocardial infarcation Dissecting aortic aneurysm

Deep venous thrombosis

A 1-year-old girl is brought to the physician because of multiple bacterial infections since birth. She is at the 40th percentile for length and 40th percentile for weight. She has white hair, pale skin, blue irises, and prominent red pupils. A complete blood count shows neutropenia. A peripheral smear shows giant granules in the neutrophils. This patient's disorder is most likely due to which of the following pathogenetic mechanisms? Bone marrow suppression Defect in leukocyte adhesion NADPH oxidase deficiency Myeloperoxidase deficiency Defect in phagolysosome function

Defect in phagolysosome function

A 3-year-old girl is brought to the emergency departmentbecause of left leg pain after falling at preschool 2 hoursago. She has a history of fractures after minor trauma. Shehas consistently been at the 10th percentile for height andweight since birth. Examination shows blue sclerae. Thereis an obvious deformity of the left thigh. An x-ray shows anew fracture of the left femur and evidence of previousfracturing. Which of the following is the most likely causeof these findings? Hypophosphatemia Vitamin D deficiency Deficient sulfate ion transport Child abuse Defective type I collagen Hypocalcemia

Defective type I collagen

A 10-year-old girl has fatigue and appears pale. Her father underwent splenectomy as a youth for "low blood" and recently had a cholecystectomy. Her complete blood count shows: Hemoglobin 6.4 g/dL (N = 11.9-15 g/dL) Mean corpuscular volume 90.2 μm 3 (N = 76-90 μm 3) Leukocyte count 6000/mm 3 (N = 4500-13,500/mm 3) Neutrophils 58% Eosinophils 2% Lymphocytes 35% Monocytes 5% Platelet count 188,000/mm 3 (N = 150,000-400,000/mm 3) Reticulocyte count 9% of red cells (N = 0.5-1.5°/o of red cells) A peripheral blood smear is shown. Which of the following is the most common underlying abnormality that accounts for these clinical manifestations? Abnormal hemoglobin structure Impaired oxidative enzyme function Abnormal iron absorption Deficiency of erythrocyte spectrin Inadequate erythropoietin production

Deficiency of erythrocyte spectrin

Two days after beginning primaquine for malaria prophylaxis, a 17-year-old African American boy is brought to the physician because of dark urine and yellowing of his eyes. His pulse is 88/min, respirations are 12/min, and blood pressure is 124/68 mm Hg. Examination shows scleral icterus and pale mucous membranes. His hemoglobin concentration is 10 g/dL. A blood smear shows erythrocyte fragments and Heinz bodies. Which of the following is the most likely explanation for these findings? Decreased production of β-globin chains Deficiency of glycosylphosphatidylinositol Deficiency of glucose 6-phosphate dehydrogenase Fragmentation of erythrocytes by fibrin strands Decreased porphobilinogen deaminase activity

Deficiency of glucose 6-phosphate dehydrogenase

A previously healthy 48-year-old man is brought to the emergency department because of a 1-hour history of nausea and crushing chest pain that radiates to his left arm. An ECG and evaluation of serum cardiac enzyme activity confirm a diagnosis of a myocardial infarction of the anterior wall. Fourteen hours after arrival, he has a cardiac arrest. Resuscitation efforts are successful after approximately 30 minutes. He then immediately develops severe oliguria. Serum studies show increased urea nitrogen and creatinine concentrations. Microscopic examination of this patient's urine is most likely to show which of the following? Degenerating WBCs and colorless hyaline casts Dysmorphic RBCs and yellow granular casts WBCs and RBC casts Degenerating epithelial cells and dirty brown granular casts Numerous eosinophils and red hyaline casts Lymphocytes and RBC casts

Degenerating epithelial cells and dirty brown granular casts

A 43-year-old woman comes to the physician because of progressive difficulty walking during the past 3 months. Neurologic examination shows weakness and decreased muscle bulk of the lower extremities. Patellar and achilles tendon reflexes are diminished. Sensations of joint position, pain, and temperature in the lower extremities are normal. Which of the following is the most likely cause of the findings in this patient? Acute peripheral neuropathy Degeneration of motoneurons of the lumbar cord Demyelination of the corticospinal pathways Loss of afferent la axons innervating muscle spindles Myotonic muscular dystrophy

Degeneration of motoneurons of the lumbar cord

Ten years ago, a 60-year-old woman underwent an aortic valve replacement with a porcine heterograft. She now has shortness of breath with exertion. Examination and x-ray of the chest show evidence of congestive heart failure. Which of the following is the most likely explanation for these findings? Development of mitral stenosis Development of a ventricular septal defect Degeneration of the valve Development of an ascending aortic aneurysm Embolus to the coronary circulation

Degeneration of the valve

Four days following a hike through the forest, a 40-year-old man develops multiple intensely pruritic areas on his skin similar to the lesions shown in the photograph. The involved skin was not covered by clothing during the hike. He applies a topical ointment containing 1%, hydrocortisone to the affected areas and obtains moderate relief. The lesions subside over the next 1 to 2 weeks. These findings are most characteristic of which of the following conditions? Chronic granulomatous disease Insect bites Arthus reaction Sunburn Delayed type 4 hypersensitivity Localized anaphylaxis

Delayed type 4 hypersensitivity

Case of severe intellectual disability, ataxia, seizures, and inappropriate laughter. Which of the following is the most likely cause of this patient's condition? Deletion of maternal 15q11-13 Partial deletion of the short are of chromosome 5 Deletion of paternal 15q11-13 Translocation of maternal 15q11-13 Translocation of paternal 15q11-13 Deletion of maternal 22q11

Deletion of maternal 15q11-13

A 63-year-old woman is brought to the emergency department because of a 2-day history of fever, abdominal tenderness, and painful urination. She appears agitated. Her temperature is 38.8°C (101 .8 °F). Laboratory studies show a leukocyte count of 14,000/mm3. She is admitted to the hospital. Two hours later, the nurses note that she has torn up four breakfast menus because she found them to be too confusing. She is highly educated, and her husband says that she does not behave this way at home. Which of the following is the most likely cause of this patient' s current mental status? Mild cognitive impairment Dementia Delirium Occult cerebral infarction Amnesia

Delirium

A 71-year-old man with prostate cancer is admitted to the hospital for a prostatic resection. On admission, his mental status is normal. Two days after the operation, he is confused and restless. He complains that he cannot sleep because little men are coming through his window at night. Which of the following is the most likely diagnosis? Delirium Schizophrenia Dementia Brief psychotic disorder Delusional disorder

Delirium

A 72-year-old man is brought to the physician by his daughter because of a 2-day history of confusion, disorientation, and lethargy. He has had no weakness. He had a cerebral infarction 1 year ago and has been treated with daily aspirin since then. He is awake but lethargic. His temperature is 38.9°C (102°F), pulse is 82/min, respirations are 16/min, and blood pressure is 144/88 mm Hg. Physical examination shows no other abnormalities. He is disoriented to place and time but recognizes his daughter. Laboratory studies show: Hemoglobin 11.1 g/dL Leukocyte count 12,200/mm3 Segmented neutrophils 60% Eosinophils 2% Lymphocytes 30% Monocytes 8% Serum Na+ 130 mEq/L Serum Cl- 92 mEq/L Serum K+ 4.1 mEq/L Serum HCO3- 21 mEq/L Serum Urea nitrogen 29 mg/dL Serum Creatinine 1 mg/dL Urine pH 7 Specific gravity 1.020 WBC 10/hpf RBC 6/hpf Bacteria positive Nitrates positive Which of the following is the most likely diagnosis? Delirium Vascular dementia Major depressive disorder Parkinson disease Dementia, Alzheimer type

Delirium

A 63-year-old man is brought to the physician by his daughter because she is concerned about his memory loss during the past year. Yesterday he could not remember his 18-month-old granddaughter's name. Although he denies that there is any problem, she says he has been forgetful and becomes easily confused. There is no history of alcohol abuse. His temperature is 37°C (98.6°F), pulse is 77/min, respirations are 12/min, and blood pressure is 118/84 mm Hg. On mental status examination, his mood is normal. He is oriented to person and place but initially gives the wrong month, which he is able to correct. He recalls memories from his youth in great detail but only recalls one of three words after 5 minutes. He has difficulty recalling the names of common objects and does not remember the name of the current US president. Physical examination, laboratory studies, and thyroid function tests show no abnormalities. Bipolar disorder, depressed Alcohol withdrawal Dementia, Alzheimer type Apathetic hyperthyroidism Dementia, alcohol-related Generalized anxiety disorder

Dementia, Alzheimer type

A 63-year-old man is brought to the physician by his daughter because she is concerned about his memory loss during the past year.Yesterday he could not remember his 18-month-old granddaughter's name. Although he denies that there is any problem, she says hehas been forgetful and becomes easily confused. There is no history of alcohol abuse. His temperature is 37°C (98.6°F), pulse is77/min, respirations are 12/min, and blood pressure is 118/84 mm Hg. On mental status examination, his mood is normal. He isoriented to person and place but initially gives the wrong month, which he is able to correct. He recalls memories from his youth ingreat detail but only recalls one of three words after 5 minutes. He has difficulty recalling the names of common objects and does notremember the name of the current US president. Physical examination, laboratory studies, and thyroid function tests show noabnormalities. Bipolar disorder, depressed Dementia, alcohol-related Alcohol withdrawal Apathetic hyperthyroidism Generalized anxiety disorder Dementia, Alzheimer type

Dementia, Alzheimer type

A 1-year-old boy is brought to the physician by his mother because of a rash for the past 2 weeks. He is at the 10th percentile for length and weight. Physical examination shows a scaly, seborrheic eruption over the scalp, palms, back, diaper region, and soles of the feet. There is also generalized lymphadenopathy and hepatosplenomegaly. X-rays of the skull show a few osteolytic lesions. Electron microscopy of a biopsy specimen of a skin lesion shows a tennis racket-shaped bilamellar granule in the cytoplasm of cells. lmmunohistochemical studies of the abnormal cells show CD1 a antigen expression. The abnormal cells in this patient are most likely derived from which of the following? T lymphocytes Mast cells Dendritic cells Merkel cells Keratinocytes

Dendritic cells

Which of the following best explains why deoxygenated blood can carry more carbon dioxide for a given Pco2 than oxygenated blood? Deoxyhemoglobin has a lower pKa than oxyhemoglobin Deoxyhemoglobin has a lower capacity to form carbamino compounds than oxyhemoglobin Deoxyhemoglobin is a better buffer of hydrogen ions than oxyhemoglobin Oxyhemoglobin binds nitric oxide with a higher affinity than deoxyhemoglobin Deoxyhemoglobin does not bind to 2,3-bisphosphoglycerate as efficiently as oxyhemoglobin Oxygen and carbon dioxide compete for the same binding site in hemoglobin

Deoxyhemoglobin is a better buffer of hydrogen ions than oxyhemoglobin

A 35-year-old man has a 6-year history of weight loss, chronic abdominal pain, bloating, and frequent voluminous, greasy, malodorous stools. He had similar symptoms as an adolescent, but they resolved spontaneously within 2 years. His mother has a similar illness that is controlled by restricting wheat, barley, and rye in her diet. In addition to the gastrointestinal symptoms, the patient has a chronic pruritic rash shown in the photograph of his elbow. Which of the following is the most likely diagnosis of this patient's skin condition? Dermatitis herpetiformis Pemphigus vulgaris Erythema nodosum Bullous pemphigoid Epidermolysis bullosa

Dermatitis herpetiformis

A 12-year-old girl is brought to the physician by her father because of progressive fatigue and generalized weakness during the past 5 months. Physical examination shows an erythematous, maculopapular rash over both elbows and proximal muscle weakness. She has difficulty lifting her upper extremities, but she can move her fingers with precision. She has trouble standing from a seated position. Serum studies show an increased creatine kinase activity; a serum antinuclear antibody assay is positive. Which of the following is the most likely diagnosis? Dermatomyositis Juvenile rheumatoid arthritis Systemic lupus erythematosus Mucocutaneous lymph node (Kawasaki) syndrome Viral cardiomyopathy and failure Muscular dystrophy

Dermatomyositis

An 83-year-old woman with systemic arteriosclerosis undergoes repair of an 8-cm, infrarenal abdominal aortic aneurysm. A prosthetic endog raft is inserted in the femoral artery, threaded up the aorta, and then expanded when in place. The graft extends from just below the renal arteries to the bifurcation of the aorta. Because of the position of the graft, which of the following organs is most likely to lose its primary blood supply and subsequently must rely on collateral circulation? Stomach Spleen Pancreas Descending colon Adrenal gland

Descending colon

A 31-year-old woman is brought to the emergency department by her husband because of difficulty breathing and severe muscle weakness for 10 minutes. She has a 3-year history of myasthenia gravis treated with neostigmine. The husband reports that she doubled her dosage 2 days ago because she was feeling extraordinarily weak, but her weakness has increased since then. Which of the following events is the most likely cause of the increased muscle weakness in this patient? Autoimmune hyperactivation of nicotinic receptors Autoimmune inactivation of muscarinic receptors Excessive degradation of acetylcholine Desensitization of nicotinic receptors Hypersensitization of muscarinic receptors

Desensitization of nicotinic receptors

A 60-year-old man comes to the physician because of skin blistering for 1 week. Six months ago, he had blisters in his mouth that easily ruptured. Physical examination shows scattered blisters over the upper and lower extremities and trunk intermingled with crusted denuded areas. A biopsy specimen of an early lesion shows intraepidermal vesicle formation. Direct immunofluorescence microscopy of intact skin shows deposits of lgG on keratinocytes. Which of the following structures is most likely involved in the skin disorder in this patient? Adherens junctions (zonulae adherentes) Desmosomes (maculae adherentes) Tight junctions (zonulae occl udentes) Hemidesmosomes Gap junctions

Desmosomes (maculae adherentes)

A 28-year-old woman is brought to the hospital by her boyfriend because of severe lethargy for 2 hours. Her temperature is 37.1 C (98. 7°F), pulse is 1 1 0/min, respirations are 24/min, and blood pressure is 126/78 mm Hg. Physical examination shows obtundation. Her serum glucose concentration is 25 mg/dl. She revives immediately upon intravenous administration of 50% dextrose in water. She does not have type 1 or type 2 diabetes mellitus, and she takes no medications. Multiple nonfasting glucose concentration measurements are within the reference range, and results of a 75-g, 2 -hour glucose tolerance test show no abnormalities. Corresponding serum insulin concentrations are within the reference range. Six hours later, she is unresponsive. Her serum glucose concentration is 32 mg/dl. She is again resuscitated with 50% dextrose in water. The physician suspects surreptitious use of insulin. Which of the following is the most appropriate next step? Obtain a psychiatric consultation Search the patient's personal belongings for insulin and syringes Confront the patient about the physician's suspicion Measure hemoglobin A1c Determine the C-peptide concentration in the latest blood specimen drawn

Determine the C-peptide concentration in the latest blood specimen drawn

A 42-year-old man comes to the physician because of multiple lesions over his body. Physical examination shows flaccid bullous erosions involving the upper and lower extremities and torso. A biopsy specimen of one of the lesions shows extensive epidermal acantholysis resulting in the formation of an intraepidermal blister. An intact basal layer of keratinocytes adherent to the basement membrane is identified. Which of the following is most likely related to the pathogenesis of this disorder? Deposition of lgA- containing immune complexes within dermal papilla Infection with a herpesvirus Mutation of an anchoring filament protein Development of autoantibodies against desmosomal proteins Immediate (type I) hypersensitivity reacti on to a topicall y applied agent

Development of autoantibodies against desmosomal proteins

A 23-year-old woman comes to the physician because of a 3-week history of persistent cough that is worsened by the cold air in her drafty apartment. She does not smoke, takes no medications, and has no history of asthma or chronic pulmonary disease. Physical examination shows no abnormalities. Short-term cough suppressant treatment is initiated with a drug that does not cause constipation and has a low potential for substance abuse. This drug is most likely which of the following? Codeine Hydromorphone Tramadol Oxycodone Dextromethorphan

Dextromethorphan

After a brief patient history, it is revealed that diuretics are not the cause of polyuria in this patient. Upon this discovery, which of the following is the next most prevalent cause of polyuria? Polydipsia Diabetes mellitus Nephrogenic diabetes insipidus (NDI) Central diabetes insipidus (CDI)

Diabetes mellitus

A 45-year-old man with an ulcer in the anterior superior portion of the fundus of the stomach is brought to the emergency department because of acute pain in the left upper quadrant and the left shoulder for 4 hours. Physical examination shows a rigid abdomen. Perforation of the stomach wall by the ulcer is suspected. Irritation of which of the following structures by gastric contents best explains the left shoulder pain? Duodenum Pancreas Diaphragm Spleen Liver Gallbladder

Diaphragm

A 52-year-old man comes to the physician because of a 3-month history of epigastric abdominal pain; he also has had an unintentional 6.8-kg (15-lb) weight loss during this period. He has osteoarthritis treated with naproxen as needed. He immigrated to the USA from Japan 6 months ago. He eats mostly traditional Japanese food prepared by his wife. He has smoked 2 packs of cigarettes daily for 30 years and drinks three to four glasses of wine daily. He is 170 cm (5 ft 7 in) tall and now weighs 82 kg (180 lb); BMI is 28 kg/m 2. Physical examination shows epigastric tenderness. Upper gastrointestinal endoscopy shows a 4-cm ulcer in the stomach. Examination of a biopsy specimen of the lesion confirms adenocarcinoma. Which of the following is the strongest predisposing risk factor for this patient's condition? Naproxen use Diet Ethnicity Alcohol use Tobacco use

Diet

A 60-year-old man comes to the physician because of progressive weakness of his hands during the past 6 months. He works as a carpenter and has had difficulty handling his tools. He has smoked 1 pack of cigarettes daily for 45 years. He drinks two beers daily during the workweek and approximately six beers daily on weekends. Physical examination shows no lymphadenopathy. There is decreased strength (4/5) in the upper and lower extremities and atrophy of the muscles of the hands. He has diffuse hyperreflexia. Fasciculations are noticed on the muscles of the hands and upper extremities. Sensory examination is normal. Over the course of this disease, which of the following findings is most likely? Dementia Loss of facial sensation Loss of peripheral vibratory sensation Nystagmus Difficulty swallowing

Difficulty swallowing

A 51-year-old female presents with a 2-month history of a persistent cough and worsening dyspnea. She denies chest pain or other illnesses. Her blood pressure is 114/72 mm Hg, and pulse is 120/min. Chest auscultation reveals diffuse basilar crackles bilaterally, an S3 gallop, and no detectable murmur. ECG shows is consistent with tachycardia and nonspecific ST-T wave changes. Echocardiography show a dilated left ventricle and a diffusely hypokinetic heart. Which of the following represents the most likely diagnosis? Hypertrophic cardiomyopathy Dilated cardiomyopathy Acute pericarditis with effusion Silent subendocardial infarction Severe mitral valve prolapse

Dilated cardiomyopathy

A 4-month-old boy is diagnosed with a rare autosomal recessive skeletal dysplasia involving abnormal endochondral bone formation. Genetic analysis shows null mutations in a gene for a protein that controls the traffic of vesicles into the Golgi complex. Electron microscopy of this patient's cells will most likely show which of the following findings? Increased smooth endoplasmic reticulum Dilated rough endoplasmic reticulum Dilated rough endoplasmic reticulum Decreased smooth endoplasmic reticulum Small lysosomes Large lysosomes

Dilated rough endoplasmic reticulum

A 1045-g (2-lb 5-oz) male newborn delivered at 28 weeks' gestation develops rapid breathing, grunting, and subcostal retractions shortly after birth. Coarse rhonchi are heard on auscultation of the chest. A chest x-ray shows fine densities with a reticulogranular appearance bilaterally. Arterial blood gas analysis on room air shows decreased pH and Po 2 and increased Pco2. A deficiency of which of the following is the most likely cause of the pulmonary disorder in this patient? Surfactant protein D Phosphatidylserine Sphingomyelin Dipalmitoylphosphatidylcholine Diacylglycerol

Dipalmitoylphosphatidylcholine

One week after receiving 4 units of packed red blood cells for acute hemorrhage due to a gunshot wound, a 17-year-old male develops fatigue, jaundice. Laboratory results show the following:Hemoglobin 8.8 g/dL Hematocrit 28%, Serum total bilirubin 5 mg/dL Serum lactate dehydrogenase 120 U/L. Liver and kidney function tests are otherwise within normal limits. Which of the following is the most appropriate next step in diagnosis? Ultrasonography of the gallbladder Monospot test Serology for hepatitis B markers Direct and indirect antiglobulin (Coombs) tests Cytomegalovirus antibody titer

Direct and indirect antiglobulin (Coombs) tests

The sequence surrounding the first two exons of the human '3- globin gene is shown, with the exons given in bold capital letters. The translation start codon is underlined. A mutation from G-+A at position 355 (bold italics) is most likely to lead to '3-thalassemia by which of the following mechanisms? Disruption of polyadenylation of the mRNA Creation of a missense mutation that leads to the synthesis of a defective '3 -globin Disruption of normal splicing by creation of a new 3' splice site Inhibiti on of the replication of this gene

Disruption of normal splicing by creation of a new 3' splice site

A 28-year-old man is brought to the emergency department 30 minutes after the sudden onset of shortness of breath. He has a 3-year history of cocaine abuse. His temperature is 38.1°C (100.6°F), pulse is 100/min, and blood pressure is 150/45 mm Hg. Physical examination shows diminished pulses in the left upper extremity. Crackles are heard over all lung fields. A grade 2/6 diastolic murmur is heard best at the left sternal border. A chest x-ray shows a widened aortic arch. Which of the following is the most likely diagnosis? Atherosclerotic aneurysm Saccular aneurysm Pseudoaneurysm Mycotic aneurysm Dissecting aneurysm

Dissecting aneurysm

A 72-year-old man is brought to the emergency department because of a 1-week history of increasingly severe abdominal pain, diarrhea, and loss of appetite. He has had a fever for 2 days. His temperature is 38.3°C (101 °F), and pulse is 106/min. Abdominal examination shows distention and rebound tenderness localized to the left lower quadrant. His leukocyte count is 18,000/mm3 (80% segmented neutrophils, 10%, bands, and 10%, lymphocytes). An abdominal CT scan with contrast shows a 7-mm, thickened colonic wall, and pericolic fat inflammation; there is a collection of contrast in this region. Which of the following is the most likely diagnosis? Amebiasis Ulcerative colitis Diverticulitis Colon cancer Collagenous colitis

Diverticulitis

A 27-year-old man who is being treated for Hodgkin lymphoma receives a bone marrow transplant from his first cousin. Two weeks after the transplant, he develops a generalized erythematous, maculopapular rash, diarrhea, and elevated concentrations of liver enzymes and bilirubin. No evidence of infection or drug reaction is found. Which of the following mechanisms is the most likely cause of the clinical symptoms? Donor T lymphocytes reacting against host cells Donor macrophages secreting cytokines and affecting host cells Host macrophages secreting cytokines and affecting donor cells Host T lymphocytes reacting against donor cells Donor plasma cells elaborating antibodies against host cells Host plasma cells elaborating antibodies against donor cells

Donor T lymphocytes reacting against host cells

A 16-year-old girl comes to the physician because of a 2-year history of intermittent fainting that occurs while she is standing; these episodes have increased in frequency during the past 6 months. Her blood pressure is 110/80 mm Hg while supine and 60/40 mm Hg while standing. Neurologic examination shows no other abnormalities. Plasma studies show an undetectable norepinephrine concentration and a marked increase in dopamine concentration when rising from a supine position to standing. This patient most likely has a deficiency of which of the following? Dopamine beta hydroxylase Norepinephrine transporter Tyrosine hydroxylase Monoamine oxidase Amino acid decarboxylase

Dopamine beta hydroxylase

A 2-month-old female infant is admitted to the hospital because of a temperature of 38.9°C (102°F), vomiting, diarrhea, and dehydration. Examination of a stool suspension by negative-stain electron microscopy shows viral particles with a wheel-like shape. Which of the following best describes the physical and chemical properties of the virus involved? Double stranded RNA, segmented, no envelope, icosahedral

Double stranded RNA, segmented, no envelope, icosahedral

A 19-year-old woman comes to the physician for a follow-up examination. She has scarring acne vulgaris that has not been responsive to topical medication. A bacteriostatic drug that acts by inhibiting the binding of aminoacyl-tRNA to the 30S subunit of bacterial ribosomes is prescribed. This drug is most likely which of the following? Penicillin V potassium Doxycycline Erythromycin Rifampin Ciprofloxacin

Doxycycline

A 76-year-old man comes to the physician for a follow-up examination. He has hypertension treated with a beta-adrenergic antagonist. He lives on a farm in central California and says he has always distilled his own liquor. Before retiring 10 years ago, he worked in a hat factory and subsequently in a textile factory. He has smoked 2 packs of cigarettes daily for the past 55 years. He tells the physician that he has had several episodes of painful swelling of his right great toe. Physical examination shows several lesions consistent with gouty tophi over the elbows bilaterally. Laboratory studies show: The most likely cause of this patient's condition is which of the following? Drinking home-distilled liquor Working in a hat factory Cigarette smoking Farming in central California Working in a textile factory

Drinking home-distilled liquor

A 30-year-old woman with seasonal rhinitis has been treated with therapeutic doses of chlorpheniramine for 4 weeks. Which of the following adverse effects is most likely? Atrioventricular block Miosis Decreased myocardial contractility Tinnitus Seizures Intermittent claudication Chronic cough Biliary colic Increased blood glucose concentration Drowsiness

Drowsiness

An 82-year-old man is admitted to the hospital because nursing staff in his skilled nursing care facility report that he has appeared sad and depressed during the past 2 months. It is reported that he has a history of psychiatric illness, but details are not provided. He has been taking olanzapine, paroxetine, and haloperidol for 2 years. He does not appear to be in acute distress. He is 160 cm (5 ft 3 in) tall and weighs 48 kg (105 lb); BMI is 19 kg/m2. Physical examination shows 2+ cogwheel rigidity of the upper extremities. Neurologic examination shows psychomotor retardation. On mental status examination, he is alert and generally pleasant and cooperative. His affect has little intensity or range. He says he does not feel depressed or anxious. Laboratory findings are within the reference range. Which of the following is the most likely cause of this patient's current symptoms? Adjustment disorder Akathisia Dementia, Alzheimer type Drug-induced parkinsonism Major depressive disorder

Drug-induced parkinsonism

A 4-year-old boy is brought to the physician by his parentsbecause of a 4-month history of difficulty running andfrequent falls. His parents report that his calves have beengradually increasing in size during this period. Examinationshows diffusely enlarged muscles of the calves and lumbarlordosis. Sensation is intact. He has difficulty arising froma supine position. Which of the following is the most likelydiagnosis? Juvenile idiopathic arthritis Lumbosacral radiculopathy Duchenne muscular dystrophy Rhabdomyosarcoma Spina bifida

Duchenne muscular dystrophy

A 70-year-old man is admitted to the hospital for elective coronary artery bypass grafting. On the day of his operation, an asymptomatic carotid bruit is found. Which of the following is the most appropriate immediate next step in diagnosis? Cerebral angiography Duplex scan of the neck CT scan of the head PET scan of the brain MRI of the brain

Duplex scan of the neck

A 3-year-old girl is brought to the physician because of fever and rapid breathing for 1 day. She has had episodes of otitis media and sinusitis since the age of 1 year. Her temperature is 39°C (102°F), and respirations are 24/min. She is at the 10th percentile for height and weight. Physical examination shows tachypnea. Crackles and rhonchi are heard in all lung fields. Heart tones are heard best on the right side of the chest. A chest x-ray shows bilateral infiltrates, dextrocardia, and stomach bubble on the right. A deficiency of which of the following is the most likely cause of this patient's disorder? Wiskott- Aldrich syndrome protein (WASp) Fibrillin-1 Adenosine deaminase activity Dynein Cystic fibrosis transmembrane regulator Bruton tyrosine kinase activity

Dynein

A 35-year-old man comes to the physician for evaluation of recurrent episodes of sinusitis and bronchitis throughout his entire life. One of his sisters has a similar history of chronic upper respiratory problems. He is married but has no children. Cardiac examination shows point of maximum impulse at the fourth intercostal space within the midclavicular line on the right. The hepatic margin is palpable on the left. Endoscopy shows nasal polyps on the left middle and inferior turbinates. Examination of tissue obtained on biopsies of the polyp and adjacent mucosa shows thickened, ciliated, pseudostratified epithelium with small patches of squamous metaplasia and mild lymphoid infiltration. Which of the following structures is most likely to be absent on electron microscopic examination of the biopsy specimen of the epithelium? Microtubules Lysosomes Actin filaments Gap junctions Basal lamina Basal bodies Nuclear pores Desmosomes (maculae adherentes) Dynein arms

Dynein arms

A 5-year-old boy is brought to the physician by his parentsbecause of an 8-month history of difficulty walking. Hisparents say that he limps when he walks and has a waddlinggait; he also has difficulty standing. When getting up from asitting position, he uses his hands to walk up his thighs andpush his body into a standing position. His parents have notnoticed any weakness of his arms. His mother is an onlychild, but she has an uncle who became bedridden as a childand died of respiratory arrest. Physical examination showsprominent calf muscles. Muscle strength is 4/5 at both hipsbut normal elsewhere. This patient most likely has amutation in the gene coding for which of the followingproteins? Dystrophin Frataxin Myelin Actin Myosin

Dystrophin

A previous healthy 26-year-old woman comes to the physician because of flank pain, palpitations, sweating, and recurrent headaches during the past 2 weeks. Her blood pressure is 180/95 mm Hg. Physical examination shows no other abnormalities. A 24-hour urine specimen shows increased concentrations of metanephrine. A CT scan of the abdomen shows a retroperitoneal mass. The mass most likely originated in which of the following regions? E

E

A 50-year-old man has a 1-hour history of unremitting chest pressure and "gassiness." He has no history of cardiac problems but does have a history of peptic ulcer disease. Physical examination shows no abnormalities except for a blood pressure of 140/80 mm Hg. Which of the following is the most appropriate initial step in diagnosis? Test of the stool for occult blood Echocardiography ECG X-ray series of the upper gastrointestinal tract Endoscopy of the upper gastrointestinal tract

ECG

An 83-year-old man is brought to the emergency department after being found at home bedridden and confused. He takes no medications. Temperature is 35.6°C (96°F), pulse is 100/min, and blood pressure is 85/50 mm Hg. Blood pressure is unchanged after intravenous infusion of 1 liter of isotonic saline. A pulmonary artery catheter is inserted and the following findings are obtained: Cardiac output is high, pulmonary capillary wedge is pressure low, systemic vascular resistance is low. Which of the following is the most likely cause of the hypotension? Early septic shock Hypothyroidism Massive pulmonary embolism Gastrointestinal bleeding Silent myocardial infarction

Early septic shock

A 22-year-old man who is a professional cyclist undergoes extensive physiologic testing as part of his training regimen. His resting pulse is 33/min, and blood pressure is 110/62 mm Hg. Echocardiography shows dilated ventricles with normal function and a left ventricular ejection fraction of 75%. Which of the following best describes the findings in this patient? Congestive cardiomyopathy Hypertrophic cardiomyopathy Eccentric hypertrophy Diastolic dysfunction Increased myocardial stiffness

Eccentric hypertrophy

A 22-year-old primigravid woman at 34 weeks' gestation isbrought to the emergency department by ambulance after being found unconscious by her husband. Paramedics report that she was having tonic-clonic movements thathave now stopped. Her last visit to the physician was2 weeks ago. Pregnancy had been uncomplicated. Onarrival, she is awake, lethargic, and mildly confused. Shesays that she had a headache and did not feel well earlier inthe day. She has no history of serious illness. Her onlymedication is a prenatal vitamin. Her temperature is 37°C(98.6°F), pulse is 80/min, respirations are 18/min, andblood pressure is 170/110 mm Hg. Examination shows anontender, soft uterus consistent in size with a 34-weekgestation. Cranial nerves are intact. Motor function isnormal. Deep tendon reflexes are 3+. Which of thefollowing is the most likely diagnosis? Absence seizures Cerebral infarction Eclampsia Migraine Pheochromocytoma

Eclampsia

A 9-month-old boy is brought to the emergency department by his mother because of severe respiratory distress for 1 hour. His respirations are 80/min. He is at the 3rd percentile for length and weight. Physical examination shows cyanosis. Despite appropriate measures, the patient dies. Autopsy findings show a ventricular septal defect and persistent truncus arteriosus. Which of the following populations of cells was most likely absent during early fetal cardiac development in this patient? Splanchnopleuric mesoderm Mesothelial cells Mantle layer neuroblasts Ectodermal neural crest Extraembryonic mesoderm

Ectodermal neural crest

A 24-year-old woman is admitted to the hospital because of acute pain in the left lower quadrant of the abdomen. She has leukocytosis and an increased human chorionic gonadotropin concentration. A photograph of the surgically removed specimen is shown. Which of the following is the most likely diagnosis? Choriocarcinoma invading myometrium Endometriosis Papillary carcinoma of the ovary Ectopic pregnancy Hydatidiform mole

Ectopic pregnancy

A 32-year-old woman, gravida 2, para 1, at 7 weeks' gestation comes to the emergency department because of vaginal bleeding for 3 days and increasingly severe left abdominal pain for 18 hours. Abdominal examination shows severe direct and rebound tenderness with guarding in the left lower quadrant. The cervicalos is closed. Her serum -hCG concentration is 6000 mlU/mL. Ultrasonography shows an empty uterus. Which of the following is the most likely diagnosis? Placenta previa Incomplete abortion Abruptio placentae Vasa previa Hydatidiform mole Threatened abortion Preterm labor Ectopic pregnancy

Ectopic pregnancy

A 2-year-old girl is brought to the physician by her mother because of fever and a sore throat for 3 days. She has difficulty swallowing and has become irritable, listless, and uninterested in eating. She was recently adopted from a developing country, and her immunization history is unknown. Her temperature is 38.5°C (101 .3°F), pulse is 150/min, and respirations are 40/min. Physical examination shows a dull red throat. A gray exudate is present on the tonsils, pharynx, uvula, and tongue, and the exudate can be lifted with a tongue blade. There is marked cervical lymphadenopathy. The most likely causal organism produces a toxin that causes ADP-ribosylation of which of the following targets? Acetylcholi nesterase Guanylyl cyclase Adenylyl cyclase GTPase G.subunit Elongation factor 2 Ribosomal protein S 1

Elongation factor 2

A newborn undergoes surgical repair of esophageal atresia. Pathologic examination of resected tissue shows that the esophagus is fibrotic and has no lumen. Which of the following embryonic germ layers is the most likely origin of the cells that fill the lumen? Paraxial mesoderm Surface ectoderm Endoderm Intermediate mesoderm Neural crest

Endoderm

A previously healthy 48-year-old nulligravid woman comes to the physician because of excessive uterine bleeding for 3 months. She says the bleeding occurs both during her menses and at irregular intervals. Menses had occurred at regular 28- to 38-day intervals. She is 165 cm (5 ft 5 in) tall and weighs 72 kg (160 lb); BMI is 27 kg/m2. During pelvic examination, the adnexae are non palpable. Endometrial curettage shows abundant tissue. Which of the following is the most likely cause of this patient's symptoms? Endometrial hyperplasia Endometriosis Endometritis Endometrial metaplasia Adenomyosis

Endometrial hyperplasia

A 15-year-old girl is brought to the physician by her mother because of a 1-year history of monthly cramps that begin 2 days before menses and last 3 days. The cramps have increased in severity during the past 6 months. She is unable to practice with her volleyball team because of the pain and typically misses 2 days of school monthly. Menarche was at the age of 12 years. Menses occur at regular 30-day intervals and last 4 days with normal flow. She has no history of serious illness and takes no medications. She has never been sexually active. Physical examination shows no abnormalities. Which of the following is the most likely cause of this patient's pain? Bladder wall permeability Endometrial prostaglandin production Intestinal hypermobility Degenerating myoma Escherichia coli infection

Endometrial prostaglandin production

A 15-year-old girl is brought to the physician by hermother because of a 1-year history of monthly cramps thatbegin 2 days before menses and last 3 days. The crampshave increased in severity during the past 6 months. She isunable to practice with her volleyball team because of thepain and typically misses 2 days of school monthly.Menarche was at the age of 12 years. Menses occur atregular 30-day intervals and last 4 days with normal flow.She has no history of serious illness and takes nomedications. She has never been sexually active. Physicalexamination shows no abnormalities. Which of thefollowing is the most likely cause of this patient's pain? Endometrial prostaglandin production Bladder wall permeability Intestinal hypermobility Degenerating myoma Escherichia coli infection

Endometrial prostaglandin production

A 27-year-old nulligravid woman has had severe pain with menses that has caused her to miss at least 2 days of work during each menstrual cycle for the past year. She has occasional pain during sexual intercourse. She weighs 50 kg (110 lb) and is 160 cm (5 ft 3 in) tall; BMI is 20 kg/m2. Pelvic examination shows a normal-appearing vulva and vagina. The cervix is pink with minimal endocervical gland eversion. The uterus is normal in size. The left ovary is 2 × 3 cm; the right is 4 × 6 cm. Which of the following is the most likely cause of her condition? Pelvic congestion syndrome Polycystic ovarian syndrome Premenstrual syndrome Chronic appendicitis Endometriosis

Endometriosis

A 27-year-old nulligravid woman has had severe pain withmenses that has caused her to miss at least 2 days of workduring each menstrual cycle for the past year. She hasoccasional pain during sexual intercourse. She weighs 50 kg(110 lb) and is 160 cm (5 ft 3 in) tall; BMI is 20 kg/m2.Pelvic examination shows a normal-appearing vulva andvagina. The cervix is pink with minimal endocervical glandeversion. The uterus is normal in size. The left ovary is2 × 3 cm; the right is 4 × 6 cm. Which of the following is themost likely cause of her condition? Pelvic congestion syndrome Premenstrual syndrome Endometriosis Chronic appendicitis Polycystic ovarian syndrome

Endometriosis

A 50-year-old woman with chronic obstructive pulmonary disease comes to the physician because of a 3-month history of progressive shortness of breath. Physical examination shows jugular venous distention. A loud pulmonary component of S2 is heard on auscultation. Pulmonary function tests show an FEV 1: FVC ratio of 20% and a severely decreased diffusing capacity for carbon monoxide. Based on these findings, which of the following is most likely decreased in this patient's pulmonary vascular smooth muscle? Cytosolic phospholipase A2 activity Endothelial nitric oxide synthase production Voltage-gated calcium channel a-subunit production Endothelin expression Adventitial collagen matrix deposition

Endothelial nitric oxide synthase production

A 50-year-old woman comes to the physician because of progressive shortness of breath during the past 2 years. Her respirations are 20/min. Physical examination shows cyanosis and ankle edema. Her pulmonary artery pressure, pulmonary vascular resistance, and right atrial pressure are increased; her pulmonary capillary wedge pressure is 9.3 mm Hg (N=8- 16). Her ventilation improves when inhaled nitric oxide is administered. Treatment with oral bosentan is begun for long-term therapy. Which of the following mediators of pulmonary resistance in this patient will most likely be antagonized by this drug? Endothelin C-reactive protein Prostacyclin (PG12) Adenosine Calcitonin gene-related peptide

Endothelin

A previously healthy 35-year-old woman comes to the physician because of a 3-month history of progressive shortness of breath with exertion. Her respirations are 26/min. Physical examination shows jugular venous distention with a prominent a wave. The lungs are clear to auscultation. Cardiac examination shows a loud pulmonic component of S2 and a right-sided S4 gallop. This patient's condition most likely involves an increased pulmonary expression of which of the following? Thrombomodulin Endothelin 1 Nitric oxide Prostacycl in ( PG I synthase) Voltage-gated potassium channels

Endothelin 1

A male newborn at 34 weeks' gestation is delivered vaginally. Physical examination shows no abnormalities except for a 1-cm, raised, red area on the anterior section of the neck. His mother is concerned, and the physician explains that the lesion is a hemangioma, a benign growth. This neoplasm most likely originated in which of the following tissues? Neural crest cells Endothelium Yolk sac Notochord Ectoderm

Endothelium

A female newborn is delivered at term to a 35-year-old primigravid woman. Pregnancy was complicated by untreated maternal Graves disease. Her respirations are 66/min. Physical examination shows stridor, nasal flaring, intercostal retractions, and an asymmetric neck mass. Which of the following is the most likely cause of the newborn's stridor? Thyroglossal duct cyst Vascular ring Cystic hygroma Enlarged thyroid gland Tracheomalacia

Enlarged thyroid gland

A 40-year-old woman comes to the emergency department because of a 1-week history of abdominal pain and watery, bloody diarrhea. She returned from a 2-week vacation in Kenya 1 month ago. Her vital signs are within normal limits. Abdominal examination shows distention and diffuse tenderness. Microscopic examination of a stool specimen shows 12-μm trophozoites displaying erythrophagocytosis. Which of the following is the most likely causal organism? Taenia saginata Leishmania donovani Entamoeba histolytica Toxoplasma gondii Trypanosoma cruzi

Entamoeba histolytica

A 45-year-old man with poorly controlled type 2 diabetes mellitus comes to the physician because of a 1-month history of low-grade fever. He is receiving hemodialysis for end-stage renal disease. His temperature is 37°C (98.6° F), pulse is 72/min, and blood pressure is 144/92 mm Hg. Physical examination shows a subclavian catheter inserted below the right clavicle. The lungs are clear to percussion and auscultation. Cardiac examination shows no murmurs. Two of two blood cultures grow nonhemolytic, catalase-negative, gram-positive cocci in pairs and chains. Which of the following is the most likely causal organism? Listeria monocytogenes Staphylococcus aureus Enterococcus faecalis Streptococcus pneumoniae Staphylococcus epidermidis

Enterococcus faecalis

An 8-month-old girl is brought to the physician by her mother because of a 3-week history of poor feeding, chronic diarrhea, and pale, foul-smelling stools. The mother says that the symptoms began with colicky abdominal pain following the introduction of solid food to the infant's diet. There is a family history of gluten sensitivity. She is at the 60th percentile for length and 25th percentile for weight. Physical examination shows no other abnormalities. Dysfunction of which of the following cells is the most likely cause of this patient's symptoms? Paneth cells Parietal cells Goblet cells Enterocytes Serosal cells

Enterocytes

A previously healthy 6-year-old girl is brought to the emergency department because of severe abdominal pain, nausea, and vomiting that began 4 hours after attending a friend's birthday party. Cake and other pastries were served at the party. She appears acutely ill and mildly dehydrated. Her temperature is 37°C {98.6°F), pulse is 104/min, respirations are 20/min, and blood pressure is 96/60 mm Hg. Examination shows no abnormalities of the skin. Bowel sounds are hyperactive. Which of the following is the most likely cause of these findings? Enterotoxin ingestion Viremia Villous atrophy Lead toxicity Intestinal mucosa inflammation Bacteremia

Enterotoxin ingestion

A laboratory technician wipes down the workbench with alcohol after an experiment. This treatment will successfully inactivate viruses with which of the following characteristics? Icosahedral capsid DNA genome Helical capsid RNA genome Enveloped virion Naked virion

Enveloped virion

A 20-year-old man is brought to the emergency department because of a 6-hour history of difficulty breathing and vomiting. He has a 10-year history of type 1 diabetes mellitus currently treated with insulin twice daily. His pulse is 90/min, respirations are 30/min and deep, and blood pressure is 90/60 mm Hg. Physical examination shows dehydration. Laboratory studies show: Serum Na+ 130 mEq/L K+ 6.5 mEq/L HC03- 5 mEq/L Glucose 500 mg/dl Arteri al blood gas analysis on room air: pH 7.2 PC02 25 mm Hg Which of the following compounds most likely stimulated hormone-sensitive lipase in adipocytes that resulted in the accumulation of metabolites causing the acidosis? Epinephrine Insulin Thyroid-stimulating hormone Thyroxine (T .J) Testosterone

Epinephrine

A homeless 38-year-old man has a 3-month history of decreased appetite, generalized wasting, a low-grade fever, and worsening cough productive of bloody, mucopurlent sputum. He has a 3-year history of alcohol abuse. X-ray films of the chest show consolidation and cavitary lesions in the apices of the lungs. Which of the following is the most likely inflammatory and/or immune response in the lung? Epitheliod cells, multinucleated giant cells, caseation necrosis Fibrobasts, collagen deposition, replacement of intrinsic tissue Neutrophils, vascular congestion, exudate Endothelial cell proliferation, vascular budding, fibroblasts Macrophages, lymphocytes, hypergammaglobulinemia

Epitheliod cells, multinucleated giant cells, caseation necrosis

A 19-year-old man has had fever, headache, sore throat,and swelling of the cervical lymph nodes for 5 days. Histemperature is 40°C (104°F), pulse is 120/min, respirationsare 20/min, and blood pressure is 125/85 mm Hg. Thepharynx is red and swollen with enlarged tonsils andexudate. There is tender cervical adenopathy and palpablelymph nodes in the axillary and inguinal areas. The spleentip is palpable. Leukocyte count is 14,000/mm3 (25%segmented neutrophils, 60% atypical lymphocytes, and15% monocytes). A throat culture is obtained. Which ofthe following is the most likely pathogen? Adenovirus Toxoplasma gondii Epstein-Barr virus Group A streptococcus Cytomegalovirus

Epstein-Barr virus

A 70-year-old man from China has a poorly differentiated monoclonal carcinoma of the nasopharynx. DNA probes of neoplastic cells are most likely to detect the genome of which of the following viruses? HTLV-1 Herpesvirus 8 Hepatitis B virus Cytomegalovirus Epstein-Barr virus

Epstein-Barr virus

A 66-year-old man dies 7 days after being admitted to the hospital for treatment of a myocardial infarction. The gross appearance of the heart at autopsy is shown. Which of the following findings is most likely on histologic examination of tissue around the lesion shown? Extensive fibrous connective tissue Contraction band necrosis of myocardial fibers Lymphocytes, plasma cells, and giant cells Erythrocytes, cellular debris, macrophages, and early granulation Coagulation necrosis of myocardial fibers with no inflammatory cell infiltrate

Erythrocytes, cellular debris, macrophages, and early granulation

A 2-year-old boy is brought to the physician because of a 2-day history of fever, sore throat, and rash. His temperature is 38.7°C(101 . 7° F). There is a widespread, red, sandpaper-like rash over the extremities. A purulent exudate is seen over the tonsils. He receives the diagnosis of Streptococcus pyogenes (group A) infection. Which of the following streptococcal components is the most likely cause of the skin findings in this patient? Streptolysin S Capsular polysaccharide Erythrogenic toxin Hyaluronidase Hemolysin

Erythrogenic toxin

A 50-year-old man with a long-standing history of type 2 diabetes mellitus comes to the physician because of several fainting episodes and fatigue during the past month. He appears chronically ill. His temperature is 3 7°C (98.6°F), pulse is 95/min, respirations are 17/min, and blood pressure is 155/95 mm Hg. Physical examination shows pallor. Laboratory studies show: Hemoglobin 8 g/dL (N: 13.5 - 17.5 g/dL) Hematocrit 24% (N: 40 to 54%) Mean corpuscular hemoglobin 30 pg/cell (N: 27 - 31 pg/cell) Mean corpuscular hemoglobin concentration 33% Hb/cell (N: Mean corpuscular volume 85 fL (N: 80 - 100 fL) Leukocyte count 8000/mm3 Platelet count 200,000/mm3 (N: 150,000 - 400,000/mL) BUN: 60 mg/dL Serum Creatinine: 6 mg/dL Total bilirubin: 0.9 mg/dL (N: Which of the following is the most appropriate pharmacotherapy for this patient? Oprelvekin Deferoxamine Erythropoietin Filgrastim Vitamin B12 (cyanocobalamin) Cyclosporine Folic acid

Erythropoietin

A 27-year-old nulligravid woman comes to the physician because of a 24-hour history of pain with urination and increased urinary frequency. She has no history of serious illness and takes no medications. She is sexually active with one partner and takes an oral contraceptive. She does not use condoms. Her temperature is 37°C (98.6° F). Physical examination shows no abnormalities. Pelvic examination shows suprapubic tenderness. Which of the following is the most likely cause of these findings? Neisseria gonorrhoeae Treponema pallidum Escherichia coli Human papillomavirus Candida albicans

Escherichia coli

A 45-year-old man has fever, chills, dysuria, and a tender, enlarged prostate. Which of the following is the most likely causal organism? Adenovirus Ureaplasma urealyticum Chlamydia trachomatis Escherichia coli Pseudomonas aeruginosa

Escherichia coli

A 65-year-old man comes to the physician because of a 6-week history of fatigue and difficulty swallowing; he alsohas had a 6.8-kg (15-lb) weight loss during this period. Tenyears ago, he underwent operative resection of squamouscell carcinoma of the floor of the mouth. He takes nomedications. He has smoked 2 packs of cigarettes daily for40 years and drinks 60 oz of alcohol weekly. Physicalexamination shows a nontender abdomen with bowelsounds present. The physician suspects a second primarycancer. This cancer is most likely located at which of thefollowing sites? Stomach Lymph nodes Liver Esophagus Brain

Esophagus

A 30-year-old nulligravid woman comes to the physician because she has been unable to conceive for 2 years. She has had a 36-kg(80-lb) weight gain during this period. Her last menstrual period was 4 months ago. Menses had occurred at regular 28-day intervals until the age of 25 years. They now occur every 90 to 120 days. She is 168 cm (5 ft 6 in) tall and now weighs 98 kg (215 lb); BMI is 35 kg/m2. Her blood pressure is 130/90 mm Hg. Physical and pelvic examinations show no other abnormalities. A pregnancy test result is negative. Serum studies show: Prolactin 1 5 ng/ml Thyroid-stimulating hormone 2 μU/ml Follicle-stimulating hormone 10 mlu/ml Luteinizing hormone 28 mlu/ml The patient has withdrawal bleeding after a progestin challenge test. The most likely cause of this patient's oligomenorrhea is an increased production of which of the following hormones? Androgen in the adrenal glands Estrogen in the adipose tissue Gonadotropin in the pituitary gland Progesterone in the ovaries Gonadotropin releasing hormone in the hypothalamus

Estrogen in the adipose tissue

A 45-year-old woman comes to the physician because she recently was diagnosed with cholelithiasis. She asks the physician why her abdominal pain is intermittent and not constant. The physician explains that pain is produced when the gallbladder contracts against the gallstone that is obstructing the cystic duct. Secretion of hormones from which of the following cells is the most likely cause of this contraction? Chief cells of the gastric mucosa Centroacinar cells of the exocrine pancreas Gastric parietal cells Vasoactive intestinal polypeptide-producing cells of the jejunal mucosa Eteroendocrine cells of the small intestine

Eteroendocrine cells of the small intestine

A 28-year-old African American man comes to the physician because of a 3 -month history of mild fatigue and weakness; he has had a 4.5-kg (10- lb) weight loss during this period. He also has a 6-month history of dry cough associated with chest pain and shortness of breath. The patient is married with two children, ages 2 and 3 years. He says that no one else at home has similar symptoms. He is a carpenter. He has several pets, including a dog, two cats, and a bird. He spent a week in Mexico approximately 1 year ago. He has smoked one-half pack of cigarettes daily for 1 0 years. He is 178 cm (5 ft 1 0 in) tall and weighs 70 kg (154 lb); BMI is 22 kg/m2• Mild wheezes are heard. Physical examination shows no other abnormalities. A chest x ray shows bilateral hilar adenopathy and right paratracheal node enlargement. Biopsy specimens obtained via fiberoptic bronchoscopy show noncaseating granulomas. Test results of the biopsy specimen for acid-fast bacilli and fungi are negative. Which of the following is the strongest predisposing risk factor for this patient's condition? Gender Smoking history Employment Ethnicity Living with pets Recent travel to Mexico

Ethnicity

A 25-year-old man develops a temperature of 38.6°C (101.5° F) after running his first 10-kilometer (6.2-mile) race. This man's temperature is most likely to return to normal because of which of the following mechanisms? Increased minute ventilation Increased muscle tone Peripheral arteriolar vasoconstriction Central arteriolar vasodilation Peripheral venous vasoconstriction Evaporation of sweat

Evaporation of sweat

X-ray of a newborn shows ribs associated with the seventh cervical vertebra. This apparent transformation of the seventh cervical segment to a thoracic identity may have resulted from which of the following alterations in HOX gene expression at the seventh cervical level? Overexpression of a HOX gene normally expressed at C-7 Underexpression of a HOX gene normally expressed at C-7 Expression of a hox gene normally expressed only caudal to C-7 Expression of a HOX gene normally expressed only cranial to C-7 Lack of expression of a HOX gene normally expressed at C-7

Expression of a hox gene normally expressed only caudal to C-7

A 72-year-old woman comes to the physician because of a 6-month history of increased bruising on her forearms. She appears alert and well nourished. Physical examination shows extensive wrinkling, scaly erythematous patches on the face, and irregularly shaped brown macules on the face and forearms. There are ecchymoses in various stages of healing on both forearms; the ecchymoses are more numerous on the right side. Laboratory studies, including a complete blood count and coagulation studies, are within the reference ranges. She has noticed no bleeding from her gums after brushing her teeth . Which of the following is the most likely cause of the ecchymoses in this patient? Ultraviolet destruction of Langerhans cells in the epidermi Exocytosis of lymphocytes Extensive solar elastosis Impaired platelet function

Extensive solar elastosis

A previously healthy 26-year-old woman comes to the physician because of a 1-day history of severe rectal pain with no bleeding. She takes no medications and does not smoke cigarettes or drink alcohol. She has been sexually active, and she and her partner use condoms inconsistently. A photograph of the rectal area is shown. Which of the following is the most likely diagnosis? Anal carcinoma Anal fissure External hemerrhoid Human papill omavirus infection Skin tag

External hemerrhoid

A 60-year-old woman with a 3-year history of hyperlipidemia comes to the physician for a follow-up examination. A low-cholesterol diet and exercise program were found to be ineffective in treating her disease 1 year after diagnosis. Treatment with lovastatin was initiated then, and the dose has been slowly increased to control her serum LDL-cholesterol concentration. She is unable to tolerate greater than 20 mg daily of this drug because of its adverse effects. Physical examination shows no abnormalities. Serum studies show: Cholesterol, total 240 mg/dL H DL-cholesterol 35 mg/dL LDL-cholesterol 173 mg/dL Triglycerides 160 mg/dL An additional cholesterol-decreasing drug is added to her regimen that inhibits the transport of cholesterol through the intestinal wall. This drug is most likely which of the following? Ezetimibe Cholestyramine Gemfibrozil Simvastatin Niacin

Ezetimibe

During the following experiment, limb bud development was monitored overtime to characterize limb patterning. Which of the following was most likely to identify the apical ectodermal ridge (AER)? Hoxb2-specific antibody FGF8-specific antibody Tbx3-specific antibody Retinoic acid-specific stain FGF10-specific antibody

FGF8-specific antibody

Which of the following regulates bone growth by limiting the formation of bone from cartilage (ossification), particularly in long bones? FGFR3 PKD2 COL4A4 FGFR2 ANK1

FGFR3

A 6-month-old girl is brought to the physician by her mother 3 days after she noticed that her daughter's smile seemed lopsided. She is at the 50th percentile for length and weight. Her vital signs are within normal limits. Physical examination shows left-sided facial weakness and decreased hearing in the left ear. A high-resolution CT scan of the head shows stenosis of the internal acoustic meatus on the left. This patient most likely has a developmental anomaly involving which of the following cranial nerves? Oculomotor Facial Abducens Trigeminal Trochlear

Facial

A 3-year-old boy and his 5-year-old brother bruise easily and have recurrent hemarthrosis. Both parents are healthy, but the mother has two younger brothers with the same symptoms and a maternal uncle who died at 8 years of age after mild head trauma. Partial thromboplastin time is prolonged. The most likely defect involves which of the following? Thrombin Factor XIII (transglutaminase) Factor VIII (antihemophilic factor) Factor V (proaccelerin) von Willebrand factor

Factor VIII (antihemophilic factor)

A 6-month-old boy is brought to the physician because of left knee swelling for 24 hours. Three months ago, he had three large hematomas on his forehead that resolved without treatment (??). His two sisters and his mother have no history of similar symptoms. His mother's maternal uncle and her brother died (x-linked recessive) before the age of 30 years of massive cerebral hemorrhages. Physical examination shows deep ecchymosis over the buttocks and severe swelling of the left knee. The most likely explanation for these findings is a deficiency of which of the following? Factor V (proaccelerin): Factor V Leiden (FVL) results from a point mutation in the F5 gene, which encodes the factor V protein in the coagulation cascade. FVL renders factor V (both the activated and inactive forms) insensitive to the actions of activated protein C (aPC), a natural anticoagulant. As a result, individuals who carry the FVL variant are at increased risk of venous thromboembolism (VTE). However, FVL is extremely common in the population, and many individuals with the variant will never have a VTE. von Willebrand factor Factor VIII (antihemophilic factor) Factor XIII (transglutaminase) Factor I (fibrinogen): An abnormality of fibrinogen may be suspected if there is prolongation of the prothrombin time (PT), activated partial thromboplastin time (aPTT), and/or thrombin time (TT). In congenital afibrinogenemia (absence of detectable fibrinogen), bleeding often occurs in the neonatal period. Umbilical cord bleeding, which can be fatal, is the initial presentation in approximately 60 to 85 percent of cases. Other individuals with afibrinogenemia may have a later age of onset with bleeding in the skin, gastrointestinal tract, urinary tract, or central nervous system. Joint bleeding is relatively rare.

Factor VIII (antihemophilic factor)

A 60-year-old man comes to the physician because of weakness and lightheadedness for 3 months. His diet consists of a healthy balance of fruits, vegetables, whole grains, and lean chicken and fish. He has a 10-year history of chronic autoimmune (Hashimoto) thyroiditis treated with thyroxine. Physical examination shows pallor. Serum studies show increased concentrations of methylmalonic acid and total homocysteine. A peripheral blood smear shows megaloblastic anemia. Which of the following is the most likely cause of the findings in this patient? Helicobacter pylori gastritis Folic acid deficiency Failure of intrinsic factor production Methylmalonyl-CoA mutase deficiency Cystathionine 13-synthase deficiency

Failure of intrinsic factor production

A previously healthy 48-year-old man comes to the physician because of a 2-month history of increasing abdominal girth and inability to achieve an erection. He has smoked 1 pack of cigarettes daily for 20 years; he also has drunk 1 pint of liquor daily during this period. His temperature is 37°C (98.6° F), pulse is 98/min, respirations are 14/min, and blood pressure is 120/76 mm Hg. Physical examination shows scleral icterus and spider angiomata. The lungs are clear to auscultation. Cardiac examination shows no abnormalities. There is gynecomastia. Abdominal examination shows ascites and a prominent umbilical venous pattern. His testes are small. Which of the following is the most likely cause of this patient's gynecomastia? Inadequate gonadotropin secretion by the pituitary gland Excessive testosterone degradation by the liver Excessive estrogen production by the adrenal glands Failure of the liver to degrade estrogen Failure of the liver to conjugate testosterone to its carrier molecule Inadequate testosterone production by the adrenal glands

Failure of the liver to degrade estrogen

An otherwise asymptomatic 52-year-old woman comes to the physician because of hot flashes. Her menses have been irregular for the past 6 months. Which of the following is the most likely physiologic cause of her symptoms? Excessive producti on of adrenal androgens Excessive conversion of androstenedione to estrone Failure of the ovaries to secret 17 beta estradiol Decreased hypothalamic secretion of gonadotropin-releasing hormone Failure of the anterior pituitary to secrete gonadotropins

Failure of the ovaries to secret 17 beta estradiol

Adolescent male presents with breathing difficulties and poor feeding. He coughs, chokes, and spits up milk soon after initiating to suckle. Physical examination and radiographs reveal the presence of the most common type of tracheoesophageal fistula. This defect most likely resulted from which of the following? patent thyroglossal duct failure of the buccopharyngeal membrane to rupture incomplete formation of the septum secundum incomplete recanalization of the larynx Failure of the tracheoesophageal ridges to fuse

Failure of the tracheoesophageal ridges to fuse

A previously healthy 35-year-old man has become increasingly depressed, impulsive, and difficult over the past year. He grimaces intermittently and has rigid, jerking, purposeless movements of the fingers. Which of the following historical factors is most relevant in establishing a diagnosis? Family history with a similar illness Exposure to environmental toxins Travel to a foreign country Tick bite Dietary deficiency Pet with an unexplained illness

Family history with a similar illness

Influenza virus strains are characterized by differing sensitivity to growth inhibition by the weak base ammonium chloride (AC). Influenza A San Francisco (SF) produces virus yields in AC-treated cells that approximate those in untreated cells, whereas influenza A New York (NY) grows poorly in AC-treated cells. To identify viral genes that segregate with differences in the sensitivity of these strains to AC, reassortant viruses are isolated by coinfecting cells with both virus strains. The ratios of the growth of the reassortant viruses in the presence and absence of AC are shown in the table. Fasting insulin: increased testosterone: increased LH: increased Fasting insulin: testosterone: LH

Fasting insulin: increased testosterone: increased LH: increased

Which of the following laboratory tests is typically used in the diagnosis of diabetes mellitus? HbA1c Fasting plasma glucose (FPG) Urine glucose dipstick test Oral glucose tolerance test

Fasting plasma glucose (FPG)

A 3-year-old boy is brought to the physician by his parents because of a 1-week history of nonproductive cough, wheezing, and nausea. Coarse bilateral crackles are heard on auscultation of the chest. Physical examination shows no other abnormalities. Stool culture shows a 1.6-cm roundworm larva. A chest x-ray shows bilateral infiltrates. This disease was most likely caused by ingestion of which of the following? Pork products Improperly canned beans Raw shrimp Feces contaminated soil Undercooked meat

Feces contaminated soil

An 82-year-old woman comes to the physician because of constant severe lower abdominal pain and fever for 24 hours. Laparoscopic examination shows severe diverticulosis and perforated diverticulitis. In spite of appropriate therapy, she dies 2 days later. Her liver at autopsy is shown. Which of the following is the primary component of the material shown on the hepatic surface? Collagen, type I Proteoglycans Fibrin Fibronectin Collagen, type Ill

Fibrin

A 44-year-old man with a 1-year history of angina pectoris comes to the emergency department because of increasingly severe chest pain during the past 2 days. He has had five previous similar episodes, which required treatment with increasing doses of nitroglycerin to resolve. His temperature is 37°C (98.6°F), pulse is 105/min , respirations are 16/min, and blood pressure is 150/90 mm Hg. Cardiac examination shows an S4. An ECG shows ST-segment depression in the precordial leads. In addition to aspirin, heparin, and nitroglycerin, he is given a dose of a monoclonal antibody against the platelet lib/Illa receptor. This antibody will most likely prevent binding of which of the following substances to platelets? Serotonin Thromboxane A2 Adenosine ADP Fibrinogen Thrombin

Fibrinogen

A 62-year-old man develops a pericardial friction rub 3 days after an acute myocardial infarction. Which of the following is the most likely cause of the friction rub? Left ventricular failure MitraI valve prolapse Cardiac tamponade Fibrinous pericarditis Purulent pericardial effusion

Fibrinous pericarditis

A previously healthy 20-year-old woman has had a palpable lump in her right breast for the past 4 months. She has not had pain, swelling, or a nipple discharge. The lump is smooth, firm, round, mobile, nontender, and well delineated from underlying tissue. There are no changes in the skin around the lesion. Which of the following is the most likely diagnosis? Cancer Mastitis Fibroadenoma Gynecomastia Cysts

Fibroadenoma

A 41-year-old woman is evaluated because of increasingly severe headaches for 6 weeks. Her blood pressure is 160/100 mm Hg while standing and supine. A bruit is heard over the left costovertebral angle. Urinalysis shows no abnormalities. An angiogram of the left renal artery shows alternating areas of stenosis and aneurysmal dilatation ("string of beads" sign). Which of the following conditions of the renal artery is the most likely diagnosis? Periarterial fibroplasia Hyaline arteriolosclerosis Intimal fibroplasia Fibromuscular dysplasia Perimedial hyperplasia

Fibromuscular dysplasia

A 21-year-old woman comes to the physician because of a 2-day history of urinary frequency and pain with urination. Her temperature is 39°C (102.2° F), pulse is 125/min, and blood pressure is 96/60 mm Hg. Physical examination shows right flank and suprapubic tenderness. Urine culture grows gram-negative bacteria. Which of the following virulence factors of the causal organism increases its adherence to the bladder epithelium? Endotoxin K antigens Fimbriae Flagella Hemolysin

Fimbriae

A 4-day-old male newborn who was born at home is brought to the emergency department because of respiratory distress and cyanosis. The mother reports that she found him in his crib not breathing. He began to breathe again after she picked him up. Examination shows a narrow thorax. His ears have peri auricular skin tags. He also has micrognathia, glossoptosis, a mandibular cleft, and a short palate. Tracheostomy relieves his respiratory distress. Defects such as these are consistent with altered development of which of the following pharyngeal arches? First Fourth Third Sixth Second

First

A 15-year-old girl is brought to the physician by her mother because of a 1-day history of redness and painful skin following sunbathing. She reports that she used sunblock every few hours. She takes no medications. Physical examination shows severe erythema of the back and extremities. There are no blisters. Which of the following best describes her condition? Third-degree burn Rhus dermatitis First degree burn Second-degree burn Allergic reaction to the sun

First degree burn

A 54-year-old man who lives in New York City develops severe pain and swelling in his lower left leg upon returning from a business trip in London. Ultrasound reveals a thrombosis in his posterior tibal vein. He is immediately administered intravenous heparin and admitted to the hospital. Twelve hours later, he develops left-sided weakness and facial droop. CT scan of the head reveals a right middle cerebral artery stroke. The patient if examined further. Which of the following physical examination findings is most likely to be present in this patient? Fixed, wide splitting of S2 that does not change with inspiration Systolic hypertension in the upper extremities Holosystolic or blowing high pitched systolic murmur Systolic murmur that is best heard in the interscapular region Fixed, wide splitting of S2 that changes with inspiration

Fixed, wide splitting of S2 that does not change with inspiration

A 19-year-old woman comes to the physician because of a 2-day history of pain in her left index finger. She injured it during a softball game when catching a ball. Physical examination shows erythema of the left index finger. The patient is unable to flex the distal phalanx when the proximal interphalangeal joint and metacarpophalangeal joints are restrained. Laboratory studies and x-rays of the left hand show no abnormalities. Which of the following structures is most likely injured in this patient? Flexor digitorum profundus tendon Flexor digitorum superficialis tendon Ulnar nerve Median nerve Extensor digitorum indicis Extensor digitorum tendon

Flexor digitorum profundus tendon

An asymptomatic 24-year-old primigravid woman at 36 weeks' gestation comes for a routine prenatal visit. A grade 2/6, systolic ejection murmur is heard at the upper left sternal border. The S2 varies with inspiration, and the pulmonic component is soft; diastole is clear. Which of the following is the most likely diagnosis? Patent ductus arteriosus Flow murmur Atrial septal defect Anomalous pulmonary venous return Pulmonary valve stenosis

Flow murmur

A 3-year-old girl is brought to the emergency department 2 days after the onset of fever, profuse watery diarrhea, and progressive lethargy. On arrival, she appears ill and is poorly responsive. Her temperature is 39.4°C (103°F), pulse is 180/min, respirations are 40/min, and blood pressure is 80/45 mm Hg. Examination shows poor skin turgor and cool extremities. Mucous membranes are dry. Capillary refill time is delayed. Which of the following is the most appropriate next step to decrease the risk of acute renal failure in this patient? Vasopressor therapy Diuretic therapy Monitoring urine output Fluid resuscitation Ceftriaxone therapy

Fluid resuscitation

A 3-year-old girl is brought to the emergency department2 days after the onset of fever, profuse watery diarrhea, andprogressive lethargy. On arrival, she appears ill and ispoorly responsive. Her temperature is 39.4°C (103°F),pulse is 180/min, respirations are 40/min, and bloodpressure is 80/45 mm Hg. Examination shows poor skinturgor and cool extremities. Mucous membranes are dry.Capillary refill time is delayed. Which of the following isthe most appropriate next step to decrease the risk of acuterenal failure in this patient? Fluid resuscitation Ceftriaxone therapy Monitoring urine output Diuretic therapy Vasopressor therapy

Fluid resuscitation

A 2778-g (6-lb 2-oz) male newborn is born at 37 weeks' gestation to a 27-year-old woman, gravida 3, para 2, after an uncomplicated labor and delivery. The mother has no medical insurance and did not receive prenatal care. She says she did not have any health problems during pregnancy, but she continued to consume two bottles of beer weekly during her pregnancy. She does not take any medications, vitamins, or herbal supplements. Her diet consists mostly of rice and beans. Examination of the newborn shows spina bifida. Which of the following measures during the mother's pregnancy is most likely to have prevented this child's deformity? Glucose tolerance test Increase dietary intake of omega-3 fatty acids Screening for group B streptococcal infection TORCH titer screening Folic acid supplementation Abstinence from alcohol

Folic acid supplementation

A 2778-g (6-lb 2-oz) male newborn is born at 37 weeks'gestation to a 27-year-old woman, gravida 3, para 2, afteran uncomplicated labor and delivery. The mother has nomedical insurance and did not receive prenatal care. Shesays she did not have any health problems duringpregnancy, but she continued to consume two bottles ofbeer weekly during her pregnancy. She does not take anymedications, vitamins, or herbal supplements. Her dietconsists mostly of rice and beans. Examination of thenewborn shows spina bifida. Which of the followingmeasures during the mother's pregnancy is most likely tohave prevented this child's deformity? Folic acid supplementation Increase dietary intake of omega-3 fatty acids Glucose tolerance test TORCH titer screening Screening for group B streptococcal infection Abstinence from alcohol

Folic acid supplementation

A 3-year-old boy is brought to the emergency departmentbecause of a 2-week history of persistent cough andwheezing. His mother states that his symptoms occur whenhe laughs or runs. He has not had any other symptoms, andno one else at home is sick. Three weeks ago, he had anepisode of choking and coughing while eating dinner. Histemperature is 37.6°C (99.7°F), pulse is 90/min,respirations are 20/min, and blood pressure is 70/45mm Hg. Expiratory wheezes are heard on the right withnormal airflow. The remainder of the examination showsno abnormalities. His symptoms do not improve afteradministration of nebulized albuterol. An expiratory chestx-ray shows hyperinflation of the right lung; there is nomediastinal or tracheal shift. Which of the following is themost likely diagnosis? Foreign body aspiration Laryngotracheobronchitis (croup) Asthma Psychogenic cough Tension pneumothorax

Foreign body aspiration

An 8-year-old male presents with a productive cough that produces yellow sputum for the past two days. He has a history of bilateral lower-lobe pulmonary infiltrate, several episodes of pneumonia, and a positive chloride sweat test. Which of the following is most likely cause of this patient's symptoms? Nonsense mutation In-frame deletion Silent mutation Frameshift mutation Missense mutation

Frameshift mutation

A 28-year-old woman at 18 weeks' gestation has palpitations. Laboratory studies show an increased serum total thyroxine T3 concentration. Which of the following tests is best to confirm a diagnosis of hyperthyroidism in this patient? Free T4 Radioactive iodine uptake Fine-needle aspiration biopsy of the thyroid gland Thyroid antibodies Serum total triiodothyronine {T

Free T4

An autopsy is done on a 50-year-old man who died of pneumonia despite 5 days of antibiotic therapy in the intensive care unit. He had a 15-year history of alcoholism. A photograph of a sagittal section of the brain is shown. Based on this pathology, which of the following findings was most likely present on neurologic examination of the patient prior to his death? Essential tremor Dysmetria on finger-nose testing Present Romberg sign Dysdiadochokinesia Gait ataxia

Gait ataxia

A 5-year-old boy is stung on his foot by a bee. Within 30 minutes the local area is edematous. The extravascular accumulation of fluid is most directly related to which of the following? Vasodilation Demargination of leukocytes Vasoconstriction Gap formation between endothelial cells Fibrin thrombi

Gap formation between endothelial cells

A 35-year-old man comes to the physician because of a 6-month history of burning abdominal pain that occurs 1 to 2 hours after he eats. He also has had black stools for 2 days. Use of over-the-counter antacids and histamine-2 (Hi}-receptor blocking agents has not been effective in relieving his symptoms. He is sweating profusely and has light-headedness when he stands. His blood pressure is 105/70 mm Hg while sitting. Physical examination shows epigastric tenderness. A CT scan of the abdomen shows a 1-cm mass in the pancreas. lmmunohistochemical labeling of neoplastic cells in a biopsy specimen is most likely to involve the use of antibodies directed to which of the following substances? Somatostatin Insulin Gastrin Glucagon Serotonin Vasoactive intestinal polypeptide Amylase Human pancreatic polypeptide Lipase

Gastrin

A previously healthy 72-year-old man is brought to the physician because of weakness and fatigue. His hemoglobin concentration is 9.2 g/dl, leukocyte count is 5400/mm3, and platelet count is 350,000/mm3. A peripheral blood smear is shown. Which of the following is the most likely cause of these findings? Beta-Thalassemia major Gastrointestinal blood loss Aplastic anemia Vitamin B12 (cobalamin) deficiency Chronic myelogenous leukemia

Gastrointestinal blood loss

A previously healthy 25-year-old woman comes to the physician for a routnine pelvic exam. Physical examination shows an enlarged right ovary. The ovary is surgically removed. Examination of the excies ovary shows a cystic lesion filled with hair and thick, oily, yellow-tan subastance. Microscopic examination of the lesion shows hair follicles, cartilage, smooth muscle, and brain tissue. Which of the following cell types is the most likely source of the neoplasm in this patient? Ganulosa cells Stromal cells Germ cells Sex cord cells Surface epithelial cells

Germ cells

A 23-year-old man comes to the physician because of a low-grade fever, nasal congestion, and muscle aches for 1 week. He states that his girlfriend recently told him that his eyes appeared yellow. He has not had clay-colored stools or dark urine. His only medication is ibuprofen. He has no recent history of foreign travel, tattoos, use of intravenous drugs, or exposure to blood products. He is 183 cm (6 ft) tall and weighs 77 kg (170 lb); BMI is 23 kg/m2. Physical examination shows mild scleral icterus but no subungual icterus. The remainder of the examination shows no abnormalities. Serum studies show: Total bilirubin: 2.6 mg/dL Direct bilirubin: 0.3 mg/dL Alkaline phosphatase 30 U/L AST 12 U/L ALT 10 U/L Which of the following is the most likely diagnosis? Hepatitis D Gilbert Syndrome Steatohepatitis Idiopathic cirrhosis Hereditary spherocytosis

Gilbert Syndrome

A 45-year-old man comes to the physician for a follow-up examination 1 week after he underwent an emergency appendectomy. Physical examination shows mild scleral icterus and a well healing surgical incision. Laboratory studies show: Which of the following best explains these findings? Gilbert syndrome Liver failure Hepatitis Surgical bile duct trauma Cholelithiasis

Gilbert syndrome

An investigator breeds a transgenic strain of mice that develops severe fasting hypoglycemia. Administration of glucagon does not correct the hypoglycemia, but administration of epinephrine results in an increase in the serum glucose concentration. Which of the following proteins is most likely defective in this mouse strain? Protein kinase A Adenylyl cyclase Glucagon receptor Glycogen phosphorylase Phosphorylase kinase Heterotrimeric G protein

Glucagon receptor

A 23-year-old woman comes to the physician because of a 3-week history of frequent thirst and urination; she also has had a 3-kg (6.6-lb) weight loss during this period. Physical examination shows dehydration and tachypnea. Serum studies show a glucose concentration of 330 mg/dl, 2+ ketones, and pH of 7.2. Following the administration of intravenous fluids and insulin, there is marked improvement. The activity of which of the following enzymes has most likely increased in this patient's hepatocytes because of this treatment? Phosphoeno/pyruvate carboxykinase Glucokinase Phosphorylase kinase Glucose 6-phosphatase Glycogen phosphorylase

Glucokinase

A 25-year-old woman comes to the physician because of a2-day history of muscle cramps and profuse, watery stools.She returned from a trip to Pakistan 3 days ago. Hertemperature is 37°C (98.6°F), pulse is 120/min, and bloodpressure is 80/50 mm Hg. Stool culture shows numerouscurved, gram-negative bacteria; there are no erythrocytes orleukocytes. Oral rehydration is initiated. The blood pressureincreases, and the pulse decreases. The oral hydrationformula most likely promotes sodium absorption via the gutby allowing cotransport with which of the following? Fatty acid Potassium Glucose Albumin Magnesium

Glucose

A 6-year-old girl is brought to the physician for a follow-up examination. She has been receiving treatment with potassium citrate monohydrate since the diagnosis of renal tubular acidosis was made at the age of 2 years. Physical examination shows no abnormalities. Serum studies show: Na+ 142 mEq/L K+ 3.5 mEq/L Cl- 115 mEq/L HC03- 18 mEq/L Urea nitrogen 9 mg/dl Creatinine 0.9 mg/dl A defect in renal ammoniagenesis is suspected. Which of the following substrates is the most likely source of ammonia production in this patient? Glycine Glutamine Leucine Creatinine Urea

Glutamine

A 2-month-old male infant is brought to the emergency department by his mother because of a 2-day history of generalized tonic-clonic seizures, myoclonus, and hiccuping. His mother says he has fed poorly and has been floppy since birth. Physical examination shows decreased deep tendon reflexes and hypotonia. A signaling defect in an inhibitory neurotransmitter is suspected. Which of the following neurotransmitter receptors most likely contains the defect? Glutamate receptor Glycine receptor Acetylcholine receptor Aspartic acid receptor Epinephrine receptor

Glycine receptor

A 79-year-old woman is admitted to the hospital after sustaining a fracture of the right hip when she fell down the stairs in her two-story home. She worked as a librarian for 40 years until she retired 14 years ago. Her hobbies include reading, knitting, and gardening. She also volunteers at the local hospital, reading to patients. She occasionally visits a local health club to participate in a swimming pool-based exercise class. She drinks one glass of wine daily with her evening meal. She is 155 cm (5 ft 1 in) tall and weighs 50 kg (110 lb); BMI is 21 kg/m2. Which of the following recommendations by the physician is most likely to decrease this patient's risk for additional fractures in the future? Go for a long walk outside daily Increase participation in swimming pool-based exercise classes to at least three times weekly Discontinue gardening Discontinue consumption of all alcoholic beverages Lose weight

Go for a long walk outside daily

A 27-year-old woman comes to the physician because of fever, malaise, abdominal pain, and vaginal discharge for 4 days. Her last menstrual period ended 5 days ago. She had an ectopic pregnancy 1 year ago. Her temperature is 38.3°C (101°F). Abdominal examination shows bilateral lower quadrant tenderness with rebound and guarding. Pelvic examination shows cervical motion tenderness and bilateral adnexal tenderness. Her leukocyte count is 18,000/mm3. A pregnancy test is negative. Which of the following is the most likely diagnosis? Gonorrhea Bacterial vaginosis Diverticulitis Appendicitis Trichomoniasis Chancroid Herpes genitalis

Gonorrhea

A previously healthy 42-year-old woman comes to the emergency department because of progressive shortness of breath and intermittent cough productive of blood-tinged sputum for 10 days. She has no history of major medical illness and takes no medications. Her respirations are 24/min. Diffuse rhonchi are heard in both lung fields on auscultation. A chest x-ray shows patchy opacities bilaterally. Laboratory studies show: Serum creatinine 2.5 mg/dL, Urine protein 3+ RBC 3+ RBC casts numerous. A serum anti-glomerular basement membrane autoantibody assay is positive. Which of the following is the most likely diagnosis? Wegener granulomatosis Goodpasture syndrome Löffler syndrome Hemolytic uremic syndrome Mucocutaneous lymph node syndrome(Kawasaki disease)

Goodpasture syndrome

A 22-year-old woman comes to the physician because of a10-day history of pain in multiple joints. She first had pain inher right elbow, and then her right shoulder, and now haspain, redness, and swelling in her left knee that began 2 daysago. She currently has no pain in the right shoulder andelbow. There is no history of trauma. She is sexually active,and she and her partner use condoms for contraceptioninconsistently. Examination of the left knee shows warmth,erythema, tenderness, and soft-tissue swelling. Range ofmotion of the knee is limited to 10 degrees of flexion. Theremainder of the examination, including pelvic examination,shows no abnormalities. Arthrocentesis of the knee jointyields 10 mL of cloudy fluid with a leukocyte count of18,300/mm3 (97% segmented neutrophils). Microscopicexamination of the leukocytes within the joint fluid is mostlikely to show which of the following? Gram-negative diplococci Needle-shaped negatively birefringent crystals Cuboidal positively birefringent crystals Acid-fast bacteria Gram-positive cocci in clusters

Gram-negative diplococci

A 45-year-old woman is admitted to the hospital for treatment of pneumococcal pneumonia and bacteremia. She is intubated and mechanically ventilated. Her temperature is 39.4 •c (102.9°F), pulse is 120/min, and blood pressure is 90/50 mm Hg. Bilateral crackles are heard on auscultation. Laboratory studies show: Hemoglobin 13 g/dL Hematocrit 40% Leukocyte count 500/mm3 Platelet count 250,000/mm3 Production of which of the following proteins is most likely necessary for the leukocyte count to return to the reference range in this patient? Granulocyte colony stimulating factor Erythropoietin lnterleukin-6 (IL-6) von Willebrand factor Angiogenesis factor Protein C

Granulocyte colony stimulating factor

A 22-year-old female presents with palpitations for >2 months. She describesfeeling shaky and nervous most of the time. She is a singlemother and notes that her boyfriend has not helped withtheir infant daughter as much as he had promised. She ishaving difficulty taking care of her daughter and managingher schoolwork. She also has had a 4.5-kg (10-lb) weightloss despite a normal appetite. She is 165 cm (5 ft 5 in) talland weighs 55 kg (120 lb); BMI is 20 kg/m2. Hertemperature is 37.4°C (99.4°F), pulse is 120/min,respirations are 18/min, and blood pressure is 150/70mm Hg. Examination shows moist palms. The globes ofthe eyes are prominent with ptosis bilaterally. Neurologicexamination shows a fine resting tremor of the hands.Deep tendon reflexes are 3+ with a shortened relaxationphase. Which of the following is the most likely diagnosis? Chronic lymphocytic thyroiditis(Hashimoto disease) Generalized anxiety disorder Panic disorder Graves disease Acute stress disorder Pheochromocytoma

Graves disease

A 25-year-old woman comes to the physician because of fatigue for 3 weeks. She also has had intermittent fever during the past 7 days. She had her teeth cleaned 1 month ago, but she did not receive prophylactic antibiotics. She had rheumatic fever as a child and had an episode of endocarditis 4 years ago. A grade 2/6 murmur is heard on auscultation. Ultrasonography shows an abnormal mitral valve. A photomicrograph of the causal organism grown in blood cultures is shown. The causal organism will most likely demonstrate which of the following? Inhibition by optochin Greening reaction on blood agar Resistance to novobiocin Coagulase production Production of indole Susceptibility to bacitracin

Greening reaction on blood agar

A previously healthy 39-year-old woman is brought to the physician because of a tingling sensation in her fingers and toes for 2 days and rapidly progressive weakness of her legs. She had an upper respiratory tract infection 2 weeks ago. She was unable to get up from bed this morning. Examination shows weakness of all four extremities, distal greater than proximal. Deep tendon reflexes are absent. Sensation is mildly decreased over the feet. Which of the following is the most likely diagnosis? Multiple sclerosis Guillain-Barré syndrome Poliomyelitis Tick paralysis Myasthenia gravis

Guillain-Barré syndrome

A 17-year-old boy is brought to the physician by his mother because she is concerned about his physical development. She says that he is not developing the way his two older brothers did. He has no history of major medical illness. He is at the 10th percentile for height and 25th percentile for weight. Serum studies show a follicle-stimulating hormone concentration of 5 mlU/mL and testosterone concentration of 8 nmol/L (N=10-35). Which of the following physical findings is most likely in this patient? Excess body hair Enlarged testes Low-pitched voice Strong muscular development Gynecomastia

Gynecomastia

A 57-year-old man with alcoholism has a distended abdomen with shifting dullness and a fluid wave. He has caput medusae, palmar erythema, and spider angiomata. Which of the following is the most likely additional finding? Flame-shaped retinal hemorrhages Gynecomastia Jugular venous distention Enlarged inguinal lymph nodes Digital clubbing

Gynecomastia

Following a stroke, a 68-year-old man has a language problem. His speech is fluent but contains many grammatical errors, word substitutions, and neologisms. He is unable to repeat words after the examiner and is apparently unable to comprehend other verbal requests. Which of the following labeled sites on the photograph of the left hemisphere is most likely to be damaged? H

H

Influenza virus strains are characterized by differing sensitivity to growth inhibition by the weak base ammonium chloride (AC). Influenza A San Francisco (SF) produces virus yields in AC-treated cells that approximate those in untreated cells, whereas influenza A New York (NY) grows poorly in AC-treated cells. To identify viral genes that segregate with differences in the sensitivity of these strains to AC, reassortant viruses are isolated by coinfecting cells with both virus strains. The ratios of the growth of the reassortant viruses in the presence and absence of AC are shown in the table. Which of the following viral gene segments segregates with progeny viruses that are resistant to ammonium chloride? HA

HA

A 2-month-old boy is given a vaccine that has been manufactured to convert T-independent antigens to T-dependent forms to enhance protection in young children. Which of the following vaccines has been given? Measles Oral polio Diphtheria Haemophilus influenzae type b Tetanus

Haemophilus influenzae type b

A pale 62-year-old man has had increasing fatigue and indigestion over the past few months and decreasing appetite over the past few weeks. He has moderate splenomegaly but no lymphadenopathy. Laboratory studies show: Hemoglobin Hematocrit Leukocyte count Neutrophils Lymphocytes Monocytes Platelet count 9 g/dL 27% 3100/mm 3 20% 75% 5% 75,000/mm3 The lymphocytes have cytoplasmic projections and positivity for acid phosphatase even in the presence of tartrate. Which of the following is the most likely diagnosis? Acute myelogenous leukemia Infectious mononucleosis Chronic lymphocytic leukemia Acute lymphoblastic leukemia Chronic myelogenous leukemia Hairy cell leukemia

Hairy cell leukemia

A previously healthy 15-year-old boy is brought to the emergency department 30 minutes after the onset of headache, nausea, and dizziness. He had been practicing football twice daily in outside temperatures of 100°F. On arrival, he is alert and cooperative. His temperature is 39°C (102.2°F), pulse is 120/min, respirations are 28/min, and blood pressure is 90/60mm Hg. Examination shows profuse diaphoresis. Mental status examination shows no abnormalities. Which of the following is the most likely diagnosis? Heat exhaustion Heat cramps Malignant hyperthermia Heat stroke Neuroleptic malignant syndrome

Heat exhaustion

A 56-year-old woman comes to the physician because of a 3-week history of progressive difficulty swallowing. When her symptoms began, she felt that bread and larger pieces of meat were getting stuck in her chest before passing through completely, but now she is having difficulty swallowing all foods and some liquids. Physical examination shows normal oral coordination and a nontender abdomen. An x-ray obtained after a barium swallow shows an irregular mass at the gastroesophageal junction. A photomicrograph of a biopsy specimen obtained via esophagogastroduodenoscopy is shown. Chronic infection with which of the following pathogens is the most likely cause of the histologic findings in this patient? Entamoeba histolytica Clonorchis sinensis Helicobacter pylori Mycobacterium tuberculosis Cytomegalovirus

Helicobacter pylori

A healthy 25-year-old man lives at 10,000 feet above sea level for 4 weeks. Two weeks after returning to sea level, ventilation has returned to normal, but oxygen delivery to muscles is greater than before acclimatization. An increase in which of the following best explains the oxygen delivery finding in this man? Arterial PO2 Cardiac output Pulmonary artery pressure Arterial pH Hematocrit

Hematocrit

A 50-year-old man underwent a bowel resection for colon cancer 2 years prior to his death. A photograph of his liver as seen at autopsy is shown. Which of the following is the most likely cause of these findings? Tumor seeding of the abdominal cavity from the colonic tumor Direct extension of tumor for the colonic primary turmo Development of hepatocellular carcinoma Multiple granulomata from disseminated fungal infection Hematogenous spread of colonic tumor via portal venous system

Hematogenous spread of colonic tumor via portal venous system

A 40-year-old man is evaluated because his skin is extremely sensitive to sunlight. Exposure to the sun causes the formation of vesicles and blisters on the skin, which frequently take weeks to heal. He is diagnosed with a disorder caused by the increased synthesis of compounds in the skin that are subject to excitation by visible light. Which of the following biochemical pathways is most likely defective in this patient? Riboflavin metabolism Melanin synthesis Heme synthesis Bilirubin degradation Bile acid synthesis

Heme synthesis

A 70-year-old man comes to the physician because of a rash for 7 days. Physical examination shows multiple 4-to 6-cm tense bullae filled with clear fluid over the inner aspects of the forearms, lower abdomen, groin, and thighs. Examination of a skin biopsy specimen shows a non-acantholytic blister with separation of the epidermis from the basement membrane. Serologic studies are most likely to show an autoantibody directed against which of the following? Desmosomal proteins Collagen, type IV Hemidesmosomal proteins Anchoring filaments Intracellular adhesion molecule-1

Hemidesmosomal proteins

A 70-year-old man comes to the physician because of skin blistering for 1 week. He has had no oral lesions. Physical examination shows tense bullae in the joint folds of the upper and lower extremities. A biopsy specimen of the skin shows subepidermal blister formations. Direct immunofluorescence microscopy shows antibodies directed against proteins at the dermal-epidermal junction. Which of the following structures is most likely being targeted by the antibodies in this patient? Adherens junction (zonula adherens) Hemidesmosome Desmosome (macula adherens) Tight junction (zonula occludens) Gap junction

Hemidesmosome

Three days after undergoing elective laparoscopic cholecystectomy for cholelithiasis, a 42-year-old woman has the onset of hematomas at all surgical sites. She was treated for deep venous thrombosis 3 years ago but was not taking any medications at the time of this admission. Results of preoperative laboratory studies were within the reference range. Prior to the operation, she received heparin and underwent application of compression stockings. Her initial postoperative course was uncomplicated. Heronly medication is ibuprofen. She is 163 cm (5 ft 4 in) tall and weighs 87 kg (192 lb); BMI is 33 kg/m2. Her temperature is 37.3°C(99.2°F), pulse is 94/min, respirations are 16/min, and blood pressure is 112/74 mm Hg. Examination shows mild hematomas at all surgical sites. The abdomen is soft and nontender. There is no organomegaly. Bowel sounds are normal. Laboratory studies show:Hemoglobin 10.3 g/dL Hematocrit 30%Leukocyte count 12,000/mm3 Platelet count 45,000/mm3 Which of the following is the most likely cause of this patient's current findings? Hypersplenism Acute intermittent porphyria Inhibition of cyclooxygenase von Willebrand disease Heparin-induced thrombocytopenia

Heparin-induced thrombocytopenia

A 55-year-old man of northern European descent comes to the physician because of a 2-month history of weakness, altered skin color, and bilateral knee pain. Two of his siblings have type 2 diabetes mellitus and cirrhosis. Physical examination shows bronzed skin, hepatomegaly, and arthritis of both knees. Serum studies show increased saturation of transferrin and increased ferritin concentration. A liver biopsy specimen shows increased iron content and cirrhosis. This patient is at greatest risk for which of the following? Pigment gallstones Hepatocellular carcinoma Basal ganglia atrophy Pulmonary fibrosis Diabetes insipidus

Hepatocellular carcinoma

A 10-year-old boy has had anemia since birth. His spleen is five times normal size. Splenectomy is indicated if the anemia is caused by which of the following disorders? Fanconi anemia Hereditary spherocytosis Glucose 6-phosphate dehydrogenase deficiency Thalassemia major Sickle cell disease

Hereditary spherocytosis

A 23-year-old woman is brought to the emergency department by her roommate because she was found unresponsive on the living room floor. Examination shows constricted pupils and stupor. Which of the following substances is most likely responsible for these effects? LSD Nicotine Amphetamine Marijuana Phencyclidine (PCP) Androstenedione Caffeine Heroin Cocaine

Heroin

A male newborn is delivered at 38 weeks' gestation following an uncomplicated pregnancy. Twelve-hours later, he begins sucking frantically and crying inconsolably. He overreacts to stimuli around him and has a marked startle response. The symptoms slowly resolve over the next 2 to 3 weeks. The most likely explanation for these symptoms is maternal use of which of the following substances during pregnancy? Alcohol Heroin Marijuana LSD Psilocybin

Heroin

Three days after unprotected sex with a new partner, a sexually active 24-year-old man noticed the development of small tender blisters on his penis. Some of the blisters broke and left open sores. He does not use condoms. A photograph of the lesions is shown. Which of the following is the most likely causal agent? Treponema pallidum Herpes simplex virus type 2 Calymmatobacterium granulomatis Haemophilus ducreyi Chlamydia trachomatis

Herpes simplex virus type 2

A 40-year-old woman comes to the physician for genetic counseling because of a family history of muscle weakness. She says that she has not sought treatment because her symptoms are not severe, but her children have profound muscle weakness and fatigue. Sequence analysis of mitochondrial DNA shows a mutation in 50% of the mother's mitochondrial DNA and in 100% of each of her children's mitochondrial DNA. A pedigree is shown. Which of the following genetic principles best explains the findings in this family? Heteroplasmy Genetic drift Anticipation Imprinting Pseudodominant inheritance

Heteroplasmy

A 19-year-old man is evaluated for signs of gastrointestinal bleeding. At laparotomy, a 5-cm blind outpouching on the antimesenteric side of the terminal ileum, about 15 cm from the ileocecal valve, is resected. Pathologic examination of the resected segment will most likely show which of the following? Crohn disease Henoch-Schonlein vasculitis Heterotopic gastric mucosa Ulcerative colitis Angiodysplasia of the colon

Heterotopic gastric mucosa

A 65-year-old man is brought to the emergency department 1 hour after the sudden onset of generalized tonic-clonic seizures. His wife says that he has had a change in personality during the past 6 months; he used to be mild mannered, but he has recently become verbally abusive. A CT scan of the head shows a single mass that enhances with contrast in the right frontal lobe and crosses over to the left hemisphere through the corpus callosum. Examination of a biopsy specimen of the mass is most likely to show which of the following conditions? Metastatic carcinoma Malignant schwannoma Malignant meningioma Metastatic melanoma High grade fibrillary astrocytoma

High grade fibrillary astrocytoma

A 42-year-old woman undergoes biopsy of suspicious calcifications seen on routine mammograms. Light microscopy shows a poorly demarcated region consisting of poorly cohesive cells growing in sheets. The nucleus to cytoplasmic ratio approaches 1:1, and prominent nucleoli are observed. There is microscopic evidence of invasion into stromal tissues. She has no adenopathy. CT scans show no evidence of hepatic, pulmonary, or bone metastases. These findings are most consistent with which of the following? Low-grade, high-stage neoplasm High-grade, low-stage neoplasm Low-grade, low-stage neoplasm High grade, high stage neoplasm

High grade, high stage neoplasm

A 43-year-old woman is admitted to the hospital 40 minutes after experiencing a transient hypoxic-ischemic event caused by an episode of ventricular fibrillation. Her cardiac rhythm was successfully stabilized using an automated external defibrillator. Two days later, she is unable to remember new events in her life. She has no other neurologic sequelae. Injury to which of the following is the most likely underlying cause of this patient's memory deficit? Right parietal lobe Amygdaloid nuclei Hippocampus Wernicke area Frontal lobes

Hippocampus

A mouse melanoma, B-16, is highly metastatic. A clone of the B-16 melanoma cells is injected intravenously into the tail vein of mice, and 3 weeks later tumor nodules are found in the lungs, liver, and ovaries. When an antibody to a liver cell surface antigen is injected 1 hour prior to injection of the B-16 melanoma cells, formation of tumor nodules in the liver is prevented, but tumor nodules develop in the ovaries and lungs. In vitro studies do not show any cytotoxic effect of the antibody on B-16 or hepatic cells. Which of the following processes is most likely to be affected by the antibody? O C) Invasion Homing O E) Progression 0 D) Motility 0 B) Initiation O F) Promotion

Homing

A 6-year-old girl is brought to the emergency department by her father because of a 15-minute history of severe shortness of breath. The father says that she was diagnosed with a throat tumor 3 years ago. Her respirations are 32/min. Physical examination shows nasal flaring. Laryngoscopy shows multiple raised, finger-shaped lesions arising from the vocal cords and epiglottis and partially occluding the larynx. The lesions are excised. Microscopic examination of one of the lesions shows finger-shaped fibrovascular cores lined with benign squamous epithelium. Which of the following is the most likely causal virus? Herpes simplex virus 1 Epstein-Barr virus Cytomegalovirus Herpes simplex virus 2 Human papillomavirus, type 6

Human papillomavirus, type 6

A 65-year-old man is brought to the emergency department by his wife because of a 4-hour history of intermittent severe pain that originates in the flank area and radiates to the genital region. He has a history of hypercalciuria and renal calculi. Which of the following drugs is most likely to decrease the urinary excretion of calcium in this patient, decreasing his risk for future renal calculus formation? Spironolactone Furosemide Acetazolamide Triamterene Hydrochlorothiazide

Hydrochlorothiazide

An 80-year-old woman comes to the emergency department because of a 2-day history of "feeling funny." She says, "I have lost my pep." She has never been admitted to the hospital, but she has a history of poorly controlled hypertension. She just started a medication 2 weeks ago but does not recall the name. Her blood pressure is 130/85 mm Hg. Physical examination shows no abnormalities. Her serum potassium concentration is 3 mEq/L. Which of the following drugs is the most likely cause of this patient's condition? Clonidine Metoprolol Hydrochlorothiazide Triamterene Lisinopril

Hydrochlorothiazide

A 22-year-old nulligravid woman comes to the physician for counseling prior to conception. Menses have occurred at irregular 26- to 32-day intervals since menarche at the age of 14 years. She is 163 cm (5ft 4in) tall and weighs 85 kg (187 lb); BMI is 32 kg/m2. Her blood pressure is 130/80 mm Hg. Physical examination shows mild hirsutism and velvety brown, thickened skin at the base of the neck and around the axillae. This patient most likely also has which of the following conditions? Hyperinsulinemia Hypoprolactinemia Hypoandrogenism Hypoglycemia Hyperthyroidism

Hyperinsulinemia

At autopsy, the heart of a 76-year-old woman shows concentric enlargement of the left ventricle. Microscopic examination of the heart shows enlarged myocardial cells with large nuclei. Which of the following diseases was the most likely cause of the cardiac enlargement? Pericarditis Hypertrophic cardiomyopathy: Concentric, Hypertension Myocardial infarction Cardiac amyloidosis: Concentric? South American trypanosomiasis

Hypertension

A 29-year-old man is brought to the emergency department because he has a severe bilateral headache and irritability. His pulse is 120/min, respirations are 30/min, and blood pressure is 200/120 mm Hg. Ophthalmoscopic examination shows blurring of the optic discs. Deep tendon reflexes are 3+ and symmetric. Which of the following is the most likely diagnosis? Hyperthyroidism Panic disorder Alcohol intoxication Hypertensive encephalopathy Heavy metal poisoning

Hypertensive encephalopathy

Heavy metal poisoning Alcohol intoxication Hyperthyroidism Hypertensive encephalopathy Panic disorder

Hypertensive encephalopathy

A 6-week-old male newborn has persistent non-bilious projectile vomiting. Physical examination shows prominent peristalsis. Laboratory findings are indicative of a metabolic alkalosis. Which of the following is the most likely mechanism of this condition? Intestinal obstruction by meconium Hypertrophy of the pyloric sphincter Congenital defect in the diaphragm Congenital atresia of the bile ducts Aganglionosis of the large bowel

Hypertrophy of the pyloric sphincter

An 8-year-old boy with severe mental retardation is brought to the physician because of a 1-year history of self-mutilating behavior, including biting his fingers. His male cousin has similar symptoms. Physical examination shows spasticity, chorea, and mutilation of the digits. Laboratory studies are most likely to show which of the following abnormalities? Hypophosphatemia Hypokalemia Hyponatremia Hypercalcemia Hyperuricemia Hypercalciuria

Hyperuricemia

A 24-year-old woman with Marfan syndrome comes to the physician one week after a home pregnancy test result was positive. Previous evaluations showed mild coarctation of the thoracic aorta with mild dilation of the ascending aorta, and mild aortic regurgitation, but no significant heart failure. If her pregnancy were to proceed, which of the following mechanisms would most likely increase the amount of aortic regurgitation in this patient? Increased metabolic rate Decreased vascular resistance Weight gain Increased cutaneous blood flow Hypervolemia

Hypervolemia

A male newborn is born to a 32-year-old woman by cesarean delivery at 38 weeks' gestation and weighs 4309 g (9 lb 8 oz). The pregnancy was complicated by gestational diabetes that was difficult to manage. Apgar scores are 6 and 8 at 1 and 5 minutes, respectively. This newborn is at increased risk for developing which of the following within the next 24 hours? Hypoglycemia Hypercalcemia Hyperkalemia Hypokalemia Hyperglycemia Hypothermia

Hypoglycemia

A male newborn is born to a 32-year-old woman bycesarean delivery at 38 weeks' gestation and weighs4309 g (9 lb 8 oz). The pregnancy was complicated bygestational diabetes that was difficult to manage. Apgarscores are 6 and 8 at 1 and 5 minutes, respectively. Thisnewborn is at increased risk for developing which of thefollowing within the next 24 hours? Hypercalcemia Hyperglycemia Hypoglycemia Hypothermia Hyperkalemia Hypokalemia

Hypoglycemia

A 64-year-old man is evaluated for cough, dyspnea, and chest pain. He is afebrile. An x-ray of the chest shows an abnormal density in the hilar region. Lung tissue biopsy shows round cells with little cytoplasm that are about twice the size of lymphocytes. These cells are arranged in infiltrating sheets that have neither glandular nor squamous organization. Which of the following abnormalities in serum is most likely in this patient? Hypoalbuminemia Hyponatremia Hyperlipidemia Hypermagnesemia Hyperkalemia Hypoglycemia

Hyponatremia

A 18-year-old male is found unconscious in his room. The patient has a history of IV heroin use. On arrival, he appears cyanotic, unresponsive to verbal and painful stimuli. normal temperature of 98.8°F, pulse is 80/min, respiratory rate is 4/min, and blood pressure is 110/60 mm Hg. Arterial blood gas analysis on room air shows: pH 7.20 PCO2 of 80 mm Hg PO2 of 50 mm Hg HCO3 of 29 mEq/L. Which of the following is the best explanation for this patient's hypoxemia? Impaired diffusion capacity of the lung for carbon monoxide Hypoventilation Decreased inspired oxygen tension Right-to-left shunt Ventilation-perfusion mismatch

Hypoventilation

A 27-year-old man is brought to the emergency department20 minutes after his roommate found him unconscious ontheir bathroom floor. The patient has a history ofintravenous heroin use. He has no history of serious illnessand takes no medications. On arrival, he appears cyanotic.He is unresponsive to verbal and painful stimuli. Histemperature is 37.1°C (98.8°F), pulse is 80/min,respirations are 4/min, and blood pressure is 110/60mm Hg. Examination shows new and old needle tracksover the upper and lower extremities. Cardiopulmonaryexamination shows no abnormalities. Arterial blood gasanalysis on room air shows:pH 7.20PCO2 80 mm HgPO2 50 mm HgHCO3- 29 mEq/LWhich of the following is the best explanation for thispatient's hypoxemia? Decreased inspired oxygen tension Hypoventilation Right-to-left shunt Impaired diffusion capacity of the lung forcarbon monoxide Ventilation-perfusion mismatch

Hypoventilation

A previously healthy 54-year-old man comes to the clinic because of light-headedness for 6 hours. His symptom began after skiing at a resort at an altitude of 274 3 m (9000 ft). He has been taking a carbonic anhydrase inhibitor since 2 days before arrival at the resort. His blood pressure is 110/60 mm Hg while sitting and 95/50 mm Hg while standing. Physical examination shows no other abnormalities. Which of the following is the most likely cause of his orthostatic hypotension? Hypovolemia High-altitude sickness Respiratory alkalosis Hypoxia Impaired sympathetic nerve activity

Hypovolemia

A 27-year-old nulligravid woman and her husband have been unable to conceive for 12 months. She has never used contraception. Menses occur at 28-day intervals, and her last menstrual period was 2 weeks ago. She had a single episode of pelvic inflammatory disease 4 years ago and was treated with oral antibiotics. Vaginal examination shows no abnormalities. Cervical cultures are normal. Which of the following is the most appropriate next step in diagnosis? Reevaluation in 6 months Ultrasonography of the abdomen Hysterosalpingography Sperm penetration assay Endometrial biopsy

Hysterosalpingography

A 27-year-old nulligravid woman and her husband have beenunable to conceive for 12 months. She has never usedcontraception. Menses occur at 28-day intervals, and her lastmenstrual period was 2 weeks ago. She had a single episodeof pelvic inflammatory disease 4 years ago and was treatedwith oral antibiotics. Vaginal examination shows noabnormalities. Cervical cultures are normal. Which of thefollowing is the most appropriate next step in diagnosis? Endometrial biopsy Ultrasonography of the abdomen Hysterosalpingography Reevaluation in 6 months Sperm penetration assay

Hysterosalpingography

A 12-year-old male with a preliminary diagnosis of a muscular disorder is referred to a pediatric neurologist that specializes in muscular dystrophy. Family history reveals a maternal uncle has was diagnosed with Becker muscular dystrophy (BMD) at 16 years of age. Using the known polymorphisms in four restriction enzyme sites within the DMD gene, family members where typed for polymorphisms in these sites. A polymorphism in the HindII site that mapped to the 5'-end of intron 12 within the DMD was amplified by PCR, digested with HindII, separated by gel electrophoresis. Results are given in figure 1. Which of the following is the most accurate conclusion? II-3 and III-1 are homozygous for the recessive allele II-2 and III-2 are heterozygous II-2 and III-2 are homozygous for the recessive allele II-3 and III-1 are homozygous for the dominant allele II-3 and III-1 are heterozygous

II-3 and III-1 are heterozygous

A previously healthy 42-year-old man is brought to the emergency department by his wife because of a 5-hour history of fever, chills, and severe pain and swelling of his left arm. His wife says he scratched his left arm on a nail while repairing a fence in his yard yesterday. He appears confused. His temperature is 40°C (104° F), pulse is 140/min, respirations are 25/min, and blood pressure is 71/38 mm Hg. Examination of the left upper extremity shows edema and erythema in the area of the scratch. Laboratory studies show: L-4 and IFN gamma lnterleukin- 1 (IL-1) and interferon (IFN) alpha IL-4 and TNF-a IL-4 and IFN alpha IL-1 and IFN gamma IL-1 and tumor necrosis factor (TNF)- a

IL-1 and tumor necrosis factor (TNF)- a

A 54-year-old man comes to the physician because of a 2-week history of burning epigastric pain. He is recently divorced. His diet mainly consists of fast food. He smokes cigars and drinks one to two 12-ounce cans of beer on the weekends. He was treated for Helicobacter pylori infection 1 year ago. He uses high doses (600 mg daily) of over-the-counter ibuprofen for chronic joint pain. Abdominal examination shows tenderness to deep palpation of the epigastric area. A peptic ulcer is suspected. Which of the following is the strongest predisposing risk factor for this patient's condition? Ingestion of alcohol Ibuprofen use Smoking cigars Recent psychosocial stressor High-fat diet

Ibuprofen use

A 66-year-old man comes to the physician because of a 1-month history of a nonproductive cough and a 6-month history of progressive shortness of breath. He has no history of major medical illness. Bilateral end-inspiratory crackles are heard on auscultation of the chest. Heart sounds are normal. A CT scan of the chest shows diffuse reticular opacities that are more pronounced at the periphery and bases of the lungs. Microscopic examination of a biopsy specimen of the lung shows patchy interstitial fibrosis, several fibroblastic foci, and no granulomas. Which of the following is the most likely diagnosis? Hypersensitivity pneumonitis Cryptogenic organizing pneumonia Sarcoidosis Berylliosis Idiopthic pulmonary fibrosis

Idiopthic pulmonary fibrosis

An 18-month-old boy is brought to the physician because of progressive facial abnormalities and slow growth during the past 6 months. He has an older brother with similar abnormalities who also has mental retardation. The patient is below the 3rd percentile for length and at the 10th percentile for weight. Physical examination shows coarse facial features, hepatosplenomegaly, and joint stiffness. His urine concentrations of dermatan sulfate and heparan sulfate are increased. A deficiency of which of the following enzyme activities is the most likely cause of the findings in this patient? Galactosidase Iduronate sulfatase Lipoxygenase Hexosaminidase A Glycosylase

Iduronate sulfatase

A 35-year-old male presents with mild swelling of the hands, feet, and face. History reveals that patient started taking Azthyromycin (Zpack) to treat bronchitis. Physical examination reveals normal lung and heart ascultations and a rash near the groin area and axillary zone. History reveals no prior reaction to antibiotics. A penicillin skin test is positive. The immediate allergic reaction is most likely mediated by which of the following immunoglobins? IgG IgE IgM IgA IgD

IgE

A 16-year-old girl comes to the physician because of a 3-day history of fever, nonproductive cough, and fatigue. Her temperature is 38.3°C (100.9°F), pulse is 88/min, respirations are 18/min, and blood pressure is 102/70 mm Hg. Physical examination shows pale conjunctivae. A chest x-ray shows bilateral interstitial infiltrates. A sample of blood drawn for testing spontaneously agglutinates while awaiting transport to the laboratory. Which of the following antibody isotypes is the most likely cause of the agglutination? IgD IgE lgA IgM IgG

IgM

A 26-year-old primigravid woman delivers a 2948-g (6- lb 8-oz) male newborn at term. Six months ago, she received a vaccine, Streptococcus agalactiae (group B), serotype Ill, during a clinical trial. Screening indicated that she had generated an antibody response to this polysaccharide antigen within 1 month of the vaccination. This antibody was detected in her serum immediately prior to the onset of labor. Eight hours after delivery, the newborn appears progressively lethargic, and his temperature is 38.2C (100.8F). Cerebrospinal fluid and blood cultures grow S. agalactiae (group B), serotype Ill. It is suspected that the mother produced only one antibody isotype in response to the vaccine. This isotope is most likely which of the following? lgE lgG IgM lgD lgA

IgM

A 6-week-old girl is brought to the physician by her mother because of a 6-day history of vomiting a small amount of milk 2 to 3 times daily. She appears well. She is at the 50th percentile for length and weight. Physical examination shows no abnormalities. Which of the following is the most likely cause of this patient's findings? Pyloric stenosis Gut malrotation Neuromuscular abnormality of the esophagus Tracheoesophageal fistula Immature lower esophageal sphincter Esophageal spasm

Immature lower esophageal sphincter

An investigator wishes to test the hypothesis that a number of CD44 splice variants correlate with aggressive disease progression and risk of relapse in patients with Hodgkin disease, especially if the variant protein CD44v10 is expressed. The investigator plans to retrospectively screen archives of fresh frozen tissue and paraffin-embedded biopsy specimens from documented Hodgkin disease patients. Assuming the necessary reagents are available, which of the following methods would be the most effective in screening the archives for the variant protein? Affinity column chromatography Density gradient centrifugation Reverse transcriptase PCR Immunohistochemistry Southern blot Northern blot Polymerase chain reaction (PCR)

Immunohistochemistry

An 11-year-old girl with cystic fibrosis is admitted to the hospital 18 hours after the onset of shortness of breath. During the past 11 years, she has had more than 20 episodes of respiratory exacerbations of her cystic fibrosis that have required hospitalization. Current medications include an inhaled bronchodilator, inhaled corticosteroid, oral pancreatic enzyme, and oral multivitamin. Her temperature is 38°C (100.4°F), pulse is 96/min, respirations are 28/min, and blood pressure is 92/68 mm Hg. Diffuse wheezes and crackles are heard bilaterally. A sputum culture grows Burkholderia cepacia. Which of the following is the most likely cause of this patient's recurrent respiratory tract infections? Impaired clearing of airway secretions Increased exposure to infectious organisms Interstitial fibrosis Immune suppression Restrictive changes of the small airways

Impaired clearing of airway secretions

An 11-year-old girl with cystic fibrosis is admitted to thehospital 18 hours after the onset of shortness of breath.During the past 11 years, she has had more than20 episodes of respiratory exacerbations of her cysticfibrosis that have required hospitalization. Currentmedications include an inhaled bronchodilator, inhaledcorticosteroid, oral pancreatic enzyme, and oralmultivitamin. Her temperature is 38°C (100.4°F), pulse is96/min, respirations are 28/min, and blood pressure is92/68 mm Hg. Diffuse wheezes and crackles are heardbilaterally. A sputum culture grows Burkholderia cepacia.Which of the following is the most likely cause of thispatient's recurrent respiratory tract infections? Interstitial fibrosis Restrictive changes of the small airways Immune suppression Impaired clearing of airway secretions Increased exposure to infectious organisms

Impaired clearing of airway secretions

A 6-year-old boy is found to have almond-shaped eyes, temperature instability, mental retardation, hyperphagia, and obesity. Cytogenetic analysis shows a balanced translocation involving chromosome 15q11-q13. His father and paternal aunt are found to have an identical balanced translocation. His aunt's 5-year-old daughter has a history of inappropriate laughter, abnormal gait, and mental retardation. Which of the following genetic principles best explains the findings in this family? Genetic drift Heteroplasmy Anticipation Imprinting Pseudodominant inheritance

Imprinting

In a nuclear transplantation experiment, mouse embryos are produced with two pronuclei, both of which are of the same parental origin. When the two pronuclei are both of maternal origin, the products of conception have poorly developed extraembryonic structures. When the two pronuclei are both of paternal origin, the products of conception have poorly developed embryonic tissue. These results are most compatible with which of the following genetic mechanisms? Expansion of trinucleotide repeats Pleiotropy Loss of heterozygosity Anticipation Imprinting

Imprinting

A 32-year-old woman develops a new neurologic finding while being treated for an acute infection involving the sphenoid sinus. Imaging studies show cavernous sinus thrombosis on the left. Which of the following additional findings is most likely present on the left? Decreased sense of smell Enlarged blind spot Homonymous hemianopia Central facial weakness Inability to abduct the eye

Inability to abduct the eye

A 23-year-old woman with type 1 diabetes mellitus is brought to the physician for a follow-up examination. She has a 2-year history of recurrent yeast infections. Genetic analysis shows a deficiency of myeloperoxidase. Which of the following is the most likely cause of the increased susceptibility to infections in this patient? Decreased oxygen consumption after phagocytosis Inability to produce hydrogen peroxide Inability to produce hydroxy-halide radicals Defective neutrophil degranulation Defective production of prostaglandins

Inability to produce hydroxy-halide radicals

A 52-year-old male has intermittent episodes of supraventricular tachycardia. During these episodes, he has a pulse of 190 to 200/min and systolic blood pressure of 60 to 80 mm Hg. Which of the following is the most likely cause of the hypotension? Erroneous blood pressure recordings Inadequate sympathetic tone Intravascular volume depletion Inadequate ventricular filling Depressed myocardial contractility

Inadequate ventricular filling

A female newborn delivered at 36 weeks' gestation is in respiratory distress. Apgar scores are 3 and 5 at 1 and 5 minutes, respectively. Physical examination shows cyanosis. Endotracheal and nasogastric tubes are placed. The x-ray shown depicts the nasogastric tube in the left hemithorax, displacement of the mediastinum to the right, and absence of bowel gas in the abdomen. This condition most likely results from which of the following embryologic events? Failure of epimere to migrate Incomplete formation of pleuroperitoneal membrane Failure of hypomere to migrate Hypoplasia of the lungs Malrotation of bowel

Incomplete formation of pleuroperitoneal membrane

A 27-year-old primigravid woman at 18 weeks' gestation comes to the physician for a routine prenatal examination. The uterus is consistent in size with an 18-week gestation. Ultrasonography shows a male fetus. The collecting system and pelvis of the left kidney is dilated, and the renal cortex appears compressed. The left and right ureters are not dilated. The right kidney appears normal. Amniotic fluid volume is normal. Which of the following is the most likely cause of the renal findings in this patient's fetus? Ectopic ureter Incomplete recanalization of proximal ureter Congenital urethral stricture Posterior urethral valves Abnormal ureteral insertion into the bladder wall Agenesis of the upper urinary tract

Incomplete recanalization of proximal ureter

A 28-year-old man with a history of intravenous drug use comes to the physician because of a 6-week history of fever, nonproductive cough, chills, and progressive shortness of breath. His temperature is 39°C (102.2°F), pulse is 110/min, respirations are 32/min and regular, and blood pressure is 120/80 mm Hg. Physical examination shows a white, patchy, loosely adherent exudate on the buccal mucosa bilaterally. A chest x-ray shows bilateral interstitial infiltrates. After receiving treatment for pneumonia, he agrees to participate in a clinical study of the effects of interleukin-2 (IL-2). After administration of IL-2, which of the following hematologic changes is most likely in this patient? Decreased CD4+ T lymphocytes Decreased erythrocytes Increased CD4+ T lymphocytes Increased erythrocytes Increased platelet count Decreased platelet count

Increased CD4+ T lymphocytes

A 15-year-old girl is brought to the emergency department 12 hours after she ingested an entire bottle (100 capsules) of vitamin D in a suicide attempt. The patient says that she is not in any pain. Physical examination shows no abnormalities. Laboratory studies are within the reference ranges. At a follow-up visit 1 month later, her serum calcium concentration is 10.4 mg/dl. Which of the following mechanisms is the most likely cause of the increased serum calcium concentration in this patient? Increased osteoblast activity in bone Increased 1-hydroxylase activity in the kidney Decreased osteoclast activity in bone Increased absorption of calcium in the gastrointestinal tract Decreased excretion of calcium from the gastrointestinal tract

Increased absorption of calcium in the gastrointestinal tract

A 50-year-old man who has smoked 2 packs of cigarettes a day for 34 years has experienced gradually increasing shortness of breath on exertion, chronic cough productive of thick sputum, and frequent episodes of wheezing. The anteroposterior diameter of his chest is increased. Diminished breath sounds and scattered rhonchi are audible throughout the lung fields. Which of the following laboratory abnormalities is expected? Increased urinary excretion of HC03 Decreased blood Pco2 Increased blood HCO3- Increased blood pH Increased urinary pH

Increased blood HCO3-

A 63-year-old man comes to the physician because of a 3-month history of difficulty sleeping. He indicates that he sleeps better if he sits upright in bed. His pulse is 90/min, and blood pressure is 110/60 mm Hg. Physical examination shows increased jugular venous pressure and mild ankle edema. Which of the following is the most likely cause of the edema in this patient? Increased capillary hydrostatic pressure Decreased capillary hydrostatic pressure Increased capillary permeability Decreased capillary permeability Increased plasma colloid osmotic pressure Decreased plasma colloid osmotic pressure

Increased capillary hydrostatic pressure

A 48-year-old woman comes to the emergency department because of palpitations. Pulse is 180/min, and blood pressure is 104/68 mm Hg. ECG shows an atrioventricular (AV) nodal reentrant tachycardia. Carotid sinus massage is performed and sinus rhythm is restored. Which of the following is the most likely mechanism of the termination of the tachycardia in this patient? Activation of carotid chemoreceptors Blockade of L-type calcium channels in the AV node Blockade of '3-adrenergic receptors in the AV node Increased cardiac sympathetic activity Increased cardiac parasympathetic activity

Increased cardiac parasympathetic activity

A 64-year-old man comes to the physician because of a 2-month history of progressive shortness of breath with minimal activity. His temperature is 37°C (98.6° F), respirations are 30/min, and blood pressure is 125/80 mm Hg. Physical examination shows clubbing of the fingers. His total lung capacity is decreased. A chest x -ray shows a coarse reticular pattern. Which of the following is the most likely underlying cause of these findings? Loss of elastic support to the walls of the bronchioles Increased mucus secretion in the bronchioles Constriction of the terminal bronchioles Destruction of the alveolar walls Increased fibrosis in the insterstitium

Increased fibrosis in the insterstitium

A 75-year-old man comes to the physician because of a 2-year history of decreased force of his urinary stream and a need to urinate several times throughout the night. His serum urea nitrogen (BUN) concentration is 55 mg/dl, and serum creatinine concentration is 5.0 mg/dl. Ultrasonography of the urinary tract shows bilateral hydronephrosis and dilated ureters. Which of the following is the most likely mechanism of this patient's renal failure? Precipitation of proteins in the renal tubules Decreased renal plasma flow Precipitation of uric acid in the renal tubules Decreased hydrostatic pressure in glomerular capillary Increased hydrostatic pressure in Bowman's space

Increased hydrostatic pressure in Bowman's space

A 48-year-old man comes to the physician because of increasing bronzing of his skin, weakness, and fatigue during the past 3 months. Physical examination shows bronzing of the skin, hepatomegaly, and small testes. Serum studies show: Which of the following best explains the findings in this patient? Ferrochelatase deficiency Increased erythropoietin production Ineffective erythropoiesis Transferrin deficiency Increased intestinal iron absorption

Increased intestinal iron absorption

A 50-year-old man comes to the physician because of a 2-month history of pain of his wrists, changes in skin color, and progressive fatigue. His brother has type 2 diabetes mellitus and cirrhosis. Physical examination shows bronze-colored skin, tenderness of the metacarpophalangeal joints in both hands, and hepatosplenomegaly. Serum studies show: AST 100 U/L ALT 110 U/L Ferritin 1200 ng/ml Total iron-binding capacity 200 μg/dl (N=250-400) Transferrin saturation 80% (N=20-50) Analysis of a liver biopsy specimen shows a markedly increased iron concentration and cirrhosis. Which of the following is the most likely cause of the findings in this patient? Increased erythropoietin action Decreased erythropoiesis Decreased serum transferrin concentration Increased oral iron intake Increased intestinal iron absorption Decreased iron excretion

Increased intestinal iron absorption

A 3-year-old girl is brought to the emergency department by her mother 1 hour after she was found with a half-empty bottle of her grandmother's diabetes medication. The mother tells the physician that the child consumed approximately 25 metformin tablets. Physical examination shows no abnormalities. This patient is at greatest risk for which of the following serum abnormalities? Decreased sodium concentration Increased AST and ALT activities Increased creatinine concentration Decreased glucose concentration Decreased calcium concentration Increased lactic acid concentration

Increased lactic acid concentration

A 55-year-old woman who is obese has a greater risk forendometrial carcinoma than a 55-year-old woman with thesame health history and status who is not obese. Which of thefollowing best explains this increased risk? Association of obesity with smoking Impairment of immune surveillance byT lymphocytes Greater average number of pregnancies Increased production of estrogen by adiposetissue Accelerated catabolism of antioxidants Carcinogenic effects of dietary fats

Increased production of estrogen by adiposetissue

A previously healthy 40-year-old man is brought to the emergency department 1 hour after the sudden onset of severe pain in his left leg while playing tennis. He is found to have ruptured the left Achilles tendon and undergoes operative repair and long leg cast immobilization. Six months later, the left calf shows a 2-cm decrease in circumference compared with the right calf. Which of the following is the most likely cause of this decrease? Increased protein degradation Decreased myosin light chain phosphatase activity Decreased glycogen synthesis Necrosis of muscle fibers Increased phosphatidyl degradation Mitochondria damage

Increased protein degradation

A 47-year-old man is admitted to the hospital for treatment of a myocardial infarction. On admission, pulse oximetry on 30% oxygen shows an oxygen saturation greater than or equal to 95%. Three hours later, the patient develops shortness of breath. Pulse oximetry now shows an oxygen saturation of 90%. Crackles are heard at the lung bases, and a grade 2/6 systolic murmur is heard. Arterial blood gas analysis on 30% oxygen shows: Which of the following is the most likely cause of this patient's hypoxemia? Increased plasma colloid osmotic pressure Increased permeability of pulmonary capillaries Decreased erythrocyte transit time in pulmonary capillaries Increased pulmonary capillary pressure Decreased alveolar ventilation Decreased lymphatic drainage

Increased pulmonary capillary pressure

A 28-year-old woman comes to the physician because of a3-month history of shortness of breath with exertion. Shetakes an oral contraceptive. There are no occupationalexposures to birds or grain dusts. Her respirations are20/min. The lungs are clear to auscultation. Cardiacexamination shows a regular rate and rhythm; S2 is slightlylouder than S1. Cardiac catheterization shows a pulmonaryartery pressure of 78/31 mm Hg (N=15-30/3-12) with anormal left ventricular end-diastolic pressure. Which of thefollowing is most likely changed in this patient? Decreased left ventricular afterload Increased muscle tone Decreased alveolar ventilation Increased pulmonary vascular resistance Increased pulmonary compliance

Increased pulmonary vascular resistance

A 40-year-old man with interstiti al pulmonary fibrosis has a greater maximal expiratory flow rate than predicted. Which of the following best explains this finding? Hypoxic bronchodilatation Hypercapnic bronchodilatation Increased surfactant secretion Hyperinflation of the lungs Increased radial traction on airways

Increased radial traction on airways

A protein found in the brown adipose tissue of mice causes a leak of H+ ions inward across the inner mitochondrial membrane. Which of the following is the most likely effect of this protein on oxidative phosphorylation and energy metabolism? Increased ratio of oxygen consumption to CO2 production Decreased ratio of oxygen consumption to CO2 production Decreased ratio of oxygen consumption to ATP generation Increased ratio of oxygen consumption to ATP generation No change in the ratios of oxygen consumption to ATP generation and oxygen consumption to CO2 production

Increased ratio of oxygen consumption to ATP generation

A 67-year-old male with severe congestive heart failure and edema has gained 9.1 kg. Which of the following is most likely to contribute to renal sodium retention? Increased renal sympathetic nerve activity Increased effective arterial blood volume Decreased plasma aldosterone concentration Increased plasma atrial natriuretic peptide concentration Decrease extracellular fluid volume

Increased renal sympathetic nerve activity

A 26-year old woman is brought to the emergency department because of a 4-day history of flu-like symptoms accompanied by vomiting following each attempt to eat or drink. Her temperature is 38.5°C (101.3°F), pulse is 93/min, respirations are 24/min, and blood pressure is 105/70mm Hg. Physical examination shows no other abnormalities. Which of the following additional findings is most likely in this patient? Increased serum atrial natriuretic peptide Increased serum aldosterone concentration Increased urine sodium and chlorideconcentrations Decreased serum ADH (vasopressin)concentration Increased urine volume

Increased serum aldosterone concentration

A 21-year-old man loses 15% of his total blood volume within 2 minutes after a motor vehicle collision. Which of the following findings is most likely? Chemoreceptor-mediated vasoconstriction Increased tonic release of atrial natriuretic peptide from the atria Increased vagal efferent nerve activity to the sinoatrial node Increased sympathetic nerve traffic to the sinoatrial node Decreased sympathetic nerve activity to the veins

Increased sympathetic nerve traffic to the sinoatrial node

A 60-year-old female is brought to the emergency department by her husband because of chest pain that began three-hours ago. She is dyspneic and diaphoretic, and she appears anxious. Her pulse is 110/min and regular, respirations are 35/min, and blood pressure is 190/100 mm Hg. An ECG shows an acute myocardial infarction. Which of the following is the most likely cause of this patients' pain? Increased sympathetic stimulation Increased diastolic pressure Increased systolic pressure Abnormalities of ion transport in myocytes Increased parasympathetic stimulation Accumulation of metabolic products in heart tissue

Increased sympathetic stimulation

A 55-year-old man comes to the physician for a routine health maintenance examination. His father and his older brother both died of a myocardial infarction at the ages of 50 and 55 years, respectively. The patient is 175 cm (5 ft 9 in) tall and weighs 89 kg (196 lb); BMI is 29 kg/m2. Physical examination shows no other abnormalities. Serum studies show a total cholesterol concentration of 300 mg/dl, HDL-cholesterol concentration of 40 mg/dl, and LDL-cholesterol concentration of 230 mg/dl. Treatment with lovastatin is begun. This treatment is most likely to result in which of the following adaptive responses at the cellular level in this patient? Decreased hepatic expression of LDL-cholesterol receptors Increased transcription of HMG-CoA reductase Decreased transcription of HMG-CoA reductase Increased mevalonic acid synthesis Increased mevalonic acid degradation

Increased transcription of HMG-CoA reductase

Treatment of inherited NDI includes adequate free water intake, restriction of dietary salt and protein, and administration of the diuretic hydrochlorothiazide. Which of the following best explains the rationale behind the administration of a diuretic to a person that cannot concentrate their urine? Reduce serum Na+ Increased water reabsorption along the collecting duct Increased water reabsorption along the proximal tubule Decreased water reabsorption along the proximal tubule Reduce urine Na+

Increased water reabsorption along the proximal tubule

A 45-year-old man comes to the physician for a preemployment examination. He has a 10-year history of hypertension and does not adhere to his medication regimen. His temperature is 37°C (98.6° F), pulse is 80/min, respirations are 14/min, and blood pressure is 160/100 mm Hg. Cardiac examination shows that the apical impulse is displaced laterally. A loud S2 and an S4 gallop are heard on auscultation. Echocardiography shows thickening of the left ventricular wall. The most likely mechanism causing the changes in this patient's left ventricle is best represented by which of the following changes in Transcription Factor c-Jun; beta-myosin heavy chain; Endothelin? Increased, increased, increased

Increased, increased, increased

A 63-year-old man comes to the physician because of an abnormal blood pressure reading taken during a health fair 1 week ago. He has no history of major medical illness. At a previous examination 6 months ago, his blood pressure was 135/85 mm Hg. His blood pressure today is 170/98 mm Hg. A bruit is heard over the left renal artery. Spiral CT angiography shows left arterial stenosis. Which of the following sets of changes in (TPR, renin, and aldosterone) is most likely in this patient? Increased, increased, increased

Increased, increased, increased

During a study of gastric parietal cells, an investigator attempts to elicit maximum hydrochloric acid secretion from the stomach of an experimental animal. Which of the following combinations of substances is most likely to lead to this desired effect? Acetylcholine, Gastrin, Histamine, Secretin Decreased decreased decreased decreased Increased decreased decreased increased Increased increased increased increased Decreased increased increased increased Increased, increased, increased, decreased Decreased decreased increased increased

Increased, increased, increased, decreased

A 66-year-old man is brought to the emergency department 30 minutes after the sudden onset of substernal chest pain that radiates to the neck and left arm; the pain is associated with weakness, nausea, and profuse sweating. He was jogging during the onset of pain. He has a 5-year history of hypertension treated with indapamide. His pulse is 90/min, and blood pressure is 150/90 mm Hg. Cardiac examination shows an S4. Treatment with a sublingual medication resolves his pain within 2 minutes. Which of the following mechanisms of action of the drug most likely led to the relaxation of vascular smooth muscle in this patient? Decreasing intracellular calcium Increasing cGMP Stabilizing depolarization of the vascular smooth muscle cell membrane Inhibiting the sodium pump Increasing cAMP

Increasing cGMP

A 20-year-old man is brought to the physician because of a 4-hour history of abdominal pain, nausea, and vomiting. He says that he had been drinking ethanol heavily all weekend, and he took three doses of acetaminophen within 2 hours after the onset of a severe headache Monday morning. This patient is at increased risk for liver injury because of which of the following actions of ethanol? Decreased acetaminophen clearance via glucuronidation Metabolic acidosis due to an increased NADH:NAD+ ratio Increased bioavailability of acetaminophen Induction of cyt p450 enzymes that activate acetaminophen to a hepatotoxic metabolite Activation of lgE-mediated mast cell degranulation

Induction of cyt p450 enzymes that activate acetaminophen to a hepatotoxic metabolite

A previously healthy 42-year-old Asian woman is brought to the emergency department because of a 24-hour history of nausea, vomiting, and progressive lethargy. She has smoked 1 pack of cigarettes daily for 25 years and drinks four glasses of wine daily. She uses high-dose acetaminophen daily for headaches. She does not use illicit drugs. She is 155 cm (5 ft 1 in) tall and weighs 50 kg (110 lb); BMI is 21 kg/m2. She is responsive to painful stimuli. Initial laboratory studies show increased hepatic aminotransferase. Which of the following effects of alcohol most likely contributed to this patient's condition? Increased hepatic glutathione stores Induction of cytochrome p450 Decreased generation of N- acetyl-p-benzoquinoneimine Increased sulfation Increased glucuronidation

Induction of cytochrome p450

A 42-year-old man comes to the emergency department because of a 10-day history of progressive fever, shortness of breath, and nonproductive cough. He also has had a 20-kg (44-lb) weight loss during the past 2 years. He immigrated to the USA from the Ivory Coast 4 years ago. His temperature is 38°C (100.4 "F), and respirations are 32/min. The lungs are clear to auscultation. A chest x-ray shows a diffuse interstitial infiltrate. A silver stain of a specimen obtained via bronchoscopy shows Pneumocystis jiroveci (formerly P. carinii). Treatment with high dose prednisone and trimethoprim-sulfamethoxazole is initiated, and workup for HIV infection is done. Results show: Which of the following best explains these laboratory results? Infection with HIV-2

Infection with HIV-2

A 27-year-old woman is brought to the emergency department 30 minutes after she was ejected through the windshield during a motor vehicle collision. She was the unrestrained front-seat passenger. Physical examination shows marked edema and tenderness of the jaw. A Panorex x-ray of the mouth is shown. Which of the following additional structures is most likely injured by this abnormality? Inferior alveolar nerve Parotid gland Levator labii superioris Tongue Maxillary artery

Inferior alveolar nerve

A 21-year-old man comes to the emergency department because of excruciating anal pain for the past 4 hours. Examination shows a 15-mm, blue-tinged, rounded mass at the anal margin. This mass most likely represents a thrombosis in a tributary of which of the following blood vessels? Superior rectal artery Superior rectal vein Middle rectal artery Inferior rectal vein Middle rectal vein Inferior rectal artery

Inferior rectal vein

A 3-year-old girl with Down syndrome is brought to the physician because of a 1-week history of frequent nosebleeds, decreased appetite, and lethargy. She takes no medications. Her temperature is 38°C (100.4°F), pulse is 100/min, respirations are 20/min,and blood pressure is 80/45 mm Hg. Examination shows no other abnormalities except for pallor. Laboratory studies show:Hemoglobin 6.5 g/dL Hematocrit 19%Leukocyte count 100,000/mm3 Segmented neutrophils 15%Atypical lymphocytes 85%Platelet count 45,000/mm3 Which of the following is the most likely mechanism of these laboratory findings? Bone marrow hypoplasia Iron deficiency Reticuloendothelial phagocytosis of platelets Serum immunoglobulin A deficiency Infiltration of bone marrow by leukemic blasts Lymphocyte inhibitors of erythropoiesis

Infiltration of bone marrow by leukemic blasts

A 20-year-old woman at 27 weeks' gestation is admitted to the hospital because of a 12-hour history of intense uterine contractions occurring every 8 minutes. Her membranes ruptured 32 hours ago. Her temperature is 39.1 °C (102.4 °F), and pulse is 115/min. Physical examination shows tenderness of the uterus. Pelvic examination shows a closed cervix that is not effaced. The fetal heart rate is 210/min. Which of the following is the primary stimulus for her uterine muscle contractions? Maternal adrenocorticosteroid release Direct response to maternal hyperthermia Decreased myometrial intracellular calcium Inflamed maternal decidua release of prostaglandin Stressed fetal production and release of oxytocin

Inflamed maternal decidua release of prostaglandin

A 46-year-old woman comes to the physician because of a 1-week history of low-grade fever and joint pain. She has chronic headaches for which she takes ibuprofen several times daily. Physical examination shows a diffuse maculopapular rash. Urinalysis shows 2+ protein, 10-20 WBC/hpf, and eosinophils. A renal biopsy specimen is most likely to show which of the following in this patient? Inflammatory infiltrates in the interstitium Mesangial cell proliferation Crescent-shaped glomeruli Sclerosed glomeruli Subepithelial deposits in the glomerular capillary walls

Inflammatory infiltrates in the interstitium

A previously healthy 67-year-old woman comes to the physician because of a 3-day history of fever, shortness of breath, nonproductive cough, headache, and muscle pain. Her temperature is 38.9°C (102° F). The lungs are clear to auscultation. Examination shows no other abnormalities. Which of the following vaccines is most likely to have prevented these findings? Measles-mumps-rubella Influenza vaccine Diphtheria-tetanus-acellular pertussis Typhoid Hepatitis B Inactivated poliovirus Varicella 23-Valent pneumococcal Haemophilus influenzae type b

Influenza vaccine

A 35-year-old man comes to the physician because of a 4-day history of a high-grade fever, severe muscle aches, malaise, loss of appetite, and a nonproductive cough. His wife and his two children had a similar illness. His temperature is 39.2°C (102.5°F). Physical examination shows no other abnormalities. A complete blood count and a chest x-ray show normal findings. The causal virus replicates its genome within the cell's nucleus. Which of the following is the most likely causal organism? Vaccinia virus Reovirus Poliovirus Rabies virus Respiratory syncytial virus Influenza virus

Influenza virus

A 24-year-old woman was stung on her right thumb by a bee 30 minutes ago. She develops an urticarial lesion at the site of the sting. Six hours later, the area has become indurated and firm. Which of the following explains the induration at the site of the sting? Vasoconstriction Influx of macrophages producting IL-1, IL-6, and tumor necrosis factor alpha Decrease in serum C-reactive protein concentration Lysis of endothelial cells by the alternative complement pathway Decrease in the expression of adhesion molecules on vascular endothelial cells

Influx of macrophages producting IL-1, IL-6, and tumor necrosis factor alpha

A 63-year-old man with mild emphysema who has smoked 1 pack of cigarettes daily for 45 years has the sudden onset of headache, myalgia, and rising temperature. A dry cough develops, with chills and chest pain. Examination of Gram stain of sputum discloses neutrophils, but bacteria are not evident. An x-ray of the chest is consistent with severe pneumonia. A culture of the sputum is negative, but a culture of a bronchoscopy specimen on a highly specialized bacteriologic medium yields gram-negative rods (intracellular). The identity of these bacteria is confirmed by staining with specific fluorescent antibodies (legionella). The patient responds to therapy with a macrolide antibiotic (what bacteria). This infection was most likely acquired from which of the following? Inhalation of aerosols from an environmental source Inhalation of respiratory secretions from an infected animal Inhalation of respiratory secretions from another infected person The patient's normal flora Inhalation of dust from bird droppings

Inhalation of aerosols from an environmental source

One week after a "breathing treatment" in the emergencydepartment for an initial episode of coughing andwheezing, a 10-year-old girl is brought to the physician fora follow-up examination. She has a 3-year history of nasalallergies; both her parents have allergic rhinitis. She isafebrile. The lungs are clear to auscultation. An x-ray ofthe chest shows normal findings. Which of the followingmedications is most appropriate on an as-needed basis? Oral β-adrenergic agonist Inhaled corticosteroids Inhaled β-adrenergic agonist Inhaled cromolyn sodium Oral theophylline

Inhaled β-adrenergic agonist

A 43-year-old man comes to the physician because of progressive shortness of breath during the past year. He is a stockbroker and has had no environmental exposure to toxins. When asked about his smoking habits, he says he smoked marijuana in college and afterwards, until the age of 28 years. He has not smoked any substance since then. He was adopted as a child and does not know his family history. His respirations are 29/min. Diffuse wheezing is heard on auscultation of the chest. Abdominal examination shows mild hepatomegaly. Pulmonary function tests show a decreased FEV 1 and decreased diffusing capacity. A chest x-ray shows hyperinflation consistent with air trapping mostly in the lung bases. A CT scan of the chest is shown. Which of the following is the most likely cause of these findings? History of smoking marijuana sprayed with a toxic herbicide Continuous marijuana use Asthma Diffuse idiopathic interstitial fibrosis Inherited protease deficiency

Inherited protease deficiency

A 42-year-old man comes to the physician because of a 4-week history of muscle cramping and pain. Two months ago, he began treatment with simvastatin (80 mg daily) for hypercholesterolemia. After 1 month, marked improvement was noted in his serum LDL-cholesterol concentration, but serum triglyceride concentration remained increased. At that time, gemfibrozil was added to his regimen to decrease his triglyceride concentration. Physical examination today shows no abnormalities. This patient's myalgia is most likely related to which of the following effects of gemfibrozil on simvastatin? Decreased bioavailability Inhibition of hepatic glycosylation Increased absorption Inhibition of cytochrome P450 metabolism Inhibition of hepatic sulfation

Inhibition of cytochrome P450 metabolism

A 67-year-old man has urinary urgency after placement of a urinary bladder catheter during transurethral resection of the prostate for carcinoma. The most appropriate therapy is a drug with which of the following actions? Bladder contracting>alpha antagonist/agonist Inhibition of muscarinic receptors Activation of adenosine receptors Activation of nicotinic receptors Inhibition of adenosine receptors Activation of muscarinic receptors Inhibition of nicotinic receptors

Inhibition of muscarinic receptors

A 50-year-old woman comes to the physician because of a 1-year history of hot flashes and irregular menses. Bone mineral density is decreased. As part of her therapy, alendronate is prescribed. Which of the following is the mechanism of the beneficial action of this drug in this patient? Stimulation of calci tonin secretion Stimulati on of estrogen secretion Inhibition of osteoclast mediated bone resorption Inhibiti on of 1,25-dihydroxycholecalci ferol hydroxylation Inhibition of parathyroid hormone secretion Stimulation of osteoblast-mediated bone formation

Inhibition of osteoclast mediated bone resorption

A 68-year-old man comes to the physician because of a 6-month history of erectile dysfunction. Physical examination and laboratory evaluation show no abnormalities. If pharmacotherapy is indicated, the most appropriate is a drug with which of the following mechanisms of action? Stimulation of 13 1-adrenergic receptors Blockade of a2-adrenergic receptors Inhibition of nitric oxide release Inhibition of phosphodiesterase Increased activity of 5a-reductase

Inhibition of phosphodiesterase

A 76-year-old man with congestive heart failure comes to the physician because of a 1-month history of shortness of breath after walking up a flight of stairs. He has had frequent nausea, and he has had to use two pillows to sleep during this period. Diffuse, moist crackles are heard over both lungs. There is pitting edema above the ankles. Which of the following best describes the function of the product secreted by the atrial myocytes in this patient? Decreased glomerular filtration rate Inhibition of renin release Stimulation of aldosterone release Induction of vasoconstriction Increased sodium reabsorption

Inhibition of renin release

A 45-year-old man is diagnosed with gastroesophagealreflux disease. Omeprazole treatment is initiated. Which ofthe following is the mechanism of action of this drug? Binds to H2 receptors Inhibits H+-K+ ATPase Stimulates serotonin receptors Creates a cryoprotective layer in the stomach Prevents absorption of dietary acids

Inhibits H+-K+ ATPase

A 38-year-old homeless man comes to the emergency department because of stomach cramps for 2 hours. He appears drunk and says that he began feeling ill after ingesting a homemade alcoholic beverage provided by a friend. His respirations are 33/min. Physical examination shows no other abnormalities. Fifteen minutes after arrival, he has visual disturbances. Laboratory studies show: Serum HC03-: 5 mEq/L Anion gap: 40 mEq/L (N=8-16) Osmolality: 370 mOsmol/kg H20 Arterial blood gas analysis on room air: pH: 7.10 Pco2: 18 mm Hg Po2: 120 mm Hg A loading dose of an antidote is administered. Which of the following best describes the mechanism of action of the antidote in this patient? Blocks tubular reabsorption and enhances urinary elimination of methanol Enhances metabolism of methanol by alcohol dehydrogenase and cytochrome P450 3A4 Inhibits alcohol dehydrogenase, blocking the conversion of methanol to formaldehyde Inhibits cytochrome P450 3A4, blocking the conversion of formaldehyde to formic acid Binds methanol in the gastrointestinal tract and decreases its absorption

Inhibits alcohol dehydrogenase, blocking the conversion of methanol to formaldehyde

A 33-year-old woman comes to the physician for a follow-up examination. Three weeks ago, she underwent oophorectomy because of epithelial ovarian cancer. At today's visit, the physician recommends adjuvant chemotherapy including paclitaxel. Which of the following best describes the mechanism of action of paclitaxel in this patient? Prevents microtubule polymerization Alkylates DNA Destroys asparagine Inhibits dihydrofolate reductase Inhibits microtubule disassembly

Inhibits microtubule disassembly

A hormone is known to activate phospholipase C with subsequent release of calcium from internal stores. The release of calcium most likely occurs as a result of an increase in the concentration of which of the following intracellular second messengers? Inositol 1,4,5-trisphosphate cAMP Calcium cGMP Diacylglycerol

Inositol 1,4,5-trisphosphate

A 79-year-old man is brought to the emergency department by his wife 30 minutes after he lost consciousness for 30 seconds. On arrival, he is alert, but he says that he is dizzy. There is no urinary or fecal incontinence. His pulse is 40/min and regular, and blood pressure is 92/56 mm Hg. Physical examination shows no evidence of tongue biting. The lungs are clear to auscultation. Cardiac examination shows a variable intensity S1. He is oriented to person, place, and time. An ECG shows a third-degree atrioventricular block. Which of the following is the most appropriate next step in management? Cardiac catheterization with stent placement Transthoracic echocardiography Insertion of transvenous pacemaker Transesophageal echocardiography Cardiac catheterization with angioplasty Insertion of pulmonary artery catheter

Insertion of transvenous pacemaker

A 47-year-old man comes to the physician because of a 4- week history of increased thirst and urination. He has had a 23-kg (50-lb) weight gain during the past 2 years. He has no history of serious illness and takes no medications. His mother and maternal grandfather have type 2 diabetes mellitus. The patient does not smoke and drinks one beer every night. He is 175 cm (5 ft 9 in) tall and now weighs 104 kg (230 lb); BMI is 34 kg/m2. His pulse is 90/min, and blood pressure is 150/88 mm Hg. The remainder of the examination shows no abnormalities. His serum glucose concentration is 330 mg/dL. Which of the following is the most likely underlying cause of this patient's increased serum glucose concentration? Pancreatic cancer Chronic pancreatitis Autoimmune destruction of islet cells Exogenous production of corticosteroids Insulin resistance

Insulin resistance

A 58-year-old man with supraventricular tachyarrhythmia refractory to pharmacotherapy undergoes ablation of an accessory excitatory pathway in the atrial endocardium. Which of the following areas should be avoided to leave the sinoatrial (pacemaker) node intact in this patient? Interatrial septum near the opening of the coronary sinus Within the fossaovalis Junction of the superior vena cava and right atrium The opening of the inferior vena cava as it enters the right atrium The joining of the pulmonary vein and the left atrium Inferior to the opening of the coronary sinus

Interatrial septum near the opening of the coronary sinus

A 6-year-old boy is brought to the physician by his mother because of a 2-day history of fever, sore throat, and occasional vomiting. His temperature is 39°C (102.2°F). Physical examination shows erythema and purulent exudate over the posterior pharyngeal wall and tonsils. The anterior cervical glands are enlarged and tender. A throat culture grows group A Streptococcus. Because he is allergic to penicillin, treatment with erythromycin is begun. His condition improves during the next 2 days. The effectiveness of this antibiotic is most likely due to which of the following actions in protein synthesis? Interferes in aminoacyl-tRNA binding Inactivates elongation factor-2 Functions as an analog of aminoacyl-tRNA Interferes with translocation Causes misreading of mRNA i nformation

Interferes with translocation

A 54-year-old woman who works as a nurse comes to the physician for routine tuberculosis screening. A sample of her whole blood is incubated with a mixture of synthetic peptides representing two proteins of the suspected pathogen. Three days later, supernatant is collected for evaluation. Measurement of which of the following cytokine concentrations is most likely to indicate a positive result? Interferon alpha Interleukin-4 (IL-4) Interferon gamma Granulocyte-macrophage colony-stimulating factor IL-10 Transforming growth factor

Interferon gamma

A 3-year-old girl is brought to the physician because of a 2-week history of diarrhea. Her temperature is 37.6°C (99.8°F), pulse is 70/min, respirations are 18/min, and blood pressure is 110/70 mm Hg. Physical examination shows generalized lymphadenopathy. A CT scan of the chest and abdomen shows enlarged lymph nodes in the mesentery and para-aortic region. Examination of a lymph node biopsy specimen shows marked proliferation of histiocytes and numerous segmented neutrophils. Granulomata are absent, and special stains show numerous acid- fast bacilli, which are subsequently identified as Mycobacterium avium- intracellular. Serum studies show normal concentrations of lgA, lgG, lgM, B lymphocytes, T lymphocytes, and CD4+ and CD8+ T lymphocytes. This patient most likely has defective function or expression of which of the following proteins? Interferon gamma receptor Class I MHC molecules Leukocyte function- associated antigen-1 lnterleukin-2 (IL-2) receptor NADPH oxidase

Interferon gamma receptor

A 20-year-old man has cramping peri umbilical pain. After several hours, the pain shifts to the right lower quadrant and becomes constant. His temperature is 40°C (104°F), and his leukocyte count is 20,000/mm3 with 90%, neutrophils. Which of the following mediators is most likely causing this patient's fever and leukocytosis? Leukotriene C4 Leukotriene B4 lnterleukin-1 (IL-1) Interleukin-1 (IL-1) lnterleukin-2 (IL-2) Prostaglandin D2

Interleukin-1 (IL-1)

A 77-year-old woman dies in the hospital after a long illness. Her vertebral column, obtained at autopsy, is shown in the photograph. The process shown is most likely associated with an increase in which of the following? Calcium Estrogen Interleukin-1 (IL-1) Vitamin D Monoclonal immunoglobulin

Interleukin-1 (IL-1)

A previously healthy 3-year-old boy is diagnosed with bacterial colitis caused by Salmonella enterica serovar arizonae. Which of the following factors best accounts for the recruitment of neutrophils to the inflammatory site by intestinal epithelial cells in this patient? Toll-like receptor-3 Interleukin-a (IL-8) Histamine Granulocyte colony-stimulating factor Leukocyte function-associated antigen-1

Interleukin-a (IL-8)

A 22-year-old woman, gravida 1, para 1, is brought to the emergency department because of a 2-day history of fever and severe vaginal bleeding. Four days ago, she delivered a healthy male newborn spontaneously at term. Her temperature is 38.1 C (100.6°F). Pelvic examination shows an open cervix and heavy vaginal bleeding. Ultrasonography of the uterus shows no retained placental tissue or large thrombi. If an operation is required to control the bleeding, ligation of a branch of which of the following arteries is most appropriate? Internal iliac Internal pudendal Obturator Median sacral External iliac

Internal iliac

A 32-year-old woman comes to the physician for a follow-up examination after atypical cells were noted on a recent Pap smear. Physical examination shows a 1x1-cm area of leukoplakia on the cervix. A biopsy specimen of the lesion shows invasive squamous cell carcinoma. Malignant cells from this site will most likely drain first to which of the following lymph nodes in this patient? Lumbar Inferior mesenteric Internal iliac Superficial inguinal Femoral

Internal iliac

An 18-year-old woman comes to the physician because of progressive fever, general malaise, and blood in her urine since she began oral antibiotic therapy for a urinary tract infection 5 days ago. She also has a 3-day history of a rash. Her temperature is 38°C (100.4° F), pulse is 75/min, respirations are 12/min, and blood pressure is 125/80 mm Hg. Physical examination shows a petechial rash over the chest, back, and upper and lower extremities. Urinalysis shows: Which of the following is the most likely diagnosis? Acute tubular necrosis Glomerulonephritis Interstitial nephritis lgA nephropathy Papillary necrosis

Interstitial nephritis

A 5-year-old girl is brought to the physician because of increasingly severe pain of her left leg 1 day after she fell off a sled. She recently immigrated to the USA from Iceland with her family. She has a history of three bone fractures since birth. Her parents are vegan and do not give the patient any dairy products. Physical examination shows tenderness, swelling, and limited range of motion of the left lower extremity. An x-ray of the left lower extremity shows a fracture of the fibula. This patient most likely has a vitamin deficiency that directly affects which of the following processes? Collagen synthesis Intestinal calcium absorption Osteoclastic activity Parathyroid hormone production Folate metabolism

Intestinal calcium absorption

A 73-year-old female presents to the emergency room after collapsing at home. She is 5' 4" and weighs 198 lbs with a BMI of 34. Medical records reveal a long-standing history of hypertension. She reported experiencing severe, excruciating, stabbing-like chest pain that radiated to her left shoulder before she collapsed. Upon admission, her pulse is 112/min and blood pressure is 162/84 mm Hg. Auscultation reveals distant heart sounds and a drop in systolic blood pressure of >10 mm Hg upon inspiration. ECG reveals ST-T wave changes. Chest X-ray shows a widening of the mediastinum. Which of the following most likely represents the mechanism of injury? Intimal tear

Intimal tear

A 67-year-old woman is hospitalized because of abdominal pain and persistent copious vomiting for 24 hours. Two weeks ago, she was hospitalized for treatment of atrial fibrillation; after cardioversion to a normal sinus rhythm, she began treatment with warfarin. Yesterday, at a follow-up visit, her INR was 6 (?? normal), and her medication was discontinued. She takes no other medications. Her temperature is 37°C (98.6°F), pulse is 120/min and regular, respirations are 20/min, and blood pressure is 100/78 mm Hg. The abdomen is distended and moderately tender; there is voluntary guarding in the epigastrium (??? source of pain) There are no masses, organomegaly, or obvious hernias. Rectal examination shows no abnormalities. Test of the stool for occult blood is negative. Her hemoglobin concentration has decreased from 13 g/dL yesterday to 7.8 g/dL today. An ECG shows a normal sinus rhythm. Which of the following is the most likely explanation for this patient's abdominal symptoms? Intramural hematoma of the proximal small bowel Internal small-bowel herniation Malrotation of the small bowel Intestinal ischemia from a cardiac embolus Intussusception of the small bowel

Intramural hematoma of the proximal small bowel

A 45-year-old man is admitted to the hospital because of congestive heart failure. Two first-degree relatives died in their 40s with dilated cardiomyopathy and cirrhosis. Physical examination shows cardiac enlargement and generalized hyperpigmentation. His serum glucose concentration is 320 mg/dl. Histologic examination of endomyocardial tissue is most likely to show an excess of which of the following? Iron Mucopolysaccharide Amyloid a1 -Antitrypsin Eosinophils Copper Cerebroside Lysosomal glycogen

Iron

A 38-year-old woman is brought to the emergency department after being found lying on the sidewalk. She is conscious but says she is too weak to stand. Physical examination shows a red, swollen, tender tongue; angular stomatitis; and spoon-shaped nails. Hemoglobin and mean corpuscular volume are decreased. Which of the following is the most likely diagnosis? Vitamin B 1 (thiamine) deficiency Sickle cell disease A) Vitamin B 12 (cobalamin) deficiency Folic acid deficiency Iron deficiency

Iron deficiency

A 22-year-old woman comes to the physician because of a 1-year history of intermittent lower abdominal cramps associated with bloating and mild nausea. The cramps are occasionally associated with constipation and bowel movements relieve the pain. She has not had any other symptoms. She has no history of serious illness and takes no medications. Her last menstrual period was 2 weeks ago. She appears well. Abdominal examination shows mild diffuse tenderness to palpation. Which of the following is the most likely diagnosis? Peptic ulcer disease Meckel diverticulum Irritable bowel syndrome Ulcerative colitis Crohn disease

Irritable bowel syndrome

During an experiment on the cough reflex in humans, avolunteer inhales air containing different amounts ofparticles that will impact and adhere to mucus primarily inthe trachea. Blockade of which of the following receptorswould most likely prevent this volunteer's reflex to initiate acough? Chemoreceptors Stretch receptors Proprioceptors Irritant receptors J receptors

Irritant receptors

A 72-year-old woman comes to the emergency department because of bloody diarrhea for the past 12 hours and diffuse abdominal pain for the past 4 days. History includes a cerebral infarction 3 years ago from which she recovered completely. Examination shows watery stool with red blood. Colonoscopy shows patchy ulceration at the area near the splenic flexure. Which of the following is the most likely diagnosis? Ischemic colitis Ulcerative colitis Pancreatic insufficiency Celiac sprue Irritable bowel syndrome Regional enteritis

Ischemic colitis

A 5-year-old boy is brought to the physician by his mother because of progressive clumsiness and fatigue during the past 6 months. He says that his legs are tired. He was delivered at term after an uncomplicated pregnancy. He has met all developmental milestones, although there was some delay compared with other child ren his age. He is alert. He has difficulty rising from the chair; he uses his arms to push himself into a standing position. He is unable to jump with both feet together. Physical examination shows hypertrophy of the calf muscles. This patient most likely has weak hip adduction as a result of dysfunction of the muscle inserting onto the femur from which of the following locations? Ischium Lateral ilium Iliac crest Iliac spine Anterior sacrum

Ischium

A 52-year-old man is brought to the emergency department 30 minutes after the onset of chest pain and shortness of breath. He had played tennis all day, and he does not remember how much fluid he had consumed. His temperature is 36.7°C (98° F), pulse is 122/min, respirations are 28/min, and blood pressure is 90/50 mm Hg. Physical examination shows dry skin and decreased capillary refill. An ECG and evaluation of cardiac enzymes show no abnormalities. Which of the following findings in the nephron (proximal tubule, macula densa, and medullary collecting duct) best describes the tubular osmolality, compared with serum, in this patient? Isotonic; hypotonic; hypertonic

Isotonic; hypotonic; hypertonic

A 15-year-old girl is brought to the physician by her mother because of lack of improvement in facial acne. Topical clindamycin and benzoyl peroxide therapy was initiated 3 months ago. Physical examination shows severe acne vulgaris. The patient says, "Please help me. The medication you gave me isn't working, and I'm embarrassed to go out with my skin looking like this." She has never been sexually active. Which of the following drugs is the most appropriate treatment at this time? Methotrexate Metronidazole Triamcinolone Chloramphenicol Isotretinoin

Isotretinoin

Stomach Descending colon First part of the duodenum Sigmoid colon Jejunum

Jejunum

A 62-year-old man comes to the physician because of pain and swelling of the right foot for 24 hours. There is no history of trauma. One month ago, he was diagnosed with hypertension and started treatment with hydrochlorothiazide. His temperature is 37.3°C (99.1°F), pulse is 86/min, and blood pressure is 150/90 mm Hg. Examination shows tenderness, swelling, erythema, and warmth of the right first metatarsophalangeal joint; range of motion is decreased. There is mild hallux valgus. Pedal pulses and sensation are intact. He walks with an antalgic gait. X-rays of the right foot show mild joint space narrowing and periarticular bony erosions of the metatarsophalangeal joint. Which of the following is the most appropriate next step in diagnosis? Joint aspiration Technetium 99m bone scan MRI of the right foot Indium-labeled white blood cell scan Venous Doppler ultrasonography

Joint aspiration

A 62-year-old man comes to the physician because of painand swelling of the right foot for 24 hours. There is nohistory of trauma. One month ago, he was diagnosed withhypertension and started treatment withhydrochlorothiazide. His temperature is 37.3°C (99.1°F),pulse is 86/min, and blood pressure is 150/90 mm Hg.Examination shows tenderness, swelling, erythema, andwarmth of the right first metatarsophalangeal joint; rangeof motion is decreased. There is mild hallux valgus. Pedalpulses and sensation are intact. He walks with an antalgicgait. X-rays of the right foot show mild joint spacenarrowing and periarticular bony erosions of themetatarsophalangeal joint. Which of the following is themost appropriate next step in diagnosis? Venous Doppler ultrasonography Technetium 99m bone scan Indium-labeled white blood cell scan Joint aspiration MRI of the right foot

Joint aspiration

The joining of the pulmonary vein and the left atrium Junction of the superior vena cava and right atrium Within the fossaovalis Inferior to the opening of the coronary sinus Interatrial septum near the opening of the coronary sinus The opening of the inferior vena cava as it enters the right atrium

Junction of the superior vena cava and right atrium

A 60-year-old man with no history of bleeding problems has coagulation testing prior to coronary artery bypass grafting. His prothrombin time is 11.5 seconds (IN R=1.0), and activated partial thromboplastin time is 160 seconds. Which of the following inflammatory responses is most likely to be abnormal in this patient? CSa generation Platelet aggregation Histamine release Kallikrein formation Phagocytosis

Kallikrein formation

A 12-year-old African American boy is brought to thephysician by his mother because of a swollen right earlobefor 3 weeks. He had the ear pierced at a local mall 6 weeksago. The swelling has persisted despite removal of theearring 3 weeks ago. The mother developed a thick rubberyscar on her abdomen after a cesarean delivery 12 years ago.His temperature is 37°C (98.6°F). Physical examinationshows a nontender, flesh-colored swelling of the rightearlobe. The skin is intact over the swelling, and it is softand nontender. There is no cervical lymphadenopathy.Which of the following is the most likely cause of theswelling in this patient? Keloid Lipoma Foreign body Contact dermatitis Bacterial infection

Keloid

A 30-year-old woman comes to the physician for a health maintenance examination. She is training for a marathon and has been running up to 20 miles daily. She states that as long as she has adequate caloric intake, she feels well on long-distance runs of up to 20 miles. Physical examination shows no abnormalities. Her fasting serum glucose concentration is 60 mg/dl. After her glucose stores have been depleted, which of the following organs, in addition to the liver, is most likely to release newly produced glucose in this patient? Thyroid gland Stomach Pancreas Adrenal glands Kidney

Kidney

A 65-year-old woman with a 20-year history of osteoarthritis of the hands now has pain radiating down the distal anterior thigh, knee, medial leg, and foot. Bony outgrowth of vertebrae compressing one of the spinal nerves is suspected. Compression of a nerve root in which of the following intervertebral foramina is the most likely cause of her symptoms? T-10 to 11 L3 to 4 S-4 to 5 L-1 to 2 S-1 to 2

L3 to 4

A 36-year-old man who is heterozygous for an LDL-receptor deficiency has a total serum cholesterol concentration of 330 mg/dL. After taking a drug that inhibits cholesterol synthesis, his total serum cholesterol concentration decreases to 200 mg/dL. Which of the following proteins would be upregulated as a result of drug therapy? LDL receptor Lysosomal cholesterol esterase Lipoprotein lipase Hepatic lipase Cholesterol acyltransferase

LDL receptor

A 15-year-old boy is brought to the emergency department2 hours after the onset of severe left scrotal pain that beganwhile he was playing ice hockey. There is no history of trauma.Over the past year, he has had several episodes of mild leftscrotal discomfort that have resolved spontaneously. The leftside of the scrotum is enlarged, erythematous, and exquisitelytender. The left testicle is higher in the scrotum compared withthe right. Which of the following is the most likely underlyingcause of this patient's condition? Presence of an appendix testis Overactive cremasteric reflex Lack of fixation of the testicle to the tunicavaginalis Patent processus vaginalis Failure of the testicle to fully descend

Lack of fixation of the testicle to the tunicavaginalis

A 32-year-old man comes to the physician because of a 12-day history of abdominal cramps and bloating, diarrhea, and flatulence. He says that he started a new exercise program 2 weeks ago and has been consuming a high quantity of yogurt bars, peanut butter, and protein- and calorie-enriched milk shakes to "bulk up." He has no history of serious illness and takes no medications. His temperature is 37°C (98.6°F). The abdomen is distended, nontender, and tympanitic to percussion. Bowel sounds are increased. The remainder of the examination shows no abnormalities. Which of the following is the most likely cause of this patient's symptoms? Allergy to peanuts Fungal overgrowth in the small bowel Incarcerated hernia Lactase deficiency Irritable bowel syndrome

Lactase deficiency

An 8-year-old boy is brought to the physician because of persistent aching pain on the back of his head since he fell off a swing 2 days ago. He has no vision difficulties. Physical examination shows a 2.5-cm tender, swollen mass over the right occiput. There is no adenopathy. Neurologic examination shows no abnormalities. A CT scan of the head shows an osteolytic and soft tissue mass in the skull, with mild, smooth inward displacement of the dura. A biopsy specimen of the lesion shows replacement of soft tissue by a sheet-like infiltrate of pale eosinophilic cells with bean-shaped nuclei; there are increased admixed eosinophils. The cells test positive for CD1 a on immunostaining. Transmission electron microscopy of the cells shows Birbeck granules. The most likely cause of these findings is an abnormal proliferation of which of the following cell types? Glial cells Langerhans cells Endothelial cells Squamous cells Meningeal cells

Langerhans cells

A 4-year-old male with a unique form of Charcot-Marie-Tooth syndrome is included in a study to identify the genetic cause of his variation of Charcot-Marie-Tooth syndrome. Linkage analysis, following by DNA sequencing, located the duplication to 17q11.2. Which of the following is most likely involved in this disorder? Large-segment duplications Short tandem repeats Tandem repeats Staellite Variable tandem repeats SINE

Large-segment duplications

Twenty subjects participate in a study of cardiopulmonary physiology. The pulmonary circulation of these subjects most likely has which of the following characteristics compared with their bronchial circulation? Larger volume of nutrient blood to the conducting airways Lower-volume, higher- pressure system Larger percentage of cardiac output Larger number of anastomoses with the intercostal arteries Greater number of branches supplying the visceral pleura

Larger percentage of cardiac output

A previously healthy 17-year-old boy is brought to the emergency department 1 hour after having a syncopal episode. He has had fever, nausea, muscle aches, and progressive confusion during the past 12 hours. His temperature is 39.7°C (103.5°F), and palpable blood pressure is 80 mm Hg. Physical examination shows a rash on the lower extremities and feet. His leukocyte count is 26,000/mm 3. Blood and cerebrospinal fluid cultures grow an oxidase-positive, gram-negative diplococcus. The family reports that the patient's older brother had a similar infection at the age of 16 years. The patient and his brother most likely have which of the following immune system disorders? X-linked hyper-lgM syndrome Late component of complement deficiency X-linked hypogammaglobulinemia Purine-nucleoside phosphorylase deficiency Adenosine deaminase deficiency

Late component of complement deficiency

70-year-old man comes to the emergency department 30 minutes after the onset of weakness, nausea, and lower abdominal and back pain that radiates to the lower extremities. He appears pale and has a cold sweat. Pulse is 110/min and blood pressure is 90/60 m m Hg. Pulses in the lower extremities are weak. A faint bruit is heard in the lower abdomen. Muscle stretch reflexes are normal in both lower extremities. Which of the following is the most likely diagnosis? Herniated L-3 to L-4 intervertebral disc Leaking aortic aneurysm Acute compression fracture of the L-3 vertebral body Leaking left renal artery aneurysm Perforated rectum Perforated sigmoid colon

Leaking aortic aneurysm

A 19-year-old man is admitted to the hospital following a motor vehicle collision. Physical examination shows a penetrating wound to the right cerebral cortex with complete paralysis of the left lower extremity, fracture of the right mid humerus with severing of the radial nerve, and a fracture of the right tibia. Treatment includes cast immobilization of the right upper extremity and right knee and ankle. After 10 weeks, the casts are scheduled to be removed from the right upper and lower extremities. At this point, the deep tendon reflex is most likely to be strongest in which of the following locations in this patient? Left biceps tendon Right patellar tendon Right brachioradialis tendon Left achilles tendon

Left achilles tendon

A 68-year-old man comes to the physician because of difficulty swallowing solids for 2 months. He has a history of dilated cardiomyopathy. X-rays of the esophagus with barium contrast show indentation and posterior displacement of the esophagus. Enlargement of which of the following structures is the most likely cause of the dysphagia? Right ventricle Right atrium Superior vena cava Left atrium Left ventricle

Left atrium

A 72-year-old woman is admitted to the hospital because of an acute myocardial infarction. She undergoes cardiac catheterization. Angiography shows a left dominant circulation, and a 90% narrowing of the artery supplying the diaphragmatic surface and atrioventricular node of the heart. A balloon angioplasty is scheduled during which a stent will be inserted in the narrowed vessel. The catheter and the balloon must be passed through which of the following vessels (stated in order) to reach the narrowed vessel? Left coronary, posterior interventricular (posterior descending), circumflex Right coronary, circumflex, anterior interventricular (left anterior descending) Left coronary, circumflex, posterior interventricular (posterior descending) Left coronary, circumflex, anterior interventricular (left anterior descending) Left coronary, circumflex, posterior descending Right coronary, posterior interventricular (posterior descending), circumflex Right coronary, circumflex, posterior interventricular (posterior descending)

Left coronary, circumflex, posterior descending

A full-term newborn has respiratory distress. Imaging studies show herniation of abdominal contents into the left pleural cavity. Maldevelopment of which of the following structures is most likely to have caused the defect of the diaphragm? Left pleuroperitoneal membrane Esophageal mesoderm Septum transversum Left pleuropericardial fold Left diaphragmatic crus

Left pleuroperitoneal membrane

A 75-year-old woman comes to the physician because of a 3-month history of an enlarging lesion on her face. Physical examination shows a 1.5-cm, brown-black, mottled, scaly lesion with irregular borders. Microscopic examination of a biopsy specimen of the lesion shows atypical melanocytes spread along the basilar layer of the epidermis. Which of the following is the most likely cause of these findings? Acanthosis nigricans Compound nevus Seborrheic keratosis Lentigo maligna Actinic keratosis

Lentigo maligna

A pharmaceutical company is trying to develop a long-acting weight-loss agent that mimics the activity of a naturally occurring peptide that originates in adipose tissue, signals the brain about the amount of stored fat, and suppresses appetite by its action in the central nervous system. Their best strategy for identifying useful peptides would be to assess the binding of candidate peptides to the receptor for which of the following chemical mediators? Neurotensin Y Serotonin Leptin Somatostatin Substance P Cholecystokinin

Leptin

A 51-year-old man who is a farmer comes to the physician 5 days after he cut his right foot while cleaning out his barn. He has a 15-year history of poorly controlled type 2 diabetes mellitus. His temperature is 38.5°C (101.3° F), pulse is 105/min, and blood pressure is 139/86 mm Hg. Examination of the right lower extremity shows erythema up to the mid-calf, edema, and exquisite tenderness of the ankle and the area around the laceration. His leukocyte count is 57,000/mm3• Which of the following best describes the response to this patient's injury? Leukoclastic response Leukemoid reaction Leukotriene Leukoplakia Leukopenia

Leukemoid reaction

An 18-month-old girl is brought to the physician for a follow-up examination. Separation of the umbilical cord was delayed after birth. She has had four severe skin infections caused by Staphylococcus aureus since birth; there has been no pus formation at the infection sites. Previous laboratory studies have shown persistent leukocytosis in the absence of infection. Physical examination shows edematous, erythematous gums. Which of the following mechanisms is most likely impaired in this patient? Generation of terminal membrane attack complex Leukocyte adhesion and transmigration Activation of T lymphocytes to alloantigens and mitogens Ability to generate reactive oxygen species Conversion of hydrogen peroxide to hypochlorous acid

Leukocyte adhesion and transmigration

A 10-year-old girl is brought to the emergency department by her parents 30 minutes after the sudden onset of progressive shortness of breath. She has had several similar episodes in the past 3 months. Her respirations are 24/min. Diffuse wheezes are heard on auscultation of the chest. Examination of a sputum sample shows numerous eosinophils and Charcot-Leyden crystals. Which of the following substances is involved in the pathogenesis of this patient's disorder? Elastase Interferon gamma Hydrogen peroxide Leukotriene C4 lnterleukin-10 (IL-10) Lactoferrin

Leukotriene C4

A 22-year-old man comes to the physician because of a 2-year history of episodes of shortness of breath and wheezing. His respirations are 12/min. Physical examination shows no abnormalities. It is found that the symptoms occur after he takes certain over-the-counter medications containing aspirin. Laboratory studies show a serum lgE concentration within the reference range and no eosinophilia. This patient's symptoms are most likely caused by excessive production of which of the following substances? Prostaglandins Leukotrienes Thromboxane A2 Hydroxyeicosatetraenoic acid Prostacyclin (PG12) Cyclooxygenase

Leukotrienes

A 65-year-old woman comes to the physician because of progressive vulvar itching over the past 2 months; treatment with miconazole for yeast infections has been ineffective. Examination shows atrophy of the labia minora and thin, parchment-like skin over the vulva and around the anus. Which of the following is the most likely diagnosis? Urethral prolapse Human papillomavirus infection Lichen sclerosus Candidal infection Trichomoniasis

Lichen sclerosus

A 56-year-old man is scheduled for repair of an inguinal hernia under spinal anesthesia. If the spinal needle is inserted posteriorly between the L-3 and L-4 vertebral arches, which of the following ligaments must be traversed to reach the spinal canal? Posterior longitudinal Anterior longitudinal Denticulate Costotransverse Ligamentum flavum

Ligamentum flavum

A 56-year-old woman is brought to the emergency department by her husband because of fever and shortness of breath for 2 hours. Her husband says that she also has had urinary frequency and pain with urination for 2 days. She is in respiratory distress. Her temperature is 38.7°C (101.7°F), pulse is 120/min, respirations are 30/min, and blood pressure is 80/50 mm Hg. Which of the following components of the causal organism is the most likely cause of the hypotension? Flagellar H antigen Lipopolysaccharide P pilus Polysaccharide capsule Shiga- like cytotoxin Lipopolysaccharide

Lipopolysaccharide

A 57-year-old man with alcoholism dies of Klebsiella pneumoniae pneumonia. Several abscess cavities filled with a purulent exudate are found at autopsy. Which of the following patterns of necrosis would be found in lung tissue within these cavities? Liquefaction Fibrinoid Gangrenous Fat Gaseous

Liquefaction

A 75-year-old man has a cerebral infarction. He dies 1 week later. Histologic examination of the brain is most likely to show which of the following types of necrosis? Fibrinoid Gangrenous Gaseous Liquefactive Coagulation

Liquefactive

A 30-year-old primigravid woman at 22 weeks' gestation comes to the physician because of a 1-day history of fever, chills, and muscle aches. Her temperature is 39.4 °C (102.9° F), pulse is 114/min, respirations are 15/min, and blood pressure is 104/72 mm Hg. Physical examination shows a uterus consistent in size with a 22-week gestation. Fetal heart sounds are heard. Her leukocyte count is 12,000/mm3. Blood cultures grow gram-positive rods. Which of the following is the most likely causal organism? Clostridium perfringens Staphylococcus aureus Streptococcus agalactiae (group B) Bacillus cereus Listeria monocytogenes

Listeria monocytogenes

A 38-year-old woman at 32 weeks' gestation is brought to the physician because of a 2-day history of fever, nausea, vomiting, and muscle aches. She appears acutely ill. Her temperature is 39°C (102.2°F), pulse is 120/min, and respirations are 20/min. A photomicrograph of a Gram stain of an organism recovered from a blood culture is shown. Which of the following is the most likely causal organism? Listeria monocytogenes Rhodococcus equi Staphylococcus aureus Corynebacterium urealyticum Gardnerella vaginalis

Listeria monocytogenes

A 48-year-old woman with ductal carcinoma of the right breast has a modified radical mastectomy. After the operation, she has winging of the scapula when she presses her outstretched arms against a wall. Which of the following nerves is most likely injured? Thoracodorsal Suprascapular Lower subscapular Long thoracic Axillary

Long thoracic

During the following experiment, limb bud development was monitored overtime to characterize limb patterning. Experiment A, the apical ectodermal ridge (AER) was removed. Experiment B, the AER is left intact. Which of the following was most likely to occur to the limb bud mesenchyme? Loss of FGF10 expression Loss of Tbx5 expression

Loss of FGF10 expression

A 54-year-old man has a squamous cell carcinoma of the right lung. Analysis of his germline DNA shows two alleles, m and n, at the microsatellite locus Z tightly linked to the p53 gene. Similar analysis of the neoplastic cells shows the presence of the mallele only. These findings are most likely due to which of the following processes? Loss of heterozygosity Pericentric inversion involving p53 and locus Z Point mutation of an allele p53 Gene amplification Germline mosaicism

Loss of heterozygosity

A 4-year-old girl has had "puffy" eyes for the past 3 days. She has had an upper respiratory tract infection for the past week. A urine dipstick test shows 3+ protein. Urinalysis shows no red or white blood cells or casts. Which of the following mechanisms is the most likely cause of this patient's condition? Damage to the proximal renal tubule Attack by antiglomerular basement membrane antibodies Loss of negative glomerular charge Damage to the loop of Henle Presence of a posterior urethral valve

Loss of negative glomerular charge

A 35-year-old man with Chagas disease comes to the emergency department because of a 2-hour history of moderate chest pain. He has had a 2-kg (4.4-lb) weight loss during the past 2 months. His pulse is 100/min, respirations are 25/min, and blood pressure is 135/90 mm Hg. An ECG shows no abnormalities. A barium swallow shows a dilated esophagus with beak-like narrowing at the level of the lower esophageal sphincter {LES). A biopsy specimen of the LES is most likely to show which of the following? Degeneration of motor end-plates Degeneration of smooth muscle Loss of neurons in the myenteric plexus Increased vasoactive intestinal polypeptide-like immunoreactivity Absence of anticholinesterase-like immunoreactivity

Loss of neurons in the myenteric plexus

A 20-year-old man comes to the physician because of a 6-month history of diarrhea and a 16-kg (35-lb) weight loss. He is 186 cm (6 ft 1 in) tall and now weighs 75 kg (166 lb); BMI is 22 kg/m2. Physical examination shows pallor and pitting edema of the lower extremities. Laboratory studies show microcytic hypochromic anemia, hypoalbuminemia, and an increased fat concentration in stool. Serum anti-endomysium lgA and anti-tissue transglutaminase lgA antibodies are positive. A biopsy specimen of the small intestine is most likely to show which of the following in this patient? Loss of villi and increased number of intraepithelial lymphocytes Eosinophil infiltration of lamina propria and mucosa Macrophages containing periodic acid-Schiff-positive bacilli in lamina propria Normal villi with epithelial cells vacuolated with fat Granulomas in bowel wall and serosa

Loss of villi and increased number of intraepithelial lymphocytes

A 3-year-old boy with chronic otitis media is scheduled to undergo bilateral myringotomies with placement of polyethylene tubes under general anesthesia. He is asleep after only three to five breaths of inhalational anesthetic delivered by face mask. Which of the following properties of the anesthetic is the most likely explanation for the rapid onset of action? Low minimum alveolar concentration Low heptic metabolism High minimum alveolar concentration High blood solubility Low blood solubility High hepatic metabolism

Low blood solubility

A 7-year-old boy is about to undergo an appendectomy. An intravenous catheter needs to be inserted, but the patient is extremely fearful of being stuck with a needle while awake. The most appropriate anesthesia administered by mask to anesthetize this patient quickly would have which of the following characteristics? Low blood solubility Low lipid solubility High blood solubility High lipid solubility High cerebrospinal fluid solubility

Low blood solubility

A 43-year-old man comes to the physician for a routine health maintenance examination. He is 170 cm (5 ft 7 in) tall and weighs 86 kg (188 lb); BMI is 30 kg/m2. Physical examination and laboratory studies show no other abnormalities. He tells the physician, "My older brother just got diagnosed with diabetes. I don't want that to happen to me. What should I do?" Which of the following diets is most likely to be effective in decreasing this patient's risk for type 2 diabetes mellitus? Low-protein Low-calorie Low-cholesterol Low-carbohydrate Low-sodium

Low-calorie

A 16-year-old boy is brought to the physician by his father because he has no signs of puberty. Sexual development is Tanner stage 2. Physical examination shows a circumcised penis. The testes are soft and small with a volume of 5 ml (N=20-30). The prostate is firm, nontender, and of appropriate size. There are no lesions or discharge. His serum testosterone concentration is in the prepubertal range. Measurement of which of the following hormones in serum is most likely to determine whether the decreased serum testosterone concentration is a result of a lack of pituitary hormone stimulus in this patient? Follicle-stimulating hormone Luteinizing hormone Estrogen lnhibin Adrenocorticotropic hormone

Luteinizing hormone

A 33-year-old woman comes to the physician 3 days after she found a mass in her right breast on self-examination. Examination of the right breast shows a 2.5-cm, hard mass in the upper outer quadrant of the right breast. A photograph of the resected mass is shown. Which of the following characteristics of the lesion best predicts this patient's prognosis? Presence or absence of hormone receptors within the lesion DNA content of the cells comprising the lesion Degree of differentiation of the cells comprising the lesion Lymph node involvement by cells originating in the lesion Size of the lesion

Lymph node involvement by cells originating in the lesion

A 45-year-old woman comes to the physician because of a 4-month history of joint pain, muscle aches, and fatigue. She was treated for a renal calculus 6 months ago, and she underwent cholecystectomy 1 year ago. She appears fatigued and depressed. Physical examination shows mucosal pallor. All extremities have full range of motion, but movement produces mild pain. No masses are palpated in the neck or abdomen. Laboratory studies show: Erythrocyte count 3 million/mm3 Serum K + CICa2+ Phosphorus Alkaline phosphatase 4 mEq/L 106 mEq/L 13 mg/dl 2.7 mg/dl 125 U/L A technetium-99m tetrofosmin scan shows a single 1.2-cm nodule in the neck region. The pathologic process in this patient is most likely initiated by a mutation in which of the following genes? NF1 BRCA 1 MLH1 MEN1 EGFR

MEN1

A 30-year-old woman with a long-standing history of pelvic inflammatory disease has surgical resection of a scarred segment of a fallopian tube. Which of the following inflammatory cells is most likely to be found in the resected specimen? Macrophages Neutrophils Eosinophils Basophils Mast cells

Macrophages

A 34-year-old patient with AIDS develops pulmonary tuberculosis. No CD4+ T lymphocytes are detected in his peripheral blood. Which of the following cellular components is most likely to have deficient function in the tuberculous lesions in this patient's lungs? Macrophages Eosinophils Neutrophils Langhans giant cells Fibroblasts

Macrophages

A 56-year-old woman comes to the physician for a follow-up examination 8 weeks after recovering from pneumococcal pneumonia. X-rays of the chest show no abnormalities. Which of the following most likely allowed this resolution to occur? Increased angiogenesis Maintenance of basement membrane integrity Formation of granulation tissue Proliferation of fibroblasts Metaplasia of mesenchymal cells to pneumocytes

Maintenance of basement membrane integrity

A 53-year-old man, who recently returned from Africa, has fever, headache, and abdominal discomfort. He received appropriate vaccinations prior to the trip. His temperature is 39.4°C (103°F). Physical examination shows no other abnormalities. A Wright-stained peripheral blood smear is shown. Which of the following is the most likely diagnosis? Malaria Leishmaniasis Toxoplasmosis Trypanosomiasis Babesiosis

Malaria

A 45-year-old man comes to the physician because of yellow skin for 4 days. He lost his job 1 0 days ago, and he has been drinking eight to ten 12-ounce cans of beer daily since then. Physical examination shows jaundice. The liver is tender to palpation. Serum studies show: Total bilirubin 5.9 mg/dL Alkaline phosphatase 210 U/L AST 110 U/L ALT 69 U/L γ-Glutamyl transferase 35 U/L (N=0-30) Histologic examination of a liver biopsy specimen will most likely show which of the following in this patient? Glycogen Mallory hyaline Lipofuscin Hemosiderin Antitrypsin globules

Mallory hyaline

A 57-year-old woman comes to the physician 1 week after noticing a mass in her left breast during breast self-examination. Menopause occurred 6 months ago. She was receiving estrogen therapy but discontinued it 6 weeks ago; she has had no menopausal symptoms. There is no family history of breast cancer. Examination shows a 2-cm, palpable, nontender, mobile mass in the upper outer quadrant of the left breast; no nipple discharge can be expressed. Examination of the right breast shows no abnormalities. Which of the following is the most appropriate next step in management? Mammography Reexamination in 3 months Mastectomy Ductal lavage CT scan of the chest

Mammography

A 57-year-old woman comes to the physician 1 week afternoticing a mass in her left breast during breast self-examination.Menopause occurred 6 months ago. She wasreceiving estrogen therapy but discontinued it 6 weeks ago;she has had no menopausal symptoms. There is no familyhistory of breast cancer. Examination shows a 2-cm,palpable, nontender, mobile mass in the upper outerquadrant of the left breast; no nipple discharge can beexpressed. Examination of the right breast shows noabnormalities. Which of the following is the mostappropriate next step in management? CT scan of the chest Ductal lavage Mammography Mastectomy Reexamination in 3 months

Mammography

A 20-year-old woman comes to the physician because of a 1-day history of increasing urinary frequency and a burning sensation with urination. She has one sexual partner and uses condoms regularly. Her vital signs are within normal limits. Physical examination shows mild suprapubic tenderness to deep palpation. Microscopic examination of a urine specimen shows rare epithelial cells and 10 WBCs/hpf. Urine cultures grow Escherichia coli greater than 100,000 colonies/ml. The bacterial strain isolated from this culture most likely has which of the following virulence factors? Cytotoxin that increases cellular concentrations of cAMP Lipopolysaccharide wi th the 0157 specificity Pyrogenic exotoxin that acts as a superantigen Capsule composed of sialic acid Mannose-binding (type I) fimbria

Mannose-binding (type I) fimbria

A 30-year-old woman, gravida 2, para 1, comes for her first prenatal visit at 26 weeks' gestation. Uterine size is greater than expected for dates. Ultrasonography shows fetal hydrops. Which of the following is the most appropriate next step in diagnosis? Maternal Rh status with antibody screening Maternal HIV antibody test Cervical and urine cultures for group B Streptococcus Amniocentesis for measurement of α-fetoprotein concentration MRI of the fetus

Maternal Rh status with antibody screening

A 14-year-old boy is brought to the emergency department 1 hour after colliding with a teammate while playing soccer. Physical examination shows edematous tissues of the left eye and mild depression of the left zygomatic bone. The skin between the left eye and the upper lip is numb. He has double vision when asked to look upward. Damage to which of the following nerves is the most likely cause of the sensory loss? Ophthalmic division of the trigeminal nerve Buccal branch of the facial nerve Maxillary division of trigeminal nerve Mandibular branch of the trigeminal nerve Lingual branch of the glossopharyngeal nerve

Maxillary division of trigeminal nerve

A 65-year-old man has had increasingly severe headaches and diffuse muscle aches during the past 3 months. He also has a 1-month history of jaw pain when chewing food and decreasing visual acuity in his left eye. His temperature is 38°C (100.4°F). Visual acuity in his left eye is 20/100, and the left optic disc is slightly atrophic. His muscle strength is normal. Which of the following tests should be obtained next? CT scan of the head Examination of cerebrospinal fluid Measurement of erythrocyte sedimentation rate Electroencephalography Antinuclear antibody assay

Measurement of erythrocyte sedimentation rate

An otherwise healthy 23-year-old woman comes to the physician because she has not had a menstrual period for 2 months. Menarche occurred at the age of 14 years, and menses had occurred at regular 28-day intervals. Pelvic examination shows blue-tinged vaginal mucosa. The uterus is somewhat soft in the lower segment and is about the size of an orange. The adnexa are nontender with normal-sized ovaries. Which of the following is the most appropriate next step to establish the diagnosis? Measurement of serum follicle- stimulating hormone concentration Ultrasonography of the pelvis Measurement of serum thyroid-stimulating hormone concentration Measurement of human chorionic gonadotropin concentration Measurement of urine '3-human chorionic gonadotropin concentration Endometrial biopsy

Measurement of human chorionic gonadotropin concentration

A 1-week-old girl is brought to the physician for a follow-up examination after newborn screening showed a possible defect in fatty acid oxidation. Physical examination shows no abnormalities. Which of the following is the most appropriate next step in diagnosis? Measurement of serum amino acid concentrations Measurement of serum lactic acid concentration Measurement of serum electrolyte concentrations Arterial blood gas analysis Measurement of serum acylcarnitine concentrations

Measurement of serum acylcarnitine concentrations

A 77-year-old woman is brought to the emergency department by her husband because of agitation and confusion for 3 hours. He states that she has been intermittently crying out and does not appear to recognize him. A routine health maintenance examination 3 days ago showed no abnormalities except for mild memory deficits. Her current temperature is 37.8°C (100°F), pulse is 100/min, respirations are 14/min, and blood pressure is 130/60 mm Hg. Physical examination shows no abnormalities except for mild tenderness to palpation of the lower abdomen. Mental status examination shows confusion; she is oriented to person but not to time or place. Which of the following is the most appropriate next step in diagnosis? Determination of erythrocyte sedimentation rate Measurement of serum folate concentration Western blot assay Urinalysis Measurement of serum alkaline phosphatase activity

Measurement of serum folate concentration

A 77-year-old woman is brought to the emergency department by her husband because of agitation and confusion for 3 hours. He states that she has been intermittently crying out and does not appear to recognize him. A routine health maintenance examination 3 days ago showed no abnormalities except for mild memory deficits. Her current temperature is 37.8°C (100°F), pulse is 100/min, respirations are 14/min, and blood pressure is 130/60 mm Hg. Physical examination shows no abnormalities except for mild tenderness to palpation of the lower abdomen. Mental status examination shows confusion; she is oriented to person but not to time or place. Which of the following is the most appropriate next step in diagnosis? Measurement of serum folate concentration Western blot assay Urinalysis Determination of erythrocyte sedimentation rate Measurement of serum alkaline phosphatase activity

Measurement of serum folate concentration

A 28-year-old woman has palpitations that occurapproximately once a week, last 1-5 minutes, and consist ofrapid, regular heart pounding. The episodes start and stopsuddenly and have not been associated with chest discomfortor dyspnea. There is no history of heart problems. She drinkstwo to three cups of coffee daily. She rarely drinks alcoholand does not smoke. Her pulse is 96/min and regular, andblood pressure is 120/88 mm Hg. A stare and lid lag arenoted. The thyroid gland is firm and 1.5 times larger thannormal. There is a midsystolic click at the apex and a grade2/6, early systolic murmur at the upper left sternal border.An ECG is normal except for evidence of sinus tachycardia.Which of the following is the most appropriate next step indiagnosis? Echocardiography MUGA scan Measurement of serum thyroid-stimulating hormone concentration Measurement of urine catecholamineconcentration Ambulatory ECG monitoring

Measurement of serum thyroid-stimulating hormone concentration

A 16-year-old girl is brought to the emergency department6 hours after the onset of moderate lower abdominal crampsand intermittent nausea. She has not vomited during thistime. She says that her last menstrual period was 2 monthsago, but she has had intermittent bleeding since then,including spotting for the past 2 days. Menarche was at theage of 15 years. Menses occur at irregular 25- to 45-dayintervals. She is sexually active and uses condomsinconsistently. Her temperature is 38.1°C (100.6°F), pulseis 94/min, respirations are 22/min, and blood pressure is120/80 mm Hg. Examination shows a soft abdomen withlower quadrant tenderness, especially on the right. Bowelsounds are normal. Pelvic examination shows scant vaginalbleeding and a palpable, tender right adnexal mass. Thecervix appears normal. There is no cervical motiontenderness. Which of the following is the most appropriatenext step in management? Exploratory laparoscopy Ceftriaxone and azithromycin therapy Measurement of serum β-hCG concentration Abdominal x-ray Complete blood count

Measurement of serum β-hCG concentration

A 42-year-old woman, gravida 3, para 3, comes to the physician because she has not had a menstrual period for 2 months. She reports that she had an episode of spotting 3 weeks ago. She has had no other symptoms. She has no history of abnormal Pap smears; her last Pap smear was 10 months ago. She is sexually active with her husband and uses condoms. She is 163 cm (5 ft 4 in) tall and weighs 72 kg (160 lb); BMI is 28 kg/m2. On physical examination, the abdomen is nontender to palpation. Pelvic examination shows a slightly enlarged uterus; there are no palpable adnexal masses. Which of the following is the most appropriate next step in management? Oral contraceptive therapy Measurement of serum thyroid-stimulating hormone concentration Endometrial biopsy Measurement of serum β-hCG concentration CT scan of the pelvis

Measurement of serum β-hCG concentration

A 42-year-old woman, gravida 3, para 3, comes to thephysician because she has not had a menstrual period for2 months. She reports that she had an episode of spotting3 weeks ago. She has had no other symptoms. She has nohistory of abnormal Pap smears; her last Pap smear was10 months ago. She is sexually active with her husband anduses condoms. She is 163 cm (5 ft 4 in) tall and weighs72 kg (160 lb); BMI is 28 kg/m2. On physical examination,the abdomen is nontender to palpation. Pelvic examinationshows a slightly enlarged uterus; there are no palpableadnexal masses. Which of the following is the mostappropriate next step in management? Measurement of serum thyroid-stimulatinghormone concentration Measurement of serum β-hCG concentration Oral contraceptive therapy Endometrial biopsy CT scan of the pelvis

Measurement of serum β-hCG concentration

A 16-year-old girl is brought to the physician because of severe acne over her face and upper back for 6 months. She has had no itching or scaling of the acne. Treatment with topical and oral antibiotics has not resolved her symptoms. Examination shows numerous papules and pustules with widespread erythema over the face and upper back. There is no hyperpigmentation. The patient requests information about beginning isotretinoin, because she says it improved her boyfriend's acne. Which of the following is the most appropriate initial step prior to treatment with isotretinoin? Measurement of urine β-hCG concentration PPD skin test Complete blood count with differential Measurement of serum thyroid-stimulating hormone concentration 24-Hour urine collection for measurement of creatinine clearance Determination of prothrombin and partial thromboplastin times

Measurement of urine β-hCG concentration

A 16-year-old girl is brought to the physician because ofsevere acne over her face and upper back for 6 months. Shehas had no itching or scaling of the acne. Treatment withtopical and oral antibiotics has not resolved her symptoms.Examination shows numerous papules and pustules withwidespread erythema over the face and upper back. Thereis no hyperpigmentation. The patient requests informationabout beginning isotretinoin, because she says it improvedher boyfriend's acne. Which of the following is the mostappropriate initial step prior to treatment with isotretinoin? Measurement of urine β-hCG concentration PPD skin test 24-Hour urine collection for measurement ofcreatinine clearance Determination of prothrombin and partialthromboplastin times Measurement of serum thyroid-stimulatinghormone concentration Complete blood count with differential

Measurement of urine β-hCG concentration

A 42-year-old woman with multiple sclerosis is brought to the physician because of double vision that began yesterday morning when she awoke. The double images are horizontal on lateral gaze. Her only medication is interferon beta. On ophthalmologic examination, the eyes converge normally, but there is decreased adduction in both eyes during lateral conjugate gaze. Which of the following is the most likely location of a demyelinating plaque causing this patient's diplopia? Rubrospinal tract Oculomotor nerve nucleus Medial longitudinal fasciculus Superior colliculus Lateral lemniscus

Medial longitudinal fasciculus

A 22-year-old man comes to the emergency department 30 minutes after his left forearm was slashed with a pocketknife during a bar fight. Initially, the wound was bleeding profusely; however, continuous application of pressure to the affected area has resolved the bleeding. His pulse is 1 1 0/min, and blood pressure is 135/85 mmHg . Examination of the left upper extremity shows a 3-cm laceration that is oriented transversely over the pal mar aspect of the distal forearm and located 2 cm proximal to the distal palmar wrist crease. He is unable to abduct the left thumb. Sensation to pinprick is decreased over the palmar aspect of the thumb and index and middle fingers. Capillary refill time is normal for all digits. Which of the following nerves is most likely injured in this patient? Lateral antebrachial cutaneous Median Medial antebrachial cutaneous Ulna Radial

Median

A 29-year-old woman with an 11-year history of bipolar disorder comes to the physician because she is concerned about memory loss during the past 2 weeks. She has had difficulty remembering appointments that she has made, and on one occasion, she got lost going to the health club where she has been a member for years. She has taken lithium carbonate for 8 years, and she has been taking a friend's diuretic for perimenstrual weight gain during the past 3 months. Physical examination shows a resting tremor of both hands and mild ataxia. On mental status examination, she is oriented to person, place, and time, but she recalls only one of three objects after 5 minutes. Normal-pressure hydrocephalus Pseudodementia Pick disease Parkinson disease Medication toxicity Normal age-associated memory decline

Medication toxicity

A 50-year-old man comes to the physician for a routine health maintenance examination. Physical examination shows a firm thyroid nodule. The neoplasm is resected; histologic sections show sheets of granular cell s embedded in a hyaline stroma. Immunohistochemical stains show the presence of cytoplasmic calcitonin. Which of the following is the most likely type of carcinoma? Medullary Giant cell Anaplastic Follicular Papillary

Medullary

A 39-year-old man is admitted to the hospital by his brother for evaluation of increasing forgetfulness and confusion during the past month. His brother reports that the patient has been drinking heavily and eating very little, and has been slightly nauseated and tremulous. He wanders at night because he cannot sleep. On admission to the hospital, intravenous administration of 5% dextrose in water is initiated. Two hours later, the patient has ophthalmoplegia and is completely confused. Which of the following is the most appropriate next step in management? Oral anticoagulant IV fluids with magnesium Megadose of vitamin B1 by IV Megadoses of vitamin C by IV Oral diazepam

Megadose of vitamin B1 by IV

Case of newborn with Down syndrome. Which of the following most likely occurred prior to conception? Gene inactivation Meiotic nondisjunction Chromosomal deletion Trinucleotide expansion Chromosomal translocation

Meiotic nondisjunction

A 40-year-old woman comes to the physician because of a mole on her back that has increased in size during the past 4 months. Physical examination shows a raised irregular lesion with variegated black-tan pigmentation and ill-defined margins. Examination of tissue from the lesion shows pleomorphic, hyperchromatic cells within clear islands that tend to coalesce and are present at all levels of the epidermis, with extension into the papillary dermis. Which of the following is the most likely diagnosis? Cafe au lait spot Seborrheic keratosis Melanoma Intradermal nevus Basal cell carcinoma Blue nevus Lentigo simplex

Melanoma

A previously healthy 57-year-old woman comes to thephysician 1 week after noticing a lump under her right arm.She is concerned that it is breast cancer because both hermother and maternal aunt died of breast cancer. She does notsmoke, drink alcohol, or use illicit drugs. She has avoidedthe sun for the past 10 years. She notes that her skin hasnever tanned but always burned and freckled when exposedto the sun. She exercises daily on a stationary bicycle andeats a well-balanced diet. Her temperature is 37°C (98.6°F),pulse is 82/min and regular, respirations are 14/min, andblood pressure is 130/74 mm Hg. There are numerousfreckles over the entire body. Examination of the right breastshows a 0.6-cm, flat, brown lesion; the lesion is mottled withdeep purple and black areas and has an irregular border.There are no breast masses, dimpling, peau d'orange, ornipple discharge. The patient says that the lesion has beenpresent for 1 year, but she has never had it examined. Thereis a firm, nontender mass in the right axilla. Examinationshows no other abnormalities. Which of the following is themost likely diagnosis? Mastitis Fibrocystic changes of the breast Pigmented nevus Basal cell carcinoma Port-wine stain Melanoma

Melanoma

A 58-year-old woman is brought to the emergency department because of a 2-hour history of shortness of breath and chest pain that radiates to her back between the shoulder blades. Her respirations are 28/min. Physical examination shows diaphoresis. An ECG shows no abnormalities. Coronary angiography shows occlusion of the marginal branch of the left anterior descending coronary artery. A percutaneous coronary revascularization is done. Subsequent to the stent's placement, her serum concentrations of myocardial creatine kinase (CK-MB) and troponin I are increased. Which of the following mechanisms best explains these laboratory findings? Protease inactivation by cytoplasmic free calcium ions Cell shrinkage Liquefactive necrosis of the myocardium Membrane lipid peroxidation Formation of apoptotic bodies

Membrane lipid peroxidation

A 60-year-old man comes to the emergency department because of the sudden onset of acute abdominal pain and tenderness, nausea, vomiting, and bloody diarrhea 2 hours ago. He has a history of cirrhosis and hepatocellular carcinoma. His blood pressure is 99/50 mm Hg. His abdominal wall becomes rigid, with the loss of bowel sounds. During surgical intervention, half of the small intestine is found to have a dark purple-red hemorrhagic appearance. Which of the following is the most likely cause? Pseudomembranous colitis Mesenteric venous thrombosis Ulcerative colitis Bacterial enteritis Necrotizing enterocolitis Whipple disease

Mesenteric venous thrombosis

A 38-year-old man is brought to the emergency department 30 minutes after being found near his home unable to stand upright. He appears lethargic. His pulse is 110/min, and blood pressure is 90/62 mm Hg while supine. Physical examination shows dry mucous membranes and poor skin turgor. Abdominal examination shows midepigastric tenderness. Laboratory studies show: Serum Na+: 143 mEq/L Serum K+: 3.2 mEq/L Serum Cl-: 101 mEq/L Serum HC03-: 11 mEq/L Arterial blood gas analysis on room air: pH: 7.28 PCO2: 23 mm Hg PO2: 98 mm Hg Which of the following is the most likely current acid- base status in this patient? Mixed respiratory acidosis and respiratory alkalosis Metabolic alkalosis Metabolic acidosis Respiratory acidosis Mixed respiratory alkalosis and metabolic alkalosis Mixed respiratory acidosis and metabolic alkalosis Mixed metabolic acidosis and metabolic alkalosis Respiratory alkalosis

Metabolic acidosis

A 48-year-old woman has had the gradual onset of back pain over the past 2 weeks. There is no history of trauma. She does not smoke cigarettes, drink alcohol, or use illegal drugs. Hemogram, serologic studies, and urinalysis are unremarkable. An x-ray of the spine shows two lytic lesions, one in T-10 and the other in L-1. Which of the following is the most likely diagnosis Metastatic carcinoma of the breast Renal osteodystrophy Osteosarcoma Thyroid carcinoma Avascular necrosis

Metastatic carcinoma of the breast

A 4-year-old boy is brought to the physician by his parents because of fatigue and irritability over the past 2 months. The family visited relatives in rural Louisiana 5 months ago where the patient ran around barefoot. The child is active and appears normal. The conjunctivae are pale. Laboratory studies include a normal leukocyte count with 15% eosinophils. A stool preparation discloses the parasite egg shown. Which of the following is the most likely cause of fatigue and irritability in this patient? Myocarditis Circulating immune complexes Activation of lgE Central nervous system infestation Microcytic anemia

Microcytic anemia

A 20-year-old man comes to the physician because of cramping abdominal pain and diarrhea during the past 3 weeks; he has had a 4.5-kg (10-lb) weight loss during this period. The pain is exacerbated following meals. He went on a camping trip in upstate New York 3 weeks ago, swimming in the nearby lakes and hiking in the mountains. His vital signs are normal. Physical examination shows no abnormalities. Which of the following diagnostic tests is most likely to identify the causal organism of this patient's condition? Microscopic examination of the stool for ova and parasites Proctoscopy and rectal biopsy Electron microscopy of the stool for small round viruses Polymerase chain reaction test of the stool for Shiga toxin Culture of the stool for enteric bacterial pathogens

Microscopic examination of the stool for ova and parasites

A 49-year-old woman with breast cancer develops increased numbness of both hands and feet during the 3-week interval between her third and fourth rounds of chemotherapy. Sensation to pinprick and fine touch is decreased over the hands, wrists, ankles, and feet. The physician suspects that this distribution of sensory loss is most likely an adverse effect of chemotherapeutic agents. Which of the following is most likely disrupted in this patient as a result of the chemotherapy? Microtubules for axonal transport Synaptic vesicles for synaptic transmission Neurofilaments for structural support of axons Myelin sheaths for saltatory conduction of action potentials Sodium channels for membrane depolarization

Microtubules for axonal transport

A 76-year-old woman is brought to the emergency department because of a 3-hour history of severe headache, slurred speech, and confusion. She has a 2-year history of atrial fibrillation. She has difficulty understanding and answering questions, but she is cooperative. Ophthalmologic examination shows hemianopia and a tendency to gaze to the right. The pupils are normal sized and reactive to light. Neurologic examination shows left-sided numbness and paralysis that are more severe in the face and upper extremity than in the lower extremity. Which of the following arteries is most likely involved in this patient's condition? Middle cerebral Anterior cerebral Posterior cerebral Anterior spinal Vertebral

Middle cerebral

A 30-year-old man is brought to the emergency department 15 minutes after he was found unconscious in a park. On arrival, he is comatose. The pupils are 4 mm in diameter and not reactive to light. A CT scan of the head is shown. The most likely cause of the coma is bleeding from which of the following structures? Transverse sinus Bridging vein Middle meningeal artery Lenticulostriate artery Sigmoid sinus Internal cerebral vein

Middle meningeal artery

A 17-year-old female comes to the physician because of shortness of breath. A systolic murmur is heard best in the left fifth intercostal space in the midclavicular line. Which of the following valvular abnormalities is the most likely cause of the murmur? Mitral stenosis Pulmonary insufficiency Mitral insufficiency Aortic stenosis Aortic insufficiency

Mitral insufficiency

A 51-year-old male has a holosystolic murmur that is loudest over the apical impulse and radiates to the axillae. Which of the following is the most likely diagnosis? Pulmonic insufficiency Tricuspid insufficiency Mitral insufficiency Aortic stenosis Mitral stenosis

Mitral insufficiency

A 45-year-old man comes to the physician for an initial examination. He has not been examined by any physician during the past 30 years because he has not felt ill. He says that he feels well at this visit. He appears relaxed. Cardiac examination shows a grade 2/6, holosystolic murmur that is best heard at the left axillary line. Which of the following is the most likely cause of this finding? Mitral regurgitation Mitral stenosis Aortic regurgitation Tricuspid regurgitation Tricuspid stenosis Aortic stenosis

Mitral regurgitation

Case of newborn Down syndrome. Which of the following abnormalities is most likely to be present in this patient? Uniparental disomy Partial deletion Triplet expansion Aberrant genomic imprinting Mosaicism

Mosaicism

A one-day-old newborn male is found to have hemolytic disease of the newborn. Both parents are Rh-positive, mother is positive for IgG isohemagglutinins, while father is negative. Which of the following parental blood types most likely caused this condition? Mother: O and Father: AB Mother: AB and Father: O Mother: B and Father: O Mother: B and Father: A Mother: O and Father: AB

Mother: O and Father: AB

A 40-year-old man has had orthostatic hypotension and loose stools for 1 year. He has a 26-year history of type 1 diabetes mellitus. Studies of the stool show no abnormalities. Which of the following pathophysiologic mechanisms is the most likely cause of the diarrhea? Secretion Osmosis Exudation Motility disorder Generalized malabsorption

Motility disorder

A 55-year-old man comes to the physician because of a 2-month history of increasing difficulty swallowing and regurgitation of undigested food. He also has noticed unusual rumbling sounds in his voice that he feels originate in his neck. Physical examination shows halitosis. A videofluoroscopic swallowing study shows a 4-cm, posterior midline pouch protruding between the thyropharyngeus and cricopharyngeus portions of the inferior pharyngeal constrictor muscle. These muscles are most likely innervated by which of the following nerves? Hypoglossal nerve Glossopharyngeal nerve Motor fibers from the vagus nerve Parasympathetic fibers from the vagus nerve Sympathetic fibers from the superior cervical ganglion

Motor fibers from the vagus nerve

A 65-year-old woman with diabetic ketoacidosis has had a fever and proptosis of the left eye over the past 3 days. She is unable to move the left eye. A black eschar is present on the nasal mucosa. Which of the following is the most likely diagnosis? Histoplasmosis Cryptococcosis Candidiasis Coccidioidomycosis Mucormycosis Aspergillosis

Mucormycosis

A healthy 3-year-old boy has a cleft lip and palate. He has no other birth defects. Which of the following modes of inheritance is the best explanation for these abnormalities? Multifactorial Autosomal recessive X-linked recessive Autosomal dominant Mitochondrial

Multifactorial

Parents with cleft lip and child with spina bifida. Which of the following is the most likely mode of inheritance of this disorder? X-linked recessive Mitochondrial Multifactorial Autosomal recessive Autosomal dominant

Multifactorial

A 27-year-old woman comes to the physician because of a 2-year history of sporadic double vision and a 1-year history of intermittent numbness and tingling in her arms. The symptoms became more severe 5 days ago after a 4-day visit from her husband's parents. During the past year, she also has had four episodes of urinary incontinence. Physical examination shows mild dysarthria. On mental status examination, she has a mildly worried mood and a reactive affect. MRI of the brain shows a few small, scattered, nonspecific white-matter plaques. Which of the following is the most likely diagnosis? Retroviral diseases Polyarteritis nodosa Chiari malformation Behçet syndrome Multiple Sclerosis Systemic lupus erythematosus Sjögren syndrome

Multiple Sclerosis

A 62-year-old woman comes to the physician because of a 3-week history of progressive fatigue and pain in the right arm and low back that has intensified during the past 2 days. She has hypertension treated with hydrochlorothiazide. Physical examination shows pale mucous membranes and tenderness to palpation over the lower lumbar spine and right upper humerus. There is full range of motion, and muscle strength is normal. Laboratory studies show: Hemoglobin 8 g/dl Hematocrit 24°/c, Serum Ca2+ Al bumin 12.5 mg/dl 3 g/dl X-rays of the humerus and low back show lytic bone lesions. A bone scan shows multiple areas of uptake. A photomicrograph of a peripheral blood smear is shown. Which of the following is the most likely diagnosis? Osteomalacia Acute myelogenous leukemia Non-hodgkin lymphoma Multiple myeloma Osteosarcoma

Multiple myeloma

A 78-year-old man comes to the physician because of a 3-month history of increasingly severe back pain. He has had a 5.4-kg (12-lb) weight loss during this period. The pain occurs mostly at night, is exacerbated by movement, and is no longer relieved by aspirin. There has been no trauma to the area. He appears chronically ill. His pulse is 100/min, and blood pressure is 160/100 mm Hg. Physical examination shows point tenderness over the L4-5 area; there are no skin lesions. Urinalysis shows 4+ protein. A photomicrograph of findings from bone marrow aspiration are shown. Which of the following is the most likely diagnosis? Waldenstrom macroglobulinemia Multiple myeloma

Multiple myeloma

A 64-year-old female presents with the unpleasant urge to move her legs. She reports that the symptoms worsen in the evening, but improve while she is moving. She also reports that symptoms worsen after taking benadryl. She takes not other medication. Physical examination is unremarkable, except for mild hypertension (140/85 mm Hg). She self-reports of sleep-onset inxomnia and mild depression. This patient has an increased risk for which of the following? Myasthenia gravis Alzheimer's disease Hyperthyroidism Hypothyroidism Multiple sclerosis

Multiple sclerosis

A 23-year-old woman is brought to the emergency department 45 minutes after sustaining an injury to her right shoulder when she was forcefully thrown into a swimming pool. The patient is holding her right arm in external rotation. There is an area of tenderness and absent sensation over the lateral aspect of the forearm. Immediately after reduction of the glenohumeral joint, she is unable to flex the elbow against gravity. Which of the following structures is most likely injured in this patient? Thoracodorsal nerve Axillary nerve Musculocutaneous nerve Posterior cord of the brachial! plexus Radial nerve

Musculocutaneous nerve

A health inspector confiscates chickens smuggled intoTaiwan from mainland China after she discovers them in thehold of a ship. Testing shows that, although the chickensappear healthy, they are infected with the H5N1 subtype ofthe influenza A virus. Which of the following is the primaryconcern for human health from these virus-infectedchickens? Secretion of a product that will make the meatof the chicken poisonous to eat Mutation of the virus to a form that causes fatalencephalitis Secretion of a product that will cause mutationsof the virus in the fetuses of mothers that eatthe meat Mutation of the virus to a form that causes fatalrenal disease Secretion of a product that will causeimmunosuppression in those that eat the meat Mutation of the virus to a form that is highlyinfectious among humans

Mutation of the virus to a form that is highlyinfectious among humans

A 32-year-old woman comes to the physician because of a 1-month history of progressive shortness of breath and anxiety. She says that her symptoms become much more pronounced as the day progresses. She is 170 cm (5 ft 7 in) tall and weighs 70 kg (155 lbs); BMI is 24 kg/m2. Breath sounds are normal on auscultation of the chest, but the patient has difficulty taking a long, deep breath. Arterial blood gas analysis on room air shows: pH 7.33 PC02 70 mm Hg Po2 65 mm Hg Pulmonary function testing shows a decreased vital capacity, tidal volume, and expiratory reserve volume. Her residual volume is within the reference range. Which of the following is the most likely underlying cause of this patient's condition? Obesity Myasthenia Gravis 1-Antitrypsin deficiency Central nervous system neoplasm Chronic bronchitis Chronic opiate use

Myasthenia Gravis

A 19-year-old woman comes to the physician because of a 3-month history of intermittent drooping of her left eyelid each evening?? and occasional difficulty chewing and swallowing. She also has had two episodes of double vision that occurred in the evening and resolved by the following morning. Examination shows no abnormalities except for slight ptosis on the right. Which of the following is the most likely diagnosis? Which of the following is the most likely defect? Which of the following represents the most common form of treatment? Myasthenia gravis Brain stem glioma Acute intermittent porphyria Guillain-Barré syndrome-> Complex partial seizures

Myasthenia gravis

A 42-year-old man has an autoimmune disorder resulting in proximal muscle weakness of the lower extremities. Arrows in the photomicrograph shown indicate membranes that contain high concentrations of channels that are targeted by this condition. Which of the following is the most likely diagnosis? Periodic paralysis Myasthenic (Lambert-Eaton) syndrome Tetany Multiple sclerosis Myasthenia gravis

Myasthenic (Lambert-Eaton) syndrome

A 24-year-old man is brought to the physician because of a 3-day history of progressive numbness of both feet that now has ascended to the level of his thighs. In the last 24 hours, he has developed numbness and tingling of his hands. Physical examination shows an ataxic gait. Deep tendon reflexes are diminished in the upper extremities and absent at the knees and ankles. Sensation to vibration and joint position is absent in the fingertips and feet bilaterally. There is mild weakness of the distal upper extremities and moderate weakness of the lower extremities. A process involving which of the following structures is the most likely explanation for these sensory findings? Fasciculus cuneatus Fasciculus gracilis Unmyelinated primary afferents Dorsal spinocerebellar tract Myelinated primary afferents Ventral spinocerebellar tract

Myelinated primary afferents

A 63-year-old man is brought to the physician because of a 2-week history of a sensation of fullness in the left upper quadrant of the abdomen. He also has had lethargy and occasional shortness of breath during this period. He has had a 9-kg (20-lb) weight loss during the past 3 months. Physical examination shows pallor. The spleen tip is palpated. Laboratory studies show: Hemoglobin 9 g/dL Hematocrit 27% Leukocyte count 4000/mm3 Serum uric acid 15 mg/dL A peripheral blood smear shows numerous erythrocytes with abnormal shapes and sizes, nucleated erythrocytes, and myelocytes. Aspiration of bone marrow results in a dry tap. A bone marrow biopsy specimen shows markedly thickened bony trabeculae with increased reticulum. Which of the following is the most likely diagnosis? Dysmyelopoietic syndrome Chronic myeloid leukemia Acute myeloid leukemia Essential thrombocythemia Polycythemia vera Myelofibrosis

Myelofibrosis

A 9-year-old boy is brought to the emergency department because of fever, chest pain, and migratory joint pain of his shoulders, hips, and knees for 2 days. Four weeks ago, he had a febrile pharyngitis, which resolved 2 weeks later without treatment. His temperature is 39°C (102.2°F). The lungs are clear to auscultation. A pericardial friction rub and quiet heart sounds are heard. Throat cultures do not grow any pathogens. Laboratory testing shows increased antibody titers to streptolysin 0. The greatest risk for death at this time is from which of the following? Aortic stenosis Embolism Mitral insufficiency Myocarditis Septic shock

Myocarditis

A previously healthy 17-year-old girl is brought to the emergency department because of a 1-day history of shortness of breath, weakness, and muscle tenderness. She completed a triathlon the previous day. She appears restless. She is 163 cm (5 ft 4 in) tall and weighs 50 kg (110 lb}; BMI is 1 9 kg/m2. Her temperature is 38°C (100.4°F), respirations are 20/min, and blood pressure is 150/90 mm Hg. Bilateral crackles are heard in the lower lung lobes on auscultation of the chest. Physical examination shows muscle tenderness. Her serum creatinine concentration is 4 mg/dl. Urinalysis shows 3+ protein and 4+ hemoglobin . The most likely cause of this patient's condition is an increased release of which of the following substances? Hemoglobin Myoglobin TroponinI Creatine kinase Aldolase

Myoglobin

During an experiment of muscle contraction, intracellular calcium is decreased after a substance is administered to a muscle preparation obtained from an experimental animal. Which of the following best explains why contraction is inhibited in this case? Myosin binding sites on actin remain covered by troponin C Acetylcholine release is increased Sodium influx is increased Depolarization along T tubules is enhanced Tropomyosin is detached from actin

Myosin binding sites on actin remain covered by troponin C

A 19-year-old man who plays college basketball comes to the physician because of a 3-week history of shortness of breath with exertion. He says that he has been unable to complete his daily practice sessions. He also has had intermittent dizziness during this period. His pulse is 60/min, respirations are 18/min, and blood pressure is 100/60 mm Hg. Physical examination shows no abnormalities. Echocardiography shows a heterogeneous pedunculated mass in the left atrium. Which of the following is the most likely diagnosis? Rhabdomyosarcoma Myxoma Teratoma Lipoma Hemangiopericytoma

Myxoma

A 24-year-old woman comes to the physician because of a 1-day history of pain in her left leg while walking. Her temperature is 37° C (98.6°F), pulse is 84/min, and blood pressure is 135/78 mm Hg. Examination of the left lower extremity shows only a markedly decreased pulse in the left lower extremity. An intermittent faint diastolic heart murmur is heard at the apex. Angiography of the left lower extremity shows complete obstruction of the distal left femoral artery by what appears to be a thrombus. The occlusion is removed using a catheter. Microscopic examination of the removed specimen shows scattered mesenchymal cells in an abundant extracellular matrix. Echocardiography confirms a diagnosis of cardiac tumor. This patient most likely has which of the following types of cardiac tumors? Hemangioma Hamartoma Fibroma Fibroelastoma Myxoma

Myxoma

A 54-year-old woman comes to the physician 1 week after an episode of the sudden loss of vision in her left eye. Most of her vision returned within 1 day. She also has a 3-month history of progressive shortness of breath with exertion. Echocardiography shows a mass in the left atrium of the heart. Which of the following is the most likely diagnosis? Angiosarcoma Fibrosarcoma Rhabdomyoma Lipoma Fibroelastoma Myxoma

Myxoma

A 5O-year-old man comes to the emergency department because of fever and shortness of breath for 3 days. A Gram stain of his sputum shows numerous neutrophils, many of which contain gram-positive diplococci. Which of the following enzymes is most likely to initiate intracellular killing of the diplococci? Cyclooxygenase Lysosomal hydrolases Superoxide dismutase NADPH oxidase Catalase

NADPH oxidase

A 28-year-old female executive has irresistible urges tosleep during the day. She often has episodes of dropping herhead, slurred speech, and suddenly dropping things fromher hands, all lasting for seconds to minutes. In addition,she frequently has vivid, colorful dreams just before fallingasleep. Occasionally, when in certain emotionally chargedsituations, she feels like her whole body goes limp;however, she remembers everything. Which of thefollowing is the most likely diagnosis? Delayed sleep phase syndrome Narcolepsy Insomnia Night terrors Absence seizures

Narcolepsy

An investigator is studying strains of Streptococcus pneumoniae to monitor the effect of a conjugate polysaccharide vaccine on the expression of different capsule types. Several strains that express a capsular type previously found only in strains of Streptococcus mitis are identified. After laboratory incubation of a nonencapsulated strain of S. pneumoniae with the lysate from heat-killed S. mitis, colonies of S. pneumoniae expressing the new capsular type are identified. However, no colonies of S. pneumoniae expressing this capsule are isolated after the addition of DNase to the lysate. Which of the following mechanisms is the most likely cause of the transfer of the genes encoding the new capsular type? Natural transformation Bacteriophage transduction Plasmid transfer Point mutations in the capsule genes Slipped strand mispairing

Natural transformation

Two days after undergoing surgical repair of a rupturedabdominal aortic aneurysm, a 67-year-old man requiresincreasing ventilatory support. He remains intubated and isbeing mechanically ventilated at an FIO2 of 0.6 and apositive end-expiratory pressure of 7.5 cm H2O. He haschronic obstructive pulmonary disease. He had amyocardial infarction 2 years ago. His only medication is asedative. He has smoked two packs of cigarettes daily for40 years. He appears diaphoretic. His temperature is38.1°C (100.6°F), pulse is 120/min, and blood pressure is90/60 mm Hg; the ventilatory rate is 25/min. Examinationshows jugular venous distention. Breath sounds are absenton the left. The trachea is shifted to the right. Pulseoximetry shows an oxygen saturation of 82%. Which of thefollowing is the most appropriate next step in diagnosis? Needle aspiration of the left side of the chest ECG CT scan of the chest Transthoracic echocardiography

Needle aspiration of the left side of the chest

A previously healthy 4-year-old boy is brought to theemergency department 8 hours after the onset of fever and adiffuse, constant headache. He had been well until 2 daysago when he developed malaise. Yesterday, he slept morethan usual and had temperatures to 39.2°C (102.5°F).Immunizations are up-to-date. He is lethargic and irritablewhen aroused. He holds his head extended and cries whenhis neck is flexed. Muscle strength is normal, and deeptendon reflexes are symmetrically increased. A lumbarpuncture yields turbid cerebrospinal fluid. Laboratory studiesShow:Serum glucose 120 mg/dLCerebrospinal fluidOpening pressure 250 mm H2OGlucose 10 mg/dLProtein 85 mg/dLLeukocyte count 750/mm3Segmented neutrophils 95%Lymphocytes 5%Erythrocyte count 0/mm3A Gram stain of cerebrospinal fluid shows gram-negativediplococci. A CT scan of the head shows no abnormalities.Which of the following is the most likely causal organism? Haemophilus influenzae Varicella-zoster virus Streptococcus agalactiae (group B) Cytomegalovirus Neisseria meningitides

Neisseria meningitides

A 35-year-old woman has an abnormal Pap smear. Examination of tissue obtained on cervical biopsy shows microinvasive cervical carcinoma. Which of the following microscopic features led to this diagnosis? Neoplastic cells invading nerves Full thickness of the epithelium occupied by neoplastic cells Neoplastic cells invading blood vessels More than half of the epithelium occupied by neoplastic cells Neoplastic cells in sub-basement membrane connective tissue

Neoplastic cells in sub-basement membrane connective tissue

A 10-month-old boy has an abdominal mass noted during a routine physical examination. Abdominal ultrasonography shows a 10-cm solid mass on the upper pole of the left kidney. Angiographically, the mass is hypervascular. Which of the following is the most likely diagnosis? Nephroblastoma (Wilms tumor) Retinoblastoma Mesoblastic nephroma Germ cell tumor Pheochromocytoma Renal cell adenocarcinoma

Nephroblastoma (Wilms tumor)

Lab results showed that 24 hour serum and urine Na+ and osmolality did not change following the administration of exogenous ADH (i.e., desmopressin). These results are consistent with which of the following conditions? Excessive hypotonic IV fluid Nephrogenic diabetes insipidus (NDI) Osmotic diuresis Polydipsia Central diabetes insipidus (CDI)

Nephrogenic diabetes insipidus (NDI)

A 60-year-old woman is brought to the physician 3 hours after the sudden onset of pain of her right ankle. She has a 4-year history of gradually increasing serum creatinine concentrations. She began furosemide therapy 1 month ago for pedal edema. Current medications also include glipizide for type 2 diabetes mellitus. Her pulse is 120/min, respirations are 25/min, and blood pressure is 150/100 mm Hg. Physical examination shows exquisite tenderness to palpation of the right ankle joint. Analysis of joint fluid aspirate shows negatively birefringent crystals. Sensation to pinprick is decreased in the feet. The patient is at increased risk for which of the following complications of the underlying process causing the joint findings? Osteoporosis Pathologic fracture Osteoarthritis Nephrolithiasis Cholelithiasis

Nephrolithiasis

A 46-year-old man comes to the physician because of a 6-month history of sleep disturbances, difficulty walking, and pain of his extremities. He now is having trouble getting dressed. He is a retired professional boxer. Physical examination shows mask-like features, stooped posture, slow and rigid movements, shuffling gait, and resting tremor. The most likely cause of this condition is which of the following alterations in this patient's brain? Increased interstitial fibrosis Loss of Schwann cell myelin Disruption of astrocyte-endothelial association Amyloid angiopathy Neuronal degeneration

Neuronal degeneration

A 35-year-old woman, gravida 1, para 1, comes to the physician because of a 2-month history of generalized weakness and fatigability that are significantly exacerbated by exercise. The symptoms began during her pregnancy. Physical examination shows diplopia. There is reduction in arm muscle strength with repetitive movement, but preservation of deep tendon reflexes. A therapeutic trial of oral pyridostigmine provides symptomatic relief. Serum studies will most likely show antibodies to which of the following proteins? Nicotinic acetylcholine receptor Myeloperoxidase Acetylcholinesterase Muscarinic acline receptoetylchor

Nicotinic acetylcholine receptor

A 24-year-old man comes to the physician because of nasal congestion, a watery nasal discharge, and sneezing. He says that these symptoms happen each spring, when grass and tree pollens are abundant. Which of the following types of drugs is most likely to be effective in relieving these symptoms in the short term? -Adrenergic agonist a-Adrenergic agonist Nicotinic cholinergic agonist Nicotinic cholinergic antagonist a-Adrenergic antagonist -Adrenergic antagonist

Nicotinic cholinergic antagonist

A female newborn is delivered at 34 weeks' gestation in an advanced-care setting where special delivery systems are available. The diagnosis of persistent pulmonary hypertension is made. Considering that the newborn can be carefully monitored for methemoglobinemia, which of the following is the most appropriate therapy? Nitrous oxide Hyperbaric oxygen Oxygen diluted with helium Desflurane Nitric oxide

Nitric oxide

A 26-month-old boy is brought to the physician because of a 3-day history of cough and mild shortness of breath and a 1-week history of fever. He has had multiple cutaneous abscesses caused by Staphylococcus aureus during the past 9 months; each of these abscesses was debrided and treated with systemic antibiotics. At birth, he was at the 50th percentile for length and weight. Today, he is at the 10th percentile for length and 5th percentile for weight. He does not appear distressed. His temperature is 39°C (102.2° F), pulse is 140/min, respirations are 24/min, and blood pressure is 80/40 mm Hg. Pulse oximetry on room air shows an oxygen saturation of 93%. Oropharyngeal examination shows no abnormalities. Crackles are heard over the left posterior lung field. A chest x-ray shows a cavitary infiltrate in the left lingula. Cultures obtained via bronchoscopy grow a filamentous, gram-positive rod. Which of the following is the most likely causal organism? Streptococcus gal/olyticus (formerly S. bovis) Nocardia asteroides Bacillus anthracis Pneumocystis jiroveci (formerly P. carinil) Candida parapsilosis

Nocardia asteroides

A 22-year-old man is brought to the emergency department because of a 6-hour history of severe, sharp, upper back pain. He has had progressive fatigue during the past 3 weeks. He is 183 cm (6 ft) tall and weighs 79 kg (175 lb); BMI is 24 kg/m2. His temperature is 36.9°C (98.5°F), pulse is 90/min, and blood pressure is 160/55 mm Hg. Physical examination shows long, thin upper and lower extremities. Fingertip to fingertip with arms outstretched is 189 cm (74 in) wide. A high-pitched midsystolic click is heard predominantly over the apex. Which of the following best describes the primary genetic cause of this patient's condition? Nonsense mutation in fibrillin-1 gene Overexpression of fibronectin gene Mutation in keratin-14 gene Overexpression of collagen X gene Expression of genomic duplication within thefibrin gene

Nonsense mutation in fibrillin-1 gene

A healthy 22-year-old man is a subject in a study investigating the regulation of heart rate during exercise. He is sitting on a stationary bicycle and is instructed to begin pedaling in 5 minutes. One minute before he begins to pedal, his heart rate increases. An increase in which of the following transmitters acting at the indicated receptor best explains the increase in heart rate in this man? Acetylcholine nicotinic Norepinephrine beta 1 Norepinephrine beta 2 Acetylcholine muscarinic Norepinephrine alpha

Norepinephrine beta 1

A healthy 21-year-old man dies suddenly of cardiac arrest after snorting cocaine. Cocaine-induced inhibition of which of the following is the most likely explanation for the cardiac arrest? (32-Adrenergic receptors Norepinephrine reuptake Muscarinic receptors a2-Adrenergic receptors Monoamine oxidase

Norepinephrine reuptake

A 72-year-old man comes for a routine health maintenance examination. He has a 5-year history of progressive difficulty falling asleep at night and waking up early in the morning. He has not had snoring, nightmares, or changes in appetite or weight. He has felt energized since starting a new business venture 8 months ago. He has hypertension treated with hydrochlorothiazide. He does not smoke cigarettes, drink alcohol, or use illicit drugs. His blood pressure is 145/88 mm Hg. The lungs are clear to auscultation. Cardiac examination shows an S4 gallop. Laboratory studies are within normal limits. Which of the following is the most likely cause of this patient's insomnia? Malignancy Hypertension Major depressive disorder Hyperthyroidism Normal aging Sleep apnea

Normal aging

A 75-year-old woman with osteopenia comes to the physician for a follow-up examination. Current medications are occasional acetaminophen, and supplemental calcium and vitamin D. She does not smoke cigarettes or drink alcohol. She follows a high-protein diet and walks 3 miles daily. She is 157 cm (5 ft 2 in) tall and weighs 59 kg (130 lb); BMI is 24 kg/m2. Her weight has not changed during the past 10 years. Her pulse is 68/min, and blood pressure is 124/72 mm Hg. Physical examination shows no abnormalities. Laboratory studies from today and 10 years ago show: Serum creatinine: 1.0 mg/dL Urine Creatinine excretion: 1000 mg/24 h Creatinine clearance: 69.4 mL/min Which of the following is the most likely cause of this patient's decreased creatinine clearance rate? High-protein diet Over-supplementation with vitamin D Normal aging Renovascular disease Polycystic kidney disease

Normal aging

A 53-year-old homeless man with alcoholism is brought to the emergency department by police shortly after he was found collapsed and coughing on a park bench. He appears cachectic. His temperature is 39°C (1 02.2° F), pulse is 90/min, respirations are 18/min, and blood pressure is 112/78 mm Hg. Physical examination shows clubbing of the digits. Oral examination shows severe halitosis, several missing teeth, and numerous dental caries. Chest examination shows increased fremitus, dullness to percussion, and tubular breath sounds over the right lower lung field. His leukocyte count is 11,400/mm 3. Examination of a sputum sample shows gross purulence, gram-positive cocci in chains, and gram-negative bacilli. A chest x-ray shows cavitation in the right lower lung lobe with a surrounding infiltrate. Sputum cultures will most likely grow which of the following organisms? Mycoplasma pneumoniae Streptococcus pyogenes (group A Chlamydophila pneumoniae Haemophilus influenzae Normal oral flora

Normal oral flora

A 69-year-old woman is brought to the physician by her son because of a 2-month history of memory problems, difficulty walking, and urinary incontinence. He says, "She's always been very sharp, but recently she's had a lot of trouble functioning." She has forgotten on several occasions to turn off the stove when cooking, and she once forgot she had turned on the bathtub until she noticed water coming down the walls from the upstairs bathroom. She also is unsteady on her feet and has fallen down at home on several occasions. She reports that her mood is fine. Physical examination shows mild nystagmus. She has a broad-based gait and is unable to walk in a straight line without assistance. Neurologic examination shows no other focal findings. Mini-Mental State Examination score is 20/30. Which of the following is the most likely diagnosis? Parkinson disease Dementia, Alxheimer type Normal pressure hydrocephalus Pcik disease Cerebral neoplasm

Normal pressure hydrocephalus

A 73-year-old woman is brought to the physician because of a 1-month history of urinary frequency and urgency. She also has had a 3-month history of difficulties with walking, conversation, and memory. Neurologic examination shows normal strength and sensation. Her Mini-Mental State Examination score is 21/30. She walks with a wide-based, shuffling gait. A CT scan of the head shows marked enlargement of the ventricles. Which of the following is the most likely diagnosis? Parkinsons disease Huntington disease Dementia, Alzheimer type Normal pressure hydrocephalus Small vessel disease Atherosclerosis of the carotid artery

Normal pressure hydrocephalus

A 25-year-old woman with polycystic kidneys has a 3-month history of progressive weakness and fatigue, headaches, hypertension, loss of appetite, and itching. Her serum creatinine concentration is 4 mg/dl. Which of the following laboratory abnormalities in serum HCO3-, inorganic phosphate, and PTH is expected in this patient? Normal, increased, increased

Normal, increased, increased

A demonstration is performed during a lecture on muscle physiology in which a student is asked to fully extend his right arm with the palm up. Two large textbooks are placed on his palm, one at a time. Which of the following facilitates the maximum amount of tension that allows the student to keep his arm extended in place under the increasing weight of the books? Number of motor units recruited Amount of muscle phosphocreatine Amount of Ca2+ released from thesarcoplasmic reticulum Rate of cross-bridge recycling Amplitude of the action potential

Number of motor units recruited

A 2-month-old infant has a 5-cm strawberry hemangioma on the cheek that is increasing in size. No other lesions are noted. Which of the following is the most appropriate next step in management? Observation of the lesion Surgical removal of the lesion Intralesional corticosteroid treatment Solid carbon dioxide application to the lesion Laser therapy to the lesion

Observation of the lesion

A 2-month-old infant has a 5-cm strawberry hemangioma on the cheek that is increasing in size. No other lesions are noted. Which of the following is the most appropriate next step in management? Surgical removal of the lesion Laser therapy to the lesion Observation of the lesion Solid carbon dioxide application to the lesion Intralesional corticosteroid treatment

Observation of the lesion

A 34-year-old female is brought to the emergency department after she collapsed when she returned from a nonstop flight from New Delhi to New York City. She has smoked 1 pack of cigarettes daily for 16 years. She is sexually active and uses an oral contraceptive. Her pulse is 125/min, and blood pressure is 75/50 mm Hg. Physical examination shows jugular venous distention. The lungs are clear to auscultation. Pulmonary artery catheterization shows a pulmonary systolic/diastolic arterial pressure of 50/25 mm Hg (N: 15 to 30/3 to 12) and a pulmonary artery wedge pressure of 5 mm Hg (N: 8 to 16). Which of the following is the most likely cause of her pulmonary hypertension? Occlusion of pulmonary arteries Increased pulmonary venous resistence Pericardial tamponade Occulsion of left ventricular outflow Increased pulmonary blood flow

Occlusion of pulmonary arteries

A 23-year-old woman comes to the physician because of a 3-day history of a drooping right eyelid. She also has a 2-week history of visual discomfort when in a room with bright lights. On examination, the right pupil is larger than the left; it also seems like the right eye is looking somewhat down and to the right. These findings most likely suggest dysfunction to which of the following structures? Medial longitudinal fasciculus Trochlear nerve Oculomotor nerve Right superior cervical ganglion Abducens nerve

Oculomotor nerve

A 30-year-old woman comes to the physician because of a 1-week history of left eye pain and blurred vision and a 2-month history of progressive weakness of her right arm and both legs. Ophthalmologic examination shows swelling of the optic disc on the left. Physical examination shows spasticity and hyperreflexia of the involved extremities. An MRI of the brain shows multiple T2-weighted bright signal abnormalities in white matter. Analysis of cerebrospinal fluid obtained from a lumbar puncture shows increased mononuclear cells and oligoclonal banding. Cytokines released by T-lymphocytes infiltrating this patient's central nervous system are most likely causing inflammation and destruction of which of the following? Schwann cells Astrocytes Microglia Neurons Oligodendrocytes Ependymal cells

Oligodendrocytes

A 15-month-old girl is brought to the emergency department after a generalized tonic-clonic seizure at home. The seizurestopped spontaneously after 2 minutes, and she seemed sleepy afterward. Her temperature prior to arrival was 39.6°C (103.2°F),and paramedics administered rectal acetaminophen. Her parents state that yesterday she had a mild runny nose but otherwise hasbeen well. There is no personal or family history of serious medical illness or seizures. Development has been appropriate forage. On arrival, she is afebrile, alert, and interactive. Physical examination shows a supple neck. Neurologic examination showsno focal findings. Which of the following is the most appropriate next step in management? EEG Oral administration of phenobarbital Lumbar puncture CT scan of the head Reassurance

Oral administration of phenobarbital

A 55-year-old man who is a farmer is brought to theemergency department 30 minutes after his wife found himunresponsive in their barn. She reports that he was foamingat the mouth and had evidence of tearing of the eyes,vomiting, and diarrhea. He is unresponsive to painfulstimuli. His pulse is 45/min, and blood pressure is 90/60mm Hg. Bilateral diffuse wheezes are heard on auscultationof the chest. This patient has most likely sustained poisoning by which of the following? Ethylene glycol Methanol Organophosphate Carbon monoxide Ethanol

Organophosphate

A 3-year-old boy with sickle cell disease has the insidious onset of fever and persistent pain in his left foot over the past 3 weeks. Hematocrit is stable. Leukocyte count is 15,000/mm3 with marked predominance of neutrophils. Which of the following is the most likely explanation for these findings? Hemarthrosis Tuberculous arthritis Osteomyelitis Aseptic necrosis Acute sickle cell crisis

Osteomyelitis

A 73-year-old woman is brought to the emergency department because of severe back pain for 1 day. She has had no recent falls or trauma to the area. Menopause occurred at the age of 52 years. Her temperature is 37°C (98.6°F), pulse is 92/min, and blood pressure is 140/92mm Hg. Physical examination shows spinal tenderness at T8. A spinal x-ray shows fractures at T8 and T10. Which of the following is the most likely underlying cause of this patient's condition? Osteomalacia Spinal metastasis Osteoporosis Pott disease Osteitis deformans (Paget disease)

Osteoporosis

A previously healthy 15-year-old girl comes to the physician because of increasing left ear pain during the past 3 days. Vital signs are within normal limits. Examination of the left ear shows edema and erythema of the auditory canal with a greenish discharge. Manipulation of the left pinna elicits pain. The tympanic membrane can only partially be visualized, and examination of what can be seen appears normal with normal mobility. Examination of the right ear shows no abnormalities. Which of the following is the most likely diagnosis? Otitis externa Tympanic membrane perforation Acute otitis media Mastoiditis Furunculosis

Otitis externa

A previously healthy 15-year-old girl comes to thephysician because of increasing left ear pain during thepast 3 days. Vital signs are within normal limits.Examination of the left ear shows edema and erythema ofthe auditory canal with a greenish discharge. Manipulationof the left pinna elicits pain. The tympanic membrane canonly partially be visualized, and examination of what canbe seen appears normal with normal mobility. Examinationof the right ear shows no abnormalities. Which of thefollowing is the most likely diagnosis? Acute otitis media Furunculosis Tympanic membrane perforation Otitis externa Mastoiditis

Otitis externa

A Gram stain is performed on a sputum specimen. Pink rods and blue cocci are observed. Which of the following features is characteristic only of the pink rods? Polysaccharide capsule Peptidoglycan cell wall Endoplasmic reticulum Growth inhibition by ciprofloxacin Outer membrane

Outer membrane

An otherwise healthy 35-year-old man comes to the physician because of several episodes of dizziness and fainting during the past 2 months. His father and several paternal uncles died suddenly. Physical examination shows no abnormalities. Serum lipid studies are within the reference range. Angiography shows no blockage of the coronary arteries. An ECG shows a prolonged QT interval. A decreased activity of which of the following is the most likely cause of these findings? Outward (delayed) rectifying potassium channel Inward activating potassium channel Voltage-gated L-type calcium channel Voltage-gated fast sodium channel Voltage-gated T-type calcium channel

Outward (delayed) rectifying potassium channel

A 5-year-old boy is brought to the physician by his mother because of a 3-month history of headache, clumsiness, and fatigue. He had a generalized tonic-clonic seizure 1 month ago. His mother says that he often falls while running. Ophthalmologic examination shows ptosis of the left eye and bilateral external ophthalmoplegia. Physical examination shows weakness of the left lower extremity. Serum studies show an increased lactate concentration. Analysis of a calf muscle biopsy specimen obtained from the left lower extremity shows ragged red fibers. Which of the following metabolic processes is most likely impaired in this patient? Fatty acid oxidation Pentose phosphate pathway Glycolysis Gluconeogenesis Oxidative phosphorylation

Oxidative phosphorylation

An experimental model of treatment for sickle cell disease involves reactivating the genes that code for the chains of fetal hemoglobin. This treatment is most likely to increase the affinity of hemoglobin for which of the following? Oxygen 2,3-Bisphosphoglycerate Protons Chloride ions Carbon dioxide

Oxygen

A 22-year-old nulli gravid woman comes to the physician because she has been unable to conceive for 18 months. She has had irregular menses since menarche at the age of 13 years. She is 165 cm (5 ft 5 in) tall and weighs 88 kg (195 lb); BMI is 32 kg/m2. Her temperature is 36.8° C (98.2°F), pulse is 88/min, respirations are 14/min, and blood pressure is 140/90 mm Hg. Physical examination shows moderate hirsutism and mild acne over the face and back. Pelvic examination shows no abnormalities of the exterior genitalia; a bimanual examination is limited by the patient's obesity. Which of the following is the most likely cause of this patient's infertility? Primary Ovarian Failure Hypothyroidism PCOS Klinefelter Syndrome Pituitary Adenoma Androgen-secreting tumor

PCOS

Which of the following is most likely to be associated with acute myelogenous leukemia? PML-RARα oncogene LIMK1 gene CCND2 gene ABL-BCR oncogene LIS1 gene WT1 gene

PML-RARα oncogene

A previously healthy 35-year-old woman develops hypoxemia 35 minutes after ingesting a near-lethal dose of barbiturates. She has not aspirated. Which of the following sets of arterial blood gas values (in mm Hg) is most likely in this patient? PO2: 50, PCO2: 80, A-a: 10

PO2: 50, PCO2: 80, A-a: 10

Six hours after undergoing surgical excision of the distalcolon for cancer, a 77-year-old man has decreased urine output. His urine output has been 10 mL/h during the past 3 hours. During the 4-hour operation, he lost 500 mL of blood and underwent transfusion of 1 unit of packed red blood cells. His initial postoperative course was uncomplicated. Two years ago, he had a myocardial infarction and underwent coronary artery bypass grafting. He has a 10-year history of hypertension well controlled with atenolol. Current medications include morphine and labetalol. He currently appears pale and diaphoretic. His temperature is 37°C (98.6°F), pulse is 85/min, respirations are 14/min, and blood pressure is 90/60 mm Hg. Pulse oximetry on 2 L/min of oxygen via nasal cannula shows an oxygen saturation of 89%. Cardiopulmonary examination shows no abnormalities. The abdomen is nondistended with mild tenderness over the incision. His hematocrit is 24%,serum sodium concentration is 140 mEq/L, and serum creatinine concentration is 2.1 mg/dL. The patient is switched to administration of 100% oxygen by a nonrebreathing face mask. The most appropriate next step inmanagement is administration of which of the following? 5% Dextrose in water Furosemide Bumetanide 0.45% Saline Packed red blood cells Fresh frozen plasma

Packed red blood cells

Ten days after admission to the hospital because of acute pancreatitis, a 56-year-old man with alcoholism develops chills and temperatures to 39.4°C (103°F). Examination shows a tender abdomen with hypoactive bowel sounds. Which of the following is the most likely diagnosis? Splenic vein thrombosis Retroperitoneal hemorrhage Pancreatic insufficiency Perforated duodenal ulcer Pancreatic abscess

Pancreatic abscess

A 70-year-old man has a 4-month history of weight loss, abdominal pain, and diarrhea. Stool analysis shows increased excretion of neutral fat and muscle fiber. A d-xylose test for carbohydrate absorption shows no abnormalities. Examination of tissue obtained on intestinal biopsy shows no abnormalities. This patient is most likely to respond favorably to administration of which of the following agents? Azathioprine Pancreatic enzymes Intrinsic factor Prednisone Antibiotics

Pancreatic enzymes

A 52-year-old woman comes to the physician because of a 3-month history of diarrhea and intermittent abdominal pain that radiates to her back. The pain is exacerbated by eating. She describes her stools as greasy, foul-smelling, and difficult to flush. She has had a 4.5-kg (10-lb) weight loss during the past 4 months. She has a history of chronic alcohol abuse. Examination shows mild epigastric tenderness. An x-ray of the abdomen shows calcifications in the epigastrium. Which of the following is the most likely diagnosis? Celiac disease Lactose intolerance Pancreatic insufficiency Bacterial overgrowth Malabsorption of bile salts

Pancreatic insufficiency

A 44-year-old woman has had pain, swelling, and stiffness in both hands over the past 6 months. The pain and stiffness are worst in the mornings, but symptoms improve with activity. Examination shows painless nodules on the extensor surfaces of the arms and tenderness and swelling of the metacarpophalangeal and proximal interphalangeal joints. Which of the following is involved in the pathologic process affecting her joints? Necrotizing vasculitis Pannus formation Crystal deposition Bacterial invasion Caseating necrosis

Pannus formation

An autopsy is done on a 46-year-old woman who died of adenocarcinoma of the colon. Examination of the neck shows a 5-cm, rounded mass next to the bifurcation of the carotid artery. A section of the mass is shown in the photomicrograph. Immunohistochemistry of the section is positive for synaptophysin, chromogranin and neuron-specific enolase. Electron microscopy shows numerous electron-dense, membrane-bound neurosecretory granules. Examination of the adrenal glands shows no masses. Which of the following is the most likely diagnosis? Metastatic colonic adenocarcinoma Paraganglioma Papillary carcinoma of the thyroid gland Parathyroid adenoma Metastatic squamous cell carcinoma of the larynx

Paraganglioma

An 18-month-old girl is brought to the physician by her mother because of a 2-day history of fever and a nonproductive, barking cough. Her temperature is 38.5°C (101.3°F), pulse is 90/min, respirations are 28/min, and blood pressure is 104/64 mm Hg. Stridor is heard on expiration. A chest x-ray shows no abnormalities. Which of the following is the most likely causal virus? Parainfluenza virus Measles virus Influenza virus Rhinovirus Coxsackievirus

Parainfluenza virus

An 18-month-old girl is brought to the physician by her parents because of a 2-day history of progressive cough and hoarseness. Her temperature is 39°C (102.2°F), pulse is 88/min, respirations are 24/min, and blood pressure is 100/70 mm Hg. Pulse oximetry on room air shows an oxygen saturation of 95%. Physical examination shows mild erythema of the oropharyngeal and laryngeal mucosa but no exudate. A harsh, barking cough is heard. Laboratory studies are within the reference range. A rapid streptococcal test result is negative. The patient's condition improves within 4 days. Which of the following infectious agents is the most likely cause of these findings? Staphylococcus aureus Mycoplasma pneumoniae Respiratory syncytial virus Bordetella pertussis Parainfluenza virus Influenza A virus Streptococcus pyogenes (group A) Corynebacterium diphtheriae

Parainfluenza virus

A 31-year-old woman is evaluated because of a 1-year history of infertility. Her hysterosalpingogram is shown. The most likely cause of these findings is malformation of which of the following embryonic tissues? Mesonephric (wolffian) ducts Germ cells Paramesonephric (mullerian) ducts Urogenital sinus Urogenital folds Gonadal ridges

Paramesonephric (mullerian) ducts

A 30-year-old woman comes to the physician for a routinehealth maintenance examination. She takes no medications.Physical examination shows no abnormalities. Serum studiesshow a calcium concentration of 12 mg/dL. An increase inwhich of the following substances is the most likely cause ofthe serum finding in this patient? Vitamin A Parathyroid hormone Integrins Calcitonin Bone morphogenic protein

Parathyroid hormone

A 59-year-old man undergoes a total thyroidectomy and central lymph node dissection for a 4-cm follicular carcinoma of the thyroid. Twelve hours after the procedure, he has paresthesias of his hands and feet. Vital signs are stable, but carpal spasm is noted on inflation of the blood pressure cuff. Which of the following sets of laboratory findings in serum is most likely in this patient? Parathyroid hormone: decreased, calcium: decreased

Parathyroid hormone: decreased, calcium: decreased

A 35-year-old man with a long-standing history of chronic atrophic gastritis comes to the physician because of increased lethargy and dizziness during the past 2 weeks. Physical examination shows pale skin and mucous membranes. There is no palpable adenopathy. The lungs are clear to auscultation. Heart sounds are normal. The liver edge is palpable. Test of the stool for occult blood is negative. Neurologic examination shows decreased vibratory sensation. Laboratory studies show a hematocrit of 27%, mean corpuscular volume of 112/fL, and platelet count of 200,000/mL. The absence of sufficient numbers of which of the following cell types best explains these findings? Goblet Paneth Mucous Parietal Undifferentiated crypt Enteroendocrine Chief Columnar absorptive

Parietal

A 60-year-old man comes to the physician because of a 6-month history of fatigue. Four years ago, he had a subtotal gastrectomy after he sustained a gunshot wound to the abdomen. He drinks six to eight beers daily. Physical examination shows paresthesias of both hands. Laboratory studies show: Hemoglobin 8 g/dL Hematocrit 24% Mean corpuscular volume 115 μm3 Leukocyte count 5000/mm3 Platelet count 165,000/mm3 RBC folic acid 500 ng/mL (N=1200) Serum vitamin B12 (cobalamin) 10 pg/ml (N=160-195) The most likely cause of his anemia is the absence of which of the following? Parietal cells Transcobalamin II Chief cells Cardiac glands G (gastrin) cells

Parietal cells

Elderly patient with a history of progressive loss of facial expression, tremors, rigidity, and slowing of voluntary movement. Histological analysis reveals loss of pigmented cells within the substantia nigra. Which of the following represents the most likely diagnosis? Corticobasal degeneration Essential tremor Ideopathic familial ganglia calcification Progressive supranuclear palsy Parkinson disease

Parkinson disease

A 28-year-old woman comes to the physician for advice on how to lose weight. She tells the physician that she binges on high-carbohydrate foods two to three times weekly, usually forcing herself to vomit after a binge. She is 168 cm (5 ft 6 in) tall and weighs 63 kg (140 lb); BMI is 23 kg/m2. Which of the following additional physical findings is most likely in this patient? Sparse axillary and pubic hair Decreased vibratory sensati on at the ankles Parotid gland enlargement Bradycardia Sluggish deep tendon reflexes

Parotid gland enlargement

A 60-year-old man comes to the physician because of a 2-week history of fatigue and exercise intolerance. One month ago, he was diagnosed with stage 1 hypertension and began treatment with propranolol. His pulse is 56/min, compared with 72/min 1 month ago. His treatment is switched to pindolol. His symptoms resolve within 1 month, and his pulse is 68/min. Which of the following mechanisms of action of pindolol best explains its effect on this patient's heart rate? Partial agonism Competitive agonism Inverse agonism Competitive antagonism

Partial agonism

A 25-year-old primigravid woman at 12 weeks' gestation comes to the physician 1 hour after she passed a small amount of tissue vaginally at home. Gross examination of this tissue shows an obvious fetus. Microscopic examination of tissue obtained via dilatation and curettage shows chorionic villi with focal edema and trophoblastic proliferation. Which of the following is the most likely diagnosis? Placenta accreta Choriocarcinoma Placental site trophoblastic tumor Complete hydatidiform mole Partial hydatidiform mole

Partial hydatidiform mole

An 18-month-old male presents to the pediatric cardiologist for a follow-up examination. His was diagnosed with a continuous, grade 3/6, rumbling, machine-like murmur that is heard best over the mid to upper left sternal border. Which of the following is the most likely diagnosis? Patent ductus arteriosus Sinus of Valsalva fistula Ostium secundum defect Mitral valve regurgitation Ventricular septal defect Ostium primum defect

Patent ductus arteriosus

Following a positive diagnosis of trisomy 18 (Edwards syndrome), RFLP is performed determine how this chromosomal anomaly occurred. Given the results (see figure 1), which of the following options best explains how this mutation occurred? Paternal meiosis II Paternal meiosis I Maternal meiosis I Maternal Meiosis II

Paternal meiosis II

Description of innate immune response. Which of the following most likely represents how the innate immune response is mediated? Pattern recognition receptors Positive regulation Negative regulation Option 5

Pattern recognition receptors

A 52-year-old man is brought to the physician by a social worker because his neighbors have reported that he has been confused and not taking care of himself. He has a 4-month history of diarrhea. Physical examination shows extreme muscle wasting, stomatitis, and a diffuse rash that is worse in sun-exposed areas. The patient is mildly disoriented and has poor long-term memory and paranoid ideation. Which of the following is the most likely diagnosis? Scurvy Wernicke syndrome Pellagra Whipple disease Beri beri

Pellagra

An 80-year-old man with type 2 diabetes mellitus is brought to the physician because a 2-month history of severe constipation. Use of over- the-counter laxatives has not relieved his symptoms. Abdominal examination shows distention. Colonoscopy shows no abnormalities. This patient most likely has dysfunction of which of the following nerves? Pelvic splanchnic Sacral sympathetic Hypogastric Inferior rectal Perineal

Pelvic splanchnic

An investigator studying the molecular events in the underlying causes of an autoimmune disease develops a collection of monoclonal antibodies to ribosomal proteins. One of these monoclonal antibodies is found to inhibit the ribosome peptidyl transferase. Which of the following is most likely affected in the presence of this antibody? Release of the mRNA from ribosomes Dissociation of 80S ribosomes into 60S and 40S subunits Assembly of 80S ribosomal subunits Establishment of covalent bonding between amino acids Attachment of amino acids to tRNA Peptide bonds between amino acids

Peptide bonds between amino acids

A 57-year-old hospitalized man undergoes right subclavianvenous catheterization for hyperalimentation. He is currently being treated for a small bowel fistula. While the results of an x-ray of the chest to check the catheter position are pending, the patient suddenly becomes agitated. His pulse is 110/min, and blood pressure is 70/50 mm Hg. Examination shows jugular venous distention. The lungs are clear to auscultation. Breath sounds are equal bilaterally. The trachea is midline. An x-ray of the chest shows a catheter in the superior vena cava, transversing the right ventricle and crossing to the left of the midline. Which of the following is the most likely cause of this patient's hypotension? Pulmonary artery perforation Staphylococcal bacteremia Tension pneumothorax Air embolism Pericardial tamponade

Pericardial tamponade

An 18-year-old man is brought to the emergency department 10 minutes after he sustained a stab wound to his chest. On arrival, he is unresponsive to painful stimuli. His pulse is 130/min, respirations are 8/min and shallow, and palpable systolic blood pressure is 60 mm Hg. He is intubated and mechanically ventilated, and infusion of 0.9% saline is begun. After 5 minutes, his pulse is 130/min, and blood pressure is 70/40 mm Hg. Examination shows a 2-cm wound at the left sixth intercostal space at the midclavicular line. There is jugular venous distention. Breath sounds are normal. The trachea is at the midline. Heart sounds are not audible. Which of the following is the most likely cause of these findings? Bronchial disruption Hemothorax Tension pneumothorax Pericardial tamponade Myocardial infarction

Pericardial tamponade

A previously healthy 40-year-old man is brought to the emergency department because of constant substernal chest pain for 12 hours that is exacerbated by coughing and inspiration. The pain is relieved with sitting up and leaning forward. There is no family history of heart disease. His temperature is 38°C (100.4°F), pulse is 120/min, and blood pressure is 110/60 mm Hg. The lungs are clear to auscultation. Cardiac examination shows distant heart sounds. An ECG shows diffuse ST-segment elevation in all leads. An x-ray of the chest shows normal findings. The most likely cause of his condition is injury to which of the following tissues? esophageal sphincter Myocardium aortic intima Pericardium pleura

Pericardium

A 9-year-old girl is brought to the emergency department by her father because of severe pain in her right shoulder after a fall 1 hour ago. Physical examination shows pain on movement of the right shoulder and a tender right clavicle. An x-ray of the shoulder shows a nondisplaced fracture of the right clavicle. Which of the following structures of the clavicle will most likely assist in producing new bone to heal this fracture? Epiphysis Haversian canal Volkmann canal Lamella Periosteum

Periosteum

A 2-month-old boy is brought to the physician because of hypotonia and poor feeding since birth. Physical examination shows large fontanels, midface hypoplasia, hepatomegaly, and cryptorchidism. Serum studies show increased concentrations of very-long-chain fatty acids, phytanic acid, and pipecolic acid. If this patient's hepatocytes were analyzed using electron microscopy, results would most likely show the absence of which of the following organelles? Peroxisomes Endoplasmic reticulum Nucleoli Mitochondria Lysosomes

Peroxisomes

A 25-year-old man is brought to the emergency department because of a 2-hour history of nausea, vomiting, abdominal cramping, and difficulty passing flatus. Examination of the face shows the lesions in the photograph; similar lesions are seen on the fingers and toes. Laboratory studies show hypochromic microcytic anemia. Test of the stool for occult blood is positive. Which of the following is the most likely diagnosis? Muir-Torre syndrome Neurofibromatosis Peutz-jeghers syndrome Gardner syndrome Cowden disease

Peutz-jeghers syndrome

During which phase of the ventricular action potential does norepinephrine influence the influx of calcium ions? Phase 2 Phase 1 Phsae 0 Phase 3 Phase 4

Phase 2

A 30-year-old woman comes to the physician because of a 2-year history of episodes of numbness, blanching, and a bluish color to her ears, fingers, and toes following emotional upset or exposure to cold. Her vital signs are within normal limits. Physical examination shows no abnormalities. It is most appropriate for the physician to advise the patient that she should avoid taking which of the following drugs? Phenylephrine Acetaminophen Dextromethorphan Diphenhydramine Ibuprofen

Phenylephrine

A 3-year-old boy with a history of unexplained fever, lack of perspiration, absence of response to noxious stimuli, and self-mutilating behavior is diagnosed with congenital insensitivity to pain with anhidrosis. Genetic analysis shows a missense mutation of the tyrosine kinase domain of the TrkA gene. Assuming this is the only signaling defect in this patient, which of the following processes has most likely been disrupted which of the following? Retrograde transport of nerve growth factor from nerve terminals Formati on of TrkA homodimer in response to nerve growth factor Phosphorylation of downstream molecules in response to nerve growth factor Synthesis of nerve growth factor Binding of nerve growth factor to its receptor

Phosphorylation of downstream molecules in response to nerve growth factor

Following an operation, a 65-year-old patient has a lung region that is underventilated but well perfused. This condition will lead to an increase in which of the following? Anatomic dead space PO2 Physiologic dead space Physiologic shunt Alveolar dead space

Physiologic shunt

A 15-year-old Native American boy is brought to the emergency department 1 hour after he ingested a broth made from jimsonweed. He has hallucinations. His pulse is 128/min. Physical examination shows flushing and dry mouth. He is disoriented to person, place, and time. The pharmacologic antidote must be carefully titrated to avoid potential cardiac effects. Which of the following drugs is the most appropriate antidote? Trimethaphan Carbachol Physostigmine Neostigmine Pilocarpine

Physostigmine

A 25-year-old man comes to the physician because of a 4-day history of pain and burning with urination and frequent urinary urgency. He is sexually active and has several female partners; he does not use condoms. His temperature is 37°C (98.6°F). Urinalysis shows 2+ protein; cultures grow on chocolate agar. A Gram stain shows gram-negative diplococci. The organism causing this patient's symptoms most likely produces which of the following virulence factors? Pili Spores Free radicals Exotoxins Elementary bodies

Pili

A 6-year-old boy is brought to the physician by his mother because of a 5-day history of intermittent vomiting and a 3-month history of progressive clumsiness. His mother says that he can no longer ride his bicycle and has difficulty getting in and out of the car. Funduscopic examination shows bilateral papilledema. Neurologic examination shows impaired upward gaze and pupil response to the right. Sensation is intact. He walks with a shuffling gait. A CT scan of the head shows enlarged lateral and third ventricles and a 2-cm mass. Which of the following is the most likely location of the mass? Pineal gland Cerebellar hemisphere Lateral medulla Frontal lobe

Pineal gland

A 35-year-old woman has hypertension and truncal obesity.Serum studies show:Cortisol (AM) 100 μg/dL (N=5-20)Cortisol (PM) 100 μg/dL (N=2.5-10)ACTH (AM) 100 pg/mL (N=20-100)Cortisol 8 h after 1 mgdexamethasone 95 μg/dLCortisol 8 h after 8 mgdexamethasone 30 μg/dLWhich of the following is the most likely cause of thiswoman's increased serum cortisol concentration? Ectopic corticotropin-releasing hormoneproducing neoplasm Adrenocortical adenoma Pituitary microadenoma Ectopic corticotropin-secreting neoplasm Self-administration of synthetic glucocorticoids

Pituitary microadenoma

A 30-year-old woman, gravida 2, para 2, develops severe vaginal bleeding immediately after vaginal delivery of a male newborn at term. The hemorrhage continues during the next hour, and the patient undergoes an emergency hysterectomy to control the bleeding. Examination of the uterus shows fragments of placenta adherent to the uterine wall. There is absence of the decidua basalis. Which of the following is the most likely diagnosis? Placenta accreta Circumvallate placenta Chronic villitis Succenturiate lobe Abruptio placentae

Placenta accreta

A 35-year-old primigravid woman at approximately 36 weeks' gestation comes to the emergency department because of a 6-hour history of heavy vaginal bleeding. She has had no prenatal care. Ultrasonography shows that the placenta is located over the cervicalos. Attempts to stop the bleeding are unsuccessful, and she undergoes a cesarean delivery. Which of the following is the most likely diagnosis? Placenta previa Placental infarction Abruptio placentae Placenta accreta Placenta percreta

Placenta previa

A 77-year-old man comes to the physician because ofswelling of his legs and feet for 6 months. He has a 40-yearhistory of alcoholism and a 5-year history of hepaticdisease. Physical examination shows ascites and a 2+ edemaof the lower extremities. A decrease in which of thefollowing most likely promotes edema formation in thispatient? Plasma colloid oncotic pressure Filtration coefficient Capillary hydrostatic pressure Interstitial colloid osmotic pressure Interstitial fluid hydrostatic pressure

Plasma colloid oncotic pressure

A 62-year-old woman with chronic renal disease comes to the physician for a follow-up examination. She is 157 cm (5 ft 2 in) tall and weighs 52 kg (115 lb); BMI is 21 kg/m2. Her pulse is 78/min, and blood pressure is 140/100 mm Hg. Laboratory studies show anemia. Treatment with intravenous recombinant erythropoietin is started. Measurement of which of the following is most appropriate to monitor the effectiveness of this pharmacotherapy? Arterial blood pH Oxygen saturation of the blood Serumurea nitrogen concentration Arterial pressure of carbon dioxide Plasma hemoglobin concentration

Plasma hemoglobin concentration

A 70-year-old woman comes to the physician because of increasingly severe pain in the right knee over the past 3 months. She has fallen repeatedly. She has a 10-year history of symptomatic osteitis deformans (Paget disease). She has bowed tibias and tenderness of the proximal right tibia. An x-ray of the right knee shows a fracture of the proximal tibia with elevated periosteum and a sunburst pattern. An x-ray of the chest shows numerous pulmonary nodules of various sizes. Which of the following findings is most likely on biopsy of the proximal tibia? Clusters of mucin-producing neoplastic cells forming glands Neoplastic chondrocytes filling lacunes Pleomorphic neoplastic cells producing new woven bone Closely packed, small, round, uniform neoplastic cells Neoplastic plasma cells in sheets

Pleomorphic neoplastic cells producing new woven bone

A 60-year-old man comes to the physician because of a 1-month history of progressive shortness of breath with exertion. Examination of the chest shows: Breath sounds Percussion note Tactile fremitus Adventiti ous sounds Which of the following is the most likely diagnosis? Chronic bronchitis Pneumothorax Pleural effusion Pulmonary embolism Bullous emphysema Bronchiectasis Lobar pneumonia Asthmatic bronchitis

Pleural effusion

A previously healthy 34-year-old woman had a single rigor3 days ago. Since then she has had temperatures to 38.9°C(102°F), shortness of breath with minimal exertion, andcough productive of rust-colored sputum. An x-ray of thechest shows consolidation in the right lower lobe of thelung. Which of the following is the most likely diagnosis? Pneumococcal pneumonia Mycoplasmal pneumonia Toxoplasmosis Pseudomonal pneumonia Tuberculosis

Pneumococcal pneumonia

A 36-year-old man who is seropositive for HIV has had a nonproductive cough for 2 weeks. An x-ray of the chest shows a diffuse interstitial infiltrate. Silver stain of a lung biopsy specimen is shown. Which of the following is the most likely causal organism? Pneumocystis Jiroveci (formerly P. carinit) Mycobacterium tuberculosis Candida tropicalis O B) Coccidioides immitis O C) Cryptococcus neoformans O D) Cytomegalovirus O E) Mycobacterium tuberculosis O F) Pneumocystis Jiroveci (formerl y P. carinit) Cytomegalovrius Cryptococcus neoformans Coccidioides immitis

Pneumocystis Jiroveci (formerly P. carinit)

A 4-hour-old female newborn delivered at 30 weeks' gestation has respiratory distress. Her temperature is 36.5°C(97.7°F), pulse is 160/min, respirations are 85/min, and blood pressure is 68/40 mm Hg. Arterial blood gas analysis onroom air shows:pH 7.18PCO2 78 mm Hg PO2 55 mm Hg. Endotracheal intubation and mechanical ventilation are required. The primary cause of this patient's condition is adysfunction of which of the following cell types? Smooth muscle cells Alveolar macrophages Pneumocytes Pulmonary chondrocytes Pulmonary vascular endothelial cells

Pneumocytes

A 22-year-old basketball player who uses cocaine recreationally suddenly develops severe shortness of breath after a routine scrimmage. Physical examination shows respirations of 25/min. The left lung field is hyperresonant and breath sounds are diminished. Which of the following is the most likely diagnosis? Pneumothorax Corpulmonale Acute respiratory distress syndrome Pleural effusion Flash pulmonary edema

Pneumothorax

A 52-year-old man with a family history of renal failure comes to the physician because of abdominal pain over the past 6 months. He has been treated in the past for renal calculi and hypertension. Physical examination shows bilateral palpable masses in the upper abdomen. Urinalysis shows crenated erythrocytes and mild proteinuria. A CT scan of the abdomen identifies the cause of the patient's problem. Which of the following is the most likely diagnosis? Henoch-Schonlein purpura Glomerulonephritis Polycystic kidney disease Renal cell adenoma Adrenal adenoma

Polycystic kidney disease

A 28-year-old woman has hirsutism and irregular menstrual periods. Which of the following is the most likely underlying abnormality? Benign cystic teratoma of ovary Brenner tumor Polycystic ovarian disease Uterine endometriosis Dysgerminoma

Polycystic ovarian disease

A 21-year-old nulligravid woman who is not using contraception has had irregular menstrual periods since menarche at age 13 years. She has noted increased hair growth on her face and lower abdomen. On pelvic examination, there is copious cervical mucus and slightly enlarged irregular ovaries. Which of the following is the most likely cause of these findings? Polycystic ovarian syndrome Idiopathic hirsutism Adrenal adenoma Ovarian tumor Pituitary adenoma

Polycystic ovarian syndrome

Case of early-onset baldness. Which of the following is the most likely inheritance pattern of the hair loss in this patient's family? Autosomal dominant Polygenic Sporadic Mitochondrial Autosomal recessive

Polygenic

A 12-year-old African American girl is brought to the emergency department by her mother because of a 6-hour history of fever and severe rib pain. She has had numerous episodes in the past. Her paternal aunt and uncle have had similar symptoms since youth (autosomal recessive). Her temperature is 39°C (102.2°F). The lungs are clear to auscultation. Abdominal examination shows tenderness to palpation. A peripheral blood smear is shown in the photomicrograph. Which of the following is the most likely underlying cause (molecular/genetic cause) of this patient's symptoms? Abnormal folding of the hemoglobin molecule caused by increased Pco2 Abnormal hydrogen bonding in the hemoglobin caused by amino acid substitution on the a chain Decreased oxygen affinity caused by an amino acid substi tution in the hemoglobin a chain Polymerization of hemoglobin with hypoxic conditions due to amino acid substitution in the beta-chain Erythrocyte fragility caused by an amino acid substitution in glycoproteins on the cell surface

Polymerization of hemoglobin with hypoxic conditions due to amino acid substitution in the beta-chain

A 12-year-old girl is brought to the physician for a follow-up examination. Three months ago, she sustained a complex fracture of the left tibia, which required cast immobilization. The cast is removed. Examination of the left calf shows that it is smaller than the right calf. Which of the following processes in this patient's myocytes is the most likely cause of these findings? Glycogenolysis Polyubiquitinaton Oxidative phosphorylation Anaerobic glycolysis Fatty acid degradation

Polyubiquitinaton

Two hours after undergoing a right hepatic lobectomy, a 59-year-old woman has a distended abdomen. Her pulse is 120/min, and blood pressure is 100/60 mm Hg. Which of the following is the most likely cause of these findings? Deficiency of factor XII Poor mechanical hemostasis Deficiency of platelets Deficiency of factor III Deficiency of factor VII

Poor mechanical hemostasis

A 30-year-old man is brought to the physician by his wife because of a 1-day history of irrational behavior and severe abdominal pain. He began treatment with trimethoprim-sulfamethoxazole for a urinary tract infection 3 days ago. His urine turned a light burgundy color during a similar episode 1 year ago. His mother and his maternal grandfather have had similar symptoms. He appears anxious and restless. His pulse is 96/min. Physical examination shows diaphoresis. Serum studies show increased concentrations of 5-aminolevulinic acid (δ-ALA) and porphobilinogen. The physician suspects that a mutant allele is causing decreased activity of an enzyme involved in heme biosynthesis. This enzyme is most likely which of the following? -ALA synthase Uroporphyrinogen decarboxylase Porphobilinogen deaminase &-ALA dehydratase Ferrochelatase

Porphobilinogen deaminase

A 3-year-old boy is brought to the emergency department by his parents because of progressive fever and skin lesions during the past 24 hours. He appears ill. His temperature is 39.4 °C (102.9° F), pulse is 120/min, respirations are 20/min, and blood pressure is 110/60 mm Hg. Physical examination shows large, flaccid, bullous lesions that are prominent over the trunk and abdomen. Which of the following additional findings is most likely in this patient? Gram-negative rods in the bullous fluid Multinucleated giant cells on a scraping of an unroofed lesion Positive nares culture for toxin-producing staph aureus Elevated eosinophil count

Positive nares culture for toxin-producing staph aureus

An 18-year-old man comes to the physician because of nausea, headache, blood in his urine, and malaise for 2 days. Three weeks ago, he had severe pharyngitis that resolved spontaneously after several days without antibiotic therapy. His blood pressure is 165/88 mm Hg. Physical examination shows mild peripheral edema. His serum creatinine concentration is 2.1 mg/dL, and serum CH50 concentration is markedly decreased. Urinalysis shows: Protein 2+ RBC 25-50/hpf WBC 10-20/hpf RBC casts present. Which of the following is the most likely diagnosis? Post-streptococcal glomerulonephritis IgA nephropathy Malignant hypertension Wegener granulomatosis Goodpasture syndrome

Post-streptococcal glomerulonephritis

A 42-year-old woman is brought to the emergency department 30 minutes after the sudden onset of severe headache. She has no history of major medical illness and takes no medications. On arrival, her blood pressure is 163/90 mm Hg. The right pupil is 6 mm and nonreactive to light. Adduction of the right eye is impaired. Nuchal rigidity is noted. A CT scan of the head shows blood in the subarachnoid space. Cerebral angiography is most likely to show an aneurysm in which of the following arteries? Anterior cerebral Internal carotid Middle cerebral Posterior communicating Vertebral

Posterior communicating

A 20-year-old man comes to the physician because of a 3-month history of progressive thirst and urinary frequency; the thirst became excessive during the past 3 days. He says that he has been drinking approximately 10 quarts of ice water, in addition to approximately five large glasses of iced tea and lemonade, daily for 1 to 2 weeks. Physical examination shows no abnormalities. Urinalysis shows a specific gravity of less than 1.006. This patient most likely has dysfunction of which of the following endocrine structures? Adrenal medulla Thyroid parafollicular cells Thyroid follicular cells Adrenal cortex Anterior pituitary gland Posterior pituitary gland

Posterior pituitary gland

A 28-year-old woman, gravida 1, para 1, comes to thephysician because of progressive fatigue since delivering amale newborn 6 months ago. She is sleeping 8 to 10 hoursnightly, but she is still tired during the day. She also has nothad a menstrual period since her pregnancy. Pregnancy wascomplicated during the third trimester by severe bleedingfrom placenta previa. She required multiple bloodtransfusions during the cesarean delivery, but she did wellafter the delivery. She was unable to breast-feed hernewborn because of poor milk production. Which of thefollowing is the most likely diagnosis? Major depressive disorder Cardiomyopathy Anemia Thyroiditis Postpartum pituitary infarction

Postpartum pituitary infarction

An 81-year-old woman is admitted to the hospital because of a massive pulmonary embolism from a deep venous thrombosis. Her platelet count is 160,000/mm3. Appropriate pharmacotherapy is started. One week later, her platelet count is 55,000/mm3. The thrombocytopenia was most likely caused by a drug with which of the following mechanisms of action? Activates tissue plasminogen Selectively inhibits factor Xa Irreversibly inactivates cyclooxygenase Potentiates the action of antithrombin III Interferes with the carboxylation of coagulation factors

Potentiates the action of antithrombin III

A 25-year-old male is brought to the emergency department by friends with complainants of chest pain, fast heart rate, and dizziness. Examination reveals he is intoxicated with alcohol, but awake and responsive. His temperature is 98.2 F, blood pressure 140/96 mm Hg, heart rate is 130/min, and respirations are 12/min, SpO2 is 95% on room air. ECG show sinus tachycardia and wide QRS complex. Which of the following represents the most likely cause of the cardiac dysfunction in this patient? Blockage of Ca2+ channels Blockage of dopamine synthesis Potentiation of norepinephrine and blocking of its presynaptic uptake Potentiation of serotonin at postsynaptic sites in the atrioventricular node Stroke due to malignant hypertension

Potentiation of norepinephrine and blocking of its presynaptic uptake

A 6-year-old girl is brought to the physician by her mother because of a 4 -day history of round shiny bumps in areas where she has eczema. Her mother remembers seeing similar bumps on a playmate when they attended a pool party 3 weeks ago. The patient has no other symptoms. Physical examination shows firm, smooth, umbilicated papules 2 to 4 mm in diameter in clusters. The causal organism most likely belongs to which of the following viral groups? Herpesvirus Adenovirus Poxvirus Flavivirus Paramyxovirus

Poxvirus

An 8-year-old boy with moderate mental retardation is brought to the physician by his parents for a routine examination. The parents say that their son has recently become more stubborn and has begun to have temper tantrums. These episodes usually involve fighting about food. The parents are concerned that he will hurt himself because he picks his skin and pulls his hair when he is frustrated. The father says, "He spends his day looking for more things to eat." He is at the 10th percentile for height and 99th percentile for weight. Which of the following is the most likely diagnosis? Prader Willi syndrome Williams syndrome Fragile X syndrome Down syndrome Angelman syndrome

Prader Willi syndrome

A 25-year-old primigravid woman at 25 weeks' gestation comes to the physician for a routine prenatal examination. She uses cocaine one to two times weekly. Vital signs are within normal limits. Physical examination shows a uterus consistent in size with a 25-week gestation. Ultrasonography shows no abnormalities. This patient's fetus is at increased risk for which of the following because of her cocaine use? Down syndrome Heart defect Neural tube defect Prematurity Hypothyroidism

Prematurity

A 35-year-old woman has congestive cardiomyopathy and pitting edema. Her serum urea nitrogen concentration is 25 mg/dl, and serum creatinine concentration is 1.8 mg/dl. Furosemide therapy is started. Five days later, laboratory studies show: Serum: Na+: 130 mEq/L K+: 4.5 mEq/L Cl-: 90 mEq/L HCO3-: 30 mEq/L Urea nitrogen: 85 mg/dL Creatinine: 2.2 mg/dL Albumin: 3 g/dL Urine: Specific gravity: 1.023 RBC: 0/hpf WBC: 0-1/hpf Sediment: none The urinary fractional excretion of sodium is less than 1%. Which of the following is the most likely explanation for the observed decrease in renal function? Syndrome of inappropriate secretion of ADH (vasopressin) Obstructive renal failure Osmotic diuresis Hepatorenal syndrome Renal artery stenosis Acute tubular necrosis Glomerulonephritis Nephrogenic diabetes insipidus Diabetic nephropathy Prerenal azotemia

Prerenal azotemia

A 26-year-old man comes to the physician with his 27-year-old wife because of a 2-year history of inability to conceive. Previous evaluation of his wife showed no abnormalities. Physical examination shows long upper and lower extremities and small testes. Laboratory studies show an increased serum follicle-stimulating hormone concentration and estradiol:testosterone ratio. Cytogenetic studies show a 47,XXY karyotype. Molecular genetic studies show that one of the X chromosomes was inherited from the patient's father. An error of chromosome segregation most likely occurred during anaphase at which of the following stages of spermatogenesis in this patient's father? Type B spermatogonia Late spermatid Primary spermatocyte Primordial germ cell Spermatozoan Secondary spermatocyte Type A spermatogonia Early spermatid

Primary spermatocyte

A 3-year-old boy is brought to the emergency department by his grandmother because of a 30-minute history of severe pain in his right arm that began when his younger sister ran into him. The grandmother babysits the children daily in her home. She says, "This is the third time this year I've had to bring him here. It seems like he's always in a cast." He is at the 85th percentile for height and weight. Physical examination shows blue sclerae and normal teeth. Examination of the right upper extremity shows erythema, edema, and marked tenderness to palpation. There are no abnormal bruises or marks suggestive of trauma. An x-ray of the right upper extremity shows fracture of the humerus and osteopenia. This patient's condition is most likely due to a defect in which of the following biochemical processes? Elastin-fibrillin interaction Procollagen synthesis Mature metalloproteinase activation Transforming growth factor- secretion Endocytosis of transferrin receptor Fibroblast growth factor receptor signaling

Procollagen synthesis

A 23-year-old woman comes to the physician because of a5-month history of intermittent discharge from bothbreasts. Her last menstrual period was 6 months ago. Shetakes no medications and is otherwise healthy. She hasnever been sexually active. Physical examination showsscant white fluid expressible from the breasts bilaterally. Serum studies are most likely to show an increase in which of the following hormone concentrations? Testosterone Luteinizing hormone Follicle-stimulating hormone Progesterone Prolactin

Prolactin

An 18-year-old man comes to the physician because of a 1-year history of progressive headaches. An x-ray of the head shows a cystic tumor above the sella turcica. The serum concentration of which of the following hormones is most likely to be increased in this patient? Luteinzing Thyroid-stimulating ADH (vasopressin) Follicle-stimulating Prolactin Growth Adrenocorticotropic

Prolactin

A 23-year-old woman, gravida 1, para 1, is brought to the physician because she has not felt well since the delivery of her son 2 weeks ago; she has had ongoing fatigue, inability to breastfeed, and episodic light-headedness when in the upright position during this period. Pregnancy was complicated by preeclampsia and resulted in an urgent cesarean delivery. Immediately after delivery, she required blood transfusions because of bleeding and transient hypotension. She was discharged from the hospital after she had a stable hemoglobin concentration of 10.8 g/dl. She appears lethargic. While she is sitting, her pulse is 80/min, and blood pressure is 100/50 mm Hg; while she is standing, pulse is 85/min, and blood pressure is 86/44 mm Hg. Physical examination shows no other abnormalities. Her hemoglobin concentration is 11.6 g/dl, and hematocrit is 35%. Which of the following sets of serum hormone concentrations is most likely in this patient? Prolactin: decrease, ACTH; decrease, TSH; decrease, Aldosterone; increase

Prolactin: decrease, ACTH; decrease, TSH; decrease, Aldosterone; increase

A 20-year-old competitive cyclist taking recombinant human erythropoietin has polycythemia. Which of the following is the most likely cause? Early release of reticulocytes from bone marrow Increased erythrocyte life span Proliferation of bone marrow stem cells Proliferation of erythroid precursors

Proliferation of erythroid precursors

A 42-year-old woman comes to the emergency departmentbecause of a 2-day history of intermittent lower abdominalpain and nausea and vomiting. She has not passed flatus orstool during this time. She describes the pain as waxing andwaning. Initially, the vomitus was food that she hadrecently eaten, but it is now bilious; there has been noblood in the vomit. She underwent a hysterectomy 2 yearsago for leiomyomata uteri. Vital signs are within normallimits. Examination shows a distended tympanitic abdomenwith diffuse tenderness and no rebound. Bowel sounds areinitially quiet but then become loud with the onset of pain.Abdominal x-rays show distended loops of bowel with airfluidlevels. Which of the following is the most likelymechanism of this patient's symptoms? Inflammatory reaction to suture material leftinside the abdomen Immunologic damage to the bowel wall due toexposure to occult antigens during theoperation Chronic postoperative infection Mechanical obstruction from implantation ofleiomyomata uteri Proliferation of fibroblasts forming scar tissuein the area of the operation

Proliferation of fibroblasts forming scar tissuein the area of the operation

A 38-year-old woman comes to the physician because of a 2-year history of pain and heavy flow with menses. She stays home from work approximately 1 day every 2 months because of her symptoms. Use of ibuprofen has not been effective in treating the pain. She has one child, a 6-year-old daughter. She and her husband have not been able to conceive another child, but she says this is no longer a priority. There is a family history of infertility. Physical examination shows a mildly enlarged left ovary and retroflexed uterus. Ultrasonography of the abdomen shows substantial evidence of ovarian cysts and one peritoneal cyst. A biopsy specimen of one of the cysts is most likely to show which of the following? Ovarian germ-cell tumor Mucinous adenocarcinoma Proliferative endometrial tissue Ovarian stromal tumor Polycystic ovaries

Proliferative endometrial tissue

A 27-year-old man comes to the physician because of a 3-hour history of nausea, cramping abdominal pain, and diarrhea. His symptoms developed 24 hours after he ate at a wedding reception; several other people who ate at the reception have similar symptoms. A culture of the stool grows Salmonella enterica. He is able to maintain oral hydration. His symptoms begin to resolve within 36 hours but persist in a milder form for several more days. Which of the following is most likely to occur if this patient is treated with antibiotics? Anaphylaxis as a result of antibiotic hypersensitivity Prolonged fecal excretion of the organism Establishment of a chronic carrier state of the spleen Decreased risk for endocarditis Decreased risk for hemolytic uremic syndrome

Prolonged fecal excretion of the organism

A cluster of three cases of nosocomial pneumonia caused by a highly resistant strain of Klebsiella pneumoniae occurs in a 13-bed intensive care unit (ICU). Each patient in the ICU is in a private, enclosed room. Which of the following infection control measures is most likely to decrease the spread of this organism to other patients in the ICU and elsewhere in the hospital? Place infected patients in rooms with at least six air exchanges per hour Place infected patients in rooms with positive air pressure Place infected patients in rooms with negative air pressure Require medical personnel to wear masks when visiting infected patients Promote thorough hand cleansing before and after medical personnel visit each patient

Promote thorough hand cleansing before and after medical personnel visit each patient

A 22-year-old woman with a 10-year history of asthmacomes to the physician because she has had to increase heruse of her albuterol inhaler during the past 6 weeks. Herasthma was previously well controlled with inhaledglucocorticoids. She has a 2-year history of generalizedanxiety disorder controlled with fluoxetine and a 5-yearhistory of migraines. The migraines were well controlledwith sumatriptan until 4 months ago when she began to haveheadaches twice weekly; propranolol was added to herregimen at that time. She has been taking an oralcontraceptive for the past year. She says she has been underincreased stress at graduate school and in her personal lifeduring the past 3 months; during this period, she has beendrinking an average of four cups of coffee daily (comparedwith her usual one cup daily). She does not drink alcohol oruse illicit drugs. She appears mildly anxious but is not inrespiratory distress. Scattered end-expiratory wheezes areheard. The remainder of the examination shows noabnormalities. Which of the following is the most likelycause of the exacerbation of this patient's asthma? Sumatriptan therapy Increased caffeine intake Oral contraceptive therapy Fluoxetine therapy Propranolol therapy

Propranolol therapy

A 62-year-old man comes to the physician because of pain in the left hip for 3 weeks. An x-ray of the hip shows a 4.5-cm, destructive, osteoblastic lesion with an associated fracture in the proximal femur and two separate similar smaller lesions in the bony pelvis. A needle core biopsy of the femur shows metastatic carcinoma. Which of the following is the most likely primary site of the cancer? Pancreas Kidney Colon Prostate Lung

Prostate

An investigator conducts a study of the molecular basis of von Hippel-Lindau (VHL) disease, specifically the role of the VHL protein in the disease process. The protein forms part of a ubiquitin-ligase complex (E3) that targets hypoxia-inducible factor-1. The pathway is shown in the diagram. Step IV most likely involves which of the following organelles? Golgi complex Mitochondion Endoplasmic reticulum Lyosome Proteasome

Proteasome

A 22-year-old woman is found to be HIV positive after sexual contact with a partner with HIV infection. Combination therapy with lamivudine (3TC), ritonavir/lopinavir, and zidovudine (AZT) is initiated. Three months later, genomic typing shows that her HIV strain has become resistant to ritonavir/lopinavir. The cause of this resistance is most likely the acquisition of a mutation in a gene that is critical for which of the following viral processes? Protein processing

Protein processing

A 9-year-old boy is brought to the physician by his mother because of a 1-year history of cough productive of mucoid sputum, wheezing, and shortness of breath with exertion. He has a history of recurrent upper respiratory tract and sinus infections since birth. He is at the 25th percentile for height and weight. The mother says that his younger sibling is beginning to develop similar problems. Physical examination shows mild clubbing of the fingers. Laboratory studies show markedly increased sweat chloride and sodium concentrations. A defect of which of the following in this patient's bronchial epithelium is most likely causing these symptoms? Protein regulation Adrenoreceptors Membrane receptors Protein structure Nuclear receptors

Protein structure

A 47-year-old man is brought to the emergency department 3 hours after the sudden onset of sharp left flank pain and fever. He has had similar episodes during the past 5 years. His temperature is 38.3°C (100.9°F). Abdominal examination shows exquisite tenderness in the left costovertebral region. Laboratory studies show: Hemoglobin 16 g/dL Hematocrit 48% Leukocyte count 18,000/mm3 Segmented neutrophils 85% Bands 5% Lymphocytes 8% Monocytes 2% Urine leukocytes 2500/mm3 Imaging studies show hydronephrosis of the left kidney and a staghorn calculus. A nephrectomy is done; the calculus is composed of magnesium ammonium phosphate (struvite). Culture of a urine sample is most likely to grow which of the following organisms? Staphylococcus aureus Escherichia coli Pseudomonas aeruginosa Serratia marcescens Proteus mirabilis

Proteus mirabilis

A 23-year-old woman is treated with acetazolamide to prevent acute altitude sickness 3 days before going on a mountain-climbing expedition. Acetazolamide acts predominantly in which of the following areas of the kidney in this patient? Collecting ducts Descending loop of Henle Distal tubule Ascending loop of Henle Proximal tubule

Proximal tubule

A 25-year-old woman comes to the physician because of a 2-day history of decreasing urine output despite ample fluid intake. She underwent operative removal of a perforated appendix 3 weeks ago. She received intravenous gentamicin for 2 weeks. Physical examination shows no abnormalities. Laboratory studies show: Serum Urea nitrogen: 35 mg/dL Serum Creatinine: 2.9 mg/dL Urine Specific gravity: 1.010 Urine Protein: 2+ Urine Na+: 25 mEq/L Urine Casts: Brown granular Histologic examination of this patient's kidneys would most likely show necrotic cells in which of the following components of the nephron? Collecting tubule Proximal tubule Glomerular basement membrane Bowman capsule Mesangium

Proximal tubule

A 34-year-old man is brought to the emergency department semiconscious and combative. In addition to sedation, a short-acting neuromuscular blocking agent is administered for intubation to prevent aspiration. Within a few seconds after administration of the drug, he has transient muscle fasciculations in his face; he develops generalized paralysis within 1 minute. Forty-five minutes after completion of the procedure, he is still paralyzed. A genetic abnormality of which of the following enzymes is the most likely cause of his unusually slow recovery from paralysis? Monoamine oxidase Tyrosine hydroxylase Phenylethanolamine N-methyltransferase Angiotensin-converting enzyme Choline O-acetyltransferase Pseudocholinesterase

Pseudocholinesterase

A 24-year-old African American man comes to the physician because of a 3-month history of red bumps on his face and chin. He says that the bumps are itchy and painful. Physical examination shows hyperpigmented papules over the cheeks, jawline, and neck. Which of the following is the most likely diagnosis? Keloids Acne vulgaris Cutaneous lupus erythematosus Pseudofolliculitis barbae Rosacea

Pseudofolliculitis barbae

A 37-year-old man comes to the physician because of a 4-day history of diarrhea and abdominal pain. The condition has worsened in the past 24 hours, and he now has more frequent watery-brown stools. He has no history of recent fever, weight loss, exposure to illness, or travel. He completed a 10-day course of amoxicillin for a sinus infection 5 days ago. Laboratory studies of the stool show: Fecal fat: Negative Ova and parasites: Negative Culture for Salmonella, Shigef/a, Campylobacter, and Yersinia: Negative Clostridium difficile toxin: Positive Which of the following pathologic findings is most likely to be present in the intestinal tract of this patient? Giardia trophozoites lining the duodenal mucosa Bacterial overgrowth of the colonic surface Pseudomembranes of fibrin and inflammatory debris Flask-shaped ulcers in the colon Necrotizing granulomatous inflammation

Pseudomembranes of fibrin and inflammatory debris

An 80-year-old woman develops fever and cloudy urine 4 days after undergoing a total left knee replacement. Her temperature is 38°C (100.4° F). Physical examination shows a bandaged left knee and an indwelling Foley catheter. Microscopic analysis of a clean-catch, spun urine specimen shows 12 WBC/hpf and numerous gram-negative rods. Urine culture on brain-heart infusion medium grows greater than 100,000 bacterial colonies/ml, with a predominant colony type that is oxidase positive and produces a diffusible green pigment. Which of the following is the most likely causal organism? Streptococcus pyogenes (group A) Proteus mirabilis Pseudomonas aeruginosa Escherichia coli Klebsiella pneumoniae

Pseudomonas aeruginosa

A 58-year-old man comes to the physician because of a 4-year history of recurrent cough productive of increased sputum. Use of over-the-counter cough suppressants has not resolved his symptoms. He has smoked 2 packs of cigarettes daily for 35 years. He has no family history of lung disease. His temperature is 37°C (98.6°F), pulse is 72/min, and respirations are 18/min. Physical examination shows cyanosis. Diffuse wheezing is heard on auscultation. Which of the following pulmonary cell types is most likely to be abnormal in this patient? Alveolar endothelial cells Pseudostratified columnar epithelial cells Type I pneumocytes Alveolar macrophages Type II pneumocytes

Pseudostratified columnar epithelial cells

A 23-year-old man comes to the physician because of difficulty maintaining an erection. He was involved in an automobile collision that damaged his thoracic spinal cord, affecting motor and sensory function of the lower trunk and extremities. He is only able to obtain an erection through physical contact with his penis by his wife, and the erection does not last a long time. The best explanation for this type of erection is an intact spinal reflex arc between the sacral parasympathetic nerves and which of the following? Lumbosacral trunks Ilioinguinal nerves Obturator nerves Pudendal nerves Genitofemoral nerves

Pudendal nerves

A 45-year-old man comes to the physician because of a 6-month history of progressive shortness of breath with exertion. He has a history of frequent nosebleeds since adolescence. He is 178 cm (5 ft 10 in) tall and weighs 79 kg (175 lb); BMI is 25 kg/m 2. Inhaled albuterol does not improve his symptoms. Which of the following is the most likely cause of this patient's condition? Thromboembolism Reactive airway disease Atrial septal defect Pulmonary arteriovenous shunting Hypertension

Pulmonary arteriovenous shunting

A 62-year-old man is brought to the emergency department because of a 3-hour history of progressive difficulty breathing and mild left shoulder pain. His symptoms began after he ran up several flights of stairs. He has poorly controlled hypertension and mild angina pectoris. He has smoked one-half pack of cigarettes daily for 40 years. He appears uncomfortable and has labored breathing. Diffuse crackles, rhonchi, and scattered wheezing are heard on auscultation of the posterior lung fields. His arterial PO2 is 58 mm Hg. Which of the following is the most likely diagnosis? Pneumothorax Pneumonitis Pulmonary embolism Cardiac tamponade Pulmonary edema

Pulmonary edema

A 32-year-old nulligravid woman comes to the physician because of a 20-minute episode of shortness of breath when she awoke this morning. Her only medication is an oral contraceptive. She has smoked one pack of cigarettes daily for 10 years. She is sexually active with multiple partners. Physical examination shows erythema, swelling, warmth, and tenderness behind the right knee; a cord-like mass can be palpated. Which of the following is the most likely diagnosis? Gonococcal arthritis Steroid-induced asthma Pulmonary embolism Systemic lupus erythematosus Corticosteroid-induced arthritis

Pulmonary embolism

A 58-year-old woman comes to the physician because of a 6-month history of shortness of breath and chronic nonproductive cough. She has a 2-year history of difficulty swallowing, joint stiffness, and diffuse tightening of the skin of the face, neck, shoulders, arms, and fingers. She has had significant sensitivity to cold weather for 20 years, and she says that her hands turn white when they are exposed to the cold. Current medications include a histamine (H blocking agent for a long-standing history of esophageal reflux. One year ago, a biopsy specimen of the skin showed atrophy of the epidermis and deposition of dense collagen throughout the dermis with loss of the dermal appendages. Examination of the hands shows cutaneous ulceration, clawlike flexion deformity, and decreased joint mobility. This patient is at greatest risk for which of the following pulmonary disorders? Bronchiectasis Panacinar emphysema Pulmonary hypertension Granulomatous inflammation Pulmonary embolism Squamous cell carcinoma Adenocarcinoma

Pulmonary hypertension

A 70-year-old man is admitted to the hospital for evaluation of the recent loss of mental function. He has a history of weight loss. He is not taking any drugs. Vital signs are normal, and he is not dehydrated. Aside from mild disorientation, findings on neurologic examination are unremarkable. He has mild anemia. Laboratory studies show: Which of the following is the most likely diagnosis? Pneumococcal meningitis Adrenal carcinoma Chronic interstitial nephritis Diabetes mellitus Pulmonary neoplasm

Pulmonary neoplasm

A 65-year-old woman comes to the physician because of a 3-month history of headache, weakness of her arms, and left flank pain; she also has had a 1 4-kg (31-lb) weight loss during this period. Physical examination shows weakness of the proximal upper and lower extremity muscles. There is augmentation of strength with repetitive testing of the deltoid muscles. An MRI of the brain shows a single well-demarcated mass surrounded by edema i n the right frontal lobe. A stereotactic biopsy specimen of the lesion shows a malignant, small blue cell neoplasm that expresses cytokeratin, chromogranin, and synaptophysin. Which of the following is the most likely diagnosis? Extranodal primary central nervous system lymphoma Primary cerebral neuroblastoma Pulmonary small cell carcinoma metastatic to brain Anaplastic ependymoma Glioblastoma multiforme

Pulmonary small cell carcinoma metastatic to brain

A male newborn delivered at 34 weeks' gestation by an uncomplicated spontaneous vaginal delivery develops respiratory distress 30 minutes later. His respirations are 40/min. Physical examination shows cyanosis and lower rib retractions with respiration. Chest x-rays show fine densities with a reticulogranular appearance bilaterally. Which of the following is the most likely cause of these findings? Erythroblastosis fetalis Situs inversus intracranial hemorrhage Pulmonary surfactant deficiency Extralobar sequestration

Pulmonary surfactant deficiency

A 73-year-old female presents to the emergency room after collapsing at home. She is 5' 4" and weighs 198 lbs with a BMI of 34. Medical records reveal a long-standing history of hypertension. She reported experiencing severe, excruciating, stabbing-like chest pain that radiated to her left shoulder before she collapsed. Upon admission, her pulse is 112/min and blood pressure is 162/84 mm Hg. Auscultation reveals distant heart sounds and a drop in systolic blood pressure of >10 mm Hg upon inspiration. ECG reveals ST-T wave changes. Chest X-ray shows a widening of the mediastinum. Which of the following most likely represents the mechanism of ischemic injury in this patient? Pulsatile blood flow

Pulsatile blood flow

A 73-year-old female presents to the emergency room after collapsing at home. She is 5' 4" and weighs 198 lbs with a BMI of 34. Medical records reveal a long-standing history of hypertension. She reported experiencing severe, excruciating, stabbing-like chest pain that radiated to her left shoulder before she collapsed. Upon admission, her pulse is 112/min and blood pressure is 162/84 mm Hg. Auscultation reveals distant heart sounds and a drop in systolic blood pressure of >10 mm Hg upon inspiration. ECG reveals ST-T wave changes. Chest X-ray shows a widening of the mediastinum. Which of the following most like accounts for the drop in systolic blood pressure upon inspiration? Pulsus paradoxus

Pulsus paradoxus

The vaccine against diphtheria confers protective immunity by inducing the formation of antibodies against a preparation composed of which of the following? Purified inactivated toxin Purified capsular polysaccharide Live attenuated bacterial cells Killed bacterial cells Purified bacterial peptidoglycan

Purified inactivated toxin

A 70-year-old woman comes to the physician because of several episodes of burning pain with urination during the past year. Physical examination shows uterine prolapse. Urinalysis shows a mild increase in leukocytes. Ultrasonography of the abdomen shows bilateral dilation of the renal collecting system. This patient is at greatest risk for which of the following? Ureterolithiasis Pyelonephritis Cervical carcinoma Renal cell carcinoma Urothelial carcino

Pyelonephritis

A 3-week-old infant is brought to the physician by his mother because of a 1-week history of increasingly frequent vomiting. She says that at first, he vomited occasionally, but now he vomits after every feeding. The vomitus is nonbilious and consists of breast milk. He has had fewer wet diapers during the past 2 days. (dehydrated). He was born at term following an uncomplicated pregnancy and initially fed well. He appears lethargic and dehydrated. A 1 × 2-cm, firm, mobile, olive-shaped mass is palpated immediately to the left of the epigastrium. Which of the following is the most likely diagnosis? Congenital megacolon (Hirschsprung disease) Intussusception Duodenal atresia Midgut volvulus Pyloric stenosis

Pyloric stenosis

A 1-year-old boy is brought to the physician by his parents because of a 4-week history of progressive generalized tonic-clonic seizures and a strange odor to his urine. He has a history of delayed development. He was adopted from an orphanage in Russia at the age of 6 months. Physical examination shows fair skin and blond hair. His phenylalanine hydroxylase gene is homozygous for a point mutation (GT-+AT) in intron 12 of the affected gene that causes skipping of exon 12. Which of the following is the most likely explanation for exon skipping in this patient's affected gene? Expansion of trinucleotide repeat RNA splice error Deletion of 5' untranslated region Gene duplication Nonhomologous recombination Alternative polyadenylation site O B) Deletion of 5' untranslated region O C) Expansion of trinucleoti de repeat O D) Gene duplicati on O E) Nonhomologous recombination O F) Nonsense mutation O G) RNA splice error O H) X inacti vation X inactivation Nonsense mutation

RNA splice error

A 42-year-old man comes to the physician for cast removal. When he was traveling in another state 6 weeks ago, he sustained a fracture of the left humerus that required open reduction, internal fixation, and cast immobilization. After removal of the cast, examination of the left upper extremity shows a well-healing surgical incision over the arm with no bone deformities. Muscle strength is 2/5 with extension of the elbow and 1 /5 with extension of the wrist and fingers. This patient most likely sustained a fracture at which of the following sites on the humerus? Distal shaft Radial groove Medial epicondyle Surgical neck Coronoid fossa

Radial groove

A 23-year-old male presents with painful urination and a clear urethraldischarge for the past week. History reveals the patient had similar symptoms one month ago, for which he completed a full course of doxycycline for a chlamydialinfection. He has been sexually active with one female partnerfor 2 years. Physical examinationshows no abnormalities. A urine polymerase chain reactiontest is positive forChlamydia trachomatis. Which of thefollowing is the most likely explanation for this patient'scurrent infection? Sequestration of C. trachomatis in theepididymis Concurrent infection with Neisseriagonorrhoeae Insufficient duration of therapy Reacquisition of infection from his partner Doxycycline-resistant strain of C. trachomatis

Reacquisition of infection from his partner

Persons at risk for severe influenza are vaccinated every year because of antigenic variation. This variation can either be a result of antigenic drift (minor) or antigenic shift (major) changes in the antigenic composition of viral proteins. Which of the following is the most likely genetic mechanism of antigenic shift? Point mutation Frameshift Reassortment Amplification Recombination

Reassortment

A 3-year-old girl is brought to the physician by her parents because they are concerned about her behavior. They describe their daughter as stubborn and always on the go. She can rarely sit still for more than 10 minutes. She often refuses to comply with their requests and sometimes throws 3- to 5-minute temper tantrums. They report that she dawdles at bedtime and requires frequent direction and assistance in preparing for bed. Her preschool teacher notes that she is active and talkative without being disruptive and is beginning to demonstrate more interactive play with her peers. She generally sleeps through the night and occasionally wets the bed. Her appetite is good. Her first word was at the age of 11 months, and she began walking without assistance at the age of 14 months. Physical examination shows no abnormalities. On mental status examination, she initially hides behind her mother but warms to the interviewer after a few minutes and begins playing with toys in the office. Her speech is 90% intelligible, and her vocabulary is large for her age. Which of the following is the most appropriate next step in management? Trial of fluoxetine Reassurance Speech therapy Play therapy Trial of methylphenidate Enuresis alarm

Reassurance

At her 6-week postpartum visit, an 18-year-old woman, gravida 1, para 1, tells her physician that she has a pinkish vaginal discharge that has persisted since her delivery, although it is decreasing in amount. On physical examination, the uterus is fully involuted and there are no adnexal masses. Which of the following is the most appropriate next step in management? Measurement of serum prolactin concentration Reassurance that this is normal Quantitative β-hCG test Administration of ampicillin Dilatation and curettage

Reassurance that this is normal

At her 6-week postpartum visit, an 18-year-old woman,gravida 1, para 1, tells her physician that she has a pinkishvaginal discharge that has persisted since her delivery,although it is decreasing in amount. On physicalexamination, the uterus is fully involuted and there are noadnexal masses. Which of the following is the mostappropriate next step in management? Measurement of serum prolactin concentration Dilatation and curettage Quantitative β-hCG test Administration of ampicillin Reassurance that this is normal

Reassurance that this is normal

A postmenopausal 60-year-old woman comes to thephysician because of a 2-year history of vaginal dryness,intermittent vaginal pain, and decreased pleasure with sexual intercourse. Vital signs are within normal limits. Abdominal examination shows no abnormalities. Pelvic examination shows pale, dry vaginal mucosa. No masses are palpated. Which of the following is the most appropriate next step in management? Inform the patient that she should accept hersymptoms as part of the normal agingprocess Inform the patient that these problems are oftenpsychological and that she would benefitfrom psychotherapy Explain to the patient that it will be importantto assess the impact of her condition on herhusband as further treatment is initiated Reassure the patient that her symptoms arecommon among women her age and thatthere are treatments available Advise the patient that since these problems areexpected for a woman her age, the risksassociated with pharmacotherapy outweighany benefits

Reassure the patient that her symptoms arecommon among women her age and thatthere are treatments available

A 25-year-old woman comes to the physician for a routine follow-up examination. Two years ago, she underwent splenectomy as a result of injuries sustained in a motor vehicle collision. Physical examination shows no abnormalities. A complete blood count shows target erythrocytes. This finding is most likely the result of loss of function of which of the following portions of the spleen? Germinal centers Perivascular lymphocytes Red pulp Endothelial cells of the cortex Splenic stromal connective tissue

Red pulp

A 21-year-old African American man with sickle cell disease is brought to the emergency department 2 hours after the sudden onset of severe pain in his abdomen and knees. He has had six similar episodes within the past 4 years; he was admitted to the hospital each time for treatment of the pain. His course has been complicated by two episodes of pneumonia and a small cerebral infarction in the past 2 years. The patient asks the physician for extra pain medication to "kill me because I can't take it anymore." Which of the following initial actions by the physician regarding the patient' s request is most appropriate? Administer general anesthesia for sedation Comply with the patient's request Reevaluate the patient after his pain has been controlled Restrain the patient Reevaluate the patient after his pain has been controlled Contact the hospital ethics committee

Reevaluate the patient after his pain has been controlled

A 4-year-old girl is brought to the physician for a well-child visit. She feels well. She is at the 75th percentile for height and weight. Her temperature is 37°C (98.6°F), pulse is 100/min, respirations are 18/min, and blood pressure is 90/60mm Hg. Cardiac examination shows a normal S1 and S2. There is a grade 2/6, systolic, vibratory ejection murmur heard best at the fourth intercostal space in the left midclavicular line. Which of the following is the most appropriate next step in management? Complete blood count Reexamination in 12 months Echocardiography Antistreptolysin O titer Chest x-ray

Reexamination in 12 months

A 55-year-old male is brought to the emergency department 45-minutes after he collapsed at home. His pulse is 120/min, respirations are 16/min, and blood pressure 80/45. Physical examination shows no other abnormalities. His hemoglobin concentration is 8 g/dL, hematocrit is 24%, and platelet count is 240,000/mm3. Endoscopy shows a bleeding duodenal ulcer, which is surgically repaired. He receives 4 units of packed red blood cells. Two days later, laboratory studies show: Hemoglobin: 10 g/dL; Hematocrit: 30%; BUN: 70 mg/dL; Serum Creatinine: 3 mg/dL; Urine Blood: absent; Urine Sediment: granular casts; Urine Volume: 100 mL/24 h; Urine Total protein: 100 mg/24 h; Urine Sodium: 50 mEq/L; Fractional excretion of sodium: 2.5%; Urine Osmolality: 300 mOsmol/kg H20. Two weeks later, the urine sediment is clear and renal function is normal. Histologic examination of a renal biopsy specimen is most likely to show which of the following at this time? Granulation tissue Collagen scar Lymphocytic infiltrate and fibrogenesis Regenerating tubular epithelial cells Coagulation necrosis

Regenerating tubular epithelial cells

A 95-year-old man who is a resident of a skilled nursing care facility is brought to the emergency department because of a 1-day history of temperatures to 39.4°C (102.9°F), headache, cough, and muscle aches. Several other residents have similar symptoms. Physical examination shows no other abnormalities. Treatment with a neuraminidase inhibitor is begun. This drug most likely will inhibit which of the following processes, therefore decreasing the duration of this patient's symptoms? Nucleocapsid-matrix protein interactions Replication of genomic RNA Release of virus from infected epithelial cells Transcription of mRNA Synthesis of surface glycoproteins

Release of virus from infected epithelial cells

A 75-year-old woman is brought to the physician because of a 1-day history of fever and back pain. Her temperature is 39.5°C (103.1 °F), pulse is 111/min, respirations are 32/min, and blood pressure is 115/79 mm Hg. Physical examination shows left-sided costovertebral angle tenderness. Laboratory studies show a leukocyte count of 17,000/mm3 (with 9°/c, bands) and pyuria. Urine cultures grow Escherichia coli. It is determined that the patient's fever is partially due to interleukin-6 (IL-6), which was induced by the IL-1 produced during the immune response. Which of the following best describes the role of IKB in the nuclear factor-kappa B (NF-KB) signal transduction pathway from IL-1 binding to IL-6 induction in this patient? Translocates to the nucl eus after undergoing phosphorylation Facilitates proteolytic cleavage of NF-KB dimers Attaches to cytokine receptor Releases nf-kb after undergoing phosphorylation Phosphorylates NF-KB

Releases nf-kb after undergoing phosphorylation

A 60-year-old man comes to the physician for a routine health maintenance examination. He has had normal blood pressure measurements. His blood pressure today is 170/95 mm Hg. Physical examination shows no other abnormalities. Serum studies show hypokalemia and metabolic alkalosis. Plasma renin activity and serum aldosterone concentrations are increased. Following the administration of captopril, there is a marked increase in plasma renin activity. Which of the following is the most likely cause of the findings in this patient? Essential hypertension Chronic glomerulonephritis Aldosterone-secreting adrenal tumor Cushing syndrome Renal artery stenosis

Renal artery stenosis

An asymptomatic 32-year-old woman comes to the physician for a follow-up examination. She has a 10-month history of hypertension that has been difficult to control with medication. Current medications include metoprolol, lisinopril, hydrochlorothiazide, and nifedipine. Her blood pressure today is 170/110 mm Hg. Abdominal examination shows a bruit in the left upper quadrant. Which of the following is the most likely cause of this patient's hypertension? Pituitary adenoma Hyperthyroidism Pheochromocytoma Hyperaldosteronism Renal artery stenosis Hypercortisolism

Renal artery stenosis

A 55-year-old woman has left flank pain and gross hematuria. A mass is palpable in the left upper quadrant of the abdomen. Abdominal ultrasonography shows a 12-cm solid mass on the lower pole of the left kidney. Angiograms show a hypervascular mass. Which of the following is the most likely diagnosis? Renal cell adenocarcinoma Nephroblastoma (Wilms tumor) Squamous cell carcinoma Papillary transitional cell carcinoma Renal cell adenoma

Renal cell adenocarcinoma

A 56-year-old man comes to the physician because of vague right-sided flank pain and a 3.2-kg (7-lb) weight loss over the past 6 weeks. There has been no recent trauma or strain to his back. He has no history of genitourinary problems, but his urine has been darker than usual over the past 2 weeks. Physical examination of the back shows no abnormalities. Renal ultrasonography shows a 1 ½-cm solid mass in the upper portion of the right kidney. Microscopic examination of urine shows 50 erythrocytes/hpf, 6 leukocytes/hpf, and no bacteria. Which of the following is the most likely diagnosis? Transitional carcinoma Metastatic carcinoma Renal parenchymal hematoma Renal cell carcinoma Renal abscess

Renal cell carcinoma

Two days after admission to the hospital because of a sickle cell disease crisis, a 24-year-old man suddenly develops blood in his urine and flank pain. Laboratory studies at the time of admission showed that his hematocrit was decreased to 11% from his normal baseline of 25%. Urinalysis shows gross blood, rare WBCs, and no WBC casts. Which of the following is the most likely cause of the hematuria in this patient? Prostatitis Glomerulonephritis Renal papillary necrosis Transitional cell carcinoma Nephrolithiasis

Renal papillary necrosis

During an experiment, a cuff occlude is placed on the left renal artery of a normal rat and inflated to cause a 90% decrease in the cross-sectional area of the artery. A catheter is placed in the left and right renal veins to sample the effluent. The concentration of which of the following substances will most likely be higher in the effluent from the left renal vein than that from the right renal vein? Epinephrine Endothelin Renin Aldosterone

Renin

A healthy 70-year-old woman has participated in a longitudinal study of the effects of aging on performance during pulmonary function tests for the past 50 years. She has undergone a complete set of tests, including arterial blood gas analyses, every 5 years. Which of the following sets of changes best represents this woman now compared with her results at the age of 20 years? Residual volume: increase, arterial PO2, decrease, a-a diff: increase

Residual volume: increase, arterial PO2, decrease, a-a diff: increase

A 57-year-old woman comes to the physician because of a 2-year history of difficulty sleeping. After she gets into bed atnight, her legs feel cold and crampy, and she cannot settleinto a comfortable position. Walking around temporarilyrelieves her symptoms. She also has difficulty sitting for aprolonged period of time. She has a mild anxiety disorderbut takes no medications. Vital signs are within normallimits. Examination shows no abnormalities. Which of thefollowing is the most likely diagnosis? Stiff-person syndrome Tentanus Axial dystonia Focal limb disorder Restless legs syndrome Parkinson disease

Restless legs syndrome

A 37-year-old man is brought to the emergency department after his motorcycle ran into the back of a truck at high speed. He was wearing a helmet. On arrival, his neck is immobilized in a cervical collar, and intravenous fluids are being administered. Paramedics report that at the scene he appeared to be intoxicated. He is alert and talking. His breath smells of alcohol. His pulse is 130/min, respirations are 30/min, and blood pressure is 100/60 mm Hg. The lungs are clear to auscultation. Heart sounds are normal. The abdomen is distended and tender. There is blood at the urethral meatus. Ultrasonography shows blood in the abdomen. An x-ray of the pelvis shows fractures of the pubic rami and ischial and ilial fractures on the right. Which of the following is the most appropriate next step in management? Intravenous pyelography Cystography Retrograde urethrography Placement of a urinary catheter Diagnostic peritoneal lavage

Retrograde urethrography

A previously healthy 12-year-old girl is brought to the emergency department because of a 4-day history of sore throat that has become progressively worse during the past 2 days. She also has had malaise and an earache on the right during this time. She now has difficulty eating because of the pain but has had no vomiting or diarrhea. Her voice sounds muffled. Her temperature is 38.7°C (101.7°F). There is right cervical lymphadenopathy. Examination of the mouth is difficult to perform because of pain; the right tonsil appears enlarged, and the uvula is deviated to the left. Examination of the tympanic membranes shows no abnormalities. Which of the following is the most likely diagnosis? Tonsillar hypertrophy Tonsillitis Retropharyngeal abscess Bacterial tracheitis Peritonsillar abscess

Retropharyngeal abscess

A 32-year-old man with HIV infection comes to the physician for a follow-up examination. He has been treated with highly active antiretroviral therapy (HAART) for the past 6 years during which time his plasma HIV viral load has been undetectable. His plasma HIV viral load has now increased, and antiretroviral resistance is suspected. HIV genotype analysis confirms that the virus has resistance mutations. Based on this finding, which of the following are most likely mutated? Nef and Vpr CD4 and CCRS Reverse transcriptase and protease inhibitors Tat and Rev Nucleocapsid and gp41 Capsid and gp120

Reverse transcriptase and protease inhibitors

A 60-year-old man is brought to the emergency department because of generalized weakness for 6 hours. The onset of this symptom occurred 2 hours after he took four tablets (rather than his usual one tablet) of a medication that prevents angina pectoris. His pulse is 36/min, and blood pressure is 100/50 mm Hg. After the administration of isoproterenol, his pulse increases to 60/min. Which of the following best describes the mechanism of action of this patient's usual chronic medication? Irreversible, dose-dependent Irreversible, dose-independent Reversible, does-independent Reversible, dose-dependent

Reversible, dose-dependent

A 25-year-old woman comes to the physician because of a 6-month history of joint pain that has been poorly responsive to treatment with aspirin. Physical examination shows bilateral swelling of the proximal interphalangeal joints, metacarpophalangeal joints, and the wrists; there is weakness of grasp. A few small nodules are palpated beneath the skin around the joints of the fingers. Which of the following is the most likely diagnosis? Systemic lupus erythematosus Systemic sclerosis (scleroderma) Rheumatoid arthritis Osteoarthritis Gout

Rheumatoid arthritis

A 68-year-old man comes to the physician because of a 2-hour episode of loss of vision in his right eye 2 days ago. He underwent a left femoral-popliteal bypass 3 months ago for peripheral vascular disease. His vital signs are normal. Funduscopic examination shows cholesterol emboli. The most likely cause of his condition is an embolus in which of the following arteries? Abdominal aorta Right anterior cerebral Left common carotid Right ophthalmic Right middle cerebral

Right ophthalmic

A 56-year-old man comes to the physician 4 hours after the sudden onset of uncontrollable irregular movements of the left side of the body. Physical examination shows frequent, irregular flailing movements of the proximal appendicular muscles on the left. Which of the following nuclei is most likely damaged? Left subthalamic Left dentate Right dentate Right subthalamic Left caudate Right caudate

Right subthalamic

An otherwise healthy 45-year-old male has a 2-year history of heavy snoring and daytime sleepiness. Monitoring shows that his arterial oxygen saturation periodically decreases to 87% when he sleeps. If the man's condition is left untreated, which of the following complication is most likely? Normocytic hypochromic anemia Right ventricular hypertrophy Respiratory alkalosis Dilation of the mitral annulus Syndrome of inappropriate ADH release

Right ventricular hypertrophy

A 66-year-old woman comes to the emergency department 1 hour after the sudden onset of retrosternal chest discomfort accompanied by nausea and diaphoresis. She has hypotension, jugular venous distention, and a murmur of tricuspid regurgitation. An ECG shows ST-segment elevation in the right precordial leads. Which of the following is the most likely diagnosis? Right ventricular infarction Pulmonary emboli Dissecting aortic aneurysm Constrictive pericarditis Pericardial tamponade

Right ventricular infarction

A 42-year-old woman comes to the physician because of a 3-year history of an intermittent facial rash, including the forehead, eyelids, nose, and cheeks. She says that the rash seems to be getting worse since she moved from New York to Florida last year. She notes that she has always blushed easily. Spicy foods also precipitate a flushing reaction that seems to exacerbate the rash. Occasionally, her face burns and stings, and her skin feels dry. Physical examination shows erythema over the nose and cheeks, with scattered telangiectasias and a few papules. Which of the following is the most likely diagnosis? Pityriasis rosea Acne vulgaris Carcinoid syndrome Rosacea Seborrheic dermatitis

Rosacea

A 75-year-old man comes to the emergency department because of a 2-day history of ringing of his ears, nausea, and fatigue. His temperature is 37°C (98.6°F), pulse is 100/min, respirations are 24/min, and blood pressure is 140/85 mm Hg. Physical examination shows mild epigastric tenderness. Arterial blood gas analysis on room air shows: Which of the following is the most likely cause of this patient's condition? Salicylate poisoning Severe kyphosis Obstructive sleep apnea Pulmonary embolism lntracardiac shunt

Salicylate poisoning

A 10-year-old girl with sickle cell disease is brought to the physician because of fever, chills, and throbbing pain in her lower leg for 3 days. An x-ray of the leg shows a lytic area in the metaphysis of the tibia. A biopsy specimen of the affected bone shows numerous neutrophils and destruction of bone. Which of the following is the most likely causal organism? Haemophilus species Histoplasma species Mycobacterium species Salmonella species Cryptococcus species

Salmonella species

A 42-year-old man seeks medical attention because of the gradual onset of shortness of breath, night sweats, weight loss, and fatigue. An x-ray of the chest shows enlargement of the hilar lymph nodes and nodular infiltrates in both lungs. A PPD skin test is negative. Examination of lung tissue obtained on biopsy shows multiple noncaseating granulomas with many giant cells. No organisms are seen on sections stained with hematoxylin and eosin, acid-fast stain, or silver stain. Which of the following is the most likely diagnosis? Sarcoidosis Idiopathic pulmonary fibrosis Asbestosis Tuberculosis Pneumocystis Jiroveci (formerl y P. carinit) pneumonia

Sarcoidosis

An 18-year-old woman receives general anesthesia for extraction of wisdom teeth. After 5 minutes, she has generalized hypertonicity of skeletal muscles and increased body temperature. The most appropriate treatment is a drug that decreases which of the following? Sarcoplasmic Ca2+ release

Sarcoplasmic Ca2+ release

An otherwise healthy 60-year-old man comes to the physician for a follow-up examination 2 weeks after he was found to have a serum prostate-specific antigen concentration of 5.3 ng/ml (N<4). A biopsy specimen of the prostate shows evidence of adenocarcinoma. A CT scan of the spine shows osteoblastic lesions at L 1. The patient undergoes bilateral orchiectomy. Five months later, he dies suddenly of a massive myocardial infarction. At autopsy, which of the following microscopic changes in the prostate resulting from the orchiectomy is most likely present? Hypertrophied cells Scattered shrunken cells Widespread necrosis Edema Corpora amylacea Necrosis of the blood vessel walls

Scattered shrunken cells

A 27-year-old primigravid woman at 34 weeks' gestation is admitted to the hospital because of nausea and vomiting, and abdominal pain for 12 hours. Her pregnancy has been otherwise unremarkable. Prior laboratory studies, including a complete blood count and serologic tests for HIV and hepatitis, have shown no abnormalities. Her blood pressure now is 164/102 mm Hg. Abdominal examination shows right upper quadrant tenderness. Laboratory studies show: Hemoglobin 7.4 g/dL; Hematocrit 24%; Platelet count 72,000; Serum Creatinine 1.2 mg/dL; Total bilirubin 2.3 mg/dL; AST 112 U/L; ALT 116 U/L. Examination of a peripheral blood smear is most likely to show a predominance of which of the following in this patient? Spherocytes Schistocytes Acanthocytes Heinz bodies Target cells Howell-Jolly bodies

Schistocytes

Which of the following represents the most likely purpose for amniocentesis? Alpha-fetoprotein (AFP) Screen for genetic disorders Screen for gender Screen for paternity Screen for fetal age

Screen for genetic disorders

A newborn has male genital ducts but female external genitalia. Cytogenetic analysis shows a 46,XY karyotype, and genetic testing shows a mutation of the gene encoding 5 a-reductase. In the absence of this mutation, the apparent labia majora of this newborn would have developed into which of the following structures? Glans penis Penile shaft Scrotum Prostatic utricle Penile urethra

Scrotum

An asymptomatic 37-year-old African American mancomes to the physician for a preemployment examination.He has no history of serious illness and takes nomedications. He does not smoke. He exercises regularly.His mother and 40-year-old brother have hypertension. Heis 178 cm (5 ft 10 in) tall and weighs 73 kg (162 lb); BMIis 23 kg/m2. His pulse is 82/min, and blood pressure is144/92 mm Hg. Examination shows no otherabnormalities. Which of the following is the mostappropriate next step? Exercise stress test Decreased dietary intake of sodium Second measurement of blood pressure in2 weeks 24-Hour urine collection for measurement ofcatecholamines Routine examination in 1 year

Second measurement of blood pressure in2 weeks

A 23-year-old man has a 3-month history of mildly depressed mood, decreased energy, and dry skin. His thyroid gland is not palpably enlarged. Serum thyroxine {T 4) and thyroid-stimulating hormone concentrations are decreased. Which of the following is the most likely diagnosis? Primary hypothyroidism Secondary hypothyroidism Primary hyperthyroidism Secondary hypothyroidism

Secondary hypothyroidism

After an overnight fast, a 52-year-old man undergoesinfusion of acid through a catheter into the upper duodenum.This most likely will increase pancreatic secretion mainlythrough the action of which of the following substances? Gastrin Secretin Vasoactive intestinal polypeptide Glucagon Cholecystokinin

Secretin

A 51-year-old man with a 3-month history of hepatitis B antigen-negative polyarteritis nodosa has progressive proximal muscle weakness and myalgia. A biopsy specimen of involved muscle would most likely show which of the following? Central vacuolization with glycogen accumulation Periodic acid-Schiff-positive intramyofibrillar vacuoles Mucopolysaccharide deposition Central vacuolization with lipid accumulation Segmental ischemic necrosis Mitochondrial proliferation

Segmental ischemic necrosis

A young adult couple has been unable to conceive for the past 2 years. The woman has regular menstrual cycles and takes no contraceptives. A semen analysis shows: Quality normal range Quantity normal range Color white Fructose 5% of normal content The most likely cause of this couple's infertility is deficient activity of which of the following in the man? Adenohypophysis Bulbourethral glands Prostate Seminal vesicles Testes

Seminal vesicles

A 24-year-old man develops recurrent vesicular lesions on his left upper lip. The recurrences of the infection are most likely caused by a virus that maintains latency in which of the following cells? Lymphocytes Motor neurons Sensory neurons Dendritic Endothelial Epithelial Fibroblasts

Sensory neurons

During the following experiment, an early limb bud tip was grafted onto a late wing, which of the following most likely resulted? Sequential duplication Partial deletion Normal development Parallel duplication Complete deletion

Sequential duplication

A 55-year-old woman comes to the physician for a routine health maintenance examination. Her last menstrual period was 8 years ago. She takes no medications. Physical examination shows no abnormalities. The physician orders fasting serum lipid studies to screen for factors likely to predispose her to cardiovascular disease. Compared with a healthy 25-year-old woman, which of the following is most likely to be decreased in this patient? Serum IDL-cholesterol concentration Serum triglyceride concentrations Serum LDL- cholesterol concentration Free fatty acids Serum HDL-cholesterol concentration

Serum HDL-cholesterol concentration

A previously healthy 6-year-old boy is brought to the physician because of a 1-week history of right knee pain and swelling. He also has had occasional chills and muscle pain for 1 week. He went camping with his father in eastern Pennsylvania approximately 2 months ago. Two weeks after the trip, he had a solid red rash that slowly spread over most of his right thigh and resolved spontaneously 2 weeks later. Examination of the right knee shows swelling, an effusion, and mild tenderness to palpation. Which of the following is most likely to confirm the diagnosis? Complete blood count Serum Lyme (Borrelia burgdorferi) antibody assay Serum rheumatoid factor assay X-ray of the right knee Serum antinuclear antibody assay Determination of erythrocyte sedimentation rate

Serum Lyme (Borrelia burgdorferi) antibody assay

A previously healthy 6-year-old boy is brought to thephysician because of a 1-week history of right knee painand swelling. He also has had occasional chills and musclepain for 1 week. He went camping with his father ineastern Pennsylvania approximately 2 months ago. Twoweeks after the trip, he had a solid red rash that slowlyspread over most of his right thigh and resolvedspontaneously 2 weeks later. Examination of the right kneeshows swelling, an effusion, and mild tenderness topalpation. Which of the following is most likely to confirmthe diagnosis? Serum antinuclear antibody assay Serum rheumatoid factor assay Determination of erythrocyte sedimentationrate X-ray of the right knee Serum Lyme (Borrelia burgdorferi) antibodyassay Complete blood count

Serum Lyme (Borrelia burgdorferi) antibodyassay

Laboratory results showed fasting plasma glucose (FPG) levels to be normal, which ruled out diabetes mellitus. This leaves three potential causes of the polyuria, i.e., central diabetes insipidus (CDI), nephrogenic diabetes insipidus (NDI), and polydipsia. Which of the following tests is used to differentiate between CDI and NDI? Serum Na+ and osmolality in response to glucose intake Serum and urine Na+ concentration and 24 hour urine output in response to water intake Urine osmolality in response to water intake Serum and urine Na+ and osmolality in response to exogenous ADH (anti diuretic hormone) administration Serum and urine Na+ concentration and 24 hour urine output in response to Na+ intake

Serum and urine Na+ and osmolality in response to exogenous ADH (anti diuretic hormone) administration

A 26-year-old woman comes to the emergency departmentbecause of a 12-hour history of lower abdominal pain andvaginal bleeding. There is no history of medical illness, and she takes no medications. Her temperature is 37°C (98.6°F), pulse is 125/min, respirations are 40/min, and blood pressure is 96/64 mm Hg. Abdominal examination shows distention and tenderness. Decreased bowel sounds are heard. Pelvic examination shows an adnexal mass onthe right. Measurement of which of the following is the most appropriate next step in management of this patient? Leukocyte count Prothrombin time Platelet count Serum α-fetoprotein concentration Serum β-human chorionic gonadotropinconcentration

Serum β-human chorionic gonadotropinconcentration

A 5-year-old girl falls through the ice while skating on anoutdoor pond. She is removed from the water within1 minute, but dry clothing is not available, and she is stillcold and wet 20 minutes later. Which of the followingmechanisms helps maintain the patient's core temperatureduring the period following her rescue? Increased thermoregulatory set point Diving response Cutaneous vasodilation Shivering Release of endogenous pyrogen

Shivering

A 3-week-old male newborn is brought to the physician by his mother because of recurrent vomiting after feeding since birth. His mother says that he is eager to feed even after vomiting. He has not had fever. Abdominal examination shows a firm, mobile mass in the epigastrium to the right of the midline. Which of the following best describes this patient's condition? Multiple malformation syndrome Breech disruption sequence VATER association Single primary developmental defect Amniotic band disruption sequence

Single primary developmental defect

A 24-year-old man comes to the physician with his 25-year-old wife because they have been unable to conceive for 3 years. The wife has been evaluated for infertility, and the test results were normal. Analysis of the man's semen shows normal numbers of living sperm but they are immotile. In addition to infertility, this man is most likely to have which of the following associated conditions? Glomerulonephritis Cholelithiasis Coronary artery disease Fat malabsorption Sinusitis

Sinusitis

A 47-year-old man comes to the physician because of a 2-year history of fatigue. He has had progressive difficultywith daytime sleepiness and has intermittently fallen asleepat work. He has no difficulty falling asleep or staying asleepat night but awakens in the morning not feeling well rested.His vital signs are within normal limits. Examination of thethroat shows no abnormalities except for hypertrophiedtonsils. His hemoglobin concentration is 17.9 g/dL,leukocyte count is 8700/mm3, and platelet count is170,000/mm3. Which of the following is the most likelycause of this patient's symptoms? Erythroleukemia Chronic Epstein-Barr virus infection Sleep apnea Polycythemia vera Chronic fatigue syndrome

Sleep apnea

A 57-year-old man comes to the physician accompanied by his wife because of a 2-year history of fatigue. He reports waking up tired nearly every morning, often with a headache. He naps almost every afternoon. He thinks that the fatigue is affecting his concentration and performance at work. His wife says that he snores frequently during the night and sometimes wakes up gasping for air. She describes him as a restless sleeper. His tonsils and adenoids were removed when he was a child. He has no history of serious illness and takes no medications. He is 178 cm (5 ft 10 in) tall and weighs 115 kg (253 lb); BMI is 36 kg/m2. His pulse is 86/min, and blood pressure is 164/88 mm Hg. The nasal septum is at the midline. Examination shows no other abnormalities. Which of the following is the most likely diagnosis? Chronic fatigue syndrome Restless legs syndrome REM sleep behavior disorder Sleep apnea Narcolepsy

Sleep apnea

A 57-year-old man comes to the physician accompanied byhis wife because of a 2-year history of fatigue. He reportswaking up tired nearly every morning, often with aheadache. He naps almost every afternoon. He thinks thatthe fatigue is affecting his concentration and performanceat work. His wife says that he snores frequently during thenight and sometimes wakes up gasping for air. Shedescribes him as a restless sleeper. His tonsils and adenoidswere removed when he was a child. He has no history ofserious illness and takes no medications. He is 178 cm (5 ft10 in) tall and weighs 115 kg (253 lb); BMI is 36 kg/m2.His pulse is 86/min, and blood pressure is 164/88 mm Hg.The nasal septum is at the midline. Examination shows noother abnormalities. Which of the following is the mostlikely diagnosis? Restless legs syndrome Sleep apnea REM sleep behavior disorder Chronic fatigue syndrome Narcolepsy

Sleep apnea

A 4-year-old boy is brought to the physician by his parents because of a 2-month history of difficulty sleeping. His Parents report that he typically awakens 1 hour after going to sleep and cries loudly. When his parents come to his room, he appears frightened and is unaware of their attempts to comfort him. In the morning, he has no memory of these episodes. Physical examination shows no abnormalities.Which of the following is the most likely diagnosis? Separation anxiety disorder Oppositional defiant disorder Factitious disorder Seizure disorder Sleep terror disorder

Sleep terror disorder

A 4-year-old boy is brought to the physician by his parents because of a 2-month history of difficulty sleeping. His parents report that he typically awakens 1 hour after going to sleep and cries loudly. When his parents come to his room, he appears frightened and is unaware of their attempts to comfort him. In the morning, he has no memory of these episodes. Physical examination shows no abnormalities. Which of the following is the most likely diagnosis? Sleep terror disorder Oppositional defiant disorder Factitious disorder Separation anxiety disorder Seizure disorder

Sleep terror disorder

A 45-year-old woman has a 2-week history of a heart murmur and hepatomegaly. She has a 3-month history of flushing with hypotension and crampy diarrhea. Which of the following is the most likely site of a primary neoplasm in this patient? Bone marrow Ovary Adrenal gland Kidney Small intestine

Small intestine

A 67-year-old woman with atrial fibrillation is brought to the emergency department 30 minutes after the sudden onset of severe abdominal pain. An immediate exploratory laparotomy shows an embolus within the superior mesenteric artery producing complete occlusion of the middle colic artery. As a result of this occlusion, ischemic changes are most likely to be found at which of the following locations? Stomach, duodenum, and small intestine Splenic flexure, descending colon, and upper third of the rectum Small intestine, ascending colon, part of transverse colon Descending colon, sigmoid colon, and rectum Distal one-third of the esophagus, stomach, and duodenum

Small intestine, ascending colon, part of transverse colon

A 47-year-old woman comes to the physician for a routinehealth maintenance examination. She feels well and has nohistory of serious illness. Her mother, brother, and sisterhave hypertension. The patient's pulse is 84/min, and bloodpressure is 138/85 mm Hg. Examination shows no otherabnormalities. The most appropriate recommendation isdecreased intake of which of the following? Sodium Protein Potassium Calcium Carbohydrates

Sodium

A 60-year-old woman with a 1-year history of diabetes mellitus has had decreasing vision over the past year. Examination of the eyes shows milky opaci ty of both lenses. Which of the following aberrant metabolites of glucose is most likely responsible for the formation of this abnormality? Sucrose Lactose Fructose Glycogen Sorbitol

Sorbitol

A 35-year-old woman who is being examined because of infertility receives an injection of contrast material into her cervix to visualize the reproductive tract before undergoing radiographic tests. On the hysterosalpingogram shown, the contrast material (as indicated by the arrows) is also seen in the peritoneal cavity. Which of the following best explains this finding? Rupture of the uterine body Rupture of the fallopian tube Spillage of contrast, which is an artifact Spillage of contrast, which is normal

Spillage of contrast, which is normal

A 44-year-old primigravid woman at 15 weeks' gestation comes to the physician for a routine prenatal examination. Physical examination shows a uterus consistent in size with gestational age. Amniocentesis shows a markedly increased a-fetoprotein concentration. This patient's fetus is at greatest risk for which of the following defects? Spina bifida Congenital heart defects Malrotation of the gut Horseshoe kidney Hypergonadism

Spina bifida

A 32-year-old woman is brought to the emergency department by ambulance after a fall from a 10-foot ladder. Physical examination suggests severe hemorrhage and shock. X-rays show fractures of the left ninth and tenth ribs. Which of the following organs is most likely injured? Stomach Adrenal gland Kidney Spleen Liver

Spleen

A 43-year-old woman who has been diagnosed with immune thrombocytopenic purpura resistant to corticosteroid treatment will undergo an operation to improve this condition. Removal of which of the following labeled organs in the CT scan shown is most likely to be helpful? Spleen

Spleen

A 59-year-old man who has smoked 2 packs of cigarettes daily for 40 years develops an ulcerated laryngeal neoplasm. Examination of tissue obtained on biopsy of the lesion is most likely to show which of the following? Reactive nodule Undifferenti ated carcinoma Squamous cell carcinoma Adenocarcinoma Squamous papilloma

Squamous cell carcinoma

A 28-year-old woman, gravida 1, para 1, comes to the physician because of a 2-day history of a painful mass in her right breast. She delivered a healthy female newborn at term 3 weeks ago, and she has been breast-feeding successfully since then. Her temperature is 37° C (98.6°F). Physical examination shows a 3-cm, tender mass surrounded by an area of erythema beneath the right areola. Which of the following is the most likely causal organism? Haemophilus influenzae Bacteroides fragilis Staphylococcus aureus Escherichia coli Streptococcus agalactiae (group 8)

Staphylococcus aureus

A 55-year-old man comes to the physician because of a 3-day history of fever and shaking chills. He has end-stage renal disease treated with hemodialysis. His temperature is 39.2°C (102.6°F), pulse is 110/min, and blood pressure is 156/92 mm Hg. Physical examination shows a tunneled central venous catheter entering the right side of the chest below the clavicle; the area surrounding the catheter is not tender. Echocardiography shows a 3-cm vegetation on the tricuspid valve. Which of the following is the most likely causal organism? Staphylococcus aureus Klebsiella pneumoniae Streptococcus pyogenes (group A) Viridans streptococcus Escherichia coli

Staphylococcus aureus

A 50-year-old man comes to the physician because of a 2-week history of progressive shortness of breath while climbing stairs to his office every morning. He reports no other problems, but he is concerned because his father had a major myocardial infarction at the age of 52 years. His pulse is 110/min and regular, respirations are 16/min, and blood pressure is 135/95 mm Hg. The lungs are clear to auscultation. Cardiac examination shows normal heart sounds with a physiologic split of S2. Stress echocardiography shows hypokinesis of the posterior left ventricle with increasing activity levels. Which of the following is the most likely cause of the posterior left ventricular findings in this patient? Increased myocardial oxygen consumption Disruption of the sympathetic nerves to the left ventricle Extra vascular compression of the coronary arteries Increased left ventricular end-diastolic pressure Stenosis of right coronary artery

Stenosis of right coronary artery

59-year-old woman with metastatic renal carcinoma is receiving cyclophosphamide, paclitaxel, and interleukin-2 (IL-2). Which of the following is the reason for including IL-2 in the regimen? Decreasing the likelihood of hemorrhagic cystitis Decreasing nausea and vomiting Decreasing bone marrow suppression Stimulating T lymphocytes Decreasing the likelihood of peripheral neuropathy

Stimulating T lymphocytes

A 56-year-old male with extensive coronary atherosclerosis presents with coronary vasoconstriction and myocardial ischemia. The most likely cause of the vasoconstriction is failure of the endothelial cell layer of the diseased coronary arteries to produce a substance with which of the following actions? Stimulation of ATP accumulation Stimulation of cyclic GMP accumulation Activation of Na+ channels Closure of K+ channels Opening of L-type Ca2+ channels

Stimulation of cyclic GMP accumulation

A 58-year-old male presents to the emergency room with acute chest pain, dyspnea, and dyaporesis. His blood pressue is 158/90 mm Hg and respirations are 20/min. An ECG reveals non ST-elevation . History reveals severe coronary atherosclerosis that has been managed with statins and diet. Coronary angiography reveals significant luminal irregularities. During the proceedure, acetylcholine is infused. Results show that a significant portion coronary arteries fail to respond. Which of the following produce a substance with which of the following actions? Stimulation of ATP accumulation Opening of L-type Ca2+ channels Stimulation of cyclic GMP accumulation Activation of Na+ channels Closure of K+ channels

Stimulation of cyclic GMP accumulation

A 35-year-old man has an adenoma of the parathyroid gland, with increased serum concentrations of parathyroid hormone (PTH) and calcium. In this patient, PTH induces which of the following processes to cause hypercalcemia? Stimulation of osteoclast activity

Stimulation of osteoclast activity

A 35-year-old man has an adenoma of the parathyroidgland, with increased serum concentrations of parathyroidhormone (PTH) and calcium. In this patient, PTH induceswhich of the following processes to cause hypercalcemia? Stimulation of osteoclast activity Shift of Ca2+ from the intracellular to theextracellular fluid compartment Production of 25-hydroxycholecalciferol Suppression of renal production of 1,25-dihydroxycholecalciferol

Stimulation of osteoclast activity

While lifting weights, a 24-year-old man develops a painful swelling in the right inguinal region that cannot be reduced. The photograph shows a segment of the small intestine resected at exploratory laparotomy. Which of the following is the most likely diagnosis? Embolism Adhesions Volvulus Strangulation lntussusception

Strangulation

A 25-year-old woman with a history of rheumatic fever and mitral valve dysfunction comes to the physician because of a 2-week history of fever and fatigue. She underwent a root canal procedure 1 month ago, before which she had taken a single dose of amoxicillin. Her temperature is 38.4°C (101 .2°F). A grade 4/6 blowing murmur is heard on auscultation under the left axilla. A photomicrograph of a Gram stain of the organism recovered from a blood culture specimen is shown. On blood agar plates, the organism shows alpha hemolysis. Which of the following is the most likely causal organism? Streptococcus pneumoniae Streptococcus mitis Group A beta-h emolytic streptococci Staphylococcus aureus Enterococcus faecalis

Streptococcus mitis

A 55-year-old woman develops a headache, temperatures to 40°C (104 °F), a stiff neck, and confusion. Her spleen was removed due to injuries sustained in a motor vehicle collision 7 months ago. Which of the following organisms is most likely to be cultured from the patient's cerebrospinal fluid? Mycobacterium tuberculosis Streptococcus pneumonia Haemophilus influenzae Cryptococcus neoformans Borrelia burgdorferi

Streptococcus pneumonia

A 21-year-old man is brought to the emergency department 45 minutes after sustaining multiple injuries in a motor vehicle collision. His blood pressure is 90/50 mm Hg. Physical examination shows diffuse abdominal tenderness. He is diagnosed with laceration of the spleen and undergoes splenectomy. The splenectomy most likely predisposes this patient to develop future infection with which of the following causal organisms? Cytomegalovirus Streptococcus pneumoniae Escherichia coli Candida albicans Herpes simplex virus

Streptococcus pneumoniae

A 6-week-old boy is brought to the physician for a follow-up examination after newborn hemoglobin electrophoresis showed homozygosity for hemoglobin S. Daily treatment with penicillin is recommended. This treatment will most likely decrease this patient's risk for infection with which of the following organisms? Neisseria meningitidis Salmonella species Escherichia coli Haemophilus influenzae Streptococcus pneumoniae

Streptococcus pneumoniae

A 65-year-old woman comes to the physician because of a 2-day history of a high temperature, chills, and headache. She was diagnosed with multiple myeloma 6 months ago.Her temperature is 39°C (102.2°F), pulse is 90/min, and blood pressure is 150/80 mm Hg. Examination shows lethargy and severe neck rigidity; no other abnormalities are noted. Which of the following organisms is most likely tobe found on Gram stain of cerebrospinal fluid? Streptococcus pneumoniae Haemophilus influenzae Pseudomonas aeruginosa Neisseria meningitidis Listeria monocytogenes

Streptococcus pneumoniae

A 6-year-old girl develops edema, hypertension, hematuria, and proteinuria 2 weeks after an episode of pharyngitis. A diagnosis of acute glomerulonephritis is made. Throat cultures grow catalase-negative, gram-positive cocci. Which of the following is the most likely causal organism? Staphylococcus epidermidis Streptococcus pneumoniae Staphylococcus saprophyticus Streptococcus pyogenes (group A) Staphylococcus aureus Streptococcus mutans

Streptococcus pyogenes (group A)

A 42-year-old woman, gravida 2, para 2, comes to the physician because of increasingly frequent loss of urine during the past year. She has loss of urine when she coughs, sneezes, exercises, or plays with her children. Her incontinence is never preceded by a sudden urge to void, and she does not have loss of urine at night. Her children were born after uncomplicated vaginal deliveries. She has no history of other hospital admissions or serious illness. She takes no medications. Abdominal examination shows no abnormalities. The external genitalia, vagina, and cervix appear normal. The uterus and adnexa are normal to palpation. There is loss of a small amount of urine with Valsalva maneuver. Her post void residual volume is 50 mL. Urinalysis shows no abnormalities. Which of the followingis the most likely diagnosis? Stress incontinence Overactive bladder with incontinence Overflow incontinence Vesicovaginal fistula Urinary tract infection

Stress incontinence

A 27-year-old woman is admitted to the hospital because of a 12-hour history of fever and abdominal pain. She has a history of recurrent urinary tract infections. Her temperature is 39°C (102.2°F). Abdominal examination shows tenderness of the right flank. Abdominal x-rays show bilateral staghorn renal calculi. Urinalysis shows a pH of 8, many RBCs, WBCs, and bacteria. The calculi are most likely composed of which of the following in this patient? Calcium oxalate Uric acid Struvite Calcium phosphate Cystine Hemoglobin

Struvite

A 60-year-old woman comes to the physician because of a 2-day history of flank pain and pain with urination. She has hypertension, recurrent urinary tract infections, and hypothyroidism. Her temperature is 37°C (98.6° F), pulse is 96/min, and blood pressure is 152/92 mm Hg. Physical examination shows no other abnormalities. Urinalysis shows 3+ blood, 1+ leukocytes, and few bacteria. Ultrasonography of the abdomen shows a large calculus filling the entire right renal pelvis. The renal calculus in this patient is most likely composed of which of the following substances? Cystine Uric acid Calcium oxalate Struvite

Struvite

A 28-year-old woman comes to the physician because of a 4-day history of palpitations, severe neck pain, fatigue, and malaise. Her pulse is 120/min and regular. Physical examination shows a diffusely tender, mildly enlarged thyroid gland. There is no exophthalmos. Serum studies show a thyroid-stimulating hormone concentration of 0.01 μU/ml. Which of the following is the most likely diagnosis? Thyroid abscess Factitious thyrotoxicosis Graves disease Subacute granulomatous thyroiditis Toxic multinodular goiter

Subacute granulomatous thyroiditis

A 45-year-old man is brought to the emergency department because of a 6-hour history of severe headache and confusion. He says that it is the worst headache he has ever had. He appears lethargic. His blood pressure is 130/80 mm Hg. Physical examination shows nuchal rigidity. A CT scan of the head without contrast is shown. Which of the following is the most likely cause of this patient's condition? Hydrocephalus Diffuse cerebral edema Subdural hematoma Subarachnoid hemorrhage Large mass in the posterior fossa

Subarachnoid hemorrhage

A previously healthy 10-year-old boy is brought to the emergency department by his parents immediately after the sudden onset of difficulty breathing that began when he was stung on the arm by a bee. On arrival, he is in severe respiratory distress. His temperature is 38°C (100.4°F), pulse is 115/min, respirations are 60/min, and blood pressure is 80/40 mm Hg. Examination shows supraclavicular and intercostal retractions. There is diffuse urticaria over the trunk and upper and lower extremities. Expiratory wheezes are heard bilaterally. Which of the following is the most appropriate initial pharmacotherapy? Subcutaneous epinephrine Intravenous methylprednisolone Intravenous ranitidine Oral diphenhydramine Inhaled fluticasone

Subcutaneous epinephrine

A 47-year-old woman develops an irregular, raised, multicolored dark lesion on the area of her left forearm that is frequently exposed to direct sunlight. Examination of tissue obtained on biopsy shows malignant pigmented cells. Involvement of which of the following skin layers is associated with the greatest risk for an unfavorable prognosis? Subcutaneous tissue Reticular dermis Papillary dermis Epidermis Basement membrane

Subcutaneous tissue

A 23-year-old woman, who was diagnosed with Sjogren syndrome 6 years ago, comes to the physician because she has had burning pain in her toes for the past month. She rates the pain as a 3 on a 10-point scale. She uses capsaicin cream as needed for the pain, which has produced partial relief. Medical history is otherwise unremarkable. Vital signs are within normal limits. Neurologic examination shows decreased perception of temperature over the feet bilaterally. Sensation to vibration, proprioception, reflexes, muscle strength, muscle tone, and bulk are normal. Which of the following neurotransmitters is most likely mediating this patient's pain? Enkephalin Dopamine Serotonin y-Aminobutyric acid Substance P

Substance P

A 51-year-old woman undergoes dilatation and curettage for dysfunctional uterine bleeding. During the induction of anesthesia, isoflurane is administered along with a second compound. Thirty minutes later, she develops a temperature of 40.5°C (105°F) and muscle rigidity. Simultaneous administration of which of the following compounds is the most likely cause of the adverse effect in this patient? Succinylcholine Bupivacaine Ketamine Midazolam Neostigmine

Succinylcholine

A previously healthy 27-year-old woman is brought to the emergency department because of a 6-hour history of high-grade fever, rash, and progressive confusion. She underwent a sinus operation for removal of nasal polyps 2 days ago. On arrival, she is difficult to arouse. Her temperature is 39.2°C (102.5°F), pulse is 110/min and thready, respirations are 18/min, and blood pressure is 86/60 mm Hg. Physical examination shows a diffuse rash with erythroderma and hyperemic oral mucous membranes. Cardiopulmonary examination shows no abnormalities. The liver edge is palpable just below the right costal margin. Removal of surgical packing in the nares shows erythematous nasal mucosa. Laboratory studies show: Hemoglobin 11.5 g/dL Hematocrit 34.5% Leukocyte count 13,000/mm3 Segmented neutrophils 80% Bands 8% Lymphocytes 10% Monocytes 2% Platelet count 88,000/mm 3 Serum Creati nine AST ALT 2.2 mg/dL 55 U/L 60 U/L A Gram stain of a nasal swab specimen shows gram-positive cocci in clusters. The causal organism most likely produced the systemic syndrome in this patient via which of the following mechanisms? A-Toxin-mediated depolarization of plasma membranes Leukocidin-mediated lysis of phagocytic cells Superantigen activation of T lymphocytes Toxin B-mediated cleavage of desmoglein-1 Lipase degradation of lipids

Superantigen activation of T lymphocytes

A 63-year-old woman comes to the physician because of a 2-month history of a drooping left eyelid, intermittent pain of her left eye, and an unusually large pupil. Her husband has told her that the eye appears to be "looking sideways." Ophthalmologic examination shows ptosis, lateral strabismus, and mydriasis of the left eye. An MRI of the brain shows an aneurysm of the left posterior communicating artery. The function of which of the following muscles is most likely to remain intact in this patient? Ciliary Inferior oblique Superior oblique Superior rectus Inferior rectus Pupillary constrictor

Superior oblique

A 46-year-old man is treated with oral cyclosporine after a cardiac transplant. Cyclosporine decreases the likelihood of rejection in this patient by which of the following actions? Increasing the production of interleukin by T lymphocytes Forming 6 -thiouric acid and blocking metabolism of nucleic acid Suppressing the early response of T lymphocytes to activation Blocking recognition of antigens by mononuclear cells Suppressing the activation and production of B lymphocytes

Suppressing the early response of T lymphocytes to activation

A 28-year-old man is undergoing evaluation for infertility. He is a weight lifter and takes anabolic steroids. Which of the following mechanisms best explains his infertility? Suppression of gonadotropins Increased occupancy of testosterone receptors in the testes Increased conversion of androgen to estrogen Impaired liver metabolism Increased conversion to dihydrotestosterone

Suppression of gonadotropins

A 60-year-old man comes to the physician for an examination prior to employment. Physical examination shows no abnormalities. Laboratory studies show: Hemoglobin 1 4 g/dL Hematocrit 42°/c, Leukocyte count 12,000/mm3 Segmented neutrophils 45°/c, Small lymphocytes 50°/c, Monocytes 5%, Platelet count 250,000/mm3 Flow cytometry analysis of peripheral lymphocytes shows the following phenotype: CD3 50% CD4 40% CDS 10% CD20 50% Surface kappa 47% Surface lambda 3% Which of the following is most predictive of a clonal lymphoid proliferation in this patient? Absolute CD3+ T-lymphocyte count Surface kappa: surface lambda ratio CD4:CD8 ratio Absolute CD20+ 8 -lymphocyte count Absolute l ymphocyte count

Surface kappa: surface lambda ratio

A 57-year-old man dies of a myocardial infarction. At autopsy, examination of lung tissue shows alveoli containing epithelial cells that are cuboidal in shape with electron dense lamellar bodies 1-2 μ in diameter. These cells most likely produce which of the following? Surfactant Epinephrine Histamine Collagen, type I lgG

Surfactant

A male newborn delivered at 26 weeks' gestation develops respiratory distress immediately after a spontaneous vaginal delivery. His respirations are 40/min. Physical examination shows cyanosis and lower rib retractions with respiration. Heart sounds are normal. Bilateral breath sounds are heard on auscultation. A chest x-ray shows bilateral lung opacities. The most likely cause of this patient's breathing difficulties is insufficient production of which of the following substances? Meconium Surfactant Retinoic acid α-Fetoprotein Collagen, type III

Surfactant

A 48-year-old woman comes to the emergency department because of a 3-hour history of nausea and acute, sharp, right-sided lower abdominal pain that radiates to her back. She says the pain started 30 minutes into her aerobics class and has increased in severity since then. Physical examination shows an enlarged right ovary. The result of a urine pregnancy test is negative. Ultrasonography of the pelvis confirms the presence of an enlarged ovary, and Doppler ultrasonography shows decreased adnexal flow. Which of the following structures is most likely affected in this patient? Round ligament of the uterus Transverse cervical ligament Oviduct Suspensory ligament Mesometrium

Suspensory ligament

A 34-year-old man is evaluated after becoming lightheaded after running 12 miles of a marathon on a hot day. His pulse is 130/min and his blood pressure is 80/60 mm Hg. Which of the following changes in his autonomic nervous system most likely occurred? Sympathetic; increased, parasympathetic; decreased

Sympathetic; increased, parasympathetic; decreased

A 57-year-old woman with inoperable small cell carcinoma of the lung has had lethargy, loss of appetite, and nausea for 1 week.She received radiation therapy 2½ years ago. She has stable angina pectoris controlled with nitrates. Her pulse is 68/min,respirations are 16/min, and blood pressure is 118/72 mm Hg. There is no jugular venous distention, and skin turgor is normal. Sheis oriented to person and place but not to time. Laboratory studies show:SerumNa+ 128 mEq/LCl- 100 mEq/LK+ 4.2 mEq/LHCO3- 24 mEq/LUrea nitrogen 11 mg/dLGlucose 92 mg/dLCreatinine 0.8 mg/dLOsmolality 270 mOsmol/kg H2OUrineNa+ 78 mEq/LOsmolality 310 mOsmol/kg H2OAn x-ray of the chest shows a mass in the right upper lobe of the lung that is unchanged from an x-ray taken 3 months ago. Which ofthe following is the most likely explanation for these findings? Compulsive water drinking Surreptitious use of diuretics Congestive heart failure Hypothyroidism Syndrome of inappropriate secretion of ADH (vasopressin)

Syndrome of inappropriate secretion of ADH (vasopressin)

A 34-year-old man who is a potter has burned his hands while firing pots on several occasions over the past 3 months. He has no pain during or after a burn. Examination shows mild atrophy of the arms and hands, absence of deep tendon reflexes in the upper extremities, and decreased pain and temperature sensations in the C-4 to T-1 dermatomes. Touch sensation is preserved. Which of the following is the most likely diagnosis? Vitamin 812 (cobalamin) deficiency Cervical spondylolisthesis Tabes dorsalis Syringomyelia Amyotrophic lateral sclerosis

Syringomyelia

A 46-year-old female with a chronic illness affecting her cardiopulmonary function dies. The photograph shows her heart as seen at autopsy. This appearance is most consistent with which of the following disorders? Alcoholic cardiomyopathy Atrial septal defect Primary pulmonary hypertension Mitral stenosis Systemic hypertension

Systemic hypertension

A 20-year-old African American woman comes to the physician because of a 6-month history of diffuse joint pain, especially in her hips and knees. During this period, she occasionally has had a rash on her nose and cheeks. She has no history of serious illness and takes no medications. Her temperature is 38.1°C (100.5°F). Examination shows warmth and swelling of the knees. Laboratory studies show: Hemoglobin 10.5 g/dL Erythrocyte sedimentation rate 40 mm/h Serum Urea nitrogen 30 mg/dL Creatinine 1.8 mg/dL Which of the following is the most likely diagnosis? Septic arthritis Systemic lupus erythematosus Reactive arthritis Psoriatic arthritis Gouty arthritis Rheumatoid arthritis

Systemic lupus erythematosus

A 13-year-old boy is scheduled to receive chemotherapy for a leukemia that has the histologic features of malignant lymphocytes. This neoplasm is further typed for cell surface and intracellular markers specific for lymphocyte subsets. The neoplastic cells do not express the following markers: CD4, CDS, surface lgM, surface lgG, cytoplasmic lgM and μ-heavy chain, cytoplasmic lgG, and y-heavy chain. The leukemic cells express class I MHC molecules and show rearrangement of the T-lymphocyte receptor [3-chain gene D and J segments. Which of the following is the normal counterpart of these malignant lymphocytes? T Lymphocyte thymocytes localized to the thymic cortex Activated cytolytic effector T lymphocytes in the circulation Mature IgM-secreting B lymphocytes in the lymphnode Mature immunoglobulin-secreting plasma cells in the lymph node Pre-B lymphocyte progenitor of mature B lymphocytes in the bone marrow

T Lymphocyte thymocytes localized to the thymic cortex

A 6-month-old boy with a history of recurrent upper and lower respiratory tract infections is brought to the physician by his parents for a follow-up examination. He has hypocalcemia currently treated with calcium supplementation. He underwent operative repair of coarctation of the aorta in the newborn period. Physical examination shows a broad nasal bridge and dysplastic pinnae. Which of the following is the most likely cause of this patient's recurrent infections? Hypocellular bone marrow lgE deficiency Hyper-lgM syndrome T lymphocyte deficiency lgG deficiency

T lymphocyte deficiency

A 24-year-old man comes to the physici an because of a 2-day history of an itchy rash on his arms and legs. He returned from a camping trip in the woods 5 days ago. Examination of the upper and lower extremities shows a widespread edematous, erythematous rash with linear vesicles. The most likely cause of these findings is activation of which of the following cell types? Natural killer cells B lymphocytes T lymphocytes Mast cells Neutrophils

T lymphocytes

An 18-year-old man has had temperatures to 38.3°C (101 °F), a sore throat, and cervical lymph node enlargement for 8 days. A latex agglutination test result for Epstein-Barr virus antibody is positive. Atypical cells with abundant lacey cytoplasm in the peripheral blood smear are most likely derived from which of the following types of cells? B lymphocytes Basophils Plasma cells T lymphocytes Neutrophils

T lymphocytes

A 27-year-old female presents for prenatal genetic counseling. History reveals that her father was born with a heart defect and developed schizophrenia as an adult, while her brother has a cleft palate, a congenital heart defect, and intellectual disability. This disorder is associated with a loss-of-function in which of the following genes? T-box transcription factor (TBX10) Paired box 6 (PAX6) Lissencephaly (LIS1) Ubiquitin-protein ligase E3A (UBE3A) Catlike critical region (CLCR) Wilms tumor (WT1)

T-box transcription factor (TBX10)

A 27-year-old woman comes to the physician because of a 3-week history of fatigue and blurred vision. She occasionally has had double vision during this period. For the past year,she has had 3- to 4-day episodes of numbness and tingling of her arms and legs. She has no personal or family history of serious illness. Her only medication is an oral contraceptive. She has smoked one pack of cigarettes daily for 5 years. Vital signs are within normal limits. Funduscopic examination shows no abnormalities. Visual acuity is decreased in the left eye. Sensation to light touch is decreased over the hands and feet; sensation to pinprick is increased over the fingers and toes bilaterally. An MRI of the brain shows several hyperintense oval plaques in the periventricular region on T2-weighted images. A biopsy of the CNS plaques would reveal which of the following? T-cells and macrophages B-cells Glial cells Collagen Astrocytes

T-cells and macrophages

An 18-year-old man with Crohn disease is admitted to the hospital because of a 1-day history of severe abdominal pain and intermittent bloody diarrhea. His temperature is 38°C (100.4°F), pulse is 98/min, and respirations are 18/min. Physical examination shows a draining anal fistula. Treatment with broad-spectrum antibiotics and prednisone, along with intravenous hydration and parenteral nutrition, is initiated. He recovers over the next 3 weeks. In addition to resolving the infection, the most likely mechanism of action of this pharmacotherapy is suppression of which of the following? Complement activity Mast cell degranulation Antibody binding T-lymphocyte function Neutrophil function

T-lymphocyte function

A 20-year-old man with suspected appendicitis has peri umbilical burning and discomfort, which eventually localize to a region in the lower right quadrant about 5 cm superomedial to the anterosuperior iliac spine. The best explanation for this finding is that the peri umbilical region of the body wall and appendix are both supplied by afferent fibers with sensory cell bodies located in the dorsal root ganglia of the spinal nerves at which of the following levels? S2 T10 T5 L1 T12

T10

A 48-year-old man comes to the physician because he is concerned about his weight. He is 178 cm (5 ft 10 in) tall and weighs 91 kg (200 lb); BMI is 29 kg/m 2• His waist circumference is 103 cm (41 in). His blood pressure is 140/90 mm Hg. Physical examination shows a protuberant abdomen. Laboratory studies show fasting serum glucose and insulin concentrations that are increased and remain increased 2 hours after the patient receives 75 g of oral glucose. Which of the following sets of additional laboratory findings in serum is most likely in this patient? TG: increased HDL-chol: decreased FA: increased TG: HDL-chol: FA:

TG: increased HDL-chol: decreased FA: increased

A 29-year-old woman comes to the physician because of a 5-week history of fatigue and a 4-day history of heart palpitati ons and anxiety. She has primary hypothyroidism treated wi th triiodothyronine. She says that she has doubled the dose of the medication over the past week because of fatigue. Her pulse is 112/min, and blood pressure is 126/70 mm Hg. Physical examination shows a fine motor tremor of the hands. Deep tendon reflexes are brisk. Thyroid function testing is most likely to show which of the following sets of serum concentrations? TSH: decreased, thyroxine: decreased, triiodothyronine; increased

TSH: decreased, thyroxine: decreased, triiodothyronine; increased

A 22-year-old football player is brought to the emergencydepartment 1 hour after he sustained a left leg injuryduring a tackle. Physical examination shows mildtenderness and anterior instability of the tibia with theknee in 90 degrees of flexion (positive drawer sign).Active range of motion of the left knee is limited by pain.Which of the following best explains these findings? Tear of the anterior cruciate ligament Patellar fracture Tear of the medial ligament Tear of the medial meniscus Hemarthrosis

Tear of the anterior cruciate ligament

A 15-year-old girl who recently emigrated from India is brought to the physician because of several lesions on her neck for 2 weeks. Physical examination shows a hypopigmented, hypoesthetic area over the left side of the forehead and 4-cm lesions on the neck. A biopsy specimen of a lesion shows acid-fast bacilli. Which of the following best explains why the causal organism results in dermal rather than in visceral infections? Temperature sensitivity Lipid content Innate antibacterial compounds CO2 requirement Oxygen tolerance

Temperature sensitivity

A 71-year-old woman is brought to the emergency department 1 hour after the sudden onset of double vision. She has a 4-day history of progressive fever, fatigue, headache, left-sided facial pain, and periodic episodes of blurred vision. Her temperature is 38.3° C (101 °F). Physical examination shows mild swelling and tenderness on the left side of the face. Laboratory studies show a mild normochromic anemia and an erythrocyte sedimentation rate of 60 mm/h. Which of the following is the most likely diagnosis? Amyotrophic lateral sclerosis Trigeminal neuralgia Cerebral infarction Multiple sclerosis Temporal arteritis

Temporal arteritis

A 78-year-old woman is brought to the physician becauseof a headache and visual problems for 4 days. Theheadache is more severe on the left side, in the area aboveand in front of her ear. She has had intermittent fevers,difficulty chewing, and a 4.5-kg (10-lb) weight loss duringthe past 6 weeks. Her temperature is 37.4°C (99.4°F).Visual fields are decreased. There is tenderness topalpation on the left side of the scalp. Which of thefollowing is the most likely diagnosis? Migraine Temporal arteritis Acute meningitis Cluster headache Subarachnoid hemorrhage Idiopathic intracranial hypertension

Temporal arteritis

A 62-year-old woman is admitted to the hospital because of a 3-day history of fever, shaking chills, and left flank pain. On admission, she receives the diagnosis of acute pyelonephritis, and treatment with ciprofloxacin is begun. After 3 days of treatment, her fever and chills have resolved, and her pain has decreased. Five days after admission, she has the sudden onset of watery diarrhea and lower abdominal cramps. Her current temperature is 38.3°C (100.9° F), pulse is 80/min, respirations are 18/min, and blood pressure is 124/68 mm Hg. Cardiopulmonary examination shows no abnormalities. Abdominal examination shows moderate tenderness to palpation in the lower quadrants, especially on the right. Bowel sounds are increased. The stool is brown, and test for occult blood is negative. Which of the following is the most appropriate next step in diagnosis? Barium enema Stool culture Test of the stool for Clostridium difficile toxin Assessment of her clinical response after ciprofloxacin is discontinued for 3 days 24-Hour stool collection for measurement of fat concentration Examination of the stool for ova and parasites

Test of the stool for Clostridium difficile toxin

A 32-year-old man comes to the physician because of a 3-month history of swelling and tenderness of both breasts. He has been receiving thyroid hormone and corticosteroid replacement therapy since undergoing surgical removal of a pituitary adenoma 2 years ago. He began human chorionic gonadotropin (hCG) injections 4 months ago. Which of the following is the most likely binding site and action of hCG that is causing the development of gynecomastia in this patient? Pituitary gland Tissue Adrenal gland Breast Liver Pituitary gland Testicle Effect 17-hydroxyprogesterone synthesis direct stimulation testosterone metabolism prolactin secretion estradiol producti on Liver Adrenal gland Effect Breast Testicle, estradiol production 17-hydroxyprogesterone

Testicle, estradiol production

A 40-year-old man comes to the physician because of a 6-month history of difficulty maintaining an erection during sexual intercourse. He has consumed ½ L of bourbon daily for 15 years. His pulse is 88/min, and blood pressure is 130/80 mm Hg. Examination shows scleral icterus and spider angioma over the trunk. The liver span is 5 to 6 cm in the midclavicular line. The spleen tip is palpated 5 to 6 cm below the left costal margin. Decreased serum concentrations of which of the following is the most likely cause of this patient's erectile dysfunction? Follicle-stimulating hormone Human chorionic gonadotropin Estradiol Estrone Luteinizing hormone Testosterone

Testosterone

A 60-year-old man has had mild shortness of breath with exertion for 6 months. He has smoked two packs of cigarettes daily for 40 years. Examination shows mildly decreased breath sounds and prolonged expiratory time. Pulmonary function tests show an FEV1 of 70%. If this patient stops smoking, which of the following best describes the 5-year course of his FEV1? The FEV1 will decrease at the same rate as a nonsmoker The FEV1 will return to normal within 5 years The FEV1 will return to normal within 6 months The FEV1 will decrease at the same rate as a smoker The FEV1 will remain the same over time

The FEV1 will decrease at the same rate as a nonsmoker

A 60-year-old man has had mild shortness of breath withexertion for 6 months. He has smoked two packs of cigarettesdaily for 40 years. Examination shows mildly decreasedbreath sounds and prolonged expiratory time. Pulmonaryfunction tests show an FEV1 of 70%. If this patient stopssmoking, which of the following best describes the 5-yearcourse of his FEV1? The FEV1 will decrease at the same rate as asmoker The FEV1 will decrease at the same rate as anonsmoker The FEV1 will return to normal within 5 years The FEV1 will remain the same over time The FEV1 will return to normal within 6months

The FEV1 will decrease at the same rate as anonsmoker

A 43-year-old woman with metastatic colon cancer has maintained remission by taking bevacizumab as a single agent. The ability to administer this agent despite it being a foreign protein is most likely a result of which of the following? B-cell dysfunction The agent containing the variable region of a murine anti body molecule directed against vascular endothelial growth factor The agent being a humanized antibody T-cell dysfunction The immunosuppressive effect of previously administered chemotherapy

The agent being a humanized antibody

A 20-year-old woman comes to the physician because her menstrual period is 1 week late. Menses had occurred at regular 28-day intervals. She had unprotected sexual intercourse with her boyfriend 20 days ago. A urine pregnancy test result is positive. Which of the following best describes the stage of development of the embryo at this time? Placental development is complete, but the embryo is resistant to teratogens The cytotrophoblast is present, but the syncyti otrophoblast has not yet formed Gastrulation is complete, but there are only two germ layers The sclerotome cells have begun migrating, but the somites have not yet formed The neural plate is present, but the neural tube is not yet complete

The neural plate is present, but the neural tube is not yet complete

A 30-year-old woman has had Hodgkin disease that was diagnosed 10 years ago and was treated with radiation and chemotherapy for a 6-month period. She has remained in remission for 10 years. She now feels weak and has petechiae and ecchymoses. A diagnosis of acute myelogenous leukemia is made. Which of the following best explains this course of events? The patient has a genetic defect in the ability to repair DNA damage The patient had both Hodgkin disease and acute leukemia at the time of initial diagnosis, but the leukemia remained dormant for several years Therapy for Hodgkin disease causes mutations in hematopoietic stem cells, and one mutant cell progresses to leukemia The patient has inherited a predisposition to develop malignancies caused by Epstein-Barr virus Hodgkin disease was not cured by the treatment, and surviving neoplastic cells changed to acute leukemia

Therapy for Hodgkin disease causes mutations in hematopoietic stem cells, and one mutant cell progresses to leukemia

6-Mercaptopurine (6-MP) is used to treat acute lymphoblastic leukemia. In humans, 6-MP is acted on by a series of cellular enzymes to produce 6-thioguanine nucleotides (6-TGN). These modified nucleotides can then be incorporated into DNA. Both the efficacy and toxicity of 6-MP therapy are correlated with the final serum 6-TGN concentration. High serum concentrations of 6-TGN are toxic and can lead to life-threatening complications. Exogenously administered 6-MP also can be acted on by xanthine oxidase (XO), thiopurine methyltransferase (TPMT), or both (shown). Both of these pathways lead to the production of inactive and nontoxic metabolites. Which of the following are the clinical consequences for patients with acute lymphoblastic leukemia who are homozygous for low-activity alleles of TPMT? They must be given increased doses of 6-MP They may be given normal doses of 6-MP They must be given injections of XO if treated with 6-MP They must be given decreased doses of 6-mp

They must be given decreased doses of 6-mp

During the fourth through eighth weeks of gestation, there are five pairs of aortic arches. These will ultimately develop into components of the great arteries. Which of the following pairs of aortic arches develops into the common carotid arteries? Fifth Sixth Second First Fourth Third

Third

A 50-year-old man is admitted to the hospital within 2 hours of the onset of nausea, vomiting, and acute crushing pain in the left anterior chest. He has a family history of early coronary artery disease. The pain does not subside with the administration of nitroglycerin, sublingually. An ECG shows ST-segment elevation in leads aVL and V2 through V4. Which of the following is the most appropriate management to decrease myocardial damage and mortality? Administration of digitalis Administration of lidocaine Thrombolytic therapy Administration of quinidine Coronary artery bypass grafting within 1 week

Thrombolytic therapy

A 5-month-old boy is brought to the physician because of a 5-day history of difficulty feeding. Physical examination shows macroglossia and enlargement of the anterior neck at the mid line. A CT scan of the neck shows a 4-cm, well-defined, cystic structure. A remnant of which of the following is the most likely cause of this patient's condition? Thryglossal duct First pharyngeal pouch Fourth pharyngeal pouch Cervical sinus Occipital myotome

Thryglossal duct

A 30-year-old woman has ptosis, ophthalmoplegia, and diplopia. Examination of serum shows an autoantibody with affinity for the acetylcholine receptor at the postsynaptic neuromuscular junction. This patient is most likely to have which of the following neoplasms? Glioma Neuroblastoma Rhabdomyosarcoma Thymoma Multiple myeloma

Thymoma

A 3-year-old girl is brought to the physician because of a 1-month history of a lump in her upper neck. Physical examination shows a 2-cm mass in the mid line. The mass moves upward with swallowing and protrusion of the tongue. Radionucleotide imaging with technetium 99m pertechnetate shows uptake in the mass. If a biopsy specimen of this mass were obtained, it would most likely show which of the following? Thyroid follicles Serous and mucousacini Chief and oxyphil cells Taste buds Chondroid matrix

Thyroid follicles

A 26-year-old woman comes to the physician's office because of a 13.5-kg (30-lb) weight gain, irregular menstrual periods, constipation, and fatigue for the past 6 months. She also complains of increased difficulties with concentration and memory for the past month. Physical examination shows a blood pressure of 145/84 mm Hg, dry skin, and decreased muscle stretch reflexes at the ankles. Serum cholesterol concentration is 310 mg/dl, and serum potassium concentration is 4.0 mEq/L. Determination of which of the following serum concentrations is most likely to confirm the diagnosis? Testosterone Cortisol Luteinizing hormone Prolactin Thyroid stimulating hormone Hemoglobin A1c Dehydroepiandrosterone

Thyroid stimulating hormone

A 32-year-old woman comes to the physician because of lethargy and boredom since the birth of her son 5 months ago. She worries about her ability to care for him and has had frequent palpitations. She is unable to fall back asleep after nighttime feedings. She stopped breast-feeding 1 month ago. Her son is healthy, and growth and development are normal for his age. Her pulse is 58/min, respirations are 18/min, and blood pressure is 122/80 mmHg. Physical examination shows no abnormalities. She remembers one of three objects after 5 minutes. Her serum cholesterol concentration is 265 mg/dL. The most appropriate next step in diagnosis is measurement of which of the following serum concentrations? Cortisol Prolactin Estrogen Progesterone Thyroid-stimulating hormone

Thyroid-stimulating hormone

A newborn female is diagnosed with hypothyroidism 2 days after birth. Ultrasonography of the neck shows the absence of any thyroid gland tissue. Physical examination shows no abnormalities, including normal root, suck, grasp, and Moro reflexes. Maternal to fetal transfer of which of the following best explains the normal development in this newborn? Iodine Thyrotropin-releasing hormone Thyroxine (T4) Thyroglobulin Thyroid-stimulating hormone

Thyroxine (T4)

A previously healthy 20-year-old woman comes to the emergency department because of a 2-week history of fever, shaking chills, headaches, fatigue, and joint and muscle pain. She recently spent the summer working as a lifeguard on Long Island, New York. She has never traveled outside the USA. She underwent splenectomy for injuries sustained in a motor vehicle collision at the age of 6 years. Her temperature is 39.1°C (102.4°F). Physical examination shows no other abnormalities. A peripheral blood smear shows small intraerythrocytic rings; the result of a polymerase chain reaction test for Plasmodium is negative. This patient most likely acquired the causal organism via a bite from which of the following? Tick Black fly Mite Mosquito Bedbug

Tick

In a steady state, the difference in CO2 content between thevenous blood leaving a tissue and the arterial blood enteringthe tissue is determined by which of the following ratios? Alveolar ventilation to tissue blood flow Tissue CO2 production to venous PCO2 Tissue CO2 production to tissue O2consumption Alveolar ventilation to tissue O2 consumption Tissue CO2 production to tissue blood flow

Tissue CO2 production to tissue blood flow

A 62-year-old man comes to the physician because of blood in his urine for 24 hours. Vital signs are within normal limits. Examination shows no abnormalities. Tissue obtained on bladder biopsy shows transitional cell carcinoma. Abstinence from which of the following is most likely to have prevented this condition? Diet soda Tea Coffee Tobacco Alcohol

Tobacco

A 64-year-old man is brought to the emergency department 3 hours after he developed shortness of breath with exertion and extreme fatigue. He has ischemic heart disease. His pulse is 125/min and irregular, and blood pressure is 105/60 mm Hg. An ECG shows atrial fibrillation. Intravenous ibutilide is administered. Ten minutes later, an ECG shows a normal sinus rhythm. This patient is at greatest risk for which of the following adverse drug effects during the next 6 hours? Sinus arrest Sinus tachycardia Torsades de pointes Atrioventricular nodal reentry Atrioventricular block

Torsades de pointes

A 6-year-old boy has had three systemic infections with Neisseria meningitidis over the past 2 years. He has been healthy otherwise. Which of the following laboratory tests is most likely to be abnormal in the child? Neutrophil myeloperoxidase concentration Absolute neutrophil count Total hemolytic complement concentration Serum lgA concentration CD4+ T- lymphocyte count

Total hemolytic complement concentration

A 27-year-old man with AIDS is brought to the hospital 30 minutes after having a seizure. He currently takes no medications. A CT scan of the head shows multiple ring-enhancing lesions. Which of the following is the most likely diagnosis? Cysticercosis Amoebiasis Trichinosis Toxoplasmosis Echinococcosis Schistosomiasis

Toxoplasmosis

A 30-year-old woman who is at 20 weeks' gestation comes to the physician for a scheduled examination. Her pregnancy has been uncomplicated, and her due date was established by ultrasonography at 10 weeks' gestation. Fundal height is greater than expected for gestational age. Ultrasonography shows markedly increased amniotic fluid volume. Which of the following abnormalities is the most likely cause of these findings? Urethral obstruction Tracheoesophageal atresia Encephalocele Cleft lip Renal agenesis

Tracheoesophageal atresia

A 4030-g (8-lb 14-oz) newborn has internal rotation of the left upper extremity at the shoulder, extension at the elbow, pronation of the forearm, and flexion of the fingers following a low forceps delivery. He was born at term following an uncomplicated pregnancy. Passive range of motion of the left upper extremity is full; the newborn does not cry or grimace when the left arm, shoulder, or clavicle is palpated. Examination shows no other abnormalities. Which of the following is the most likely underlying mechanism of this condition? Posterior dislocation of the cervical vertebrae An inherited skeletal dysostosis Traction on the brachial plexus during delivery Anterior dislocation of the left shoulder Intracerebral hemorrhage

Traction on the brachial plexus during delivery

An investigator is studying mechanisms of wound healing in a 24-year-old patient who sustained a second-degree burn. Two weeks after the initial injury, the affected tissue shows increased fibroblast migration and proliferation, increased synthesis of collagen and fibronectin, and decreased degradation of extracellular matrix by metalloproteinases. These changes were most likely caused by enhanced cellular production of which of the following? Transforming growth factor beta CD94 Peptidoglycan C5a Bradykinin

Transforming growth factor beta

An investigator is studying bacterial virulence factors. Ferric uptake regulation protein (Fur) binds tightly to DNA a short distance from the 5' end of the genes for these virulence factors. The concentrations of the virulence factors of the bacteria are observed under low-iron and high-iron conditions. Results show that the efficiency of Fur binding increases as the concentration of iron in the culture media increases. Based on these findings, Fur most likely regulates which of the following processes in these bacteria? Splicing Post-transcriptional modification Translation Methylation Transcription

Translation

An increase in the amount of iron inside cultured hepatocytes leads to an increase in the amount of ferritin gene products detected by Western blotting. It does not change the amount of ferritin gene products detected by Northern blotting or by the polymerase chain reaction. These results suggest that iron increases the frequency of which of the following events? Transport of ferritin mRNA out of the nucleus Maturation of the ferritin gene transcript Transcription of the ferritin gene Translation of ferritin mRNA

Translation of ferritin mRNA

A 5-day-old newborn has been cyanotic since birth and has a soft systolic murmur. An x-ray of the chest shows decreased pulmonary vascular markings indicating a decrease in pulmonary blood flow. Which of the following is the most likely diagnosis? Tetralogy of Fallot Ventricular septal defect Patent ductus arteriosus Transposition of the great vessels Coarctation of the aorta

Transposition of the great vessels

Five days after falling and hitting her chest, a 55-year old woman has acute midsternal chest pain that radiates to the back and is exacerbated by deep inspiration. Immediately following the accident, she had acute sternal pain that resolved in 1 day. Her temperature today is 37.7°C (99.9°F). A three-component scratchy sound is heard across the precordium. An x-ray of the chest shows a normal cardiac silhouette. An ECG shows diffuse ST segment elevation and T-wave inversion. Which of the following is the most likely diagnosis? Viral pericarditis Purulent pericarditis Traumatic pericarditis Acute myocardial infarction Pleurodynia

Traumatic pericarditis

A 32-year-old nulligravid woman comes to the physician because of a 6-week history of persistent foul-smelling vaginal discharge and vaginal itching. Her symptoms have not improved despite 2 weeks of treatment with over-the counter antifungal medications and fluconazole. She has been sexually active and monogamous with her boy friend during the past year, and they use condoms consistently. Examination shows excoriated labia and erythematous vaginal mucosa. There is a frothy, watery-gray discharge in the posterior vaginal vault. A wet mount preparation of the discharge shows numerous multi-flagellated organisms the size of erythrocytes. Which of the following is the most likely causal organism? Neisseria gonorrhoeae Treponema pallidum Pseudomonas aeruginosa Haemophilus ducreyi Trichomonas vaginalis

Trichomonas vaginalis

A 32-year-old nulligravid woman comes to the physician because of a 6-week history of persistent foul-smelling vaginal discharge and vaginal itching. Her symptoms have not improved despite 2 weeks of treatment with over-the counter antifungal medications and fluconazole. She has been sexually active and monogamous with her boyfriend during the past year, and they use condoms consistently. Examination shows excoriated labia and erythematous vaginal mucosa. There is a frothy, watery-gray discharge in the posterior vaginal vault. A wet mount preparation of the discharge shows numerous multi-flagellated organisms the size of erythrocytes. Which of the following is the most likely causal organism? Trichomonas vaginalis Haemophilus ducreyi Pseudomonas aeruginosa Treponema pallidum Neisseria gonorrhoeae

Trichomonas vaginalis

A 13-year-old girl has a grade 2/6, holosystolic murmur that is heard best over the left fifth intercostal space adjacent to the sternum; it increases with inspiration. This murmur is most consistent with an abnormality of which of the following valves? Pulmonary Aortic Tricuspid Mitral

Tricuspid

A 63-year-old woman comes to the physician because of a 2-week history of daily episodes of severe, lancinating, left-sided facial pain. The pain, which occurs suddenly and lasts 30 to 60 seconds, shoots from the left ear down along the jawline. The pain is sometimes precipitated by chewing or brushing teeth. Vital signs are normal. Physical examination shows no abnormalities except for moderate dental caries. Which of the following is the most likely diagnosis? Temporomandibular joint syndrome Temporal lobe epilepsy Dental abscess Cluster headache Trigeminal neuralgia Temporal arteritis

Trigeminal neuralgia

Prenatal testing of 35-year-old female. Which of the following conditions is the most likely cause of this patient's analyte pattern? Omphalocele Fetal alcohol syndrome Gastroschisis Trisomy 21 Myelomeningocele

Trisomy 21

A 27-year-old woman comes to the physician because of vaginal bleeding for 3 weeks. Her first pregnancy ended with a spontaneous abortion 8 months ago. She has not been able to afford medical treatment since undergoing dilatation and curettage at that time. Physical examination shows an enlarged uterus. Her serum -human chorionic gonadotropin concentration is markedly increased. Ultrasonography of the pelvis shows material within the endometrial cavity and no intrauterine pregnancy. A CT scan shows a necrotic intrauterine mass and metastatic nodules in the lungs. Which of the following is the most likely site of origin of this patient's condition? Trophoblastic tissue Blastocyst Allantois Decidua basalis Decidua capsularis

Trophoblastic tissue

An 18-year-old primigravid woman at approximately 16 weeks' gestation comes to the physician because of a 3-month history of nausea and vomiting. She has not had any prenatal care. Physical examination shows a uterus consistent in size with a 20-week gestation. Her serum 13-human chorionic gonadotropin concentration is markedly increased. Urinalysis shows 4+ protein. Ultrasonography of the abdomen shows echogenic material within the endometrial cavity but no fetus. The most likely cause of her condition is proliferation of which of the following? Decidua capsularis Allantois Decidua basalis Trophoblastic tissue Blastocysts

Trophoblastic tissue

A 65-year-old man who recently emigrated from Brazil comes to the physician because of an 8-month history of shortness of breath and fatigue. Physical examination shows edema of the lower extremities. A chest x-ray shows cardiomegaly. An endomyocardial biopsy specimen shows individual myofiber necrosis with a mixed inflammatory infiltrate of neutrophils, T lymphocytes, macrophages, and eosinophils. Which of the following is the most likely causal organism? Trypanosoma cruzi Trypanosoma brucei Trichinella spiralis Leishmania tropica Wuchereria bancrofti Toxocara canis Leishmania major

Trypanosoma cruzi

A 24-year-old woman comes to the physician for a follow-up examination. She has an 8-year history of seizure disorder that is refractory to medication. Physical examination shows flesh colored papules in the nasolabial folds accompanied by hypopigmented ash-leaf macules. An MRI of the brain shows subependymal nodules. Which of the following is the most likely cause of these findings? Fabry disease Ehlers-Danlos syndrome Tuberous sclerosis Pseudoxanthoma elasticum Neurofibromatosis

Tuberous sclerosis

A 9-year-old boy has mild mental retardation and a 4-year history of increasingly frequent major motor seizures. Physical examination shows normal vital signs, clear lungs, a moderate systolic ejection murmur, and discrete pink-to-yellow papules in a butterfly distribution on the bridge of his nose, the malar prominences, and along his nasolabial folds. A CT scan shows multiple, occasionally calcified nodules on the crests of some gyri in the subependymal regions. Echocardiography shows left ventricular hypertrophy associated with marked narrowing of the aortic valvular outflow tract due to a subvalvular mass arising from the septum. Which of the following is the most likely diagnosis? Tuberous sclerosis Glycogen storage disease, type 1 Multiple endocrine neoplasia, type 1 Familial hypertrophic cardiomyopathy Gardner syndrome

Tuberous sclerosis

A 70-year-old man dies in a motor vehicle collision. He had been undergoing evaluation for occult blood in the stool. A photograph of a section of the transverse colon obtained at autopsy is shown. Which of the following is the most likely diagnosis? Juvenile polyp Tubular adenoma Inflammatory pseudopolyp Peutz-Jeghers syndrome Hyperplastic polyp

Tubular adenoma

A 4-year-old boy undergoes radiographic imaging studies of the urinary tract after having two bacterial urinary tract infections during the past year. His left kidney is found to be abnormally small and nonfunctional. The right kidney appears normal. He undergoes a left nephrectomy and ureterectomy. A photograph of the resected specimen is shown. Microscopic examination of the kidney is most likely to show which of the following changes? Diffuse cortical necrosis Tubular atrophy Thin glomerular basement membranes Neoplastic changes in the tubular epithelium Fibromuscular hyperplasia of the arterioles

Tubular atrophy

A 62-year-old woman with recurrent non-small cell carcinoma of the lung is treated with a chemotherapy combination regimen that includes vincristine. Which of the following intracellular targets is most likely affected by this drug? Tubulin DNA polymerase Cyclin D 1 DNA topoisomerase II DNA topoisomerase I DNA polymerase a Dihydroorotate dehydrogenase Ribonucleotide reductase Dihydrofolate reductase

Tubulin

A 45-year-old man comes to the physician because of intermittent bloody diarrhea and abdominal pain during the past month. Sigmoidoscopy and rectal biopsy specimens show inflammatory bowel disease. A trial of a monoclonal antibody preparation is begun. The antibody in the preparation is most likely to be directed against which of the following components? Bradykinin Fibrin Prostaglandin E2 C3a Class I MHC antigens Tumor necrosis factor

Tumor necrosis factor

A 39-year-old woman with rheumatoid arthritis comes to the physician for a follow-up examination. Treatment with multiple medications, including prednisone and methotrexate, has not been effective at slowing the progress of her condition. The most appropriate next step in pharmacotherapy is a drug that blocks the effects of which of the following cytokines? lnterleukin-2 {IL-2) IL-4 IL-10 Tumor necrosis factor-A Interferon gamma

Tumor necrosis factor-A

Type 1 diabetes mellitus Tetenus Axial dystonia Parkinson disease Ankylosing spndylitis

Type 1 diabetes mellitus

A 15-year-old boy is brought to the physician by his mother because of a 2-month history of sneezing, coughing, and watery eyes immediately after he mows the lawn. Fifteen minutes after his symptoms first begin, he develops a red rash containing areas that are soft to the touch. Physical examination shows boggy, soft, raised, pruritic, erythematous lesions with wheal and flare eruptions over the face, trunk, and back. Which of the following describes the most likely underlying mechanism of the rash in this patient? Type IV (delayed) hypersensitivity Type II {complement-mediated cytotoxic) hypersensitivity Type III (immune complex-mediated) hypersensitivity) Type I (immediate) hypersensitivity

Type I (immediate) hypersensitivity

A 39-year-old man comes to the physician because of a 1-week history of red spots on his shins, joint pain, and fatigue. His temperature is 37°C (98.6°F), and pulse is 74/min. Physical examination shows purpura over the lower extremities. The liver edge is palpated 4 cm below the right costal margin. Laboratory studies show: Leukocyte count 10,000/mm 3 Serum AST 142 U/L ALT 154 U/L Hepatitis C virus RNA pos Anti-hepatitis C virus antibody pos Cryoglobulins pos C4 120 (N = 350-360) Urine Protein 4+ Urine RBC numerous Which of the following types of hypersensitivity reactions is the primary cause of the renal damage in this patient? Type III (immune complex-mediated) Type II (complement-mediated cytotoxic) Type IV (delayed) Type I (immediate)

Type III (immune complex-mediated)

A 32-year-old man with an X-linked recessive disease has deafness, hematuria, and progressive renal failure. He most likely has an abnormality in which of the following proteins? Fibronectin Laminin Fibrillin Thrombospondin Tropoelastin Osteocalcin Keratin Type IV collagen

Type IV collagen

Case of Duchenne muscular dystrophy. This patient's gene mutation has most likely resulted in which of the following RNA codon changes? UAC >> CAC CUU >> AUU UUU >> UUC UCA >> UGA UAA >> UAG

UCA >> UGA

A 44-year-old woman comes to the physician for a follow-up examination after two separate Pap smears showed atypical squamous cells of undetermined significance. Results of a molecular diagnostic test show the presence of the viral E6 protein of human papillomavirus. This protein is known to promote cell growth and malignancy by causing cellular p53 protein degradation. This degradation most likely begins when the p53 protein is targeted to which of the following types of cellular enzymes? Lysine acetyltransferase Serine protease Lysosomal protease Tyrosine kinase Ubiquitin ligase

Ubiquitin ligase

A 29-month-old boy is brought to the physician by his mother because of a 2-week history of a green nasal discharge and cough. He has a history of frequent respiratory tract illnesses and pneumonia. The mother states that her son has a 6-month history of large, foul-smelling stools and a salty taste when she kisses him. There is no family history of a similar condition. He is at the 5th percentile for height and below the 3rd percentile for weight. His temperature is 38.3°C (100.9° F). Abdominal examination shows distention. Genetic testing shows a mutation in the Phe Residue at position 508 in the cystic fibrosis transmembrane conductance regulator gene. Which of the following types of modification is required prior to destruction of misfolded proteins in this patient's proteasomes? Ubiquitination Glycosylation Sulfation Phosphorylation Dimerization

Ubiquitination

A 32-year-old woman, gravida 4, para 0, aborta 3, delivers a female newborn at term with dysmorphic features and numerous organ anomalies. The mother has had multiple spontaneous abortions, and this is her first liveborn child. Which of the following is the most likely explanation for the abnormalities seen in this newborn? Gonadal mosaicism Autosomal recessive inheritance Autosomal dominant inheritance Submicroscopic deletion Unbalanced chromosome rearrangement

Unbalanced chromosome rearrangement

A 22-year-old nulligravid woman develops right flank pain 3 days after undergoing an appendectomy and right ovarian resection because of an appendiceal abscess. Menses occur at regular 28-day intervals. Vital signs are within normal limits. Physical examination shows mild right flank tenderness. The most likely cause of this patient's pain is the accidental ligation of which of the following structures on the right? Renal artery Ovari an artery Renal vein Ureter Uterine ligament Ovari an vein

Ureter

A 25-year-old man is brought to the emergency department because of severe abdominal pain, nausea, and vomiting for 1 hour. The pain originates in the left flank and radiates to his groin. His pulse is 100/min, respirations are 18/min, and blood pressure is 150/100 mm Hg. Physical examination shows tenderness of the left flank and the left lower quadrant of the abdomen. Bowel sounds are mildly hypoactive. Test of the stool for occult blood is negative. Which of the following best explains these findings? Epididymitis Colon neoplasm Renal infarction Torsion of the testis Ureteral calculus Diverticulitis

Ureteral calculus

A 65-year-old woman comes to the physician because of a 1-week history of a swollen, painful left knee. Her temperature is 38°C (100.4°F), and blood pressure is 110/65 mm Hg. Examination of the left knee shows erythema and swelling of the joint and decreased range of motion. A photomicrograph of synovial fluid obtained by arthrocentesis is shown. This patient's synovial fluid most likely contains which of the following? Lymphocytes Hydroxyapatite Neisseria gonorrhoeae Treponema pallidum Uric acid

Uric acid

A 30-year-old man has had nausea, vomiting, and severe colicky right flank pain radiating into the thigh for 4 hours. He is afebrile. There is right costovertebral angle tenderness. Urinalysis shows RBCs too numerous to count and no bacteria. Which of the following is the most likely diagnosis? Bladder carcinoma Renal cell carcinoma Urinary tract tuberculosis Acute glomerulonephritis Benign prostatic hyperplasia Urolithiasis

Urolithiasis

A 15-year-old girl comes to the physician's office for a health maintenance examination. Her mother was recently diagnosed with squamous cell carcinoma of the face, and her maternal grandfather died of metastatic melanoma. During the visit, the physician advises her about methods of photoprotection, including daily use of a sunscreen. In a patient of this age, which of the following factors is most likely to predict compliance with photoprotection? Ability to tan Desire to prevent sunburn Concern about premature aging of skin Family history of skin cancer Use of sunscreen by her peers Advice of the physician to wear sunscreen

Use of sunscreen by her peers

Two hours after vaginal delivery at term of a 3062-g (6-lb 12-oz) newborn, a 32-year-old woman, gravida 3, para 3, has the onset of heavy vaginal bleeding. Labor was augmented with oxytocin because of a prolonged first stage and required forceps delivery over a midline second-degree episiotomy. The abdomen is soft and nontender. Examination shows a boggy uterus palpated 4 cm above the umbilicus. The perineum is intact. Which of the following is the most likely cause of this patient's hemorrhage? Uterine rupture Episiotomy site bleeding Uterine infection Uterine atony Disseminated intravascular coagulation

Uterine atony

Two hours after vaginal delivery at term of a 3062-g (6-lb12-oz) newborn, a 32-year-old woman, gravida 3, para 3,has the onset of heavy vaginal bleeding. Labor wasaugmented with oxytocin because of a prolonged first stageand required forceps delivery over a midline second-degreeepisiotomy. The abdomen is soft and nontender.Examination shows a boggy uterus palpated 4 cm above theumbilicus. The perineum is intact. Which of the followingis the most likely cause of this patient's hemorrhage? Episiotomy site bleeding Disseminated intravascular coagulation Uterine infection Uterine atony Uterine rupture

Uterine atony

An investigator is studying beta2-adrenoreceptors in female experimental animals. During the experiment, epinephrine is injected intramuscularly into each animal, and the effects on beta 2 adrenoreceptors are then observed. Which of the following physiologic effects is most likely to be observed in these animals? Pilomotor contration Increased myocardial contractility Lipolysis Pupillary dilation Internal urethral sphincter contraction Uterine relaxation

Uterine relaxation

A 55-year-old woman is scheduled to undergo transvaginal hysterectomy and oophorectomy for dysfunctional uterine bleeding. During the procedure, the uterus must be separated from all surrounding pelvic structures. Identification and incision of which of the following structures that attaches to the cervical region and extends posteriorly is most appropriate in this patient? Uterosacral ligament O D) Ovarian ligament O E) Round ligament of the uterus O B) Mesosalpinx A) Mesometrium O C) Mesovarium

Uterosacral ligament

A 62-year-old woman with recurrent pulmonary emboli comes to the physician for a follow-up examination. Physical examination shows no abnormalities. Laboratory studies show a prothrombin time of 12 seconds. Warfarin therapy is begun. Which of the following clotting factors is the first to be decreased by 50% after the initiation of therapy? VII (proconvertin) XI (plasma thromboplastin antecedent) XII (Hageman factor) V (proaccelerin) VIII (antihemophilic factor)

VII (proconvertin)

A 49-year-old man with pancreatic cancer undergoes a Whipple procedure to resect the distal stomach, duodenum, and head of the pancreas. During the procedure, the cut end of the remaining stomach is joined to the side of the jejunum (gastrojejunostomy). Which of the following nerve structures can be cut to decrease stomach acid output and decrease the likelihood of ulcers forming in the jejunum at the site of junction with the stomach? Vagal trunks Subcostal nerves Celiac plexus Phrenic nerves iliohypogastric nerves

Vagal trunks

A 38-year-old woman comes to the physician because of a 3-day history of sore throat. During the examination, the patient is asked to protrude the tongue and say "ah," which causes elevation of the area at the tip of the arrow in the photograph shown. Function of which of the following nerves most likely is being tested by this maneuver? Vagus Hypoglossal Facial Glossopharyngeal Spinal accessory

Vagus

A 15-year-old boy has intracranial neoplasms, mental retardation, and seizures associated with tuberous sclerosis complex, an autosomal dominant disorder. His 41-year-old mother is healthy but has angiofibroma, which are benign skin lesions characteristic of tuberous sclerosis complex. Which of the following best explains the findings in the boy and his mother? Anticipation Reduced penetrance Parental germline mosaicism Alternate paternity Variable expressivity

Variable expressivity

A 64-year-old man comes to the physician because of a 3-day history of painful rash over his right flank. Three daysbefore the rash appeared, he had pain in the same area. Histemperature is 37.4°C (99.3°F), pulse is 78/min,respirations are 17/min, and blood pressure is 130/70 mmHg. Physical examination shows clustered lesions in aband-like area over the right flank. The lesions have anerythematous base and are crusted. Which of the followingis the most likely causal virus? Cytomegalovirus Varicella-zoster virus Human papillomavirus Epstein-Barr virus Kaposi sarcoma virus

Varicella-zoster virus

A 40-year-old woman is brought to the emergency department 30 minutes after sustaining severe chest injury es in a motor vehicle collision. Despite appropriate treatment, she dies 1 hour later. She had a 25-year history of a well-controlled seizure disorder. A photograph of a coronal section of the brain obtained at autopsy is shown. Which of the following is the most likely cause of this patient's seizure disorder? (see picture) Herpes simplex encephalitis Neonatal ischemic stroke Mesial temporal sclerosis Vascular malformation Glioblastoma multiforme

Vascular malformation

A 49-year-old man is brought to the emergency department 30 minutes after fainting in the street. He regained consciousness 1 minute after fainting. He says that he has had watery diarrhea during the past 5 days, which has not improved with fasting. He has not changed his diet or travelled overseas recently. His blood pressure is 90/60 mm Hg. Physical examination shows a flushed face and dehydration. Serum studies show a potassium concentration of 2 mEq/L and glucose concentration of 150 mg/dL. He is admitted to the hospital, and intravenous fluid replacement is started. Over the next day, he passes a stool with a volume of 3.5 L. Which of the following is the most likely cause of this patient's diarrhea? Vasoactive intestinal polypeptide secretion Bacterial overgrowth Lactose intolerance Inactivation of lipase Cholera

Vasoactive intestinal polypeptide secretion

A 54-year-old woman with hypertension and bilateral renal artery stenosis starts taking a nonsteroidal anti-inflammatory drug for back pain. Over the next week, her serum creatinine concentration increases from 1.0 to 5.0 mg/dL. The most likely cause of this finding is the drug's ability to inhibit which of the following? Inflammation in the glomerular capillaries Inflammation in the renal interstitium Vasodilating prostaglandins at the afferent arteriole Vasodilating prostaglandins at the efferent arteriole Vasoconstricting prostaglandins at the efferent arteriole Vasoconstricting prostaglandins at the afferent arteriole

Vasodilating prostaglandins at the afferent arteriole

A 28-year-old woman, gravida 2, para 1, comes to the physician at 36 weeks' gestation because of a 3-week history of intermittent anal itching, pain with defecation, and occasional bright red blood with wiping. She admits to intermittent constipation for which she has used mineral oil enemas. Examination of the perianal area shows the findings in the photograph. Which of the following is the most likely underlying cause of the lesion in this patient? Varicella-zoster virus infection Lymphatic obstruction Tinea cruris Venous hypertension Candidiasis

Venous hypertension

A 2-month-old boy is brought to the physician for a well-child examination. The mother has no concerns about his growth or breast-feeding habits and says that he has begun to smile. He is at the 25th percentile for length and 30th percentile for weight. Cardiac examination shows a blowing holosystolic murmur heard best over the lower left sternal border. Which of the following is the most likely cause of the cardiac findings in this patient? Patent foramen ovale Coarctation of the aorta Ventricular septal defect Functional murmur Patent ductus arteriosus

Ventricular septal defect

A 3-year-old girl is found to have a grade 4/6, loud, harsh, high-pitched holosystolic murmur that radiates over the precordium and a palpable thrill at the left sternal border. Which of the following defects is most likely in this patient? Mitral stenosis Tri cuspid regurgitation Pulmonic stenosis Atrial septal defect Patent ductus arteriosus Coarctation of the aorta Aortic regurgitation Aortic stenosis Mitral regurgitation Ventricular septal defect

Ventricular septal defect

A 60-year-old man is brought to the emergency department because of a 1-day history of fever, chills, and confusion and memory loss. He recently returned from a trip to the Gulf coast of the USA where he walked barefoot on the beach. He has a history of severe cirrhosis and portal hypertension. His temperature is 39°C (102.2°F), respirations are 22/min, and blood pressure is 90/48 mm Hg. Physical examination shows marked cellulitis of the right lower extremity with early blister formation. One day later, blood cultures grow a gram-negative, lactose-fermenting organism. Which of the following is the most likely causal organism? Streptococcus pneumoniae Vibrio vulnificus Salmonella typhi Staphylococcus aureus Listeria monocytogenes Haemophilus influenzae

Vibrio vulnificus

A 7-year-old boy is brought to the physician because of a 1-day history of fever, headache, nonproductive cough, and muscle aches. His temperature is 39.4 •c (102.9°F). Physical examination shows a mildly erythematous pharynx. A rapid antigen test of nasal secretions is positive for influenza A virus. Treatment with zanamivir is initiated. Which of the following viral activities will most likely be directly inhibited by this pharmacotherapy? Virion release RNA packaging Transcription of viral genes Protein synthesis Fusion of viral envelope and cell membrane

Virion release

A 35-year-old woman comes to the physician because of fever and sharp chest pain for 3 days. Her temperature is 38.5°C (101.3° F). A friction rub is heard on auscultation of the chest. After a thorough examination, all secondary causes of pericarditis are ruled out. Which of the following is the most likely cause of primary pericarditis in this patient? Virus Bacterium Parasite Fungus Tumor

Virus

A 14-year-old boy has had fatigue, intermittent right lower quadrant abdominal pain, diarrhea, and a 5-kg (11-lb) weight loss over the past 6 months. A diagnosis of inflammatory bowel disease limited to the terminal ileum is made. Absorption of which of the following is most likely to be impaired in this patient? Iron Vitamin B12 (cobalamin) Vitamin B1 (thiamine) Vitamin B2 (riboflavin) Folic acid

Vitamin B12 (cobalamin)

A 77-year-old woman is admitted to the hospital because of difficulty walking. She has had progressive pain and paresthesia of both feet over the past 3 weeks. She has a history of mild hypertension treated with hydrochlorothiazide and hypothyroidism treated with thyroid replacement therapy. Her pulse is 80/min, respirations are 16/min, and blood pressure is 150/80 mm Hg. Neurologic examination shows decreased ankle jerk reflexes bilaterally and decreased vibratory sense and proprioception in the lower extremities. Laboratory studies show: Hemoglobin 10 g/dL Leukocyte count 11,000/mm3 with a normal differential Mean corpuscular volume 106 μm3 Serum K+ 4.1 mEq/L Urea nitrogen 8 mg/dL Creatinine 1.1 mg/dL Glucose 110 mg/dL Which of the following is the most likely diagnosis? Diabetic neuropathy Spinal stenosis Hypothyroidism Pulmonary osteoarthropathy Vitamin B12 (cobalamin) deficiency

Vitamin B12 (cobalamin) deficiency

A 77-year-old woman is admitted to the hospital because of difficulty walking. She has had progressive pain and paresthesia of bothfeet over the past 3 weeks. She has a history of mild hypertension treated with hydrochlorothiazide and hypothyroidism treated withthyroid replacement therapy. Her pulse is 80/min, respirations are 16/min, and blood pressure is 150/80 mm Hg. Neurologicexamination shows decreased ankle jerk reflexes bilaterally and decreased vibratory sense and proprioception in the lowerextremities. Laboratory studies show:Hemoglobin 10 g/dLLeukocyte count 11,000/mm3 with a normal differentialMean corpuscular volume 106 μm3SerumK+ 4.1 mEq/LUrea nitrogen 8 mg/dLCreatinine 1.1 mg/dLGlucose 110 mg/dLWhich of the following is the most likely diagnosis? Diabetic neuropathy Pulmonary osteoarthropathy Spinal stenosis Vitamin B12 (cobalamin) deficiency Hypothyroidism

Vitamin B12 (cobalamin) deficiency

A 57-year-old woman comes to the physician for a routinehealth maintenance examination. She takes a multivitaminsupplement and calcium (500 mg/d). She has followed avegan diet for 30 years. She exercises daily for 30 minutes.She does not smoke cigarettes or drink alcohol. There is afamily history of osteoporosis. She is 168 cm (5 ft 6 in) talland weighs 60 kg (132 lb); BMI is 21 kg/m2. Examinationshows no abnormalities. Bone densitometry showsevidence of low bone density. She prefers not to be treatedwith medications. The physician recommends that thepatient increase her daily dose of the calcium supplement.The most appropriate next step in management issupplementation with which of the following? Magnesium Vitamin D Zinc Vitamin C Fish oil

Vitamin D

A 38-year-old man is admitted to the hospital after sustaining a gunshot wound to the abdomen. Broad-spectrum antibiotic therapy is initiated for complications from fecal contamination of the peritoneal cavity. He has been taking warfarin since receiving an artificial heart valve 6 years ago because of endocarditis secondary to intravenous drug use. During the next 6 weeks, the warfarin dosage required to maintain his prothrombin time progressively decreases. Which of the following best explains this finding? Vitamin K deficiency caused by bacterial overgrowth in the small intestine Decreased INR caused by hepatitis B infection Septic shock caused by a facultative anaerobe such as Escherichia coli Vitamin K deficiency caused by depletion of the normal gut flora Septic shock caused by an anaerobic commensal such as Bifidobacterium species Vitamin K deficiency caused by depletion of the normal gut flora

Vitamin K deficiency caused by depletion of the normal gut flora

A 33-year-old woman who is right-handed is brought to the physician because of a 3-day history of progressive weakness and numbness of her arms and legs. Neurologic examination shows proximal and distal weakness of the upper and lower extremities. There is areflexia. Sensation to vibration and joint position is decreased in the fingers and toes. Nerve conduction studies show a slow conduction velocity in the median, ulnar, peroneal, and tibial nerves. These electrophysiologic findings most likely indicate impaired function of which of the following ion channels? Neurotransmitter-gated Na+ channels Voltage-gated Ca2+ channels Voltage-gated K+ channels Neurotransmitter-gated K+ channels Neurotransmitter-gated Ca2+ channels Voltage-gated Na+ channels

Voltage-gated Na+ channels

A 55-year-old man comes to the physician because of a 3-day history of fever, chills, nausea, vomiting, and diarrhea. He says he has been unable to eat or drink anything without vomiting and has had little urine production during the past 12 hours. His temperature is 37.8°C (100°F). His pulse is 92/min, and blood pressure is 110/70 mm Hg while supine; pulse is 110/min, and blood pressure is 80/60 mm Hg while standing. Physical examination shows dry mucous membranes and a soft abdomen with mild, diffuse tenderness. Laboratory studies show: Leukocyte count 7200/mm 3 Serum Na+: 146 mEq/L Serum urea nitrogen: 50 mg/dL Serum creatinine: 2.2 mg/dL Serum uric acid: 10.2 mg/dL Urine Specific gravity: 1.030 Urine RBC: 0/hpf Urine Protein: negative Urine Na+: 10 mEq/L Urine creatinine: 19 mg/dL Which of the following is the most likely cause of the laboratory findings in this patient? Interstitial nephritis Acute tubular necrosis Bladder outlet obstruction Membranous glomerulonephritis Volume depletion

Volume depletion

A 14-year-old girl is brought to the physician because of a 4-month history of heavy menstrual flow. Menarche occurred at the age of 13 years. Menses have occurred at regular 30-day intervals. She has a history of frequent nosebleeds and easy bruisability. Her father has had problems with nosebleeds and clotting. Physical examination shows areas of mild gum bleeding and mild ecchymoses. Laboratory studies show: Which of the following is the most likely diagnosis? Hemophilia B Hemophilia A Thrombotic thrombocytopenic purpura Von willebrand disease Vitamin K deficiency

Von willebrand disease

Two days after sustaining an acute myocardial infarction, a 65-year-old man develops an acute atrioventricular block. Despite resuscitative efforts and the placement of a pacemaker, the patient dies. An autopsy is requested. Ischemic injury to the atrioventricular node is the suspected cause of this patient's atrioventricular block. Evaluation of a section from which of the following locations is most appropriate to evaluate this patient's atrioventricular node? Interatrial septum near the attachment of the septal cusp of the tricuspid valve Wall of the left atrium between the orifices of the right and left pulmonary veins Wall of the right atrium along the orifice of the superior vena cava Superior border of the muscular portion of the interventricular septum Wall of the right atrium along the orifice of the interior vena cava

Wall of the right atrium along the orifice of the interior vena cava

Which of the following drugs will lead to fetal nasal deformities (i.e., flattening of nose to face) and stippling of the epiphyses? Heparin Nitroglycerin Clonidine Hydrochlorothiazide Warfarin

Warfarin

A 40-year-old woman with a 6-month history of episodic sinusitis comes to the physician because of a 2-week history of intermittent headaches, fatigue, and generalized joint pain. She has had a worsening cough that has recently become productive of blood-tinged sputum. This is her fourth vi sit to the physician in the past 4 months. Therapy with antibiotics, decongestants, and nasal corticosteroids has not resulted in improvement in her sinus-related symptoms. Physical examination shows erythema of the nasal mucosa and two small ulcerations. Auscultation of the lungs shows scattered crackles. An x-ray of the chest shows patchy opacities bilaterally. Laboratory studies show: Which of the following is the most likely diagnosis? Allergic bronchopulmonary aspergillosis Sarcoidosis Tuberculosis Wegener granulomatosis Churg-Strauss syndrome

Wegener granulomatosis

A 32-year-old man begins to laugh while eating dinner with his friends. A small particle of food irritates his larynx and provokes him to cough. Which of the following best describes the position of his vocal cords throughout this sequence? While swallowing: closed, immediately after laryngeal irritation: closed, while coughing: open While Swallowing Laryngeal Irritation While Coughing

While swallowing: closed, immediately after laryngeal irritation: closed, while coughing: open

A 32-year-old man comes to the physician because of a 6-month history of low back pain and stiffness, which are worse in the morning and improve during the day; the pain radiates to his buttocks but not down his legs. He also develops back stiffness if he sits for prolonged periods. Physical examination shows that the patient is unable to bend forward completely. Which of the following is most likely to confirm the diagnosis? DEXA scan Measurement of erythrocyte sedimentation rate X-rays of sacroiliac joints Electromyography of the lower extremities Serum antinuclear antibody assay

X-rays of sacroiliac joints

Which of the following is consisent with chromosomal location of the gene responsible for this disorder? 7q31.2 4p16.3 11p15.4 Xq27.3 Xp21.1 - p21.2

Xp21.1 - p21.2

An 18-year-old man with acute lymphoblastic leukemia comes to the physician because of a 3-day history of intermittent fever and left-sided chest pain. He is in the fifth week of induction chemotherapy consisting of asparaginase, daunorubicin, prednisone, and vincristine. His absolute neutrophil count has been less than 500/mL for the past month. His temperature is 39.2°C (102.6° F). Physical examination shows dullness to percussion and decreased breath sounds on the left side. A chest x-ray shows a left lower lobe infiltrate and a moderate pleural effusion on the left. A photomicrograph of a periodic acid-Schiff stain of pleural fluid obtained via thoracentesis is shown. Which of the following is the most appropriate pharmacotherapy for this patient? amphotericin B Amphotericin B B) Ertapenem C) Ganciclovir D) infliximab AND) Rituximab

amphotericin B

A 14-year-old girl with type 1 diabetes mellitus is brought to the emergency department because of a 4-hour history of lethargy, confusion, and disorientation. She has vomited twice during this period, and she says she is thirsty and that her stomach hurts. The symptoms developed gradually overnight after she did not take her usual insulin dose during a sleepover at her friend's house. Her pulse is 110/min; respirations are 24/min, deep, and rapid; and blood pressure is 95/75 mm Hg. Laboratory studies show a serum glucose concentration of 450 mg/dl and arterial pH of 7.15. Arterial blood gas analysis on room air most likely includes which of the following additional findings? arterial pco2; decrease, arterial hco3 decrease, anion gap increase

arterial pco2; decrease, arterial hco3 decrease, anion gap increase

A 12-year-old boy is swimming in a mountain stream. He is immersed up to his neck in 60° F water for 20 minutes. Which of the following sets of physiologic changes is most likely to occur in this boy? blood vol: increase, ADH; decrease, ANP: increase

blood vol: increase, ADH; decrease, ANP: increase

A 72-year-old woman is admitted to the intensive care unit after undergoing immediate operative repair of a ruptured colonic diverticulum. During the next 2 weeks, she develops recurrent fevers. After a blood culture grows yeast, micafungin therapy is initiated. This drug most likely targets which of the following cellular structures? cell wall Transcription complex Kinetoplast Endospore Ribosome

cell wall

A 39-year-old woman with end-stage renal disease caused by systemic lupus erythematosus undergoes kidney transplantation from a living, unrelated donor. Despite aggressive immunosuppressive therapy, the transplant is rejected after 6 months. Analysis of a biopsy specimen of the transplant shows numerous infiltrating CD8+ T lymphocytes. Which of the following is the most likely cause of rejection of the transplant in this patient? Antibody synthesis Nitric oxide production Histamine release Complement activation direct cytotoxicity

direct cytotoxicity

A healthy 25-year-old man participates in a study of muscle function. The electrophysiologic observations made on a muscle biopsy specimen are shown. Via iontophoresis, 1 μM acetylcholine (ACh) was applied to the muscle surface. Extracellular Ca2+ concentration was decreased to prevent end-plate potentials (epp) from acting as a suprathreshold for muscle action potentials. The end-plate potential (epp) and miniature epp. In normal muscle the epp amplitude, mepp amplitude, and response to 1 um ACh (in mV) were recorded as follows: 10 mV, 1 mV and 1 mV, respectively. Based on these findings, which of the following electrophysiologic characteristics is expected in a muscle biopsy specimen from a patient with acute botulism? epp = 5 mV; meep = 0.2 mV; ACh = 0.1 uM epp = 15 mV; meep = 1 mV; ACh = 1.0 uM epp = 2 mV; meep = 0.2 mV; ACh = 0.2 uM epp = 1 mV; meep = 1 mV; ACh = 1.0 uM epp = 1 mV; meep = 0.5 mV; ACh = 0.5 uM

epp = 1 mV; meep = 0.5 mV; ACh = 0.5 uM

A 64-year-old woman comes to the physician because of a 6-month history of heartburn and difficulty swallowing solid food and liquid. She has not had nausea or vomiting. She also has a 2-year history of pain and swelling of her wrists and hands. She has not seen a physician in 5 years. She takes no medications. She does not smoke. Physical examination shows no wrinkles on the face and neck. Her vital signs are within normal limits. There is synovial thickening of the wrists bilaterally. A photograph of the left hand is shown. Abdominal examination shows no abnormalities. Which of the following sets of findings is most likely on esophageal manometry? Normal increased Increased increased Increased decreased Normal decreased Decreased increased esophageal peristalsis: decreased LES tone: decreased

esophageal peristalsis: decreased LES tone: decreased

A 35-year-old man comes to the physician for a routine examination. He has a 5-year history of recurrent episodes of genital herpes treated with acyclovir. He says, "I've been taking the medication, but I still occasionally get lesions. When will I be cured?" The physician informs him that herpes simplex is a lifelong infection, with no cure at the present time. Which of the following best explains the longevity of the herpes simplex virus? Continual replication in epithelial cells of the skin Continual replication in sacral root ganglia Continual replication in dendritic cells establishment of latent infection in sensory nerve cells Establishment of a latent infection in B lymphocytes Continual replication in T lymphocytes

establishment of latent infection in sensory nerve cells

A 3-year-old boy is brought to the emergency department because of multiple ecchymoses, bleeding into the joints, and bloody diarrhea 48 hours after ingesting an unknown quantity of warfarin that he took from his grandmother's purse. Prothrombin time is 80 sec (INR=20). Which of the following is the most appropriate initial therapy? Platelet concentrates fresh frozen plasma Factor IX (plasma thromboplastin component) concentrate Vitamin B12 (cyanocobalamin), parenterally Fibrinogen concentrate

fresh frozen plasma

A 65-year-old man with cancer of the cecum is found to have a metastatic lesion in the right lobe of the liver. Which of the following is the most likely venous route of metastasis? ileocolic, colic, superior mesenteric, portal, right hepatic branch of portal

ileocolic, colic, superior mesenteric, portal, right hepatic branch of portal

A 2-month-old boy dies of meningitis despite appropriate antibiotic therapy. Culture of his cerebrospinal fluid grows Mycobacterium tuberculosis. Two of his brothers died of atypical myobacterial infections. Which of the following immunodeficiencies most likely explains this familial history? X -linked agammaglobulinemia Complement deficiency Leukocyte adhesion deficiency interferon-y receptor deficiency

interferon-y receptor deficiency

A 35-year-old woman is brought to the emergency department by her husband because she passed out while washing dishes. The patient says she began to feel weak and dizzy but does not remember anything else. While the nursing staff is checking her vital signs and drawing blood, the husband tells the physician that the patient has bulimia nervosa and that she has been binging more frequently over the past 3 weeks. He suspects that she has been vomiting. Which of the following patterns of abnormal laboratory test results most likely indicates recurrent vomiting in this patient? k+ decrease, hco3-; increase, anion gap; normal, ph; increase

k+ decrease, hco3-; increase, anion gap; normal, ph; increase

A 54-year-old woman comes to the physician because of a 5-day history of severe mid-back pain. Physical examination shows point tenderness over the T6 vertebra. Serum studies show a calcium concentration of 13.4 mg/dL. Urinalysis is positive for Bence Jones proteins. Which of the following is the most likely cause of this patient's hypercalcemia? Increased fractional calcium gastrointestinal absorption local IL-1 and tumor necrosis factor effects Excessive parathyroid hormone production Unregulated 1 ,25-dihydroxycholecalciferol production

local IL-1 and tumor necrosis factor effects

A 45-year-old homeless man is brought to the emergency department by police 30 minutes after he was found unconscious. His breath and clothes smell of alcohol. His temperature is 36.8°C (98.2° F), pulse is 68/min, respirations are 14/min, and blood pressure is 110/55 mm Hg. Physical examination shows bronzed skin and spider angiomata on the chest. Laboratory studies show: Hemoglobin 10 g/dL Hematocrit 30% Mean corpuscular volume 110 μm 3 Leukocyte count 9000/mm3 Segmented neutrophils 70% Lymphocytes 20% Monocytes 10% Platelet count 160,000/mm3 Serum Ferritin: 200 ng/ml Serum Vitamin B12 (cobalamin): 500 pg/ml (N=160-950) Serum RSC folate: 20 ng/ml (N=166-640) A peripheral blood smear shows occasional hypersegmented neutrophils and 3+ oval macrocytes. Serum studies are most likely to show which of the following sets of additional findings? methylmalonic acid; normal, homocysteine; increased

methylmalonic acid; normal, homocysteine; increased

A 32-year-old man is brought to the emergency department in a coma. Drug overdose is suspected. Pulse is 68/min, respirations are 8/min and shallow, and blood pressure is 130/78 mm Hg. Lung fields are clear on percussion and auscultation. Heart rate is regular with normal heart sounds. Which of the following sets of arterial blood values are most likely to be seen in this patient while breathing room air? pH: 7.28 pO2: 50 Pco2: 60 HCO3-: 27

pH: 7.28 pO2: 50 Pco2: 60 HCO3-: 27

An 18-year-old woman is brought to the emergency department 12 hours after she ingested approximately 100 aspirin tablets in a suicide attempt. Physical examination shows tachypnea. Which of the following sets of laboratory findings would most likely be found in this patient? pH: 7.32, pCO2: 15, HCO3-: 8

pH: 7.32, pCO2: 15, HCO3-: 8

A 27-year-old man with a sleep disorder enrolls in a research study. During sleep evaluation, he is noted to snore loudly and stopbreathing for prolonged periods of 30 to 40 seconds. Toward the end of one of these apneic periods, arterial blood gas analysis isdone. Which of the following sets of findings is most likely in this patient? pH: Normal, PCO2: Decreased, PO2: Increased pH: Decreased, PCO2: Increased, PO2: Decreased pH: Increased, PCO2: Decreased, PO2: Normal pH: Decreased, PCO2: Decreased, PO2: Normal pH: Normal, PCO2: Increased, PO2: Increased

pH: Decreased, PCO2: Increased, PO2: Decreased

A 39-year-old man with polycystic kidney disease is brought to the physician for a follow-up examination. He has a 6-month history of intermittent blood in his urine. His temperature is 37°C (98.6° F), pulse is 100/min, respirations are 24/min, and blood pressure is 160/90 mm Hg. Physical examination shows no other abnormalities. His serum urea nitrogen concentration is 100 mg/dL, and serum creatinine concentration is 8 mg/dL. Urinalysis shows blood. Arterial blood gas analysis on room air is most likely to show which of the following sets of findings? pH; 7.22, pco2; 28, hco3-; 11

pH; 7.22, pco2; 28, hco3-; 11

A 22-year-old woman is admitted to the hospital because of a 10-day history of polydipsia and polyuria. She says that the urge to urinate often awakens her at night. She has been taking lithium carbonate for 2 years for bipolar disorder; her dosage was increased 6 months ago because of recurrent severe manic episodes. Her vital signs are within normal limits. Physical examination shows no abnormalities. Over the next 24 hours, urine excretion totals 6.5 L. Laboratory studies at this time show a serum sodium concentration of 148 mEq/L, serum osmolality of 315 mOsmol/kg, and urine osmolality of 75 mOsmol/kg. After administration of desmopressin, urine output and osmolality do not change. Which of the following findings in the nephron best describes the tubular osmolality, compared with serum, in this patient? proximal tubule: hypertonic jga: isotonic medullary cd: hypotonic proximal tubule: isotonic jga: isotonic medullary cd: hypotonic proximal tubule: hypotonic jga: hypotonic medullary cd: hypotonic pct: isotonic jga: hypotonic medullary cd: hypotonic proximal tubule: hypotonic jga: hypotonic medullary cd: hypertonic

pct: isotonic jga: hypotonic medullary cd: hypotonic

A 39-year-old female presents for perinatal amniocentesis at 11-weeks gestation. Karyotyping reveals 46,XX with a large satellite region on chromosome 21. Further analysis reveals a similar satellite region on parental karyotyping and part of the nucleolar organizing region (NOR). The NOR is most likely associated with which of the following? tRNA rRNA siRNA Poly A RNA mRNA

rRNA

A 15-year-old girl comes to the physician because of a 3-month history of acne. Breast and pubic hair development began at the age of 12 years. Menarche occurred at the age of 14 years. Physical examination shows scattered open and closed comedones over the cheeks and forehead. Breast and pubic hair development are Tanner stage 5. Which of the following is the most likely underlying cause of this patient's acne? inc. estrogen stimulation of the sebaceous glands decreased parasympathetic stimulation to the sebaceous glands stimulation of the sebaceous glands by androgens inc. responsiveness of the sebaceous glands to follicle-stimulating hormone inc. sympathetix stimulation to the sebaceous glands

stimulation of the sebaceous glands by androgens

A 6-year-old boy who recently emigrated from Russia is brought to the physician by his parents because of unstable gait and incoordination for 2 weeks. He has had frequent pale, bulky stools for 4 years and two episodes of bacterial pneumonia and chronic cough since the age of 1 year. He is below the 3rd percentile for height and weight. Increased rhonchi are heard over both lung fields. Neurologic examination shows ataxia, absence of deep tendon reflexes, and loss of proprioception. Stool analysis shows an increased fat concentration. Which of the following vitamins is most likely deficient in this patient? Vitamin D Biotin vitamin E Vitamin C Niacin

vitamin E


Related study sets

Pediatric Nursing Final Exam Study Set

View Set

Unit 10: Quiz 2- Human Geography

View Set

Chapter 29: Sedative-Hypnotic Drugs

View Set

C++ Chapter 13: Overloading and Templates

View Set